Sunteți pe pagina 1din 382

FCPS part 1 BCQs

GOLJAN PATHOLOGY BCQs WITH ANSWERS AND EXPLANATIONS

A 2-day-old newborn male infant with respiratory distress syndrome (RDS) has a continuous harsh murmur that is heard over the entire precordium. Which of the following sets of oxygen saturation (Sao2) values in the cardiac chambers and vessels is most likely present in this patient?

RARVPAPVLVA0 Normal SaO275 75 75 95 95 95 A 75 75 75 95 95 95 B 75 80 80 95 95 95 C 80 80 80 95 95 95 D 75 75 85 95 95 95 E 75 75 75 95 80 80 RA, right atrium; RV, right ventricle; PA, pulmonary artery; PV, pulmonary vein; LV, left ventricle; Ao, aorta.

A. Aortic regurgitation B. Aortic stenosis C. Hypertrophic cardiomyopathy D. step-up of SaO2 in pulmonary artery E. Mitral valve prolapse (MVP)

Option D (step-up of SaO2 in pulmonary artery) is correct. The patient has the classic machinery murmur (continuous murmur) of a patent ductus arteriosus (PDA). This neonate has hypoxemia (decreased arterial Po2) secondary to RDS; therefore, closure of the ductus is not stimulated. When oxygenated blood (Sao2 95%) is shunted into a chamber or vessel with venous blood (Sao2 75%), there is a step-up of Sao2 (approximately 80%) in the venous blood; this is called a left-to-right shunt. Similarly, when venous blood is shunted into a chamber or vessel with oxygenated blood, there is a step-down of the Sao2 (80%) leading to clinical cyanosis; this is called a right-to-left shunt. In PDA, there is a left-toright shunt causing blood to flow from the aorta (where pressure is high) through the PDA to the pulmonary artery (where pressure is low), which causes a step-up of Sao2 in the pulmonary artery (85% versus normal of 75%). Option A (Aortic regurgitation) is incorrect. The patient has a heart murmur consistent with a PDA; therefore, there should be a step-up of SaO2 in the pulmonary artery. Option B (Aortic stenosis) is incorrect. A step-up of Sao2 in the right ventricle (80% versus 75%) and pulmonary artery (80% versus 75%) characterizes a ventricular septal defect (VSD), a left-to-right shunt. VSD is the most common type of congenital heart disease. If a VSD is not corrected, cyanosis, or Eisenmengers syndrome, may eventually occur. This is due to volume overload in the right side of the heart from the left-to-right shunt causing pulmonary hypertension and right ventricular hypertrophy. When pressure in the right heart is greater than the pressure in the left heart, the shunt reverses and becomes right-to-left, causing a step-down in SaO2 in the left ventricle and aorta leading to cyanosis if SaO2 < 80%.

Option C (Hypertrophic cardiomyopathy) is incorrect. A step-up of Sao2 in the right atrium (80%), right ventricle (80%), and pulmonary artery (80%) characterizes an atrial septal defect (ASD, left-to-right shunt), which is most often the result of a patent foramen ovale. If an ASD is not corrected, cyanosis and Eisenmengers syndrome may eventually occur. Option E (Mitral valve prolapse (MVP)) is incorrect. A step-down of Sao2 in the left ventricle (80%) and aorta (80%) characterizes tetralogy of Fallot (right-to-left shunt), which is the most common type of cyanotic congenital heart disease. It consists of an overriding aorta (least common defect), VSD, pulmonary stenosis, and right ventricular hypertrophy. The degree of pulmonic stenosis determines the severity of the right-to-left shunt. If the stenosis is not severe, then most of the venous blood enters the pulmonary artery and is oxygenated; hence, the patient is often acyanotic (SaO2 > 80%). However, when the stenosis is severe, most of the venous blood is shunted through the VSD into the left ventricle (right-to-left shunt), leading to cyanosis. Note that the pulmonary vein has normally oxygenated blood (95%), because once blood enters the lungs through the stenotic pulmonic valve it is oxygenated.

A 45-year-old woman complains of a malodorous vaginal discharge and bleeding after coitus. She has not had a cervical Pap smear for the past 10 years and is sexually active. She has a 40-pack-year history of smoking cigarettes and has been taking oral contraceptives for 20 years. The photograph shows the cervix. Which of the following is the pathogenesis of this lesion?

A. Chlamydia trachomatis infection

B. Exposure to diethylstilbestrol (DES) C. Herpes simplex virus type 2 (HSV-2) infection D. Human papillomavirus (HPV) infection E. Unopposed estrogen stimulation

Option D (Human papillomavirus (HPV) infection) is correct. The cervix shows an ulcerated and bleeding mass on the external os. The patient's history of bleeding after coitus and the malodorous discharge are common presentations for cervical cancer. Most cervical cancers are associated with high-risk types of HPV (e.g., type 16 or 18) transmitted by sexual intercourse. Cigarette smoking and oral contraceptive pills are additional risk factors for cervical squamous cancer. A key point is that this could have been prevented if the patient had cervical Pap smears performed on a yearly basis to identify cervical squamous dysplasia, the precursor lesion for squamous cancer. Option A (Chlamydia trachomatis infection) is incorrect. C. trachomatis produces cervicitis; however, it is not a risk factor for cervical squamous cell cancer. Option B (Exposure to diethylstilbestrol (DES)) is incorrect. The daughter of a woman who received DES while pregnant is predisposed to clear cell adenocarcinoma involving the upper third of the vagina and cervix. There is nothing in the history to suggest that her mother was exposed to DES; therefore, this option can be discarded. Option C (Herpes simplex virus type 2 (HSV-2) infection) is incorrect. HSV-2 produces cervicitis; however, it is not a risk factor for cervical squamous cell cancer. Option E (Unopposed estrogen stimulation) is incorrect. Increased exposure to estrogen without progesterone predisposes a woman to endometrial adenocarcinoma not squamous cell carcinoma of the cervix. Oral contraceptives actually decrease the risk for developing endometrial adenocarcinoma.

The photograph shows a skin lesion on a mentally retarded 25-yr-old man who also has reddish-brown papules on his nose, cheeks, and chin. What additional findings may be present in this patient?

A. Acoustic neuroma B. Arteriovenous malformation in meninges C. Meningioma D. Renal cell carcinoma E. Rhabdomyoma of the heart

Option E (Rhabdomyoma of the heart) is correct. The patient has tuberous sclerosis, an autosomal-dominant neurocutaneous disorder. It is characterized by mental retardation, seizures beginning in infancy, adenoma sebaceum (red-brown papules on the face), hypopigmented skin lesions (ash leaf lesions; see photograph) and various hamartomatous lesions. Astrocyte proliferations occur in the subependyma and are the cause for seizure activity. Angiomyolipomas commonly occur in the kidneys, and rhabdomyomas may occur in the heart (almost 100% predictive of tuberous sclerosis). Option A (Acoustic neuroma) is incorrect. Neurofibromatosis is also a neurocutaneous syndrome like tuberous sclerosis and the type 2 variant is associated with acoustic neuromas, often bilateral. Option B (Arteriovenous malformation in meninges) is incorrect. Sturge Weber syndrome is a neurocutaneous syndrome with a vascular malformation of the face in a trigeminal nerve distribution and ipsilateral arteriovenous malformation in the meninges. Option C (Meningioma) is incorrect. Neurofibromatosis has an increase in meningiomas in the brain. The lesions in the

brain in tuberous sclerosis are astrocyte hamartomas. Option D (Renal cell carcinoma) is incorrect. Angiomyolipomas of the kidney do occur in tuberous sclerosis; however, they are hamartomas and do not progress into a renal cell carcinoma.

A 28-year-old woman who has been taking oral contraceptives for the past 10 years suddenly develops abdominal pain and dizziness while exercising. In the emergency department, physical examination shows that the patient has hypotension and a distended abdomen. A pregnancy test is negative. Surgical exploration shows blood in the abdominal cavity. Which of the following is the most likely cause of the intra-abdominal hemorrhage?

A. Hepatocellular carcinoma B. Ruptured abdominal aortic aneurysm C. Ruptured cavernous hemangioma of the liver D. Ruptured ectopic pregnancy E. Ruptured liver cell adenoma
Option E (Ruptured liver cell adenoma) is correct. Liver cell (hepatic) adenomas are a complication of oral contraceptive use. These benign tumors arise from hepatocytes and have a tendency to rupture and cause intraperitoneal hemorrhage. Option A (Hepatocellular carcinoma) is incorrect. The risk of hepatocellular carcinoma is not increased with the use of oral contraceptives. Furthermore, the majority of cases of hepatocellular carcinoma arise in a pre-existing cirrhosis of the liver. Option B (Ruptured abdominal aortic aneurysm) is incorrect. The majority of abdominal aortic aneurysms are due to atherosclerosis and occur in men > 50 years of age. Therefore, ruptured abdominal aortic aneurysm is not a likely cause of the intra-abdominal hemorrhage in this young woman. Option C (Ruptured cavernous hemangioma of the liver) is incorrect. Although cavernous hemangiomas are the most common benign tumors of the liver and can rupture, they do not have an association with previous use of oral contraceptives. Option D (Ruptured ectopic pregnancy) is incorrect. Ruptured ectopic pregnancy is the most common cause of intraabdominal hemorrhage in young women. However, the pregnancy test is negative, which excludes this diagnosis.

Study the following data on a test performed on 100 patients, who are known to have disease Y, and a control group containing 100 people who do not have disease Y. Which of the following conclusions can be drawn from this study?

Disease YControl Group Positive test 70 0 Negative test30 100

A. A negative test result excludes disease Y. B. A positive test result confirms disease Y. C. It is a poor test for confirming disease Y. D. The predictive value of a negative test result is 100%. E. The prevalence of disease is 60%. Option B (A positive test result confirms disease Y.) is correct. People with disease either have a true positive (TP) or a false negative (FN) test result. A TP test result is a positive test result in a person with disease, while a FN test result is a negative test result in a person with disease. People in a control group, who do not have disease, either have a true negative (TN) or a false positive (FP) test result. A TN test result is a negative test in a person without disease, while a FP test result is a positive test in a person without disease. Specificity of a test refers to the likelihood of having negative test results in people without disease. Since people without disease have test results that are either TN or FP, the formula for calculating the specificity of a test is TN/TN+FP. A test with no FPs has 100% specificity. The predictive value of a positive test result (PV+) is the likelihood that a positive test result is a TP rather than a FP; therefore, the formula for calculating the PV+ is TP/TP+FP. Tests with 100% specificity always have a PV+ of 100%; therefore, they are most useful for confirming disease. The specificity of the test for disease Y is 100/100+0 = 100%, and the PV+ is 70/70+0 = 100%. Disease YControl Group Positive test 70 TP 0 FP Negative test30 FN 100 TN

Option A (A negative test result excludes disease Y.) is incorrect. The sensitivity of a test is the likelihood of having positive test results in patients who have a selected disease. Since people with disease either have TP or FN test results, the formula for calculating the sensitivity of a test is TP/ TP+FN. The less the FN rate, the greater the sensitivity of the test. Tests with 100% sensitivity are most often used as screening tests for disease. If the test result is negative, the likelihood (predictive value of a negative test result, PV) is a TN rather than a FN is 100%, because a test with 100% sensitivity has no FNs. Since tests with 100% sensitivity always have a PV (TN/TN+FN) of 100%, a negative test result excludes disease. The sensitivity of the test for disease Y is 70/70+30 = 70%, and the PV is 100/100+30 = 77%; therefore, a negative test result does not exclude disease Y (likelihood of a FN is 23%). Option C (It is a poor test for confirming disease Y.) is incorrect. The test is an excellent test to confirm disease, because the specificity is 100% and the PV+ is 100%. Option D (The predictive value of a negative test result is 100%.) is incorrect. The PV is 100/100+30 =

77% (not 100%). Option E (The prevalence of disease is 60%.) is incorrect. Prevalence is the total number of people with disease in the population under study. The formula is TP+FN/TP+FN+TN+FP. The prevalence of disease in this study is 100+0/100+0+70+30 = 50% (not 60%).

The photograph shows a leukocyte abnormality in a 3-year-old boy, who has recurrent Staphylococcus aureus infections. Specialized studies show bactericidal and chemotaxis defects. What is the most likely diagnosis?

A. Chediak-Higashi syndrome (CHS) B. Chronic granulomatous disease (CGD) of childhood C. I cell disease D. Job syndrome E. Myeloperoxidase deficiency
Option A (Chediak-Higashi syndrome (CHS)) is correct. CHS is an autosomal recessive disease with a defect in a lysosomal transport protein that affects the synthesis and/or maintenance of storage of secretory granules in various cells (e.g., lysosomes in leukocytes, azurophilic granules in neutrophils, dense bodies in platelets). Granules in these cells tend to fuse together to become megagranules (see photograph). In addition, there is a defect in microtubule function in neutrophils and monocytes that prevents the fusion of lysosomes with phagosomes to produce phagolysosomes. This produces a bactericidal defect. In particular, there is increased susceptibility to developing S. aureus infections. Microtubular dysfunction also produces defects in chemotaxis (directed migration), which further exacerbates the susceptibility to infection. Option B (Chronic granulomatous disease (CGD) of childhood) is incorrect. CGD is an X-linked recessive disease caused by absence of nicotinamide adenosine dinucleotide phosphate oxidase in the O2-dependent myeloperoxidase (MPO) system for killing bacteria. Lack of this enzyme causes a defect in producing a respiratory (oxidative) burst in neutrophils and monocytes. There are no cytoplasmic megagranules in leukocytes in CGD.

Option C (I cell disease) is incorrect. Inclusion (I) cell disease is a rare, inherited condition where there is a defect in post-translational modification of lysosomal enzymes in the Golgi. Mannose residues on newly synthesized lysosomal enzymes coming from the rough endoplasmic reticulum are not phosphorylated due to a deficiency of phosphotransferase. Without mannose-6-phosphate to direct the enzymes to lysosomes, vesicles that pinch off the Golgi empty the unmarked enzymes into the extracellular space where they are degraded in the bloodstream. Undigested substrates (e.g., carbohydrates, lipids, and proteins) accumulate as large inclusions in the cytosol of mesenchymal cells, particularly fibroblasts (not leukocytes). Symptoms include psychomotor retardation and early death. Option D (Job syndrome) is incorrect. It is an autosomal recessive disorder of neutrophils, characterized by abnormal chemotaxis leading to cold soft tissue abscesses due to S. aureus. Patients have red hair, a leonine face, chronic eczema, and increased IgE (hyperimmune E syndrome). Megagranules are not present in leukocytes. Option E (Myeloperoxidase deficiency) is incorrect. MPO catalyzes a reaction that combines H2O2 with Cl to form hypochlorous free radicals that kill the phagocytosed bacteria. Deficiency of MPO results in a defect in microbicidal activity; however, it does not have megagranules in leukocytes or defects in chemotaxis.

The photograph shows a lesion (arrow) in the prostate gland removed at autopsy from a 72-year-old man with a history of urinary retention and a recent history of low back pain with point tenderness over the lower lumbar vertebrae. A radiograph of the lower lumbar vertebrae showed nodular densities in the vertebral bodies. Laboratory studies showed increased serum alkaline phosphatase and prostate-specific antigen. Which of the following was the most likely cause of the lower lumbar pain?

A. Osteoarthritis B. Osteoblastic metastasis C. Osteolytic metastasis

D. Osteomyelitis E. Osteoporosis
Option B (Osteoblastic metastasis) is correct. The photograph shows a yellow-colored, peripherally located cancer, whereas the central portion of the gland shows nodules consistent with prostate hyperplasia. Patients with prostate hyperplasia are more likely to present with urinary retention, because it produces urethral obstruction. Prostate cancer commonly metastasizes to the vertebral column, where it produces bone pain. Metastases are usually osteoblastic (stimulates bone formation), which increases the serum alkaline phosphatase and shows increased bone density on radiographs. The prostate-specific antigen is also increased; however, this does not differentiate prostate hyperplasia from prostate cancer. Option A (Osteoarthritis) is incorrect. Osteoarthritis (degenerative arthritis) involves the cervical and lower lumbar vertebrae and produces sclerotic changes that can simulate osteoblastic bone metastases. Osteophytes (reactive bone formation) that develop at the margins of the joints do increase the serum alkaline phosphatase levels. However, osteoarthritis does not produce point tenderness over the vertebral column. Option C (Osteolytic metastasis) is incorrect. Osteolytic metastasis does not occur in prostate cancer. Lytic lesions (lucencies) are present in bone radiographs, and serum alkaline phosphatase is not increased. Option D (Osteomyelitis) is incorrect. Osteomyelitis involving the vertebral column is most often due to Mycobacterium tuberculosis. It produces lytic lesions of the vertebral column. Option E (Osteoporosis) is incorrect. Osteoporosis decreases the amount of mineralized bone and organic matrix causing a decrease in bone density and mass. It does not increase the serum alkaline phosphatase.

The photograph shows the posterior view of a stillbirth. The mother has an increased serum -fetoprotein level. Which of the following supplement regimes could have prevented this malformation?

A. Folic acid prior to pregnancy B. Prenatal vitamins throughout pregnancy C. Vitamin B6 prior to birth D. Vitamin B12 throughout pregnancy E. Vitamin C throughout pregnancy
Option A (Folic acid prior to pregnancy) is correct. The photograph shows a stillbirth with anencephaly. The entire brain is missing, and the spinal canal is open. Mothers carrying a fetus with anencephaly have increased serum -fetoprotein levels and polyhydramnios, because cerebrospinal fluid from the spinal canal is added to the amniotic fluid. Dietary folic acid deficiency is associated with an increased risk for neural tube defects. The neural tube develops very early in the embryonic period. Current recommendations suggest that all women take adequate levels of folic acid before becoming pregnant. Option B (Prenatal vitamins throughout pregnancy) is incorrect. Prenatal vitamins are taken after pregnancy has been confirmed. They contain iron and folic acid to provide the developing fetus adequate supplies of iron for hematopoiesis and folate for DNA synthesis. Folic acid must be present in adequate amounts before pregnancy to prevent open neural tube defects.

Option C (Vitamin B6 prior to birth) is incorrect. Vitamin B6 (pyridoxine) is not required for proper development of the neural tube. Option D (Vitamin B12 throughout pregnancy) is incorrect. Vitamin B12 is not required for proper development of the neural tube. Option E (Vitamin C throughout pregnancy) is incorrect. Vitamin C is not required for proper development of the neural tube.

A 69-year-old man has a dragging sensation in the right upper quadrant and crampy abdominal pain in the left lower quadrant. Over the past 6 months, he has experienced weight loss and alternating bouts of constipation and diarrhea. He has noticed blood coating and mixed in with stools when they are more solid. Physical examination shows an enlarged, nodular liver and external hemorrhoids. Rectal examination reveals blood mixed with stool. Laboratory studies show hemoglobin 9 g/dL, WBC count 6500/mm3, platelet count 500,000/mm3, mean corpuscular volume 75 (m3, and serum ferritin 3 ng/mL. Which of the following disorders best explains the presence of blood in the stool?

A. Angiodysplasia B. External hemorrhoids C. Hepatocellular carcinoma with metastasis to the colon D. Peptic ulcer disease E. Primary colorectal cancer

Option E (Primary colorectal cancer) is correct. The patient has primary colorectal cancer complicated by iron deficiency anemia (microcytic anemia and decreased serum ferritin) and metastasis to the liver (enlarged, nodular liver). Clinical findings highly predictive of colorectal cancer in this patient include weight loss, change in consistency of stools, and blood mixed in with stools. Option A (Angiodysplasia) is incorrect. Angiodysplasia commonly causes passage of large volumes of blood per rectum (hematochezia) that are capable of producing dizziness due to hypotension or hypovolemic shock. Angiodysplasia refers to the presence of dilated vascular channels in the submucosa of the cecum, which often rupture and produce massive lower gastrointestinal bleeding. The patient has blood mixed in with stool and signs of metastatic disease to the liver, which excludes angiodysplasia. Option B (External hemorrhoids) is incorrect. External hemorrhoids usually thrombose and do not commonly bleed. In addition, they would not explain the weight loss, liver findings, and change in bowel habits in the patient. Option C (Hepatocellular carcinoma with metastasis to the colon) is incorrect. Metastasis to the colon from any site is extremely rare and usually manifests as signs of obstruction rather than bleeding.

Option D (Peptic ulcer disease) is incorrect. Peptic ulcer disease produces epigastric distress with no changes in bowel habits. Black tarry stools (melena) are more likely to occur than blood coating and mixing with stools.

The photograph is of a 50-year-old woman who has repeated sinus infections, chronic cough, and dyspnea. She does not smoke. A chest radiograph shows multiple nodular masses in the lungs, some of which have central cavitation. Urinalysis shows numerous RBCs and RBC casts. Which of the following laboratory findings is most likely present?

A. Antistreptolysin O (ASO) antibodies B. c-Antineutrophil cytoplasmic antibodies (c-ANCA) C. Hepatitis B surface antigen (HBsAg) D. p-Antineutrophil cytoplasmic antibodies (p-ANCA) E. Serum antinuclear antibodies (ANA)
Option B (c-Antineutrophil cytoplasmic antibodies (c-ANCA)) is correct. The patient has Wegeners granulomatosis involving the upper airways (sinus infections), lungs (nodular masses with cavitation), and kidneys (hematuria and RBC casts). Wegeners granulomatosis causes a necrotizing vasculitis of muscular arteries and small vessels in the lungs and kidneys and necrotizing granulomas in the upper respiratory tract and lungs. The photograph shows a saddle nose deformity due to necrotizing granulomas destroying the nasal cartilage. c-ANCA is present in Wegeners granulomatosis in >90% of cases. The antibodies are directed against neutrophil granules in the cytosol. Destruction of neutrophils by cANCA releases enzymes that contribute to the inflammatory lesions. Crescentic glomerulonephritis is the renal lesion seen in Wegeners and rapidly progresses to renal failure. Corticosteroids and cyclophosphamide have markedly improved the survival rate in this disease. Option A (Antistreptolysin O (ASO) antibodies) is incorrect. ASO antibodies occur in infections associated with group A streptococcus (e.g., rheumatic fever). Rheumatic fever is an immunologic disease that primarily targets the heart, joints, skin, and basal ganglia. Option C (Hepatitis B surface antigen (HBsAg)) is incorrect. HBsAg is associated with polyarteritis nodosa in

approximately 30% of cases. Polyarteritis nodosa causes a vasculitis of muscular arteries (e.g., renal and coronary arteries), leading to vessel thrombosis and infarctions or aneurysm formation caused by weakening of the vessels. It does not affect the pulmonary arteries. It produces renal infarctions, not glomerulonephritis. Option D (p-Antineutrophil cytoplasmic antibodies (p-ANCA)) is incorrect. p-ANCA directed against myeloperoxidase in neutrophils is associated with microscopic polyangiitis. Microscopic polyangiitis causes vasculitis of small vessels (capillaries, venules, arterioles) in multiple target organs, including the lungs and kidneys (glomerulonephritis). It is not associated with necrotizing granulomas in the upper airways and lungs. Option E (Serum antinuclear antibodies (ANA)) is incorrect. Serum ANAs are directed against nuclear antigens. These antibodies are increased in autoimmune disorders (e.g., systemic lupus erythematosus) and are not present in Wegeners granulomatosis.

A 10-year-old girl develops fever, epistaxis, and oliguria 5 days after attending a picnic at which hamburgers and hot dogs are served. Physical examination shows scattered petechiae and ecchymoses on the arms, chest, and back. Laboratory studies show: Hemoglobin 9 g/dL WBC count 8000/mm3 Platelet count 30,000/mm3 Prothrombin time (PT) 12 sec Activated partial thromboplastin time (aPTT)35 sec Serum blood urea nitrogen (BUN) 40 mg/dL Serum creatinine 4 mg/dL

A peripheral smear shows fragmented RBCs. What is the most likely diagnosis?

A. Aplastic anemia B. Disseminated intravascular coagulation (DIC) C. Hemolytic uremic syndrome (HUS) D. Idiopathic thrombocytopenic purpura (ITP) E. Thrombotic thrombocytopenic purpura (TTP)
Option C (Hemolytic uremic syndrome (HUS)) is correct. The patient has HUS, which most commonly occurs in children. HUS is due to a toxin (Verotoxin) produced by the 0157:H7 strain of Escherichia coli, causing widespread endothelial cell damage at the arteriolecapillary junction. E. coli often contaminates improperly cooked beef (e.g., hamburgers). Platelet thrombi develop at all the injury sites, causing consumption of platelets and thrombocytopenia (epistaxis, petechiae, ecchymoses, easy bruising) as well as renal failure (increased serum BUN and serum creatinine). RBCs that collide with the platelet thrombi are damaged (fragmented RBCs or schistocytes), causing a hemolytic anemia (microangiopathic hemolytic anemia; see Fig. 11-35 in Rapid Review Pathology, 3rd edition).

Option A (Aplastic anemia) is incorrect. Aplastic anemia is characterized by decreased production of WBCs, RBCs, and platelets. Aplastic anemia is excluded in this patient, because the WBC count is normal. Option B (Disseminated intravascular coagulation (DIC)) is incorrect. In DIC, fibrin clots are produced in the microcirculation that obstruct blood flow and consume coagulation factors. Factors that are consumed in a fibrin clot are fibrinogen (factor I), prothrombin (factor II), and factors V and VIII. Since coagulating factors present in the final common pathway are consumed (fibrinogen, prothrombin, factor V), there is prolongation of the PT and aPTT (not present in this patient). Platelets are trapped in the fibrin clots leading to thrombocytopenia. Fragmented RBCs (schistocytes) are present in the peripheral blood, due to damage to RBCs by the fibrin clots. There are many overlapping features of DIC with HUS; however, in the former disorder coagulation factors are consumed causing prolongation of the PT and aPTT, while in the latter, platelets are consumed in the formation of platelet thrombi and coagulation studies are normal. Option D (Idiopathic thrombocytopenic purpura (ITP)) is incorrect. ITP is autoimmune destruction of platelets, where IgG antibodies are directed against the GpIIb:IIIa fibrinogen receptors on platelets. ITP is excluded in this patient because it is not associated with anemia and renal failure. Option E (Thrombotic thrombocytopenic purpura (TTP)) is incorrect. TTP occurs in women and is uncommon in children. The pathophysiology of TTP and HUS is similar. TTP is a disorder characterized by endothelial injury with the formation of platelet thrombi that consume platelets, not coagulation factors; therefore, the PT and PTT are normal (not prolonged).

Physical examination of a 72-year-old man shows severe hypertension, an epigastric bruit, and diminished amplitude of the pedal pulses. An angiogram of the renal arteries is ordered (see photograph; the arrow points to the right renal artery). Which of the following best characterizes the findings in this patient?

A. Activation of the renin-angiotensin-aldosterone (RAA) system in the left kidney

B. Aldosterone levels are decreased in the blood C. Plasma renin activity (PRA) is increased in the left renal vein D. Plasma renin activity (PRA) is increased in the right renal vein E. Renal artery stenosis is caused by fibromuscular hyperplasia
Option D (Plasma renin activity (PRA) is increased in the right renal vein) is correct. The patient has renovascular hypertension. In men older than 50 years of age, it is most often caused by an atherosclerotic plaque narrowing the orifice of the renal artery. The photograph shows an arrow pointing to proximal stenosis of the right renal artery with poststenotic dilation. A decrease in renal blood flow activates the RAA system, resulting in hypertension caused by renal retention of sodium by aldosterone and vasoconstriction of the peripheral resistance arterioles by angiotensin II. PRA is increased in the right renal vein and should be decreased in the left renal vein. This is due to an increase in plasma volume related to increased retention of sodium by aldosterone. An increase in plasma volume increases renal blood flow to the uninvolved kidney leading to suppression of the RAA system. Narrowing of the orifice of the renal artery produces an epigastric bruit, causing atrophy of the affected kidney. Option A (Activation of the renin-angiotensin-aldosterone (RAA) system in the left kidney) is incorrect. The RAA system is suppressed in the left kidney because there is an increase in plasma volume related to increased retention of sodium by aldosterone. An increase in plasma volume increases renal blood flow to the uninvolved kidney leading to suppression of the RAA system. Option B (Aldosterone levels are decreased in the blood) is incorrect. Aldosterone levels are increased due to activation of the RAA system in the right kidney. Option C (Plasma renin activity (PRA) is increased in the left renal vein) is incorrect. PRA is suppressed in the left kidney due to the increase in renal blood flow to the left renal artery. Option E (Renal artery stenosis is caused by fibromuscular hyperplasia) is incorrect. Fibromuscular hyperplasia of the renal arteries is the primary cause of renovascular hypertension in women between the ages of 30 and 50 years. Atherosclerosis is the most common cause in men.

A 39-year-old woman has difficulty swallowing liquids and solids. She has lost weight and vomits undigested food when she lies down at night. The photograph shows an upper gastrointestinal barium study. What is the most likely diagnosis?

A. Achalasia B. CREST syndrome C. Esophageal cancer D. Gastroesophageal reflux disease (GERD) E. Myasthenia gravis
Option A (Achalasia) is correct. The patient has achalasia, which is a motor disorder of the smooth muscle cells of the mid and distal esophagus. There is incomplete relaxation of the lower esophageal sphincter (LES) due to autoimmune destruction of ganglion cells in the myenteric plexus causing a decrease in nitric oxide (NO) synthase producing neurons. This decreases proximal smooth muscle contraction (peristalsis). There is dilation of the esophagus proximal to the LES with absent peristalsis (detected by manometry). Clinical findings include nocturnal regurgitation of undigested food; dysphagia for solids and liquids; and chest pain and/or heartburn. The photograph shows her barium study. The proximal esophagus is dilated and has a beaklike tapering at the distal end (arrow). Nonpharmacologic treatment includes pneumatic dilation and esophagomyotomy. Pharmacologic options include long-acting nitrates, calcium channel blockers, and botulinum toxin injection. There is an increased risk for esophageal cancer. Option B (CREST syndrome) is incorrect. CREST syndrome is a motor disorder of the smooth muscle cells in the mid and distal esophagus. A variant of systemic sclerosis, CREST syndrome is characterized by calcinosis (dystrophic calcification of the fingers), Raynauds phenomenon (color changes in the fingers), esophageal motility dysfunction, sclerodactyly (tapered fingers), and telangiectasia (dilated vascular channels in the skin). With CREST syndrome, the LES is relaxed, and there is no vomiting of undigested food. Option C (Esophageal cancer) is incorrect. Esophageal cancer is an obstructive lesion that produces dysphagia for solids but not liquids. Risk factors for esophageal cancer that are not present in this patient include cigarette smoking and alcohol use. Option D (Gastroesophageal reflux disease (GERD)) is incorrect. GERD is caused by relaxation of the LES, and acid

reflux damages the distal esophageal mucosa producing glandular metaplasia (Barretts esophagus). Stricture formation may occur in the distal esophagus; however, there is no loss of peristalsis. Option E (Myasthenia gravis) is incorrect. Myasthenia gravis is a motor disorder (IgG antibodies inhibit acetylcholine receptors) that weakens the striated muscle of the upper esophagus, causing dysphagia for solids and liquids. Mid and distal esophageal motility is normal, because smooth muscle is not affected.

A 32-year-old man reports that he was eating peanut butter cookies and began to feel itchy. He then developed a rash over most of his body. The lesions disappeared within 3 to 4 hours. He describes the rash as roughly circular, raised lesions with central clearing and red borders. The chemical mediator responsible for these lesions is derived from which of the following cells?

A. Eosinophils B. Lymphocytes C. Mast cells D. Neutrophils E. Platelets


Option C (Mast cells) is correct. The patient has urticaria (hives). The raised wheals are clear in the center and have red borders. The patient is most likely allergic to peanuts and has developed a type I hypersensitivity reaction with the release of histamine from mast cells in the skin. The mast cells have IgE antibodies against the allergen, and reexposure to the allergen causes mast cells to release preformed chemical mediators. Histamine causes arteriolar vasodilation and increases vessel permeability, leading to the formation of raised wheals. Option A (Eosinophils) is incorrect. Eosinophilia is a peripheral blood finding in type I hypersensitivity reactions; however, it is not the cell of origin of histamine. Option B (Lymphocytes) is incorrect. Lymphocytes do not contain histamine and have no role in type I hypersensitivity reactions. Option D (Neutrophils) is incorrect. Neutrophils do not contain histamine and have no role in type I hypersensitivity reactions. Option E (Platelets) is incorrect. Platelets contain histamine; however, they do not release histamine in response to an allergen.

A 25-year-old medical student returns from a trip to Mexico on spring break. Three weeks later, he develops low-grade fever and jaundice. Physical examination shows painful hepatomegaly. The urine is dark yellow and

the stool is light colored. Which of the following serum and urine laboratory test abnormalities would you expect? % CB Urine BilirubinUrine UBGAST ALT ALP GGT Normal Negative NormalNormalNormalNormal A. <20 Negative NormalNormalNormal B. >50 Negative C. 2050 D. 2050 E. <20 Negative Normal NormalNormalNormalNormal Number of arrows relates to the degree of elevation. ALP, alkaline phosphatase; ALT, alanine aminotransferase; AST, aspartate aminotransferase; CB, conjugated bilirubin; GGT, -glutamyltransferase; UBG, urobilinogen. A. A B. B C. C D. D E. E
Option D is correct. The student most likely has hepatitis A. Hepatitis produces a mixed jaundice (CB 2050%), due to a decrease in uptake and conjugation of unconjugated bilirubin (UCB) and destruction of bile ductules among hepatocytes causing release of CB. Urine UBG is increased, because UBG that is normally recycled back to the liver is redirected into the urine. Urine bilirubin is increased, because CB is water soluble. Serum transaminases are markedly increased because of massive liver cell necrosis; serum ALT is greater than AST. Serum ALP and GGT are only slightly increased. Option A is incorrect. The laboratory studies are most consistent with extravascular hemolytic anemias (e.g., hereditary spherocytosis, antibodies blood group hemolytic disease of the newborn), where RBCs are removed extravascularly by macrophages leading to an increase in UCB and a CB < 20%. AST is present in RBCs; therefore, there is a slight increase in serum AST. Urine UBG is increased due to increased production of CB by the liver and increased conversion of CB to UBG in the colon. Fecal UBG gives the color to stool. Normally, a small portion of UBG in the colon is reabsorbed back into the blood (enterohepatic circulation) where most is taken up by the liver and the remainder is filtered into the urine to give the color to urine. Therefore, if there is more UBG in the stool (e.g., extravascular hemolytic anemia), then proportionately more UBG ends up in the urine (darker yellow than normal). Option B is incorrect. The laboratory findings indicate an obstructive type of jaundice, either due to intrahepatic cholestasis (e.g., drug-induced oral contraceptives) or extrahepatic cholestasis. Since bile is predominantly CB, in obstructive jaundice this is the predominant fraction that enters the bloodstream (CB > 50%) through ruptured intrahepatic bile ductules. CB is water soluble; therefore, it enters the urine to produce a positive dipstick test for bilirubin. Since UBG normally present in the urine derives from enterohepatic circulation of UBG initially produced in the liver, absence of UBG in the stool in obstructive liver disease (light-colored stools) leads to an absence of UBG in the

urine. Enzyme markers for obstructive liver disease are serum ALP and serum GGT. Serum AST and ALT are also increased but to a lesser degree. Option C is incorrect. The laboratory findings are compatible with alcoholic hepatitis, which produces a mixed jaundice (CB 2050%), due to decreased uptake and conjugation of UCB and destruction of bile ductules among liver cells leading to an increase in CB in the blood. Urine UBG is increased, because UBG that is normally recycled back to the liver is redirected into the urine. Serum transaminases are increased with serum AST (mitochondrial enzyme) greater than ALT (cytosol enzyme). Since AST is normally located in the mitochondria of hepatocytes, alcohol, a mitochondrial toxin, causes preferential release of AST into the blood and is higher that ALT. Serum ALP is slightly increased; however, serum GGT is markedly increased, because alcohol induces enzyme synthesis in the hepatocyte cytochrome P450 system in the smooth endoplasmic reticulum (SER). GGT is also located in the SER and is increased as well. Therefore, key markers for alcoholic hepatitis are AST > ALT and marked increase in GGT. Option E is incorrect. The laboratory findings indicate an increase in UCB. This could be due to hemolytic anemias; however, destruction of RBCs also increases AST and urine UBG is increased. Hence, the most common cause of this profile is Gilberts disease, which is an autosomal dominant disease characterized by decreased uptake and conjugation of bilirubin particularly exacerbated by the fasting state. Baseline UCB levels are increased to over twice normal in the fasting state leading to visible evidence of jaundice (CB < 20%). Next to viral hepatitis, it is the second overall most common cause of jaundice in the United States. All enzyme studies and urine studies are normal. It has no clinical significance.

Shortly after a trip to the desert in the southwestern United States, a 58-year-old archaeologist, who explores caves for Native American artifacts, develops a dry cough, fatigue, and painful nodules in the lower extremities. The photographs show a biopsy of the lung and the lesions on the lower extremities. What is the most likely diagnosis?

A. Aspergillosis B. Coccidioidomycosis C. Cryptococcosis D. Histoplasmosis

E. Tuberculosis
Option B (Coccidioidomycosis) is correct. This patient has coccidioidomycosis caused by Coccidioides immitis. This infection occurs in desert regions of the southwestern United States, particularly in Arizona, New Mexico, and southern California. It is contracted by inhalation of arthrospores in the dust. The biopsy shows an intact spherule (arrow) containing endospores. The lesions on the lower legs represent erythema nodosum, which is commonly associated with coccidioidomycosis. It is caused by inflammation of the subcutaneous fat and presents with painful, red nodules usually located on the lower extremities. Option A (Aspergillosis) is incorrect. Aspergillus fumigatus produces narrow-angled septate hyphae with fruiting bodies. Option C (Cryptococcosis) is incorrect. Cryptococcus neoformans has yeast forms with narrow-based buds and a thick capsule. Option D (Histoplasmosis) is incorrect. Histoplasma capsulatum has yeast forms phagocytosed by macrophages. Although the archaeologist was exploring caves that likely contained bats, Histoplasma do not survive in dry climates and prefer the moisture-laden caves in the Ohio River and central Mississippi River valleys. Option E (Tuberculosis) is incorrect. Mycobacterium tuberculosis is an acid-fast bacterium that is usually phagocytosed by alveolar macrophages. Acid-fast stains are necessary to visualize M. tuberculosis.

Development of colorectal cancer from a tubular adenoma involves point mutations in which of the following genes?

A. APC, ABL, and RB B. APC, RAS, and TP53 C. BCL-2 and APC D. ERBB2 and TP53 E. MYC and APC

Option B (APC, RAS, and TP53) is correct. Development of colorectal cancer from a tubular adenoma follows a sequence of gene mutations, which occurs in the following order: (1) inactivation of the APC suppressor gene; (2) activation of the RAS proto-oncogene; and (3) inactivation of the TP53 suppressor gene. This sequence accounts for 80% of cases of sporadic colorectal carcinomas. This is an important concept, because cancer is rarely due to a single mutation but a certain sequence of mutations. Recall that the TP53 suppressor gene and RAS proto-oncogene are the two most common genes associated with cancer.

Option A (APC, ABL, and RB) is incorrect. The APC suppressor gene is associated with colorectal cancer. The ABL protooncogene mutation is primarily associated with chronic myelogenous leukemia. Inactivation of the RB suppressor gene is associated with retinoblastoma, osteogenic sarcoma, and breast cancer. Option C (BCL-2 and APC) is incorrect. Activation of the BCL-2 antiapoptosis gene is associated with B-cell follicular lymphomas. Inactivation of the APC suppressor gene is associated with colorectal cancer. Option D (ERBB2 and TP53) is incorrect. Activation of the ERBB2 proto-oncogene is associated with breast cancer. Inactivation of the TP53 suppressor gene does occur in the development of colorectal cancer. Option E (MYC and APC) is incorrect. Activation of the MYC oncogene is associated with Burkitts lymphoma. Inactivation of the APC suppressor gene is associated with the development of colorectal cancer.

A 32-year-old man sees his physician because of a lesion on his elbow (see photograph). He states that the lesion becomes more prominent during periods of physical stress. He complains of pain in the distal interphalangeal joints. Which of the following pathologic changes characterizes this skin lesion?

A. Dysplastic basal cells B. Epithelial cell hyperplasia C. Liquefactive degeneration along the basement membrane D. Neoplastic lymphocytes in the epidermis E. Yeasts and hyphae in the stratum corneum
Option B (Epithelial cell hyperplasia) is correct. The patient has psoriasis. The photograph shows a salmon-colored plaque covered by loosely adherent silver-white scales. Psoriasis is a chronic inflammatory dermatosis with unregulated

proliferation of squamous epithelial cells (epithelial hyperplasia). Other microscopic findings include downward extension of the rete pegs, retention of nuclei in the stratum corneum (parakeratosis), and thinning of the epidermis overlying the tips of dermal papillae. Peeling of the scales shows bleeding points due to exposure of vessels in the dermal papillae (Auspitz sign). Pitting of the nails is another common finding. Patients who have the HLA-B27 genotype may develop seronegative (rheumatoid factornegative) spondyloarthropathy, which is characterized by inflammation in the sacroiliac joints and peripheral arthritis, as noted in this patient. Option A (Dysplastic basal cells) is incorrect. Dysplastic basal cells are seen in actinic (solar) keratosis, which is a precursor lesion for squamous cell carcinoma associated with excessive exposure to sunlight. The skin lesions show scaling papules and plaques that lack the salmon coloration and silver-white scales. Option C (Liquefactive degeneration along the basement membrane) is incorrect. Liquefactive degeneration is a characteristic finding of systemic lupus erythematosus. DNAanti-DNA immunocomplexes deposit along the dermalepidermal junction, causing immunologic destruction of the basal layer of the epithelium. Scaling red plaques mainly develop in sun-exposed areas (e.g., face). Option D (Neoplastic lymphocytes in the epidermis) is incorrect. Cutaneous T-cell lymphoma (mycosis fungoides) produces plaques on the skin due to invasion of the skin by CD4 T helper cells. These plaques are not salmon-colored and covered by silver-white scales. Option E (Yeasts and hyphae in the stratum corneum) is incorrect. Yeasts and hyphae in the stratum corneum are present in superficial dermatophyte infections (e.g., tinea corporis due to Trichophyton rubrum). Ring-shaped patches with erythematous margins and central clearing are usually present on the skin.

A 25-year-old black man has fever and pain in the lower right thigh. A radiograph of the thigh shows irregular lucency in the metaphysis and thickening of the periosteum of the distal right femur. The photograph shows his peripheral blood smear. The WBC count is 35,000 cells/mm3. Which of the following pathogens is most likely responsible for the bone lesion?

A. Pseudomonas aeruginosa

B. Salmonella paratyphi C. Staphylococcus aureus D. Streptococcus pyogenes E. Streptococcus pyogenes


Option B (Salmonella paratyphi) is correct. The patient has Salmonella osteomyelitis. The peripheral blood smear shows sickle cells (dense, boat-shaped RBCs) and numerous target cells (RBCs with a bulls-eye appearance), the latter a common sign of a hemoglobinopathy. Sickle cell disease is the most common hemolytic disease in blacks and is due to a missense mutation resulting in replacement of glutamic acid by valine in the 6th position of the -globin chain. The radiographic findings in the lower right femur (a periosteal reaction and lytic lesion in the metaphysis) are indicative of osteomyelitis, which in sickle cell disease is most often caused by S. paratyphi. This indicates that the patient most likely has autosplenectomy due to multiple infarctions of the spleen. A functioning spleen is required to remove Salmonella, Streptococcus pneumoniae, and Hemophilus influenzae from the blood. The spleen is a major site for synthesis of IgM; therefore, a decrease in IgM leads to decreased activation of the complement system and production of C3b, which is an opsonin that assists splenic macrophages remove these pathogens from the blood. Staphylococcus aureus is the usual pathogen causing osteomyelitis in patients without sickle cell disease. Option A (Pseudomonas aeruginosa) is incorrect. P. aeruginosa is a common cause of localized osteomyelitis associated with puncture of the foot in patients wearing rubber footwear. Option C (Staphylococcus aureus) is incorrect. S. aureus is the most common cause of osteomyelitis in patients without sickle cell disease. Option D (Streptococcus pyogenes) is incorrect. In sickle cell disease, S. pneumoniae is a common cause of septicemia when the spleen becomes dysfunctional from repeated infarctions; however, it is not a common cause of osteomyelitis. Option E (Streptococcus pyogenes) is incorrect. S. pyogenes is a common cause of osteomyelitis; however, in the setting of sickle cell anemia, Salmonella is the more likely pathogen.

Which of the following clinical disorders is most compatible with the distribution of affected patients shown in this pedigree?

A. Alports syndrome B. Familial hypercholesterolemia C. Familial polyposis coli D. Lebers optic neuropathy E. McArdles disease
Option D (Lebers optic neuropathy) is correct. Lebers optic neuropathy is a neurodegenerative disease in which progressive loss of central vision eventually leads to blindness. It has a mitochondrial DNA inheritance pattern, which is shown in the pedigree. Mitochondrial DNA disorders generally involve enzyme deficiencies in oxidative phosphorylation in the mitochondria. Unlike sperm, ova do not lose their mitochondria on fertilization, so affected females transmit the abnormal allele to all their children. However, affected males do not transmit it to any of their children, because mitochondria are located in the tail of the sperm, which is lost during fertilization. Option A (Alports syndrome) is incorrect. Alports syndrome is usually an X-linked dominant disorder associated with hereditary nephritis and sensorineural hearing loss. X-linked dominant disorders are characterized by a dominant allele that causes both male and female carriers to express the disease. Affected males transmit the abnormal allele to all their daughters. Symptomatic carrier females transmit disease to 50% of their sons and 50% of their daughters. Option B (Familial hypercholesterolemia) is incorrect. Familial hypercholesterolemia is an autosomal dominant disorder involving a deficiency of low-density lipoprotein receptors that leads to severe hypercholesterolemia. Autosomal dominant disorders are characterized by a dominant allele that expresses itself in either the homozygous or the heterozygous state. Only one parent must have the abnormal allele to transmit the disease to the children. A heterozygous parent with disease transmits it to 50% of the children. Option C (Familial polyposis coli) is incorrect. Familial polyposis coli is an autosomal dominant disorder characterized by the development of premalignant polyps in the colon. The disorder eventually progresses to colorectal cancer. Option E (McArdles disease) is incorrect. McArdles disease is a glycogen storage disease (type V), which all have an

autosomal recessive inheritance pattern. In McArdles disease there is a deficiency of muscle phosphorylase, which renders muscle incapable of metabolizing glycogen to glucose. Therefore, during exercise there is no glucose for anaerobic glycolysis and no accumulation of lactic acid. In autosomal recessive disorders, disease is present only in patients who are homozygous (aa) for the abnormal allele. In most cases, both parents are asymptomatic heterozygous (Aa) carriers. Approximately 25% of their children will express the disease.

A 57-year-old man presents with a 6-month history of severe constant pain in the splenic flexure area shortly after eating. He states that the pain is so intense that he is afraid of eating and has lost approximately 23 kg (50 pounds). Recently, he has noticed blood in his stools. The photograph shows a single contrast barium enema of the colonic mucosa in the splenic flexure. What is the most likely diagnosis?

A. Adenocarcinoma B. Angiodysplasia C. Intussusception D. Ischemic colitis


Option D (Ischemic colitis) is correct. Ischemic colitis involves the splenic flexure of the large bowel, which is a watershed area of two overlapping blood suppliessuperior mesenteric artery and inferior mesenteric artery. Unfortunately, there is an area of relative ischemia between the two blood supplies, which normally does not produce any problems unless there is decreased blood flow to either or both arteries, usually by atherosclerosis, as in this case. The pain in the splenic flexure after eating is classic mesenteric angina due to decreased blood flow to the splenic flexure. The pain is severe enough to cause people to lose weight, as in this patient. The bleeding is due to a localized

infarction of the mucosa. Repair by fibrosis can eventually produce an ischemic stricture with large bowel obstruction. The barium enema shows thumb-printing of the colonic mucosa due to edema of the mucosa from infarction. Option A (Adenocarcinoma) is incorrect. Adenocarcinomas of the colon do not present with pain after eating. There is either an alteration in bowel habits and/or blood coating stool or mixed in with stool. Thumbprinting is not a radiologic feature of colon cancer. Option B (Angiodysplasia) is incorrect. Angiodysplasia occurs in the cecum and presents with a sudden massive loss of blood (hematochezia) without pain. It is due to rupture of telangiectatic vessels in the cecum. Option C (Intussusception) is incorrect. An intussusception refers to the prolapse of one part of the intestine into the lumen of an immediately adjoining part. The most common type, which most often occurs in children, is when the terminal ileum intussuscepts into the cecum causing bowel obstruction (colicky pain) and bleeding. Usually there is a nidus for the intussusception (e.g., mound of hyperplastic lymphoid tissue in Peyers patches; polyp; cancer). The patient does not have colicky pain, a sign of obstructive bowel disease.

This is a photograph of a 42-year-old man with a history of alcohol abuse. The additional photograph shows a close-up of a lesion that is present on his face. Which of the following physical findings that commonly occurs in patients with his disorder has the same pathogenesis as the skin lesion?

A. Ascites B. Asterixis C. Caput medusae D. Esophageal varices E. Gynecomastia

Option E (Gynecomastia) is correct. This patient has cirrhosis of the liver. Note abdominal distention due to ascites. Signs of hyperestrinism include gynecomastia and a female distribution of hair (hair does not extend to umbilicus). The close-up of the lesion on the face shows a spider angioma (telangiectasia) with a single central arteriole and numerous radiating capillaries. Spider angiomas are associated with hyperestrinism, which is most often caused by cirrhosis. In cirrhosis, the dysfunctional liver is unable to metabolize estrogen or 17-ketosteroids (e.g., androstenedione), the latter being aromatized in the adipose into weak estrogen compounds. Hyperestrinism in men causes gynecomastia (development of breast tissue in males) and female secondary sex characteristics (palmar erythema, soft skin, female hair distribution). Option A (Ascites) is incorrect. Development of ascites in cirrhosis is multifactorial. Factors that contribute to the development of ascites include portal hypertension (increase in hydrostatic pressure), hypoalbuminemia (decrease in oncotic pressure), secondary aldosteronism (salt retention), and increased lymphatic drainage into the peritoneal cavity. Option B (Asterixis) is incorrect. Asterixis, or flapping tremor, refers to the inability to sustain posture. It is a sign of hepatic encephalopathy, which is caused by an increase in ammonia and false neurotransmitters (e.g., -aminobutyric acid). Option C (Caput medusae) is incorrect. Caput medusae, or dilated periumbilical veins, are associated with increased venous pressure caused by portal hypertension. Option D (Esophageal varices) is incorrect. Esophageal varices are dilated left gastric coronary veins. These veins normally drain the distal esophagus and proximal stomach and empty into the portal vein. An increase in portal vein pressure leads to dilation of the left gastric coronary vein and an increased risk for rupture.

A 42-year-old woman has abnormal findings on a cervical Pap smear. The photograph shows a histologic section of the cervical os obtained by colposcopy. Which of the following growth alterations is most likely present?

A. Atrophy B. Dysplasia C. Hyperplasia D. Hypertrophy E. Invasive cancer


Option B (Dysplasia) is correct. Dysplasia is disorderly cell growth related to either atypical hyperplasia or atypical metaplasia. It is a precursor to cancer. Dysplastic epithelial changes include an increase in mitoses, lack of normal cell orientation, and nuclear enlargement with atypical changes in chromatin all of which are evident in the biopsy. The normal squamous epithelium of the exocervix derives from squamous metaplasia of mucus-secreting glandular cells moving down the endocervical canal. Infection of the squamous cells by human papillomavirus types 16 or 18 initiates conversion of the metaplastic squamous epithelium to dysplastic epithelium. If the infection persists, cervical squamous cell cancer may develop. Option A (Atrophy) is incorrect. Atrophy is a decrease in tissue mass because of cellular shrinkage or loss of cells. Option C (Hyperplasia) is incorrect. Hyperplasia refers to an increase in the number of cells. Although there is likely to be an increase in number of squamous cells in this biopsy, the growth alteration that is present is dysplastic (atypical hyperplasia). Option D (Hypertrophy) is incorrect. Hypertrophy refers to an increase in the size of cells (usually muscle). Option E (Invasive cancer) is incorrect. The basement membrane of the cervical epithelium is intact, which excludes the presence of invasive cancer.

A 58-year-old man complains of frequent headaches and generalized itching after bathing. Physical examination shows congestion of the retinal vessels, a ruddy complexion, and splenomegaly. Initial laboratory studies show an RBC count of 8 million/mm3, a WBC count of 15,000/mm3, and a platelet count of 500,000/mm3. The WBC differential count shows an increase in normal-appearing segmented neutrophils. Which of the following sets of laboratory results is most likely to be reported? RBC massPlasma volume O2 saturationEPO concentration A.Increased Increased Normal Decreased B. Increased Normal Decreased Increased C. Increased Normal Normal Increased D.Normal Decreased Normal Normal EPO, erythropoietin.

A. RBC mass increased, plasma volume increased, O2 saturation normal, EPO decreased

B. RBC mass increased, plasma volume normal, O2 saturation decreased, EPO increased C. RBC mass increased, plasma volume normal, O2 saturation normal, EPO increased D. RBC mass normal, plasma volume decreased, O2 saturation normal, EPO normal
Option A (RBC mass increased, plasma volume increased, O 2 saturation normal, EPO decreased) is correct. The patient has polycythemia vera (PV), a myeloproliferative disorder (malignant disorder of stem cells). It is due to clonal expansion of the myeloid stem cell, most often due to mutation of the JAK2 gene on the short arm of chromosome 9. This results in increased production of RBCs, granulocytes (neutrophils, eosinophils, basophils), mast cells, and platelets. PV is an absolute polycythemia, meaning that the bone marrow is producing more RBCs than normal; hence, the total number of RBCs in the body in mL/kg (RBC mass) is increased as well as the RBC count. However, it is an inappropriate absolute polycythemia, because there is no hypoxic stimulus causing the release of EPO to stimulate the bone marrow to increase RBC production in PV. The RBC count (RBCs/mm3 blood) as expected is always increased if the RBC mass is increased. In PV, there is an increase in plasma volume that also accompanies the increase in RBC mass, unlike in the other types of polycythemia. The reason for this is unknown. The O2 saturation is normal, because the percentage of binding sites on heme occupied by O2 remains the same. EPO is decreased, because the O2 content of blood is increasedO2 content = 1.34hemoglobinO2 saturation + arterial PO2. The O2 content of blood has a negative feedback relationship with EPO. Absolute leukocytosis and thrombocytosis commonly accompany the increase in RBCs. This patient's pruritus after bathing is caused by the increase in histamine released from excess numbers of mast cells in the skin. This is one of the most frequent presenting signs of PV. Option B (RBC mass increased, plasma volume normal, O 2 saturation decreased, EPO increased) is incorrect. These laboratory findings are signs of an appropriate type of absolute polycythemia related to a hypoxic stimulus for EPO release (decreased O2 saturation). Examples include obstructive and restrictive lung disease, cyanotic congenital heart disease, and living at high altitudes. Note that the plasma volume is normal in other types of absolute polycythemia, unlike PV where it is increased. Option C (RBC mass increased, plasma volume normal, O 2 saturation normal, EPO increased) is incorrect. These laboratory findings are signs of an inappropriate type of absolute polycythemia (normal O2 saturation) related to ectopic or inappropriate secretion of EPO. Examples include renal disease (e.g., cancer, cystic disease, hydronephrosis) and ectopic secretion of EPO from a hepatocellular carcinoma. Note, again, that the plasma volume is normal. Option D (RBC mass normal, plasma volume decreased, O 2 saturation normal, EPO normal) is incorrect. These laboratory findings are signs of a relative, as opposed to absolute, type of polycythemia in which a decrease in plasma volume hemoconcentrates RBCs in the peripheral blood. This increases the RBC count without affecting RBC mass. Any cause of volume depletion (e.g., excessive sweating, severe diarrhea) has the potential for producing a relative polycythemia.

Which of the following clinical disorders is most compatible with the distribution of affected patients shown in this pedigree?

A. Adrenogenital syndrome B. Gouty arthritis C. Hemophilia A D. Phenylketonuria (PKU) E. Von Willebrand disease
Option E (Von Willebrand disease) is correct. Von Willebrand disease is an autosomal dominant disorder, with the inheritance pattern shown in the pedigree. It involves a combined platelet adhesion defect and coagulation factor VIII deficiency. Autosomal dominant disorders are characterized by a dominant allele that expresses itself in either the homozygous or the heterozygous state. Only one parent need have the abnormal allele for the disease to be transmitted to children. A heterozygous parent with disease will transmit the disease to 50% of the children, which is evident in the pedigree. Option A (Adrenogenital syndrome) is incorrect. Adrenogenital syndrome is an autosomal recessive disorder characterized by deficiencies of enzymes involved in the production of adrenal cortex hormone products (e.g., cortisol and mineralocorticoids). In autosomal recessive disorders, disease is present only in patients who are homozygous for the abnormal allele (e.g., aa). Both parents must have the abnormal allele to transmit the disease to their children. Option B (Gouty arthritis) is incorrect. Gout is a disorder with multifactorial (polygenic) inheritance. Multifactorial inheritance involves the additive effect of two or more gene mutations of small effect conditioned by environmental and other nongenetic factors. Option C (Hemophilia A) is incorrect. Hemophilia A is an X-linked recessive disorder that involves a deficiency of factor VIII coagulant. X-linked disorders are expressed only in males, whereas females with the abnormal allele are usually asymptomatic carriers.

Option D (Phenylketonuria (PKU)) is incorrect. PKU is an autosomal recessive disorder that involves a deficiency of phenylalanine hydroxylase.

In which of the following clinical scenarios would the patient most likely have an increase in the leukocyte shown in the photograph?

A. A 4-year-old child has whooping cough. B. A 24-year-old man, who raises hogs, has a habit of eating raw bacon. C. A 28-year-old man has a perforated acute appendicitis. D. A 45-year-old woman has severe rheumatoid arthritis. E. A 56-year-old man with polycythemia vera has flushing of his face.
Option A (A 4-year-old child has whooping cough.) is correct. Bordetella pertussis is the cause of whooping cough. The lymphotoxin in this bacteria inhibits signal transduction by chemokine receptors which prevents lymphocytes from entering lymph nodes leading to lymphocytosis. Lymphocyte counts are often >50,000/mm 3 (lymphoid leukemoid reaction). The photograph shows a small lymphocyte with scant cytoplasm surrounding a dark nucleus. Option B (A 24-year-old man, who raises hogs, has a habit of eating raw bacon.) is incorrect. This patient would most likely develop trichinosis, due to Trichinella spiralis. It is contracted by eating raw or undercooked pork. The larvae penetrate muscle producing muscle pain and tenderness. Invasive helminths produce eosinophilia (type I hypersensitivity). The photograph shows an eosinophil with cytoplasm packed with reddish-orange granules that do not cover the nucleus. Option C (A 28-year-old man has a perforated acute appendicitis.) is incorrect. A perforated acute appendicitis produces neutrophilic leukocytosis with left shift (e.g., band neutrophils) and toxic granulation (prominent azurophilic granules). Option D (A 45-year-old woman has severe rheumatoid arthritis.) is incorrect. Monocytosis is the primary leukocyte alteration in chronic inflammation (e.g., rheumatoid arthritis). Option E (A 56-year-old man with polycythemia vera has flushing of his face.) is incorrect. In polycythemia, all cell

lines except lymphocytes are increased. An increase in basophils and mast cells causes the release of histamine, which produces flushing of the face (called plethora), headaches, and pruritus after bathing. All of the myeloproliferative diseases have basophilia.

A 62-year-old man, who lives alone, has pitting edema of his lower legs. The serum total protein concentration is decreased. He primarily subsists on bread and Diet Coke. A urinalysis is negative for protein. Which of the following best explains the pathogenesis of the hypoproteinemia?

A. Acquired hypogammaglobulinemia B. Decreased intake of protein C. Malignant plasma cell disorder D. Nephrotic syndrome
Option B (Decreased intake of protein) is correct. The patients diet is lacking protein leading to a decrease in albumin and a corresponding decrease in total protein. Albumin is primarily responsible for maintaining plasma oncotic pressure; therefore, hypoalbuminemia leads to leakage of protein-poor fluid into the interstitial space thus causing pitting edema. Option A (Acquired hypogammaglobulinemia) is incorrect. Hypogammaglobulinemia is an uncommon cause of a decrease in total serum protein. Furthermore, it would not explain the pitting edema in the patient, because albumin levels are not significantly altered in hypogammaglobulinemia. Option C (Malignant plasma cell disorder) is incorrect. A malignant plasma cell disorder (e.g., multiple myeloma) produces an increase in serum total protein, not a decrease. Option D (Nephrotic syndrome) is incorrect. The nephrotic syndrome is characterized by massive losses of protein in the urine (>3.5 g/24 hours) leading to hypoalbuminemia and a decrease in total serum protein. The patient does not have proteinuria.

A newborn male infant with cyanosis develops tetany and heart failure. A chest radiograph shows an absent thymic shadow. Which of the following parathyroid hormone (PTH) and calcium findings is most likely present? PTH Serum calcium A.DecreasedDecreased B. DecreasedIncreased C. Increased Decreased D.Increased Increased E. Normal Normal

A. PTH decreased, serum calcium decreased B. PTH decreased, serum calcium increased C. PTH increased, serum calcium decreased D. PTH increased, serum calcium increased E. PTH normal, serum calcium normal
Option A (PTH decreased, serum calcium decreased) is correct. The patient has DiGeorge syndrome, which is due to failure of formation of the third and fourth pharyngeal pouches. This is associated with absence of the thymus (pure Tcell immunodeficiency) and absence of the parathyroid glands, causing primary hypoparathyroidism (decreased PTH and decreased serum calcium). DiGeorge syndrome is the only syndrome associated with absence of the thymus, hypocalcemia, and cyanotic congenital heart disease. The cyanosis is most often due to a truncus arteriosus. Hypocalcemia causes tetany, which in newborns is manifested by jitteriness, repetitive blinking, and stridor. Option B (PTH decreased, serum calcium increased) is incorrect. A decrease in PTH and an increase in serum calcium is characteristic of hypercalcemia due to malignancy (most common cause) or other nonparathyroid-gland-related disorders (e.g., sarcoidosis, hypervitaminosis D). Hypercalcemia suppresses the release of PTH by the parathyroid gland. Hypercalcemia does not produce tetany. Option C (PTH increased, serum calcium decreased) is incorrect. An increase in PTH and a decrease in serum calcium is characteristic of secondary hyperparathyroidism, which is a compensatory phenomenon that increases serum calcium to within the normal range. Hypocalcemia can produce tetany. Hypovitaminosis D is a common cause of secondary hyperparathyroidism. Option D (PTH increased, serum calcium increased) is incorrect. An increase in PTH and an increase in calcium is due to primary hyperparathyroidism, which is most often caused by a parathyroid adenoma. Hypercalcemia does not produce tetany. Option E (PTH normal, serum calcium normal) is incorrect. An absent thymic shadow, cyanosis, and signs of tetany can only be explained by DiGeorge syndrome.

The photograph is of the external genitalia of a lethargic 3-month-old infant. Physical examination reveals hypotension and dryness of the mucous membranes. Laboratory studies show a decrease in serum cortisol and an increase in plasma adrenocorticotropic hormone (ACTH). A random urine sodium is markedly increased. A chromosome study shows an XX genotype. The patient most likely has a deficiency of which of the following enzymes involved in adrenal steroid synthesis?

A. 11-Hydroxylase B. 17-Hydroxylase C. 18-Hydroxylase D. 21-Hydroxylase E. Oxidoreductase


Option D (21-Hydroxylase) is correct. The patient has adrenogenital syndrome (aka congenital adrenal hyperplasia). The genitalia is ambiguous. Note the enlarged clitoris and the partially fuse labia. The first step in working up ambiguous genitalia is a chromosome analysis to determine sex. This patient is a female, but phenotypically looks like a male (female pseudohermaphroditism). This syndrome includes a group of autosomal recessive disorders characterized by enzyme deficiencies (in this patient, 21-hydroxylase deficiency) that cause a decrease in cortisol and an increase in ACTH. There is an increase in steroid compounds proximal to the enzyme block, including the 17-ketosteroids (dehydroepiandrosterone and androstenedione), testosterone, and dihydrotestosterone. There is a decrease in compounds distal to the enzyme block, including the mineralocorticoids (e.g., corticosterone) and the 17hydroxycorticoids (11-deoxycortisol and cortisol). In the presence of excess dihydrotestosterone, female-appearing external genitalia in a fetus undergo male differentiation (e.g., clitoromegaly and fusion of the labia). The clitoris is often elongated (clitoromegaly) and the labia are often fused and appear as scrotal sacs, as in this case. A decrease in serum cortisol causes an increase in ACTH, which has melanocyte-stimulating properties that can produce increased skin pigmentation. A decrease in mineralocorticoids causes loss of sodium in the urine that may lead to hypovolemic shock, which explains why the patient is lethargic. Option A (11-Hydroxylase) is incorrect. 11-hydroxylase deficiency causes an increase in 11-deoxycortisol; 17ketosteroids (dehydroepiandrosterone and androstenedione); and 11-deoxycorticosterone, a weak mineralocorticoid. The increase in deoxycorticosterone produces salt retention (not loss in the urine) and hypertension. Option B (17-Hydroxylase) is incorrect. 17-Hydroxylase deficiency causes an increase in the synthesis of mineralocorticoids (salt retention [not loss in the urine] and hypertension) and a decrease in 17-ketosteroids and 17hydroxycorticoids. Option C (18-Hydroxylase) is incorrect. 18-hydroxylase deficiency is uncommon and causes an isolated deficiency of

aldosterone, resulting in hyponatremia, hyperkalemia, and acidosis. Disorders involving glucocorticoids and 17ketosteroids do not occur. Option E (Oxidoreductase) is incorrect. Oxidoreductase converts androstenedione to testosterone; however, there would not be any changes in mineralocorticoids or glucocorticoids.

Colonoscopic studies show a 3-cm annular mass in the sigmoid colon of a 62-year-old man. Multiple biopsies show a poorly differentiated adenocarcinoma. A colectomy is performed, and a few nodular lesions on the surface of the liver are apparent. A frozen section shows a poorly differentiated adenocarcinoma. Gross and microscopic findings of the colectomy specimen show a mucosally derived cancer that has invaded through the muscle wall and out into the serosal fat. There is metastasis in 3 of 15 mesenteric lymph nodes directly beneath the tumor. Which of the following most influences the patients prognosis?

A. Age of the patient B. Differentiation of the tumor C. Extent of invasion D. Liver involvement E. Lymph node involvement

Option D (Liver involvement) is correct. The tumor-node-metastasis (TNM) system is used to stage cancers arising from epithelial tissues. T refers to the size and nuclear features of the tumor; N refers to the presence or absence of lymph node metastasis; and M refers to the presence or absence of metastasis to sites other than lymph nodes, such as the liver. M is the most important prognostic factor. The presence of distant metastases implies that the cancer has already infiltrated through regional lymph nodes draining the cancer and has entered the bloodstream. Option A (Age of the patient) is incorrect. Age of the patient is the least important risk prognostic factor of the choices listed. Option B (Differentiation of the tumor) is incorrect. Differentiation or grade of the tumor describes the histologic appearance of the tumor. If the tumor has recognizable features, such as keratin in squamous cells or glands, then the tumor is well differentiated or low grade. If the tumor has no histologic features with characteristics identifying the tissue of origin, then the tumor is poorly differentiated, anaplastic, or high grade. The extent of differentiation is a less important prognostic factor than the M of the TNM system. Option C (Extent of invasion) is incorrect. The extent of invasion of the tumor is a less important prognostic factor than the M of the TNM system. Option E(Lymph node involvement) is incorrect. Lymph node involvement (N) is the second most important prognostic

factor; however, M is more important because it implies that the tumor has metastasized to lymph nodes and entered efferent lymphatics to gain access to the blood stream for hematogenous dissemination to other sites.

An 18-year-old black woman with a history of dysfunctional uterine bleeding complains of fatigue when exercising. Laboratory studies show a mild microcytic anemia and increased RBC distribution width (RDW). The photograph shows a representative section of the peripheral blood smear. Which of the following investigative studies would be most useful in defining the type of anemia that is present?

A. Bone marrow aspiration biopsy B. Hemoglobin electrophoresis C. Osmotic fragility test D. Serum ferritin test E. Sickle cell screening

Option D (Serum ferritin test) is correct. The patients clinical and laboratory findings are consistent with the diagnosis of an iron-deficiency anemia, which can be confirmed by the serum ferritin test. The peripheral blood smear shows RBCs with a notable increase in the central areas of pallor, which indicate decreased hemoglobin concentration. Strong evidence of iron deficiency is given by the increased RBC distribution width (RDW, increased size variation in RBCs) and the patients history of dysfunctional uterine bleeding (menorrhagia). The RDW is increased in iron deficiency, because there is a mixed population of normocytic and microcytic RBCs. The other causes of microcytic anemia (e.g., anemia of chronic disease, thalassemia) are not as likely to have this size variation in RBCs and have a normal RDW. The serum ferritin test is the best screening test for iron-related disorders, because serum ferritin levels correlate with the amount of iron stored in the bone marrow macrophages. Option A (Bone marrow aspiration biopsy) is incorrect. The patient has a microcytic anemia. A bone marrow aspiration biopsy is rarely indicated in workups for microcytic anemias, because serum tests (e.g., ferritin, hemoglobin electrophoresis) are available to identify the various causes of microcytic anemia.

Option B (Hemoglobin electrophoresis) is incorrect. Hemoglobin electrophoresis identifies changes in the concentration of normal and abnormal forms of hemoglobin. Of the microcytic anemias, the only one that is likely to show an abnormal hemoglobin electrophoresis is -thalassemia where there is a decrease in hemoglobin A (2, 2) and a corresponding increase in hemoglobin A2 (2, 2) and hemoglobin F (2, 2). The patient does not have thalassemia, because the clinical history is more compatible with iron deficiency from menorrhagia and the RDW is increased (normal RDW in thalassemia). Option C (Osmotic fragility test) is incorrect. The osmotic fragility test is used to confirm a diagnosis of hereditary spherocytosis, in which the osmotic fragility of RBCs is increased. Hereditary spherocytosis is a normocytic (not microcytic) anemia. Spherocytes are not present in the peripheral blood in this patient. Option E (Sickle cell screening) is incorrect. Although the patient is black, sickle cell anemia is unlikely, because it is usually normocytic (not microcytic) and because sickle cells are not present in the peripheral blood.

A 24-year-old man collapses at the end of a marathon. His running partner states that his friend was sweating profusely throughout the race and did not replenish his fluid losses. The physical examination shows dry mucous membranes and poor skin turgor. The blood pressure is 110/70 mm Hg and the pulse rate is 120 beats/min when lying down. When sitting up, the blood pressure drops to 80/60 mm Hg and the pulse rate increases to 150 beats/min. Which of the following serum Na+, total body sodium (TBNa+) is most likely present? Serum Na+TBNa+ A.Decreased Decreased B. Decreased Increased C. Decreased Normal D.Increased Decreased E. Normal Decreased A. serum Na+ decreased, TBNa+ decreased B. serum Na+ decreased, TBNa+ increased C. serum Na+ decreased, TBNa+ normal D. serum Na+ increased, TBNa+ decreased E. serum Na+ normal, TBNa+ decreased

Option D (serum Na+ increased, TBNa+ decreased) is correct. Excessive sweating without fluid replacement is associated with a loss of hypotonic salt-containing fluid, which produces hypernatremia ( serum Na+ = TBNa+/TBW). The patient has signs of volume depletion (TBNa+) from the loss of sodium, mainly dry mucous membranes and a positive tilt test (blood pressure dropped and pulse rate increased when sitting up). Regarding the

fluid compartments, the extracellular fluid (ECF) compartment is contracted (loss of fluid) and the intracellular fluid (ICF) compartment is contracted (osmosis effect of hypernatremia). Option A (serum Na+ decreased, TBNa+ decreased) is incorrect. This occurs when there is a hypertonic loss of fluid ( serum Na+ = TBNa+/TBW), which often occurs with a loop diuretic (inhibition of the Na+, K+, 2Cl cotransporter), Addisons disease (loss of mineralocorticoids), or 21-hydroxylase deficiency (loss of mineralocorticoids). Regarding the fluid compartments, the ECF compartment is contracted (loss of fluid) and the ICF compartment is expanded (osmosis effect of hyponatremia). Option B (serum Na+ decreased, TBNa+ increased) is incorrect. This is an example of a hypotonic gain of sodiumcontaining fluid. Examples include the edema states (e.g., cirrhosis, right-sided heart failure). In these conditions there are alterations in Starling pressuresdecreased plasma oncotic pressure and/or increase in plasma hydrostatic pressurewhich force fluid out of the vascular compartment into the interstitial space and body cavities. The cardiac output is decreased in these conditions, because fluid is trapped in the interstitial space and body cavities by the Starling pressure abnormalities. Decreased renal blood flow causes the kidney to reabsorb a slightly hypotonic saltcontaining fluid, which produces hyponatremia ( serum Na+ = TBNa+/TBW). The fluid is redirected into the interstitial space (decreased plasma oncotic pressure) and body cavities (decreased plasma oncotic pressure and increased plasma hydrostatic pressure). Regarding the fluid compartments, the ECF compartment is expanded (addition of fluid) and the ICF compartment is expanded (osmosis effect of hyponatremia). Option C (serum Na+ decreased, TBNa+ normal) is incorrect. This occurs when there is a hypotonic gain of pure water, which is present in the syndrome of inappropriate antidiuretic hormone (ADH) due to excessive stimulation of ADH release (e.g., ectopic secretion of ADH by a small cell carcinoma of lung). ADH reabsorbs electrolyte-free water from the collecting tubules causing a dilutional hyponatremia (serum Na+ = TBNa+/TBW). Since Na+ is not reabsorbed by ADH, the TBNa+ is normal; therefore, there are no signs of pitting edema. Regarding the fluid compartments, the ECF compartment is expanded (addition of fluid) and the ICF compartment is expanded (osmosis effect of hyponatremia). Option E (serum Na + normal, TBNa+ decreased) is incorrect. This occurs when there is an isotonic loss of fluid (serum Na+ = TBNa+/TBW). Excellent examples include travelers diarrhea, which is most often due to enterotoxigenic Escherichia coli and cholera. Loss of isotonic fluid does not alter the serum Na+ concentration (serum Na+ = TBNa+/TBW); however, the decrease in TBNa+ causes signs of volume depletion (dry mucous membranes, poor skin turgor, hypotension). Regarding the fluid compartments, the ECF compartment is contracted (loss of fluid) and the ICF compartment is unchanged (no osmotic effect).

A patient with a suspected HIV infection has a positive enzyme-linked immunoabsorbent assay (ELISA) test. Which of the following tests is indicated to confirm the disease? A. CD4 T-cell count B. HIV viral load C. p24 Capture assay

D. Western blot assay


Option D (Western blot assay) is correct. The Western blot assay is the confirmatory test if the ELISA test is positive or indeterminate. The ELISA assay detects anti gp120 antibodies. A positive Western blot assay requires the presence of p24 and gp41 antibodies and either gp120 or gp160 antibodies. The test has close to 100% specificity. Option A (CD4 T-cell count) is incorrect. The CD4 T-cell count is used to monitor the immune status of the patient. It is useful in determining when to initiate HIV treatment and when to administer prophylaxis against opportunistic infections. It is not a confirmatory test for HIV. Option B (HIV viral load) is incorrect. HIV viral load is used to detect actively dividing virus and is also used as a marker of disease progression. It is the most sensitive test for diagnosis of acute HIV before seroconversion. Option C (p24 Capture assay) is incorrect. The p24 antigen is an indicator of active viral replication. It is present before anti-gp120 antibodies are developed in the patient. It is positive at two different times in HIV infectionsprior to seroconversion and when AIDS is diagnosed. It is not a test to confirm disease.

The photograph shows the heart of a 25-year-old woman who died in a car accident. Which of the following best describes the valvular lesion located in the left side of the heart?

A. Diastolic blowing murmur after S2 B. Midsystolic click followed by a murmur C. Opening snap followed by a mid-diastolic rumbling murmur D. Pansystolic murmur located at the apex

E. Systolic ejection murmur located in the right second intercostal space

Option B (Midsystolic click followed by a murmur) is correct. The photograph shows the posterior leaflet of the mitral valve being prolapsed (projected into) the left atrium. Mitral valve prolapse (MVP) is the most common valvular lesion in the United States. A systolic click occurs when the valve prolapses into the left atrium during systole and is suddenly restrained by the chordae tendineae. The murmur following the click is caused by mitral regurgitation as blood leaks underneath the valve cusps into the left atrium. Most patients with MVP are asymptomatic. Redundancy of the valve leaflet (which looks like a parachute) is due to an increase of dermatan sulfate in the valve causing myxomatous degeneration. The most serious complication of MVP is rupture of the chordae leading to acute mitral regurgitation and left-sided heart failure. Option A (Diastolic blowing murmur after S 2) is incorrect. A diastolic blowing murmur in the left side of the heart that occurs after S2 characterizes aortic regurgitation. Regurgitation murmurs are problems with closing of the valve; hence, blood dripping back into the left ventricle in early diastole is responsible for the murmur. Option C (Opening snap followed by a mid-diastolic rumbling murmur) is incorrect. An opening snap followed by middiastolic rumbling murmur characterizes mitral stenosis. Recall that stenosis murmurs are problems with opening of the valve; hence, localizing the murmur of mitral stenosis in diastole. In mitral stenosis, the leaflets of the mitral valve are fibrotic and frequently calcified. It is most often caused by recurrent attacks of rheumatic fever over a period of time ranging from 210 years. Option D (Pansystolic murmur located at the apex) is incorrect. A pansystolic murmur located at the apex characterizes mitral regurgitation. Since there is no obstruction to blood flow, the blood redirected into the left atrium produces a murmur throughout systole; hence the term pansystolic. Both the left atrium and left ventricle become volume overloaded. The mitral regurgitation in MVP follows a systolic click and does occur throughout systole unless there is rupture of the chordae. Option E (Systolic ejection murmur located in the right second intercostal space) is incorrect. A systolic ejection murmur in the right second intercostal space characterizes aortic stenosis, in which the area of the valvular orifice is reduced. Ejection murmurs have a diamond-shaped appearance (crescendo-decrescendo) on a phonocardiogram. The aortic valve is normal in this heart.

An 11-year-old boy has enlarged, nontender testicles, a long face with a prominent jaw, a high arched palate, and protruding ears. The child is in a special educational program at school. Which of the following studies is most useful for evaluation of this patient?

A. Buccal smear B. DNA analysis of the X chromosome

C. Human chorionic gonadotropin D. Serum gonadotropins E. Testicular biopsy


Option B (DNA analysis of the X chromosome) is correct. The boy most likely has fragile X syndrome. In most cases, inheritance of fragile X syndrome is X-linked recessive. The fragile site or gap at the end of the long arm of the X chromosome is where there are trinucleotide repeats (CGG). DNA analysis of the X chromosome in lymphocytes identifies the trinucleotide repeats and is considered more sensitive than the fragile X chromosome study. Characteristic findings include mental retardation; enlarged, nontender testicles (present at adolescence, not at birth); a long face with a prominent jaw; a high arched palate; and protruding ears. Option A (Buccal smear) is incorrect. A buccal smear is performed to rule out deficient or extra X chromosomes. Normal women have random inactivation of one of the two X chromosomes. Hence, women have one Barr body and normal men have no Barr bodies. Patients with fragile X syndrome have no Barr bodies. Option C (Human chorionic gonadotropin) is incorrect. Human chorionic gonadotropin is not normally present in men unless they have a choriocarcinoma of the testicle. Option D (Serum gonadotropins) is incorrect. The patient shows no clinical evidence of hypogonadism (e.g., delayed puberty); therefore, gonadotropins are not indicated. Furthermore, they are normal in the fragile X syndrome. Option E (Testicular biopsy) is incorrect. Testicular biopsy is not warranted, because the patient does not have a testicular neoplasm, which is characterized by a painless mass in the testicle. Furthermore, the chances of having both testicles involved with cancer are not good.

A routine physical examination of an asymptomatic 21-year-old black woman is normal. However, urinalysis shows RBCs with no casts present in the urine sediment. The patient states that she occasionally has had blood in her urine. A urine culture is negative, a peripheral smear is normal, and renal ultrasonography is normal. Laboratory studies show a serum blood urea nitrogen of 10 mg/dL, serum creatinine of 1 mg/dL, hemoglobin of 14 g/dL, and mean corpuscular volume of 82 (m3. Which of the following is the next best step in the workup?

A. Bone marrow examination B. Cystoscopy C. Renal biopsy D. Serum ferritin E. Sickle cell screen

Option E (Sickle cell screen) is correct. The patient most likely has sickle cell trait, which causes recurrent microscopic hematuria. In sickle cell trait, the percentage of sickle hemoglobin is 40% to 45%, and the remainder of hemoglobin is hemoglobin A. There are no sickle cells in the peripheral smear in sickle cell trait; therefore, a sickle cell screen is required to induce sickling of RBCs containing sickle hemoglobin. However, the O2 tension in the renal medulla is low enough to induce sickling of RBCs in the peritubular capillaries. This causes microinfarctions in the renal medulla leading to hematuria. There is also the potential for renal papillary necrosis and subsequent loss of both concentration and dilution. This is an example where a heterozygote carrier for an autosomal recessive disease can have problems similar to those in the homozygous condition. Option A (Bone marrow examination) is incorrect. A bone marrow examination is not warranted, because the patient does not have bone marrowrelated anemia or evidence of intrinsic bone marrow disease. Option B (Cystoscopy) is incorrect. If the sickle cell screen is negative, a cystoscopy may be necessary to determine the cause of the hematuria. Option C (Renal biopsy) is incorrect. If the sickle cell screen is negative, a renal biopsy may be necessary to rule out primary renal disease, particularly IgA glomerulonephritis, which is commonly associated with episodic hematuria. However, the absence of RBC casts and dysmorphic RBCs (damaged RBCs) is against a nephritic type of glomerular disease. Option D (Serum ferritin) is incorrect. Although serum ferritin is decreased in the early stages of iron deficiency when anemia is not present and the mean corpuscular volume is normal, hematuria is not usually associated with any of the microcytic anemias.

For the past few months, a 38-year-old man, who is an intravenous drug abuser with chronic hepatitis B, has had a fever, and recently, bloody stools. He also reports a 5-kg (11-pound) weight loss and right flank pain. Physical examination shows right flank pain to gentle percussion. Abdominal examination reveals tender hepatomegaly. A stool guaiac is positive for blood. The photograph shows a mesenteric artery angiogram. A renal angiogram shows thrombosis of the right renal artery and focal, wedge-shaped areas of hypovascularity in the renal cortex. Which of the following laboratory findings is most likely to be reported?

A. Antitopoisomerase antibody B. c-Antineutrophil cytoplasmic antibodies (ANCA)

C. p-Antineutrophil cytoplasmic antibodies (ANCA) D. Hepatitis B surface antigen E. Positive serum antinuclear antibodies (ANA)

Option D (Hepatitis B surface antigen) is correct. The patient has polyarteritis nodosa, a vasculitis of small muscular arteries (except pulmonary arteries) that causes vessel thrombosis and focal aneurysmal dilation from weakening of the vessel. Hepatitis B surface antigenemia occurs in 30% of cases. Surface antigen-antibody immunocomplexes form deposits in the vessel walls and activate the complement system, which attracts neutrophils and causes vessel damage. Target organs in polyarteritis nodosa include the renal arteries (thrombosis, focal aneurysmal dilation) and mesenteric arteries (thrombosis and aneurysms). The photograph shows numerous small aneurysms (arrow) in the medium-sized vessels. The renal angiogram frequently shows thrombosis, as in this case) and cortical infarctions (areas of hypovascularity). Renal infarction is responsible for the flank pain and hematuria. Corticosteroids are the treatment of choice. Option A (Antitopoisomerase antibody) is incorrect. Antitopoisomerase antibodies (antiScl-70) occur in systemic sclerosis (scleroderma). Systemic sclerosis is characterized by replacement of subcutaneous tissue with collagen and a vasculitis involving interlobar arteries and arterioles in the kidneys causing hypertension and renal infarctions. Renal artery thrombosis and focal aneurysmal dilation do not occur. Option B (c-Antineutrophil cytoplasmic antibodies (ANCA)) is incorrect. Wegeners granulomatosis causes a necrotizing vasculitis of varying-sized vessels in the lungs and kidneys and necrotizing granulomas in the upper respiratory tract and lungs. c-ANCA is present in Wegeners granulomatosis. The antibodies are directed against neutrophil granules in the cytosol. Destruction of neutrophils by c-ANCA releases enzymes that contribute to the inflammatory lesions. Crescentic glomerulonephritis (not renal infarction) is the renal lesion seen in Wegeners granulomatosis and rapidly progresses to renal failure. Option C (p-Antineutrophil cytoplasmic antibodies (ANCA)) is incorrect. p-ANCAs occur in microscopic polyangiitis, a vasculitis of small vessels that involves multiple organ systems (including the pulmonary vessels). Glomerulonephritis, not renal artery thrombosis, is a characteristic finding. Option E (Positive serum antinuclear antibodies (ANA)) is incorrect. Antibodies directed against nuclear antigens (e.g., DNA) are not present in polyarteritis nodosa.

A 42-year-old executive of a large corporation complains of sudden onset of severe stomach pain with radiation to both shoulders. The pain does not radiate to the back. The photograph shows a plain abdominal radiograph with the patient in a supine position. Physical examination shows epigastric tenderness, absent bowel sounds, and abdominal rigidity with rebound tenderness. The stool is black and guaiac-positive. Which of the following is the most likely diagnosis?

A. Acute diverticulitis B. Acute pancreatitis C. Gastroesophageal reflux disease (GERD) D. Perforated peptic ulcer E. Small bowel infarction

Option D (Perforated peptic ulcer) is correct. The radiograph shows air collected under both diaphragms, which is most commonly due to a perforated peptic ulcer. The patient has melena (black stools) and a guaiacpositive stool, indicating bleeding from a peptic ulcer. Duodenal ulcers are more likely to perforate than gastric ulcers, causing peritonitis (rebound tenderness, absent bowel sounds) and collection of air under the diaphragms, causing referred pain to the shoulder. Melena is due to the conversion of hemoglobin to hematin by acid in the stomach. Therefore, melena indicates that the gastrointestinal bleed is in the upper gastrointestinal tract proximal to where the duodenum attaches to the jejunum. Option A (Acute diverticulitis) is incorrect. Acute diverticulitis is characterized by pain in the left lower quadrant (left-sided appendicitis) rather than the epigastrium. However, inflamed diverticula can perforate and produce peritonitis and air under the diaphragm. Option B (Acute pancreatitis) is incorrect. Acute pancreatitis is characterized by epigastric pain that radiates to the back (the pancreas is located in the retroperitoneum). It does not cause pain that radiates to the shoulder and is not associated with melena.

Option C (Gastroesophageal reflux disease (GERD)) is incorrect. GERD is characterized by reflux of acid into the esophagus through a relaxed lower esophageal sphincter. It causes a burning pain in the epigastrium associated with acid indigestion. Pain does not radiate to the shoulder. There is no melena. Option E (Small bowel infarction) is incorrect. A small bowel infarction is characterized by bloody diarrhea (not melena) and diffuse abdominal pain without rebound tenderness. Pain does not radiate to the shoulder.

A 48-year-old man with a history of bronchiectasis secondary to tuberculosis develops septicemia related to the lung condition and dies. The photograph shows an isolated lesion in the brain obtained at autopsy. Which type of necrosis is most likely present?

A. Caseous necrosis B. Coagulation necrosis C. Enzymatic fat necrosis D. Fibrinoid necrosis E. Liquefactive necrosis
Option E (Liquefactive necrosis) is correct. The cerebral cortex shows a circular mass with shaggy borders and a central area of softened tissue filled with pus (cerebral abscess). These findings indicate liquefactive necrosis, caused by the release of hydrolytic enzymes from neutrophils. In this patient, the abscess is caused by hematogenous spread of bacteria from the bronchial infection. Option A (Caseous necrosis) is incorrect. Caseous necrosis appears as a soft, cheeselike material that exudes from the center of a granuloma, unlike the lesion shown in the brain. Caseous necrosis occurs primarily in patients with

mycobacterial infection (e.g., tuberculosis) or systemic fungal infection (e.g., histoplasmosis). Option B (Coagulation necrosis) is incorrect. Coagulation necrosis does not occur in the brain. The abundant supply of hydrolytic enzymes in brain tissue causes liquefaction of brain tissue producing a cystic space. Option C (Enzymatic fat necrosis) is incorrect. Enzymatic fat necrosis occurs in the pancreas, where it appears as chalky white areas in adipose tissue in and around an area of inflammation (acute pancreatitis). It does not occur in the brain. Option D (Fibrinoid necrosis) is incorrect. Fibrinoid necrosis occurs in small muscular arteries, arterioles, venules, and glomerular capillaries. Immunologic damage to these vessels causes leakage of plasma proteins into the wall of the vessels.

A 55-year-old man with chronic ischemic heart disease complains of muscle weakness. The photograph shows the ECG in the patient. Which of the following drugs is most likely to be responsible for these findings?

A. Aldosterone blocker B. -Adrenergic blocker C. Calcium channel blocker D. Digitalis E. Loop diuretic
Option E (Loop diuretic) is correct. Loop diuretics are associated with hypokalemia (decreased serum K+ concentration), which can produce both muscle weakness and the characteristic ECG pattern of a prominent U wave (positive wave) following the T wave. Loop diuretics cause hypokalemia by blocking the Na+-K+-2 Cl cotransport system in the thick ascending limb in the renal medulla. This causes increased exchange of Na+ for K+ in the late distal tubules and collecting tubules and increased urinary loss of K+ resulting in hypokalemia. Hypokalemia causes muscle weakness by interfering with the normal repolarization of muscle.

Option A (Aldosterone blocker) is incorrect. An aldosterone blocker (e.g., spironolactone) interferes with the normal exchange of Na+ for K+ in the late distal and collecting tubules, resulting in increased urinary loss of Na+ (diuretic effect) and retention of K+ (hyperkalemia). Hyperkalemia produces a peaked T wave on an ECG. Option B (-Adrenergic blocker) is incorrect. -Adrenergic blockers inhibit the Na+, K+-ATPase pump, causing Na+ to move into cells and K+ out of cells (may produce hyperkalemia). Hyperkalemia produces peaked T waves. Option C (Calcium channel blocker) is incorrect. Calcium channel blockers produce sinus bradycardia. They do not produce electrolyte abnormalities (e.g., hypokalemia). Option D (Digitalis) is incorrect. Digitalis inhibits the Na+, K+-ATPase pump in cardiac muscle, which normally enhances the movement of Na+ into the muscle and the opening of calcium channels resulting in an increase in muscle contraction. Since K+ is pumped out of the cell, hyperkalemia may occur in digitalis toxicity.

A 62-year-old woman with chronic ischemic heart disease has dyspnea and decreased urine output. Physical examination reveals bibasilar crackles in the lungs. There is no neck vein distention or dependent pitting edema. A serum blood urea nitrogen (BUN) is 60 mg/dL and serum creatinine is 2 mg/dL. The patient is not taking diuretics. Which of the following laboratory findings is most likely present?

A. Fractional excretion of sodium (FENa+) < 1% B. Random urine sodium > 40 mEq/L C. UOsm < 350 mOsm/kg D. Urine sediment with renal tubular cell casts E. Urine sediment with waxy casts
Option A (Fractional excretion of sodium (FENa +) < 1%) is correct. The patient has left-sided heart failure (dyspnea, pulmonary edema with inspiratory crackles) causing a decrease in cardiac output leading to a decrease in renal blood flow, glomerular filtration rate, and oliguria. The serum BUN:creatinine ratio is 30 (60:2), which is consistent with a prerenal azotemia. The FENa+ is a sensitive indicator of tubular function. Values < 1% indicate intact tubular function, while those > 2% indicate tubular dysfunction. In prerenal azotemia, tubular function is intact; therefore, the FENa + < 1%. Option B (Random urine sodium > 40 mEq/L) is incorrect. Since tubular function is intact, reabsorption of sodium in the proximal and distal tubules is normal and the random urine sodium < 20 mEq/L. A random urine sodium > 40 mEq/L indicates tubular dysfunction. Option C (UOsm < 350 mOsm/kg) is incorrect. Since tubular function is intact, renal concentration is normal and the U Osm > 500 mOsm/kg. A UOsm < 350 mOsm/kg indicates a loss of urine concentration, which is the first sign of tubular dysfunction.

Option D (Urine sediment with renal tubular cell casts) is incorrect. Since tubular function is intact, there should be no casts in the urine. Renal tubular cell casts are present in renal azotemia. Option E (Urine sediment with waxy casts) is incorrect. Waxy casts are present in chronic renal failure, where the FENa+ would be >2.

A 55-year-old man with a lengthy history of chronic ischemic heart disease has difficulty breathing. Physical findings include bibasilar inspiratory crackles, distention of the neck veins, and tender hepatomegaly. The photograph shows the left ankle after finger pressure was applied to the skin. Which of the following chemical alterations is most likely operative in this patient?

A. Decreased plasma renin activity B. Decreased serum concentration of antidiuretic hormone (ADH) C. Decreased serum concentration of atrial natriuretic peptide (ANP) D. Decreased total body sodium concentration E. Increased serum concentration of aldosterone
Option E (Increased serum concentration of aldosterone) is correct. Increased serum concentration of aldosterone (secondary aldosteronism) is partly responsible for retention of sodium in this patient and the presence of pitting edema (slide). Pitting edema is due to an alteration in Starlings pressure (increased in plasma hydrostatic pressure and/or decrease in oncotic pressure). Sodium retention increases plasma hydrostatic pressure and decreases oncotic pressure (dilutes serum albumin due to an excess in plasma volume). The fluid that accumulates in the interstitial tissue

is protein-poor and cell-poor (i.e., transudate). The patient has signs and symptoms of both left-sided heart failure (dyspnea and bibasilar inspiratory crackles) and right-sided heart failure (distention of neck veins, hepatomegaly, and dependent pitting edema). Right-sided heart failure causes a backup of blood into the venous system, which increases the hydrostatic pressure and is the primary factor causing the pitting edema. In biventricular heart failure, decreased cardiac output decreases renal blood flow, which activates the renin-angiotensin-aldosterone (RAA) system, which leads to an increase in the serum concentration of aldosterone. Option A (Decreased plasma renin activity) is incorrect. In biventricular heart failure, decreased cardiac output activates the RAA system and leads to an increase (not decrease) in plasma renin activity. Option B (Decreased serum concentration of antidiuretic hormone (ADH)) is incorrect. In biventricular heart failure, a decrease in cardiac output automatically stimulates the release of ADH from the posterior pituitary gland, leading to an increase (not decrease) in serum concentration of the hormone. ADH increases renal reabsorption of free water (water without electrolytes) and produces hyponatremia, which would have been present in this patient. Option C (Decreased serum concentration of atrial natriuretic peptide (ANP)) is incorrect. Biventricular failure causes volume overload in both atria. When either atrium is dilated with blood, ANP is released, with a subsequent increase (not decrease) in the serum concentration. ANP has a diuretic effect in the kidneys, which helps the kidneys eliminate sodium. Brain natriuretic peptide would also be increased because of distention of the ventricles. Option D (Decreased total body sodium concentration) is incorrect. In biventricular heart failure, decreased cardiac output activates the RAA system. The increase in aldosterone is partly responsible for retention of sodium in this patient and contributes to an increase (not decrease) in total body sodium, which is invariably present in the pitting edema states (e.g., right-sided heart failure, cirrhosis, nephrotic syndrome). The primary source for the increase in total body sodium is increased renal reabsorption of sodium in the proximal tubules of the kidneys.

An infant born prematurely died of respiratory failure 3 days after birth. The photographs show a radiograph of the lungs and an H&E-stained section from one of the lungs removed at autopsy. Which of the following is the most likely cause of the lung disorder?

A. Bacterial pneumonia

B. Decreased production of surfactant C. Left-sided heart failure D. Pulmonary infarction E. Viral pneumonia
Option B (Decreased production of surfactant) is correct.The patient died of respiratory distress syndrome (RDS), which is caused by a deficiency of surfactant. The photograph shows dilated respiratory bronchioles and alveolar ducts lined by fibrin-rich membranes (hyaline membrane). The chest x-ray shows fine, uniform granularity (ground glass appearance) throughout both lung fields. Total collapse (atelectasis) of the subjacent alveoli is also shown. Surfactant normally decreases the surface tension in small airways, which decreases the collapsing pressure of the small airways during expiration. A decrease in surfactant causes massive atelectasis in the alveoli and widespread intrapulmonary shunting of blood, because the lungs are perfused but not ventilated. Causes of RDS of the newborn include prematurity (surfactant synthesis begins at 28 weeks), poor glycemic control in a mother with diabetes mellitus (insulin inhibits surfactant synthesis), and delivery by cesarean section (lack of stimulation of surfactant synthesis by cortisol released during the stress of a vaginal delivery). Option A (A. Bacterial pneumonia) is incorrect.Bacterial pneumonia is incorrect. Bacterial pneumonia is associated with a diffuse infiltrate of neutrophils and the formation of microabscesses in the lungs. Option C (Left-sided heart failure) is incorrect. Left-sided heart failure produces pulmonary edema with a pink-staining intra-alveolar fluid without hyaline membrane formation. Option D (Pulmonary infarction) is incorrect. A pulmonary infarction shows vague outlines of pulmonary parenchyma due to widespread coagulation necrosis. Option E (Viral pneumonia) is incorrect. Viral pneumonias are associated with interstitial inflammation by mononuclear type cells.

The photograph shows the foot of a 56-year-old man with poorly controlled type 2 diabetes mellitus. Which of the following best describes the type of necrosis that is present?

A. Caseous necrosis B. Coagulation necrosis C. Enzymatic fat necrosis D. Fibrinoid necrosis E. Liquefactive necrosis
Option B (Coagulation necrosis) is correct. Coagulation necrosis is associated with sudden cessation of arterial blood flow to the tissue. Diabetics commonly have atherosclerotic peripheral vascular disease that produces gangrene involving the lower extremities. The photograph shows toes with reddish-blue to black discoloration and ulceration. The skin is dry and parchment-like consistent with dry gangrene, which is a type of coagulation necrosis. When the dead tissue becomes infected by anaerobes (e.g., Clostridium perfringens), liquefactive necrosis predominates (wet gangrene), because of the hydrolytic enzymes released by neutrophils in the infected tissue. Option A (Caseous necrosis) is incorrect. Caseous necrosis is a variant of coagulation necrosis and appears as a soft, cheeselike material that exudes from the center of a granuloma. Caseous necrosis occurs primarily in patients with mycobacterial infection (e.g., tuberculosis) or systemic fungal infection (e.g., histoplasmosis). Option C (Enzymatic fat necrosis) is incorrect. Enzymatic fat necrosis occurs in the pancreas, where it appears as chalky white areas in adipose tissue in and around an area of inflammation (acute pancreatitis). It does not occur in the skin. Option D (Fibrinoid necrosis) is incorrect. Fibrinoid necrosis occurs in small muscular arteries, arterioles, venules, and glomerular capillaries. Immunologic damage to these vessels causes leakage of plasma proteins into the wall of the vessels. It is commonly associated with small-vessel vasculitis in hypertension and with deposits of antigen-antibody complexes in vessel walls in immunocomplex disease. Option E (Liquefactive necrosis) is incorrect. Liquefactive necrosis is associated with softening of tissue caused by the release of hydrolytic enzymes from neutrophils, often in the setting of acute bacterial infection. There is no evidence of a purulent exudate in the toes of this patient.

A 62-year-old woman complains of pain in the lower lumbar spine, pelvis, and sternum. Initial laboratory studies show a moderately severe normocytic anemia with rouleaux. Serum blood urea nitrogen (BUN) is 80 mg/dL, and serum creatinine is 8 mg/dL. The photograph shows a representative section of a bone marrow aspirate taken in the right posterior iliac crest. Which of the following additional laboratory findings is most likely to be reported?

A. Decreased erythrocyte sedimentation rate B. Decreased serum calcium level C. Increased prothrombin time D. Monoclonal protein spike on serum protein electrophoresis E. Normal bleeding time
Option D (Monoclonal protein spike on serum protein electrophoresis) is correct. The patient has multiple myeloma complicated by renal failure. The bone marrow aspirate shows immature plasma cells with eccentric nuclei and perinuclear clearing. The cytoplasm stains blue with a Wright-Giemsa stain, indicating increased synthesis of protein. A monoclonal spike in the -globulin region can be expected on serum protein electrophoresis. The spike is caused by a single immunoglobulin (usually IgG) and its corresponding light chain (usually , which are secreted by clones derived from a single neoplastic plasma cell. The other plasma cell clones are suppressed. Renal failure, a common complication of multiple myeloma, is diagnosed when serum BUN and serum creatinine levels are raised and the ratio of these values < 15:1. Excess light chains, called Bence Jones protein, are usually present in the urine. In this case, renal failure is due to the formation of casts composed of Bence Jones protein blocking the tubular lumens and inciting a foreign body giant cell reaction. Option A (Decreased erythrocyte sedimentation rate) is incorrect. The increased weight of RBCs in rouleaux (RBCs stacked together like coins) causes them to settle faster in plasma and increases (not decreases) the erythrocyte sedimentation rate (ESR). The ESR is not a very specific test and is not a key test to use in confirming the diagnosis of a plasma cell malignancy. Option B (Decreased serum calcium level) is incorrect. The patient has multiple myeloma, which frequently causes osteolytic lesions. This is due to the release of interleukin-1 (osteoclast activating factor) from malignant plasma cells. This cytokine activates osteoclasts causing lytic lesions wherever the malignant plasma cells are located. These lesions tend to increase (not decrease) the serum calcium levels. Option C (Increased prothrombin time) is incorrect. In multiple myeloma, coagulation studies, including prothrombin time, are normal. Rouleaux does not affect the prothrombin time. Option E (Normal bleeding time) is incorrect. In multiple myeloma, the bleeding time is usually prolonged, because rouleaux interferes with platelet aggregation.

A cholecystectomy is performed on a 55-year-old woman, and the incision is not healing properly. When asked about her diet, the woman says that she consumes a diet high in protein but does not eat fruits or vegetables. Which of the following events most likely accounts for the poor wound healing?

A. Decreased synthesis of granulation tissue B. Decreased synthesis of type III collagen C. Decreased tensile strength of collagen D. Defect in fibrillin in elastic tissue E. Leukocyte adhesion molecule defect
Option C (Decreased tensile strength of collagen) is correct. This patient is lacking vitamin C due to a lack of fruits and vegetables in her diet. Lack of cross-linking tropocollagen molecules causes decreased tensile strength in collagen and poor wound healing. Vitamin C (ascorbic acid) is important in the hydroxylation of proline and lysine in the initial phases of collagen synthesis by fibroblasts. The sites of hydroxylation are the anchor sites for the cross-links between the tropocollagen molecules. Option A (Decreased synthesis of granulation tissue) is incorrect. Granulation tissue is produced in patients with vitamin C deficiency. However, the type III collagen produced lacks tensile strength. Option B (Decreased synthesis of type III collagen) is incorrect. Type III collagen is produced in vitamin C deficiency; however, it lacks tensile strength. Option D (Defect in fibrillin in elastic tissue) is incorrect. A defect in fibrillin is present in Marfan syndrome, causing weakness of elastic tissue, thus weakening the aorta (e.g., aortic dissection) and ligaments. Vitamin C deficiency does produce defects in the cross-linking of fibrillin. Option E (Leukocyte adhesion molecule defect) is incorrect. A defect in neutrophil adhesion molecules (integrins and selectins) prevents neutrophils from adhering to endothelial cells and transmigrating into tissue. This does interfere with proper wound healing; however, leukocyte adhesion defects are not caused by vitamin C deficiency.

An obese 45-year-old man decides to undertake a strenuous training program to lose weight. On the morning after his first 2-hour workout, he has generalized muscle aches and pains and pink-colored urine. A urine reagent strip is positive for blood and protein. The serum blood urea nitrogen (BUN) and creatinine are normal. A CBC is normal. What is the most likely diagnosis?

A. Acute glomerulonephritis

B. Acute renal failure (ARF) C. Hemoglobinuria D. Myoglobinuria E. Renal stone


Option D (Myoglobinuria) is correct. The patient, who is most likely in poor physical condition, has damaged striated muscle (generalized muscle pain) causing the release of myoglobin, which produces a pink-colored urine and a positive urine reagent strip for blood and protein. The reagent strip for blood does not distinguish hemoglobin from myoglobin. A serum creatine kinase would be markedly elevated in this patient. Option A (Acute glomerulonephritis) is incorrect. The nephritic type of acute glomerulonephritis produces hematuria and proteinuria; however, it is not associated with generalized muscle pain. Furthermore, acute glomerulonephritis is associated with an increase in serum BUN and creatinine (normal in this patient) due to a decrease in the glomerular filtration rate. Option B (Acute renal failure (ARF)) is incorrect. Myoglobinuria is a potential cause of ARF; however, the serum BUN and creatinine are normal in this patient and it is increased in ARF. Option C (Hemoglobinuria) is incorrect. Hemoglobinuria is most commonly due to an intravascular hemolytic anemia (e.g., glucose 6-phosphate dehydrogenase deficiency). The urine reagent strip is positive for blood and protein. However, since the patient does not have anemia, this diagnosis is excluded. Option E (Renal stone) is incorrect. A renal stone presents with a sudden onset of flank pain with radiation of pain into the ipsilateral groin. Hematuria is invariably present. None of these findings are present in the patient.

A 57-year-old-man from Puerto Rico has a 40-pack-year history of smoking cigarettes and he drinks an occasional beer on weekends. He presents with ascites, dependent pitting edema, and splenomegaly. Endoscopy reveals esophageal varices. A liver biopsy shows concentric fibrosis around the portal vein. A complete blood cell count shows eosinophilia. What is the most likely diagnosis?

A. Alcoholic cirrhosis B. Clonorchiasis C. Primary sclerosing cholangitis (PSC) D. Schistosomiasis

Option D (Schistosomiasis) is correct. Schistosomes are flukes (trematodes). The schistosomal life cycle involves: eggs (human) ciliated miracidia larva infects snail (first intermediate host) produce fork-tailed cercariae larvae penetrate skin in human produce disease. This patient is infected by Schistosoma mansoni, which is found throughout Africa and Latin America including Puerto Rico. The larvae enter the superior mesenteric vein and from there into the portal vein. In the portal vein, they develop into adult worms that deposit eggs (have a sharp lateral spine) to which the host develops an inflammatory response marked by concentric fibrosis (pipestem cirrhosis) in the vessel wall. Complications of cirrhosis include portal hypertension, ascites, and esophageal varices, all of which are present in this patient. There is no association with smoking. Because it is an invasive helminthic infection, eosinophilia is present. Treatment is praziquantel. Option A (Alcoholic cirrhosis) is incorrect. Although alcohol is the most common cause of cirrhosis in the United States, biopsies do not show concentric fibrosis around the portal vein tributaries. Furthermore, eosinophilia is not a feature of alcoholic cirrhosis. Option B (Clonorchiasis) is incorrect. Clonorchis sinensis is an intestinal fluke (trematode) that, unlike schistosomes, has two intermediate hosts in its life cycle: a snail is the first intermediated host and fish are the second intermediate host. It commonly occurs in China, Japan, and Korea. It is contracted by ingesting encysted larvae in fish. The larvae enter the common bile duct and become adults. Cholangiocarcinoma is a complication. Option C (Primary sclerosing cholangitis (PSC)) is incorrect. PSC is an obliterative fibrosis of intrahepatic and extrahepatic bile ducts. It is a male dominant disease that is most often associated with inflammatory bowel disease (70%), particularly ulcerative colitis (not present in this patient). Endoscopic retrograde cholangiopancreatography shows narrowing and dilation of bile ducts (beading). Complications include cirrhosis and cholangiocarcinoma.

The photograph shows the superior view of an unopened aortic valve in a 55-year-old man. Which of the following complications commonly occurs with this type of aortic valve lesion?

A. Acute myocardial infarction (MI)

B. Aortic dissection C. Hemolytic anemia D. Hypertension E. Ischemic stroke

Option C (Hemolytic anemia) is correct. The photograph shows a bicuspid aortic valve with severe aortic stenosis due to fibrocalcific involvement of the two valve cusps. A common complication of severe aortic stenosis is an intravascular hemolytic anemia with schistocytes (fragmented RBCs; microangiopathic hemolytic anemia), which may lead to chronic iron-deficiency anemia from loss of hemoglobin in the urine. Serum haptoglobin levels are often zero, because haptoglobin combines with free hemoglobin in the plasma and is removed from the circulation by splenic macrophages. Absent haptoglobin levels is a sign for cardiovascular surgeons to replace the valve. Option A (Acute myocardial infarction (MI)) is incorrect. Aortic stenosis is the most common valvular lesion associated with angina with exercise due to concentric hypertrophy of the left ventricle. The thickened muscle wall does not receive sufficient blood flow during exercise resulting in angina from subendocardial ischemia. There is no increased risk for a myocardial infarction. Option B (Aortic dissection) is incorrect. Aortic dissection is associated with hypertension and connective tissue disorders leading to cystic medial degeneration. It is not a complication of aortic stenosis. Option D (Hypertension) is incorrect. In aortic stenosis, the diminished area of the stenotic valve orifice eventually leads to a decrease in stroke volume and cardiac output, which reduces systolic pressure but has no effect on diastolic pressure. It is not a cause of hypertension. Option E (Ischemic stroke) is incorrect. Aortic stenosis is the most common valvular lesion associated with syncope with exercise. However, it is not associated with an ischemic stroke, which is most often related to atherosclerosis or embolic disease to the middle cerebral artery.

A 25-year-old man develops hemoptysis. A few weeks later, he experienced sudden onset of acute renal failure and died. Prior to his death, urinalysis showed mild proteinuria, hematuria, and RBC casts. The photograph shows an immunofluorescence study of a representative glomerulus in a section of kidney removed at autopsy. Which of the following is the most likely diagnosis?

A. Diffuse membranous glomerulopathy B. Focal segmental glomerulosclerosis C. Goodpasture syndrome D. IgA glomerulopathy E. Minimal change disease
Option C (Goodpasture syndrome) is correct. Goodpasture syndrome is more common in men and is associated with IgG antibasement membrane antibodies that are directed against pulmonary capillary and glomerular capillary basement membranes (type II hypersensitivity reaction). The photograph shows uninterrupted (linear) smooth immunofluorescence along the glomerular basement membrane. Pulmonary involvement with hemoptysis usually occurs before renal failure. Renal failure is most often due to rapidly progressive crescentic glomerulonephritis, which is associated with a nephritic presentation (hematuria, RBC casts, mild proteinuria), as in this case. Treatment options include plasma exchange, immunosuppressive therapy with corticosteroids and cyclophosphamide, and renal transplantation. Option A (Diffuse membranous glomerulopathy) is incorrect. Diffuse membranous glomerulopathy is the most common cause of the nephrotic syndrome in adults (pitting edema, fatty casts). It is an immunocomplex disorder with deposition of immunocomplexes in a subepithelial location, which produce granular immunofluorescence. Option B (Focal segmental glomerulosclerosis) is incorrect. Focal segmental glomerulosclerosis is most often associated with AIDS and with intravenous heroin addiction. It causes the nephrotic syndrome. Glomerular injury is due to cytokine damage of the visceral epithelial cells. It is not an immunocomplex disease, therefore the immunofluorescent study is negative. Option D (IgA glomerulopathy) is incorrect. IgA glomerulonephritis is an immunocomplex type of glomerulonephritis associated with episodic bouts of microscopic or macroscopic hematuria. The immunocomplexes produce granular immunofluorescence primarily located in the mesangium. Option E (Minimal change disease) is incorrect. Minimal change disease (lipoid nephrosis) is the most common cause of the nephrotic syndrome in children. Cytokine damage to the basement membrane causes a loss of the negative charge, resulting in a selective loss of albumin in the urine. Patients usually are normotensive. It is not an immunocomplex disease; therefore, the immunofluorescent study is negative.

The upper limit of normal of the fasting blood glucose for diagnosing diabetes mellitus has been lowered from 140 mg/dL to 126 mg/dL. What effect does this change have on diagnosing diabetes mellitus?

A. Decreases the number of false positive (FP) diagnoses of diabetes B. Decreases the sensitivity and increases the specificity of the test C. Increases the number of false negative (FN) test results D. Increases the predictive value of a negative test result E. Increases the predictive value of a positive test result
Option D (Increases the predictive value of a negative test result) is correct. People with disease either have a true positive (TP) or a FN test result. A TP test result is a positive test in a person with disease. A FN test result is a negative test result in a person with disease. People in a control group, who do not have disease, either have a true negative (TN) or a FP test result. A TN test result is a negative test in a person without disease, while a FP test result is a positive test result in a person without disease. The sensitivity of a test is the likelihood of having positive test results in patients who have a selected disease. Since people with disease either have TP or FN test results, the formula for calculating the sensitivity of a test is TP/TP + FN. The less the FN rate, the greater the sensitivity of the test. Tests with 100% sensitivity are most often used as screening tests for disease. The predictive value of a negative test result (PV-) is the likelihood that a negative test result is a TN rather than a FN; therefore, the formula for calculating the PV- is TN/TN + FN. Tests with 100% sensitivity always have a 100% PV-; therefore, a negative test result excludes disease. The specificity of a test is the likelihood of having a negative test result in a person without disease. Since people without disease have test results that are either a TN or a FP, the formula for calculating the specificity of a test is TN / TN + FP. The less the FP rate, the greater the specificity of a test. A test with no FPs has 100% specificity. The predictive value of a positive test result (PV+) is the likelihood that a positive test result is a TP rather than a FP; therefore, the formula for PV+ is TP/TP + FP. Tests with 100% specificity always have a PV+ of 100%; therefore, they are most useful for confirming disease. Assume that the original cutoff point for diagnosing diabetes mellitus is set for 100% specificity (schematic A). The

normal population is under the solid curve, while the population with diabetes mellitus is under the curve with the dotted line. Note that beyond a 140 mg/dL cutoff point, all people have TP test results for diabetes. Also note that in the normal range, the number of FNs is increased, which decreases the sensitivity and the PV- of the test. Assume that the new cutoff point for diagnosing diabetes mellitus (126 mg/dL) is set for 100% sensitivity (schematic B). Note that there are no FNs in the normal range; however, beyond the cutoff point of 126 mg/dL there is an increase in the number of FPs, which decreases the specificity and PV+ of the test. Therefore, creating a test with 100% sensitivity and PVautomatically decreases specificity and PV+, while creating a test with 100% specificity and PV+ automatically decreases sensitivity and PV-. Option A (Decreases the number of false positive (FP) diagnoses of diabetes) is incorrect. In schematic B, the number of FPs increases (not decreases). Whenever the sensitivity of a test is increased by altering the normal range, the specificity of the test and the PV+ is automatically decreased. Option B (Decreases the sensitivity and increases the specificity of the test) is incorrect. Lowering the cutoff point to 126 mg/dL increases the sensitivity of the test (not decreases), and decreases the specificity of the test (not increases). Option C (Increases the number of false negative (FN) test results) is incorrect. Lowering the cutoff point to 126 mg/dL decreases the number of FN test results (not increases), which increases sensitivity and PV-. Option E (Increases the predictive value of a positive test result) is incorrect. Lowering the cutoff point to 126 mg/dL increases the number of FP test results, which decrease specificity and PV+ (not increases).

A 24-year-old woman has fever and dull, continuous pain in the right upper quadrant that radiates to the right scapula and shoulder. Physical examination shows scleral icterus, right upper quadrant tenderness, and splenomegaly. An ultrasonogram of the gallbladder shows gallstones. Laboratory studies reveal a normocytic anemia and an increased corrected reticulocyte count. The photograph shows the peripheral blood smear. Which of the following studies would most likely confirm the cause of the anemia?

A. Hemoglobin electrophoresis B. Osmotic fragility test C. Serologic tests for viral hepatitis

D. Serum ferritin test E. Serum transaminase tests

Option B (Osmotic fragility test) is correct. The patient has hereditary spherocytosis, an autosomal dominant disorder in which a defect in ankyrin causes abnormalities in the RBC membrane that result in loss of membrane and conversion of a biconcave disc to a spherocyte. The peripheral blood smear shows numerous spherocytes that have no central area of pallor. Spherocytes are inherently fragile because of the defective cell membrane. In addition, there is increased permeability of spherocytes to sodium due to a dysfunctional Na+/K+ ATPase pump in the cell membrane. The diagnosis is best confirmed by an osmotic fragility test, which shows increased fragility of the RBCs when placed in test tubes of varying tonicity when compared to normal RBCs. Spherocytes are phagocytosed by splenic macrophages, resulting in a hemolytic anemia (extravascular type) and an increase in unconjugated bilirubin from macrophage degradation of hemoglobin. Unconjugated bilirubin is converted to conjugated (direct) bilirubin in the liver. Bile becomes saturated with the water-soluble conjugated bilirubin, resulting in the formation of jet black calcium bilirubinate stones (see Fig. 18-16 in Rapid Review Pathology, 3rd edition). This may precipitate acute cholecystitis, which is the cause of this patients right upper quadrant pain. The treatment for hereditary spherocytosis is splenectomy and cholecystectomy for cholecystitis. Option A (Hemoglobin electrophoresis) is incorrect. Hemoglobin electrophoresis detects changes in the concentration of normal and abnormal forms of hemoglobin, such as hemoglobin S in sickle cell disease. Option C (Serologic tests for viral hepatitis) is incorrect. In this patient, the jaundice is due to extravascular hemolysis of RBCs with increased production of unconjugated bilirubin. Viral hepatitis produces a mixed hyperbilirubinemia with an increase in unconjugated and conjugated bilirubin. Furthermore, there is no splenomegaly or spherocytosis in viral hepatitis. Option D (Serum ferritin test) is incorrect. Ferritin is a soluble iron-binding protein that is primarily used to evaluate iron disorders, where iron is decreased (e.g., iron deficiency) or increased (e.g., anemia of chronic disease). Serum ferritin is normal in hereditary spherocytosis. Option E (Serum transaminase tests) is incorrect. Serum transaminase levels are primarily elevated in diffuse liver cell necrosis (e.g., viral hepatitis) and would not be expected to be increased in hereditary spherocytosis.

The photograph shows two children, each with a different nutritional disorder. Which of the following findings would most likely be associated with the child on the left rather than the child on the right?

A. Decrease in serum albumin B. Decrease in somatic protein stores C. Decrease in subcutaneous fat D. Decrease in total calorie intake E. Defects in cell-mediated immunity
Option A (Decrease in serum albumin) is correct. The child on the left has kwashiorkor, and the child on the right has marasmus. Both conditions are examples of protein-energy malnutrition. Kwashiorkor is caused by decreased protein intake, which decreases the serum albumin level. The total calorie intake is normal because of increased intake of carbohydrates. A decrease in albumin decreases the plasma oncotic pressure, resulting in dependent edema (see photograph, child on left). In marasmus, serum albumin levels are normal in spite of diminished intake of calories. Option B (Decrease in somatic protein stores) is incorrect. Somatic protein stores represent the protein in muscle. In marasmus, somatic protein stores are decreased, resulting in muscle wasting in the extremities (see photograph, child on right). Somatic proteins are used as substrate for gluconeogenesis. In kwashiorkor, somatic proteins are relatively intact, because excess carbohydrates provides enough carbohydrates for fuel, without having to use amino acids. However, in kwashiorkor the visceral protein stores (e.g., in the liver) are decreased. Option C (Decrease in subcutaneous fat) is incorrect. In marasmus, subcutaneous fat is absent; in kwashiorkor, it is normal. Option D (Decrease in total calorie intake) is incorrect. In marasmus, the total calorie intake is decreased; in kwashiorkor, the total calorie intake is normal but deficient in protein.

Option E (Defects in cell-mediated immunity) is incorrect. Defects in cell-mediated immunity (type IV hypersensitivity) are present in both kwashiorkor and marasmus.

A 48-year-old man with essential hypertension has muscle weakness and clinical evidence of volume depletion. He is currently taking a thiazide diuretic. An electrocardiogram (ECG) shows prominent U waves. Which one of the following electrolyte profiles is most likely to be present in this man? Serum Na+ Serum K+ Serum Cl Serum HCO3 (135147 mEq/L) (3.55.0 mEq/L) (95105 mEq/L) (2228 mEq/L) A.128 5.9 96 20 B. 146 5.5 104 18 C. 138 2.2 114 14 D.130 2.9 80 36 E. 152 2.8 110 33

A. serum Na+ 128, serum K+ 5.9, serum Cl 96, serum HCO3 20 B. serum Na+ 146, serum K+ 5.5, serum Cl 104, serum HCO3 18 C. serum Na+ 138, serum K+ 2.2, serum Cl 114, serum HCO3 14 D. serum Na+ 130, serum K+ 2.9, serum Cl 80, serum HCO3 36 E. serum Na+ 152, serum K+ 2.8, serum Cl 110, serum HCO3 33
Option D (serum Na+ 130, serum K+ 2.9, serum Cl 80, serum HCO3 36) is correct. Thiazide diuretics inhibit the Cl channel in the Na+-Cl cotransporter located in the early distal tubule. This causes an increase in the urinary loss of Na+ (hyponatremia) and Cl (hypochloremia). Increased delivery of Na+ to the late distal and collecting tubules results in augmented exchange of Na+ for K+ in the aldosterone-dependent Na+ channels resulting in increased urinary loss of K+ (hypokalemia with U waves on an ECG). When K+ is depleted, Na+ exchanges with H+ ions causing increased synthesis and reabsorption of HCO3 causing metabolic alkalosis. Option A (serum Na+ 128, serum K+ 5.9, serum Cl 96, serum HCO3 20) is incorrect. This electrolyte profile is most compatible with mineralocorticoid deficiency (e.g., Addisons disease, 11-hydroxylase deficiency). There is inhibition of Na+ reabsorption and K+ secretion by the aldosterone-dependent Na+ and K+ channels located in the distal tubule and collecting ducts causing a hypertonic loss of Na+ in the urine (hyponatremia) and retention of K+ (hyperkalemia with peaked T waves). Due to dysfunction of the aldosterone-dependent H+/K+ ATPase pump in the collecting tubules, there is retention of H+ ions and a subsequent lack of synthesis of HCO3, which produces metabolic acidosis. The excess H+ ions combine with Cl anions producing a normal anion gap type of metabolic acidosis, calculated as follows: anion gap = serum Na+ 128 (serum Cl 96 + serum HCO320) = 12 mEq/L. Option B (serum Na+ 146, serum K+ 5.5, serum Cl 104, serum HCO3 18) is incorrect. These findings are most compatible with chronic renal failure. In chronic renal failure there is tubular cell dysfunction resulting in retention of K + (hyperkalemia with peaked T waves on an ECG) and an increased anion gap type of metabolic acidosis due to retention

of organic acids like sulfuric and phosphaturic acid. The anion gap in this case is calculated as follows: anion gap = serum Na+ 146 (serum Cl 104 + serum HCO318) = 24 mEq/L (12 mEq/L ( 2). Option C (serum Na+ 138, serum K+ 2.2, serum Cl 114, serum HCO3 14) is incorrect. These findings are most compatible with a secretory type of diarrhea (e.g., travelers diarrhea due to enterogenic Escherichia coli, cholera). In these conditions a toxin activates adenylate cyclase in enterocytes causing the ion pumps in the small intestine to secrete isotonic fluid. Loss of isotonic fluid does not alter the serum Na+ concentration. Diarrheal fluid is rich in K+ and HCO3, the former resulting in hypokalemia and the latter a normal anion gap type of metabolic acidosis. The anion gap calculation is as follows: anion gap = serum Na+ 138 ( (serum Cl 114 + serum HCO314) = 10 mEq/L (12 mEq/L 2). The serum Cl is increased, because it replaces the HCO3 anions that are lost in the diarrheal fluid to maintain electroneutrality. Option E (serum Na + 152, serum K+ 2.8, serum Cl 110, serum HCO3 33) is incorrect. This profile is compatible with mineralocorticoid excess (e.g., primary aldosteronism; 11-hydroxylase deficiency). Enhanced function of aldosteronemediated Na+-H+ channels in the late distal and collecting ducts increases the synthesis of HCO3 leading hypernatremia, hypokalemia (initially exchanged with Na+ until depleted), and metabolic alkalosis (Na+ exchanges with H+; for every H+ lost in the urine there is a corresponding HCO3 entering the blood).

A 60-year-old man with a history of chronic alcoholism complains of severe chest pain with radiation of the pain down the inner side of the left arm. An ECG shows an acute anterior myocardial infarction. A complete blood count shows a macrocytic anemia with hypersegmented neutrophils and pancytopenia. Physical examination shows no neurologic abnormalities. Plasma homocysteine is increased. Which of the following is the most likely cause of the increase in plasma homocysteine?

A. Decreased activity of cystathionine synthase B. Decreased activity of dihydrofolate reductase C. Decreased activity of thymidylate synthase D. Decreased serum folate E. Decreased serum vitamin B12

Option D (Decreased serum folate) is correct. Folate deficiency produces a macrocytic anemia, hypersegmented neutrophils, and pancytopenia (anemia, neutropenia, thrombocytopenia). Alcoholism is the most common cause of folate deficiency in the United States. The pathogenesis of folate deficiency in alcoholics involves poor diet and decreased reabsorption of folate in the jejunum. In folate metabolism, vitamin B12 removes the methyl group from N5methyltetrahydrofolate (circulating form of folate) to produce methylvitamin B12 and tetrahydrofolate. The methyl group is then transferred to homocysteine to produce methionine. Therefore, a decrease in N5-methyltetrahydrofolate or vitamin B12 causes an increase in plasma homocysteine. An increase in plasma homocysteine causes damage of endothelial cells and the potential for vessel thrombosis, which may have contributed to the anterior myocardial

infarction in this patient. Option A (Decreased activity of cystathionine synthase) is incorrect. Deficiency of cystathionine synthase is present in homocystinuria, which is a rare autosomal recessive disease. Cystathionine synthase catalyzes the reaction that converts homocysteine to cystathionine causing an increase in homocysteine. Clinical findings include vessel thrombosis and mental retardation, the latter not present in this patient. Option B (Decreased activity of dihydrofolate reductase) is incorrect. Dihydrofolate reductase converts oxidized dihydrofolate to tetrahydrofolate. Inhibition of dihydrofolate reductase by methotrexate or trimethoprim causes folate deficiency and a macrocytic anemia. There is no history of the patient taking methotrexate or trimethoprim. Option C (Decreased activity of thymidylate synthase) is incorrect. Thymidylate synthase converts deoxyribose uridine monophosphate to deoxyribose thymidine monophosphate, which is used in DNA synthesis. 5-fluorouracil inhibits thymidylate synthase, causing a macrocytic anemia. There is no history of the patient taking 5-fluorouracil. Option E (Decreased serum vitamin B12) is incorrect. Vitamin B12 deficiency causes the same hematologic abnormalities as folate deficiency as well as demyelination of the posterior column and lateral corticospinal tracts in the spinal cord. Neurologic findings are absent in this patient, which rules out vitamin B12 deficiency.

A febrile 32-year-old woman with systemic lupus erythematosus (SLE) complains of fatigue and yellow eyes. She is not taking any prescription medications. Physical examination reveals splenomegaly. A complete blood cell count shows a hemoglobin of 5 g/dL and a normal mean corpuscular volume (MCV), platelet count, and WBC count. The peripheral smear shows spherocytes and polychromasia. The corrected reticulocyte count is 12%. Which of the following laboratory findings is most likely to be reported? A. Abnormal hemoglobin electrophoresis B. Absent urine urobilinogen C. Decreased serum ferritin D. Increased conjugated bilirubin E. Positive direct Coombs test

Option E (Positive direct Coombs test) is correct. The patient has SLE complicated by an autoimmune hemolytic anemia. These are subdivided into warm types (IgG-mediated) and cold types (IgM-mediated). The former type is usually an extravascular hemolytic anemia, while the latter type is usually intravascular but sometimes extravascular. The direct Coombs test detects IgG and/or C3b on the surface of RBCs and must be positive in order to confirm the diagnosis of any type of immune hemolytic anemia, warm or cold. Immune hemolytic anemias that involve IgG (e.g., SLE) have macrophage removal of RBCs coated by IgG (extravascular hemolysis). The end-product of macrophage destruction of RBCs is unconjugated bilirubin, which produces jaundice (yellow eyes). Macrophage removal of portions of the RBC membrane produces spherocytes, which are characteristic findings in extravascular hemolytic anemias. Cold autoimmune hemolytic anemias involving IgM antibodies that produce hemoglobinuria and jaundice are not as likely to be present. Option A (Abnormal hemoglobin electrophoresis) is incorrect. Hemoglobin detects differences in the concentration of normal hemoglobins (e.g., increase in hemoglobin A2) or the presence of abnormal hemoglobins (e.g., sickle hemoglobin). It is normal in autoimmune hemolytic anemias.

Option B (Absent urine urobilinogen) is incorrect. Absence of urine urobilinogen occurs in complete obstruction of bile outflow into the intestines. Colonic bacteria normally convert bilirubin in bile into urobilinogen, which is responsible for the color of stool. A small amount of urobilinogen reabsorbed from the intestine into the blood enters the urine, where it produces the normal color of urine. In autoimmune hemolytic anemia, the urine urobilinogen is increased (not absent), because greater amounts of bilirubin from the hemolytic anemia are converted to urobilinogen in the colon and proportionately more urobilinogen is recycled into the urine. Option C (Decreased serum ferritin) is incorrect. Serum ferritin correlates with the amount of ferritin stores in bone marrow macrophages. It is usually normal in autoimmune hemolytic anemias. Ferritin levels are only decreased in iron deficiency anemia. Option D (Increased conjugated bilirubin) is incorrect. Macrophage destruction of RBCs increases the production of unconjugated bilirubin, not conjugated bilirubin.

A febrile 23-year-old woman has had recurrent urinary tract infections since early childhood. She now has acute onset of right flank pain, suprapubic discomfort, dysuria, and urinary frequency. The urinary sediment examination shows clumps of WBCs, WBC casts, occasional RBCs, and numerous motile bacteria. Which of the following acute conditions is the most likely diagnosis? A. Cystitis B. Glomerulonephritis C. Pyelonephritis D. Tubular necrosis E. Urethritis

Option C (Pyelonephritis) is correct. This patient has acute pyelonephritis, which is an acute bacterial infection (usually caused by Escherichia coli) that involves the tubules and interstitium (see Fig. 19-9 in Rapid Review Pathology, 3rd edition). Classic signs include fever, flank pain, and WBC casts (see Fig. 19-2F in Rapid Review Pathology, 3rd edition) accompanied by signs of a lower urinary tract infection (e.g., suprapubic discomfort, dysuria, and urinary frequency). The majority of cases occur in women who have recurrent lower urinary tract infections and vesicoureteral reflux, which refers to the passage of urine into the ureter through an incompetent ureterovesical junction. Infection then ascends into the renal pelvis and into the renal parenchyma. Neutrophils in the tubules produce a WBC cast, which is an excellent marker of acute pyelonephritis. Fever, flank pain, and WBC casts are not present in lower urinary tract infections. Option A (Cystitis) is incorrect. Acute cystitis is characterized by lower urinary tract infection signs and symptoms but not by fever, flank pain, or WBC casts in the urine. Option B (Glomerulonephritis) is incorrect. Acute glomerulonephritis is characterized by oliguria (decreased glomerular filtration rate), proteinuria, hematuria, and urine casts (RBC, WBC, or fatty casts), depending on the type of

glomerulonephritis. Most cases are immune-mediated. Therefore, dysuria and urinary frequency are not present. Option D (Tubular necrosis) is incorrect. Acute tubular necrosis is caused by ischemia (e.g., decreased cardiac output) or nephrotoxic drugs (e.g., aminoglycosides). Acute tubular necrosis is characterized by oliguria and is associated with renal tubular cell casts rather than WBC casts. Option E (Urethritis) is incorrect. Acute urethritis is a lower urinary tract infection signs and symptoms but not fever, flank pain, or WBC casts in the urine.

A 58-year-old woman had a 30-year history of alcohol abuse and smoking cigarettes. The photograph shows the liver removed at autopsy. Which of the following is the most likely cause of these lesions?

A. Cirrhosis secondary to alcohol abuse B. Metastases from a colon carcinoma C. Metastases from a lung carcinoma D. Metastatic abscesses from a primary infection in the lungs E. Miliary spread of Mycobacterium tuberculosis
Option C (Metastases from a lung carcinoma) is correct. The liver shows multiple nodular densities with central areas of depression, which are characteristic of metastatic cancer. The tissue between the nodular masses is normal. The lung is the most likely site of the primary tumor. This patients lengthy history of cigarette smoking was a risk factor for such a lesion. Option A (Cirrhosis secondary to alcohol abuse) is incorrect. The liver shows areas of normal hepatic tissue between the lesions. The regenerative nodules of cirrhosis, regardless of its cause, involve the entire liver and leave no intervening areas of normal tissue. Option B (Metastases from a colon carcinoma) is incorrect. The lesions are unlikely to be metastases from a colon carcinoma, which has no established association with alcohol abuse and for which smoking is only a minor risk factor. Option D (Metastatic abscesses from a primary infection in the lungs) is incorrect. Abscesses are associated with

necrosis and cavitation with shaggy borders, unlike the lesions shown in the specimen. Metastatic abscesses from any primary source are uncommon in the liver. Most hepatic abscesses arise from an ascending cholangitis due to a stone impacted in the common bile duct. Option E (Miliary spread of Mycobacterium tuberculosis) is incorrect. Miliary spread of M. tuberculosis to the liver is characterized by small, seedlike lesions, unlike the large, nodular masses of metastatic cancer.

Which of the following clinical disorders is most compatible with the distribution of affected patients shown in this pedigree?

A. Essential hypertension B. Galactosemia C. Hemophilia B D. Marfan syndrome E. Wilsons disease


Option D (Marfan syndrome) is correct. Marfan syndrome is an autosomal dominant disorder that involves a missense mutation affecting the synthesis of fibrillin, a component of elastic tissue. Autosomal dominant disorders are characterized by a dominant allele that expresses itself in either the homozygous or the heterozygous state. As shown in the pedigree, only one parent must have the abnormal allele to transmit disease to the children, and a heterozygous parent with disease will transmit disease to 50% of the children. In one family, two parents who do not express disease have an affected child. In this case, one of the parents (male) has the abnormal allele but is not expressing the disease, which is an example of reduced (incomplete) penetrance of the abnormal allele. This is unique to autosomal dominant disorders.

Option A (Essential hypertension) is incorrect. Essential hypertension is a disorder with multifactorial (polygenic) inheritance. Multifactorial inheritance involves the additive effect of two or more gene mutations of small effect conditioned by environmental and other nongenetic factors. Option B (Galactosemia) is incorrect. Galactosemia is an autosomal recessive disorder involving a deficiency of galactose 1-phosphate uridyltransferase. In autosomal recessive disorders, disease is present only in patients who are homozygous for the abnormal allele (e.g., aa). Both parents must have the abnormal allele and are usually asymptomatic heterozygous carriers for the abnormal allele (e.g., Aa). This pattern is not evident in the pedigree, in which a parent with the disease is present in all families except the one family where the male has reduced penetrance. Option C (Hemophilia B) is incorrect. Hemophilia B is an X-linked recessive disorder characterized by a deficiency of factor IX. X-linked recessive disorders are expressed only in males, whereas females with the abnormal allele are usually asymptomatic carriers. Option E (Wilsons disease) is incorrect. Wilsons disease is an autosomal recessive disease associated with an inability to excrete copper in bile and to incorporate copper into ceruloplasmin, resulting in liver disease and central nervous system damage.

A 58-year-old man had a sudden onset of difficulty breathing and substernal chest pain with radiation down the inner aspects of both arms that lasted for a few hours. The patient subsequently died in a coronary care unit. The photograph shows a cross-section of the left and right ventricles. The anterior portion of the heart is at the top. Based on the gross appearance of the infarction and location, which of the following would most likely have been present on the day of his death?

A. Increased serum creatine kinase (CK)-MB B. Increased serum troponins I and T C. Q waves in leads I and aVL D. Q waves in leads I, V4V6, and aVL

E. Q waves in leads V1V4


Option B (Increased serum troponins I and T) is correct. The patient had an acute myocardial infarction (MI) involving the posterior wall of the left ventricle. Note the bright yellow area of infarction (coagulation necrosis) and the rim of dark red granulation tissue (blood vessels + collagen formation) surrounding the infarction (white arrow in photograph). These findings are present approximately 1 week after a transmural (Q-wave) infarction. The posterior wall of the left ventricle is supplied by the right coronary artery; therefore, a right coronary artery thrombosis is most likely responsible for the infarction. The serum concentration of troponins I and T begin to increase 3 to 12 hours after cardiac injury, peak within 24 hours, and return to normal within 7 to 10 days; hence, these would have been increased at the time of death. Option A (Increased serum creatine kinase (CK)-MB) is incorrect. Levels of CK-MB begin to increase 4 to 8 hours after infarction, peak in 24 hours, and return to normal within 1.5 to 3 days. Therefore, CK-MB would not have been present at the time of death. Option C (Q waves in leads I and aVL) is incorrect. Q waves in leads I and aVL are present in an infarction involving the lateral wall of the left ventricle, which is supplied by the left circumflex coronary artery. Option D (Q waves in leads I, V4V6, and aVL) is incorrect. Q waves in leads I, V4V6, and aVL occur in an anterolateral wall infarction of the left ventricle due to thrombosis of the midportion of the left anterior descending coronary artery or the circumflex coronary artery. Option E (Q waves in leads V1V4) is incorrect. Q waves in leads V1V4 are present in an anterior wall infarction of the left ventricle and are most often due to thrombosis of the left anterior descending coronary artery.

The photograph shows the cut surface of the liver removed at autopsy from a 49-year-old man with hepatomegaly. Which of the following portal vein complications may have been present in this patient?

A. Dependent pitting edema B. Esophageal varices

C. Gynecomastia D. Hemorrhagic diathesis E. Spider angiomas


Option B (Esophageal varices) is correct. The photograph shows a cirrhotic liver with numerous regenerative nodules replacing the normal architecture of the liver. Portal hypertension (PH) is a complication of cirrhosis, which is most often caused by alcohol abuse. PH is caused by increased resistance to blood flow to the liver due to compression of the sinusoids and central veins by the regenerative nodules. Anastomoses also occur between the portal vein tributaries and the arterial system. Esophageal varices are dilated left gastric coronary veins. These veins normally drain the distal esophagus and proximal stomach and empty into the portal vein. Therefore, an increase in portal vein pressure leads to dilation of the left gastric coronary veins. Other complications of portal vein hypertension include ascites, congestive splenomegaly, hemorrhoids, and caput medusae around the umbilicus. Option A (Dependent pitting edema) is incorrect. Dependent pitting edema in cirrhosis is caused by decreased synthesis of albumin and a corresponding decrease in plasma oncotic pressure. This causes leakage of a protein-poor and cell-poor fluid (transudate) into the interstitial space through capillaries and venules, which produces pitting edema. Option C (Gynecomastia) is incorrect. Gynecomastia is due to hyperestrinism related to decreased metabolism of estrogen by the cirrhotic liver. Option D (Hemorrhagic diathesis) is incorrect. A hemorrhagic diathesis (gastrointestinal bleeding, ecchymoses) in cirrhosis is due to decreased synthesis of coagulation factors, resulting in an anticoagulated state. Option E (Spider angiomas) is incorrect. Spider angiomas are arteriovenous communications that develop in hyperestrinism states (e.g., cirrhosis, normal pregnancy).

A 45-year-old man has chronic myelogenous leukemia that has progressed to an acute blast crisis. He is currently being treated with multiple antileukemic agents. After 1 week of treatment, he develops acute renal failure. A urinalysis shows numerous crystals in the sediment. Which of the following is the most likely mechanism of acute renal failure?

A. Drug nephrotoxicity B. Increased degradation of purine nucleotides C. Increased degradation of pyrimidine nucleotides D. Ischemic acute tubular necrosis

E. Leukemic infiltration of the kidneys

Option B (Increased degradation of purine nucleotides) is correct. Pharmacologic treatment of disseminated malignancies such as chronic myelogenous leukemia results in the release of purine nucleotides from the dead cancer cells. Uric acid is the end-product of degradation of purine nucleotides. Precipitation of uric acid crystals in the collecting ducts obstructs the nephrons (urate nephropathy), causing acute renal failure. Allopurinol, a xanthine oxidase that prevents conversion of xanthine to uric acid, is given before initiation of chemotherapy to prevent acute renal failure. Option A (Drug nephrotoxicity) is incorrect. Many chemotherapeutic drugs are directly nephrotoxic and cause nephrotoxic acute tubular necrosis. Renal tubular cell casts (not uric acid) are the key finding in nephrotoxic acute tubular necrosis due to drugs. Option C (Increased degradation of pyrimidine nucleotides) is incorrect. Increased release of pyrimidines by dead cancer cells increases carbon dioxide, ammonia, and amino acids. The ammonia derived from amino acids by oxidative deamination is metabolized to urea in the urea cycle. An increase in blood urea nitrogen does not produce uric acid crystals in the urine sediment or cause acute renal failure. Option D (Ischemic acute tubular necrosis) is incorrect. The main cause of ischemic acute tubular necrosis is a decrease in cardiac output, which results in decreased renal blood flow. Renal tubular cell casts, not uric acid crystals, are present in the urine sediment in ischemic acute tubular necrosis. Option E (Leukemic infiltration of the kidneys) is incorrect. Leukemic cells metastasize to all organ systems; however, the presence of uric crystals in the urine sediment is more compatible with urate nephropathy than leukemic spread to the kidneys. A 75-year-old man with point tenderness in the lower vertebral column has an increased serum alkaline phosphatase. He does not smoke cigarettes. His stool guaiac is negative. Which of the following tests or procedures would be the most cost-effective step in the initial evaluation of this patient?

A. Colonoscopy B. Digital rectal examination C. Prostate-specific antigen D. Radionuclide bone scan E. Serum -glutamyltransferase

Option B (Digital rectal examination) is correct. The patient most likely has metastatic prostate cancer to the lower vertebral column. Prostate cancer produces osteoblastic metastases (stimulates an increase in osteoblastic

activity) leading to an increase in bone density and an increase in alkaline phosphatase originating from osteoblasts. With metastasis to bone, a digital rectal examination would very likely detect prostate cancer, because it originates in the periphery of the gland. It would be the least expensive initial step in evaluating this patient. Option A (Colonoscopy) is incorrect. Prostate cancer is the most common cancer in men and the most common cause of osteoblastic metastases. Since the stool guaiac test is negative, a colonoscopy to rule out colorectal cancer as a cause of the bone lesions would not be the initial step in evaluating the patient. Option C (Prostate-specific antigen) is incorrect. Although this test is indicated and the antigen would be increased, a digital rectal examination will give more immediate information as to the cause of the patients back pain and increased serum alkaline phosphatase. Option D (Radionuclide bone scan) is incorrect. Although this test is indicated and would most likely show metastatic disease in the bone, a digital rectal examination will give more immediate information as to the cause of the patients back pain and increased serum alkaline phosphatase. Option E (Serum -glutamyltransferase) is incorrect. This test is excellent in helping to localize the source of an increased alkaline phosphatase. However, the presentation is very characteristic of prostate cancer and, although indicated, would not give immediate information regarding the cause of the patients back pain.

For the past few months, a 26-year-old man with AIDS has experienced progressive loss of visual acuity in both eyes. Intraocular pressure is normal. The CD4 helper T-cell count is 48 cells/mm3. The photograph shows the retina of this patient. Which of the following pathogens is the most likely causal agent?

A. Candida albicans B. Cytomegalovirus (CMV) C. Herpes simplex virus type 1

D. HIV E. Toxoplasma gondii

Option B (Cytomegalovirus (CMV)) is correct. The patient has retinitis due to CMV. Examination of the retina shows white areas with indistinct borders (cotton wool exudates) that represent retinal infarctions due to a CMV vasculitis. CMV retinitis is the most common cause of blindness in patients with AIDS and usually occurs when the CD4 helper Tcell count < 50 cells/mm3. Option A (Candida albicans) is incorrect. C. albicans can cause retinitis in AIDS; however, CMV is the overall most common cause of retinitis. Option C (Herpes simplex virus type 1) is incorrect. Herpes simplex virus type I can cause retinitis in AIDS; however, CMV is the overall most common cause of retinitis. Option D (HIV) is incorrect. HIV has not been implicated as a cause of retinitis leading to blindness. Option E (Toxoplasma gondii) is incorrect. T. gondii can cause retinitis in AIDS; however, CMV is the overall most common cause of retinitis.

A 35-year-old man states that when he defecates, he feels a painless mass protruding from the anus that usually reduces spontaneously; however it now remains protruded (see photograph). In the past, he has noted bright red blood on the toilet paper or in the toilet bowl and a mucoid perianal discharge that often soils his underwear. Which of the following is the most likely diagnosis?

A. Anal fissure

B. Angiodysplasia C. Colorectal cancer D. Internal hemorrhoids E. Ischemic colitis


Option D (Internal hemorrhoids) is correct. Internal hemorrhoids are a plexus of superior hemorrhoidal veins located above the dentate line, which are covered by mucosa. The most common cause is straining at stool, often causing the hemorrhoids to protrude from the anal opening (see photograph). The protrusion reduces spontaneously in most cases but sometimes cannot reduce, thus requiring surgery. Passage of bright red blood per rectum and a mucoid perianal discharge that often soils the underwear are common problems. Unless there is thrombosis of the prolapsed hemorrhoids, there usually is no pain. Option A (Anal fissure) is incorrect. Anal fissures cause acute anal pain in adults. A tearing or cutting type of pain is associated with defecation. However, anal fissures are often associated with bright red blood and do not prolapse out of the anus. Option B (Angiodysplasia) is incorrect. Angiodysplasia (dilated vascular channels in the submucosa) typically develops in the cecum of older individuals. Angiodysplasia is associated with intermittent hemorrhage (hematochezia). It is not associated with a mass protruding from the anus. Option C (Colorectal cancer) is incorrect. Colorectal cancer is associated with blood coating and mixing with stools. There is no prolapse of the cancer through the anal opening or soiling of the underwear by a mucoid discharge. Option E (Ischemic colitis) is incorrect. Ischemic colitis occurs in older individuals. Ischemic colitis is associated with pain in the splenic flexure after eating (mesenteric angina) and the presence of bloody stools.

A 32-year-old black medical missionary who recently returned to the United States from a trip overseas is diagnosed with malaria due to Plasmodium vivax. After 4 days of therapy with primaquine, he develops fever, chills, low back pain, and dark urine. A complete blood cell count shows a hemoglobin of 6 g/dL, a WBC count of 15,000/mm3, and a platelet count of 450,000/mm3. A corrected reticulocyte count is 10%. A peripheral smear shows polychromasia and numerous RBCs that are missing parts of their membrane. A urine dipstick is positive for blood. Which of the following laboratory findings is most likely to be reported?

A. Abnormal hemoglobin electrophoresis B. Decreased mean corpuscular volume (MCV) C. Decreased serum ferritin concentration

D. Positive direct Coombs test E. Positive Heinz body preparation

Option E (Positive Heinz body preparation) is correct. The patient has an acute hemolytic anemia caused by glucose-6phosphate dehydrogenase (G6PD) deficiency, an X-linked recessive disorder. G6PD deficiency is most common in persons of black and Mediterranean descent (i.e., Greeks, Italians). G6PD deficiency leads to decreased synthesis of glutathione, which is necessary to neutralize H2O2, an oxidant product in RBC metabolism. Oxidant stresses that induce hemolysis include infection (most common) and drugs (e.g., primaquine, dapsone, trimethoprim). H2O2 accumulation in the RBC damages the RBC membrane (intravascular hemolysis) and denatures hemoglobin, forming discrete inclusions called Heinz bodies. Splenic macrophages often remove damaged RBC membranes, leaving cells with membrane defects, called bite cells, circulating in the peripheral blood (see Fig. 11-30 in Rapid Review Pathology, 3rd edition). The screening test of choice in acute hemolysis is a Heinz body preparation, which requires a special supravital stain to identify the Heinz bodies (see inset in Fig. 11-30 in Rapid Review Pathology, 3rd edition). Enzyme analysis for G6PD is the confirmatory test and must be performed when active hemolysis has subsided. Option A (Abnormal hemoglobin electrophoresis) is incorrect. The patient does not have a hemoglobinopathy, in which a decrease in the synthesis of globin chains (e.g., thalassemia) or the synthesis of an abnormal hemoglobin (e.g., sickle cell anemia) is present. The development of a hemolytic anemia shortly after beginning primaquine therapy rules out a hemoglobinopathy. Option B (Decreased mean corpuscular volume (MCV)) is incorrect. The MCV, or average size of the RBCs, is normal (not decreased) in G6PD deficiency, because it is a normocytic type of anemia. A decrease in the MCV characterizes the microcytic anemias (e.g., iron deficiency). Option C (Decreased serum ferritin concentration) is incorrect. Serum ferritin, which indicates the status of the iron stores in the macrophages in the bone marrow, is normal in G6PD deficiency, decreased in iron deficiency, and increased in iron overload diseases. Option D (Positive direct Coombs test) is incorrect. The direct Coombs test, which is used to detect IgG and/or complement on RBC membranes, is the screening test of choice for diagnosis of an autoimmune hemolytic anemia. Since G6PD is not immune-mediated, the test is negative.

A 23-year-old man complains of a recurrent rash on the shaft of his penis (see photograph). He states that the rash began as an uncomfortable burning sensation. Which of the following histologic changes is most likely present in a Tzanck preparation?

A. Metaplastic squamous cells containing reticulate bodies B. Multinucleated squamous cells with eosinophilic intranuclear inclusions C. Squamous cells with basophilic intranuclear inclusions D. Squamous cells with increased mitoses and hyperchromatic nuclei E. Squamous cells with pyknotic nuclei surrounded by a clear space

Option B (Multinucleated squamous cells with eosinophilic intranuclear inclusions) is correct. The penis has multiple ulcerations surrounded by areas of erythema due to herpes simplex virus type 2 (herpes genitalis). This sexually transmitted disease often recurs, because the virus remains latent in the sensory ganglia. The virus infects the nucleus of squamous cells causing them to divide. The multinucleated cells contain ground glass nuclei, which subsequently develop into eosinophilic inclusions. A Tzanck preparation is performed by unroofing one of the vesicles and scraping the base of the ulcer. Acyclovir is used to decrease the number of recurrences but does not cure the infection. Option A (Metaplastic squamous cells containing reticulate bodies) is incorrect. Chlamydia trachomatis infects metaplastic squamous cells. Two distinct forms of the organism that develop in vacuoles within metaplastic squamous cells are the elementary body (metabolically inert but infective) and the reticulate body (metabolically active but not infective). Binary fission of the reticulate bodies in the vacuoles of the infected cell results in the production of numerous elementary bodies. Option C (Squamous cells with basophilic intranuclear inclusions) is incorrect. Basophilic intranuclear inclusions are usually present in cells infected by cytomegalovirus (CMV). CMV does not produce vesicular skin lesions in any location. Option D (Squamous cells with increased mitoses and hyperchromatic nuclei) is incorrect. Squamous cells with increased mitoses and hyperchromatic nuclei describes a dysplastic cell, which is a precursor to squamous cancer. Squamous dysplasia on the penis describes Bowens disease, which involves the shaft of the penis and/or scrotum. The lesions are white patches (not vesicular or ulcerated) and are not recurrent. They are precursor lesions for invasive squamous cell carcinoma in approximately 10% of cases. Option E (Squamous cells with pyknotic nuclei surrounded by a clear space) is incorrect. A cell with a pyknotic nucleus surrounded by a clear halo is the cytologic effect of the human papillomavirus within a squamous cell. The lesion is

called condyloma acuminatum (venereal warts). It is an exophytic lesion in the anogenital region that has a keratotic papillary appearance.

Which of the following is a characteristic of antibodies blood group (ABO) hemolytic disease of the newborn (HDN) rather than Rh HDN due to anti-D?

A. Age-adjusted normocytic anemia B. Newborn may develop anemia and jaundice in the first pregnancy C. Positive direct Coombs test on babies cord RBCs D. Severe anemia requiring an exchange transfusion E. Unconjugated hyperbilirubinemia
Option B (Newborn may develop anemia and jaundice in the first pregnancy) is correct. In ABO HDN, the majority of O mothers already have anti A, B-IgG natural antibodies that react with antigens in blood group A and B RBCs. Since IgG crosses the placenta, if the fetus is blood group A or B, the antibody will attach to the RBC and be removed by fetal splenic macrophages leading to a mild extravascular hemolytic anemia. In Rh HDN due to anti-D, the mother must first be sensitized by an Rh positive fetus before antibodies can cross the placenta and hemolyze fetal Rh positive cells in future pregnancies. Option A (Age-adjusted normocytic anemia) is incorrect. Both ABO and Rh HDN have a normocytic anemia; however, the former type is mild while the latter type is frequently severe, requiring an exchange transfusion. Option C (Positive direct Coombs test on babies cord RBCs) is incorrect. Both ABO and Rh HDN have a positive direct Coombs test on cord blood RBCs, because both types have IgG antibodies coating the fetal RBCs. In ABO HDN the antibody is anti A, B-IgG, while in Rh HDN it is anti D-IgG. Option D (Severe anemia requiring an exchange transfusion) is incorrect. Only Rh HDN is associated with an anemia severe enough to requiring a transfusion to prevent hydrops fetalis (left- and right-sided heart failure). Option E (Unconjugated hyperbilirubinemia) is incorrect. Both ABO and Rh HDN have macrophage destruction of sensitized fetal RBCs leading to an unconjugated hyperbilirubinemia.

A 38-year-old black man complains of fatigue and periodic episodes of red-colored urine during a morning void. Physical examination reveals conjunctival pallor. A CBC shows a pancytopenia with a normal mean corpuscular volume (MCV). RBC morphology is normal. A dipstick of urine is positive for blood; however, the urine sediment does not contain RBCs. The direct Coombs test is negative. A sucrose hemolysis test is positive. What is the most likely diagnosis?

A. Cold-autoimmune hemolytic anemia B. Paroxysmal cold hemoglobinuria (PCH) C. Paroxysmal nocturnal hemoglobinuria (PNH) D. Sickle cell anemia E. Warm autoimmune hemolytic anemia

Option C (Paroxysmal nocturnal hemoglobinuria (PNH)) is correct. PNH is an acquired membrane defect involving common myeloid stem cells. A gene mutation causes loss of the anchor for decay accelerating factor (DAF), which normally neutralizes complement attached to RBCs, neutrophils, and platelets at night. Loss of DAF causes intravascular complement destruction of RBCs, neutrophils, and platelets leading to pancytopenia and hemoglobinuria. There is an increased incidence of vessel thrombosis due to release of aggregating agents from destroyed platelets. Chronic loss of Hb in the urine may result in iron deficiency. The screening test for PNH is the sucrose hemolysis test (sugar water test). If this test returns positive, the acidified serum test (Ham test) is used to distinguish PNH from other causes of complement-mediated hemolysis of RBCs. Currently, there are more sophisticated tests available to confirm the diagnosis. Option A (Cold-autoimmune hemolytic anemia) is incorrect. This type of anemia is IgM-mediated and, in most cases, produces an intravascular hemolysis. Most cases are idiopathic. Known causes include infection (Mycoplasma, infectious mononucleosis) and cancer (chronic lymphocytic leukemia). The majority of immune hemolytic anemias have a positive direct Coombs test. The patient in this case has a negative direct Coombs test. Option B (Paroxysmal cold hemoglobinuria (PCH)) is incorrect. This is a very uncommon cause of hemoglobinuria. It is an acquired disease that is either idiopathic or associated with syphilis. It is caused by an IgG cold antibody with bithermal activity (called Donath-Landsteiner antibody). The antibody is directed against the P blood group antigen at cold temperatures and binds to RBCs and fixes complement. At 37C it detaches from RBCs and activates complement causing intravascular hemolysis. The sucrose hemolysis test is negative in PCH. Option D (Sickle cell anemia) is incorrect. Sickle cell anemia has an abnormal cell morphology in the peripheral blood (sickle cells, target cells). The cell morphology in this patient was normal. Option E (Warm autoimmune hemolytic anemia) is incorrect. The most common cause of a warm-type (IgG-mediated) autoimmune hemolytic anemia is systemic lupus erythematosus. The RBCs are coated by IgG and C3b causing them to be phagocytosed by splenic and liver macrophages. The direct Coombs test is positive. In PNH the direct Coombs test is negative.

A 16-year-old girl develops severe abdominal pain in the right lower quadrant. An ultrasound of the abdomen reveals an abnormality in the right ovary (see photograph). Which of the following best characterizes the lesion?

A. Choristoma B. Hamartoma C. Mixed tumor D. Sarcoma E. Teratoma

Option E (Teratoma) is correct. The photograph shows a cystic teratoma of the ovary with hair and teeth (arrow). This type of tumor is derived from germ cells or midline embryonic rests, and contains tissues derived from more than one germ cell layer (e.g., bone, muscle, glands, and neural tissue). Most teratomas are benign, but the presence of neural or immature tissue usually indicates malignancy. In a very small percentage of cases, a squamous cell carcinoma may develop from squamous epithelium in the tumor. In most cases, the tumor presents with abdominal pain, as in this patient. Option A (Choristoma) is incorrect. A choristoma, or heterotopic rest, is not a neoplasm. It is a mass of normal tissue that is present in a foreign site (e.g., pancreatic tissue in the stomach wall). Option B (Hamartoma) is incorrect. A hamartoma is not a neoplasm. It is an overgrowth of disorganized tissue that is indigenous to a particular site (e.g., bronchial hamartoma with mature cartilage in the lung). Option C (Mixed tumor) is incorrect. Mixed tumors have neoplastic cells with two different morphologic patterns that derive from the same germ cell layer. They are the most common salivary gland tumor and are most often located in the parotid gland, where they are called pleomorphic adenomas. Option D (Sarcoma) is incorrect. A sarcoma is a malignant tumor that derives from connective tissue (e.g.,

rhabdomyosarcoma derived from skeletal muscle). Although connective tissue elements are present in teratomas, they are not malignant.

The photograph is a histologic section of a pathologic finding in the left anterior descending coronary artery of a 56-year-old man. Which of the following drugs is most frequently used to prevent the pathologic finding in the coronary artery?

A. Abciximab B. Alteplase C. Aspirin D. Ticlopidine E. Warfarin

Option C (Aspirin) is correct. The histologic section shows a thrombus (red-colored) occluding the lumen of the coronary artery. Directly underneath the thrombus is a fibrous cap (blue-colored lesion), the pathognomonic lesion of atherosclerosis. Directly beneath the blue fibrous cap is a core of necrotic material containing cholesterol (clear, needleshaped spaces). There is a small fissure at the edge of the fibrous cap (disrupted plaque) that contains necrotic atheromatous debris. This debris was responsible for initiating the formation of a platelet thrombus in the lumen of the vessel. Aspirin is the most frequently used medication to prevent platelet aggregation. It does so by irreversibly inhibiting platelet cyclooxygenase activity. This prevents the production of prostaglandin H2 and its conversion to thromboxane A2 by the enzyme thromboxane synthase in the platelet. Thromboxane A 2 is a potent vasoconstrictor and platelet aggregator. Option A (Abciximab) is incorrect. Abciximab is a monoclonal antibody that is directed against the glycoprotein (Gp)

IIb/IIIa fibrinogen receptor on platelets. Fibrinogen, which is later converted into fibrin, is responsible for causing the platelet to aggregate to form a thrombus. Abciximab is very expensive and is most often used after angioplasty or stenting to prevent thrombus formation. Option B (Alteplase) is incorrect. Alteplase is a recombinant tissue plasminogen activator that is used to lyse a preexisting platelet thrombus. Plasminogen activators cause the release of plasmin within the thrombus which leads to cleavage of the fibrin strands holding the thrombus together. It is not used to prevent a platelet thrombus. Option D (Ticlopidine) is incorrect. Ticlopidine (or clopidogrel) inhibits ADP-induced expression of platelet GpIIb:IIIa receptors, which prevents fibrinogen binding and platelet aggregation. It is expensive and is most often used if patients are allergic to aspirin. Option E (Warfarin) is incorrect. Warfarin is an anticoagulant that inhibits the formation of venous clots. Venous clots frequently develop in areas of stasis (e.g., deep veins of the lower leg) and are produced by localized activation of the coagulation system leading to the formation of fibrin strands that entrap the blood that is present in the vessel lumen. Hence, the thrombus is composed of RBCs, WBCs, and platelets held together by fibrin strands. Hence, an anticoagulant like warfarin or heparin prevents activation of the coagulation system and the formation of fibrin, which, in turn, prevents a venous clot from forming. Warfarin and heparin do not prevent platelet aggregation and the formation of a platelet thrombus.

An afebrile 12-year-old boy has an episodic history of developing pink-staining urine shortly after an upper respiratory infection. The patient has borderline hypertension. The urine dipstick test is positive for blood and shows mild to moderate amounts of protein. The anti-streptolysin O titer, the anti-DNAase B titer, and the serum antinuclear antibody (ANA) test are all negative. The photographs show the urine sediment. Which of the following is the most likely diagnosis?

A. Diffuse membranous glomerulopathy B. Glomerulonephritis in systemic lupus erythematosus (SLE) C. IgA glomerulopathy

D. Minimal change disease E. Poststreptococcal glomerulonephritis

Option C (IgA glomerulopathy) is correct. The urine sediment shows an RBC cast. The other photograph (phase contrast) shows dysmorphic RBCs with protrusions from the RBC membrane, a sign of glomerulonephritis. RBC casts, hematuria with dysmorphic RBCs, and proteinuria are characteristic of a nephritic type of glomerulonephritis. The episodic history of hematuria following upper respiratory infections is characteristic of IgA glomerulopathy, which is the most common type of glomerulonephritis. Option A (Diffuse membranous glomerulopathy) is incorrect. Diffuse membranous glomerulopathy is the second most common cause of the nephrotic syndrome (proteinuria > 3.5 g/24 hr, fatty casts) in adults. The patient has a nephritic type of glomerulonephritis. Option B (Glomerulonephritis in systemic lupus erythematosus (SLE)) is incorrect. A negative serum ANA test excludes glomerulonephritis in SLE. Option D (Minimal change disease) is incorrect. Minimal change disease (lipoid nephrosis) is the most common cause of the nephrotic syndrome in children. The patient has a nephritic type of glomerulonephritis. Option E (Poststreptococcal glomerulonephritis) is incorrect. Negative DNAase B titers excludes acute poststreptococcal glomerulonephritis. Unlike rheumatic fever where anti-streptolysin O and DNAase B titers are increased, only the latter antibody is decreased.

An afebrile 28-year-old man, who recently returned from a vacation in Mexico, develops bloody diarrhea. A colonoscopy is performed, and a biopsy specimen is taken from one of many cecal ulcers. The photograph shows a representative section of the biopsy tissue. Which of the following is the most likely diagnosis?

A. Amebiasis B. Ascariasis C. Balantidiasis

D. Cryptosporidiosis E. Giardiasis

Option A (Amebiasis) is correct. The photograph shows the trophozoites of Entamoeba histolytica, many of which contain phagocytosed RBCs. The trophozoites burrow into the cecal mucosa and produce flask-shaped ulcers and bloody diarrhea. Entry into the portal vein increases the risk for developing liver abscesses. The treatment of amebiasis is metronidazole. Option B (Ascariasis) is incorrect. Ascariasis is due to the nematode Ascaris lumbricoides. Adult worms cause intestinal obstruction and nonbloody diarrhea. Option C (Balantidiasis) is incorrect. Balantidiasis is due to Balantidium coli, a ciliate protozoan. The patient develops bloody diarrhea; however, E. histolytica is the only pathogenic intestinal protozoan that phagocytoses RBCs. Option D (Cryptosporidiosis) is incorrect. Cryptosporidiosis is due to Cryptosporidium parvum, a sporozoan (type of protozoan). It is one of the most common pathogens causing diarrhea in AIDS, particularly when the CD 4 Th count < 100 mm3. It does not produce bloody diarrhea and primarily has oocysts that line the brush border of the small bowel. Option E (Giardiasis) is incorrect. Giardiasis is due to the flagellate protozoan Giardia lamblia. It produces chronic diarrhea often with malabsorption. It does not produce bloody diarrhea nor does it phagocytose RBCs.

A 22-year-old woman sees her physician because she feels a painful lump in her right breast. She is concerned because her mother had breast cancer. Upon physical examination, the physician palpates a movable mass in the right upper outer quadrant. The photographs show a gross and a microscopic section of the mass. Which of the following is the most likely diagnosis?

A. Fibroadenoma B. Fibrocystic change C. Infiltrating ductal carcinoma

D. Lobular carcinoma E. Pagets disease

Option A (Fibroadenoma) is correct. The biopsy shows a fibroadenoma with a bulging gray-white surface and a microscopic section showing compressed, benign, elongated ducts surrounded by dense, myxomatousappearing connective tissue. This benign tumor derives from the stroma (connective tissue) of the breast. It is the most common movable mass lesion in the breast in women younger than 35 years of age. Fibroadenomas are estrogen-sensitive and often increase in size during the menstrual cycle or during pregnancy. Option B (Fibrocystic change) is incorrect. Fibrocystic change is the most common breast mass in women < 50 years of age. It is considered to be a distortion of normal cyclic breast changes. Components of fibrocystic change include small and large cysts, fibrosis, proliferation of small ductules/acini in the terminal lobule (called sclerosing adenosis), and ductal hyperplasia. Option C (Infiltrating ductal carcinoma) is incorrect. The major ducts are the most common site for infiltrating ductal carcinomas. The ducts in the biopsy are compressed and do not show increased thickness of the ductal lining or invasion through the basement membrane. Option D (Lobular carcinoma) is incorrect. Lobular carcinoma is located in the terminal lobules, which are located at the end of the major ducts. They cannot be palpated unless they invade tissue. Option E (Pagets disease) is incorrect. Pagets disease is a ductal carcinoma that has infiltrated into the squamous epithelium of the nipple causing a rash and ulceration of the surface of the nipple. It is histologically similar to Pagets disease of the vulva, except it is a ductal cancer infiltrating the nipple epithelium versus an adenocarcinoma arising within the epidermis.

A test is performed on 5 normal people who have the following test results: 48 mg/dL, 52 mg/dL, 50 mg/dL, 49 mg/dL, and 51 mg/dL. The standard deviation of the test is 4 mg/dL. What is the reference interval (normal range) for the test?

A. 4060 mg/dL B. 4252 mg/dL C. 4258 mg/dL D. 4654 mg/dL E. 4856 mg/dL
Option C (4258 mg/dL) is correct. Most normal ranges are established by adding and subtracting 2 standard deviations (SD) from the mean of the test. The standard deviation for the test is 4 mg/dL; therefore, 2 SD is 8 mg/dL. The mean of the test is obtained by adding up the test values for normal people and dividing the sum by the number of people. The

mean of the test is 48 + 52 + 50 + 49 + 51 = 250 mg/dL divided by 5 = 50 mg/dL. Therefore, the lower limit of the normal range is 50 - 8 = 42 mg/dL, and the upper limit is 50 + 8 = 58 mg/dL. Option A (4060 mg/dL) is incorrect. Option B (4252 mg/dL) is incorrect. Option D (4654 mg/dL) is incorrect. Option E (4856 mg/dL) is incorrect.

A 58-year-old woman complains of stomach pain and weight loss. She also states that she has been vomiting blood and passing dark black, sticky stools. She has a 60-pack-year history of smoking cigarettes. Physical examination shows epigastric pain on deep palpation and enlarged, nontender left supraclavicular lymph nodes. Bimanual pelvic examination shows that both ovaries are enlarged and firm. The photograph shows a biopsy of one of the ovaries. A test for fecal occult blood is positive. What is the most likely diagnosis?

A. Colorectal cancer with metastasis to the ovaries B. Lung cancer with metastasis to bowel and ovaries C. Ovarian cancer with metastasis to the stomach D. Pancreatic cancer with metastasis to the ovaries E. Stomach cancer with metastasis to the ovaries
Option E (Stomach cancer with metastasis to the ovaries) is correct. The patient has primary stomach cancer that has hematogenously metastasized to both ovaries (Krukenberg tumors). A maxim in medicine states that palpable ovaries in a postmenopausal woman represent cancer until proven otherwise. The photograph shows classic signet ring cells (mucin pushes the nucleus to the periphery), which are primarily seen in the diffuse type of adenocarcinoma of the stomach. This type of cancer is not related to Helicobacter pylori infections. Stomach cancer is the most common

primary cancer that metastasizes to the left supraclavicular nodes (Virchow nodes). Common signs of stomach cancer are weight loss, epigastric pain, hematemesis, and melena. Smoking is not a risk factor for gastric cancer. Option A (Colorectal cancer with metastasis to the ovaries) is incorrect. Colorectal cancer rarely metastasizes to the ovaries and would not be expected to produce black, tarry stools, a finding that is most often associated with an upper gastrointestinal bleed. Option B (Lung cancer with metastasis to bowel and ovaries) is incorrect. Although the patient is a smoker, which is the most common risk factor for primary lung cancer, they do not produce a signet ring type of cancer. Option C (Ovarian cancer with metastasis to the stomach) is incorrect. Ovarian cancers do not metastasize to the left supraclavicular lymph nodes. These cancers usually seed the omentum and produce ascites and signs of intestinal obstruction (e.g., colicky pain). Option D (Pancreatic cancer with metastasis to the ovaries) is incorrect. Most pancreatic cancers involve the head of the pancreas and are associated with obstructive jaundice, light-colored stools, and a palpable gallbladder. Pain usually radiates to the back, because the pancreas is located in the retroperitoneum. Pancreatic cancer can metastasize to the ovaries and is most commonly associated with smoking. However, pancreatic cancers do not produce signet ring cells.

One year after receiving a cornea transplant, a 52-year-old man developed progressive dementia, abnormal jerking movements, ataxia, and somnolence. An MRI showed bilateral areas of increased signal intensity in the caudate and putamen. An EEG showed paroxysms with high voltages and slow waves. The photograph shows a section of cerebral cortex removed at autopsy. Which of the following histologic abnormalities is present in the section?

Option E (Spongiform change) is correct. Creutzfeldt-Jakob disease (CJD) is due to iatrogenic transmission of infectious agents called prions (proteins without nucleic acids). The prions were transmitted to the patient by the cornea implant. The photograph shows extensive vacuolation (spongiform change) within the neuropil between the nerve cell bodies. Spongiform change is a unique microscopic finding that is present in a group of disorders called the spongiform encephalopathies. These disorders occur in humans (and may also be transmitted from eating infected cows (bovine spongiform encephalopathy, or mad cow disease). CJD is characterized by progressive dementia, startle myoclonus (abnormal jerking movements), and ataxia. There is

no known cure, and death is inevitable. Option A (Multinucleated microglial cells) is incorrect. Multinucleated microglial cells are a characteristic finding in patients with AIDS dementia due to HIV. The virus causes the microglial cells to fuse. Microglial cells are the reservoir cell in the brain for HIV. Option B (Neurofibrillary tangles) is incorrect. Neurofibrillary tangles are present in Alzheimers disease and other central nervous system disorders. They represent filaments of tau protein coiled like a DNA helix within the cytoplasm of neurons. Option C (Neurons with Lewy bodies) is incorrect. In idiopathic Parkinsons disease, neurons in the substantia nigra contain pink-staining Lewy bodies (damaged neurofilaments) in the cytoplasm. Option D (Oligodendrocytes with viral inclusions) is incorrect. Oligodendrocytes with viral inclusions are present in progressive multifocal leukoencephalopathy, a slowly evolving encephalopathy caused by a papovavirus. Destruction of the oligodendrocytes in the white matter causes demyelination. It commonly occurs in AIDS patients.

A 48-year-old woman with type 2 diabetes mellitus is receiving chlorpropamide. She develops headaches and problems with memory. The skin turgor is normal. Which of the following serum Na+, total body sodium (TBNa+) is most likely present?

Serum Na+TBNa+ A.Decreased Decreased B. Decreased Increased C. Decreased Normal D.Increased Decreased E. Normal Decreased

A. serum Na+ decreased, TBNa+ decreased B. serum Na+ decreased, TBNa+ increased C. serum Na+ decreased, TBNa+ normal D. serum Na+ increased, TBNa+ decreased E. serum Na+ normal, TBNa+ decreased

Option C (serum Na+ decreased, TBNa+ normal) is correct. The patient has the syndrome of inappropriate antidiuretic hormone (ADH) due to excessive stimulation of ADH release by chlorpropamide, a oral hypoglycemic agent. ADH reabsorbs electrolyte-free water from the collecting tubules causing a dilutional hyponatremia (serum Na+ = TBNa+/TBW). Since Na+ is not reabsorbed by ADH, the TBNa+ is normal; therefore, there are no signs of pitting edema. The mental status abnormalities are due to hyponatremia, which creates an osmotic gradient that favors the movement of water into the cells in the brain. Regarding the fluid compartments, the extracellular fluid (ECF) compartment is expanded (addition of fluid) and the intracellular fluid (ICF) compartment is expanded (osmosis effect of hyponatremia). Option A (serum Na+ decreased, TBNa+ decreased) is incorrect. This occurs when there is a hypertonic loss of fluid ( serum Na+ = TBNa+/TBW), which often occurs with a loop diuretic (inhibition of the Na+, K+, 2Cl cotransporter), Addisons disease (loss of mineralocorticoids), or 21-hydroxylase deficiency (loss of mineralocorticoids). Regarding the fluid compartments, the ECF compartment is contracted (loss of fluid) and the ICF compartment is expanded (osmosis effect of hyponatremia). Option B (serum Na+ decreased, TBNa+ increased) is incorrect. This is an example of a hypotonic gain of sodiumcontaining fluid. Examples include the edema states (e.g., cirrhosis, right-sided heart failure). In these conditions there are alterations in Starling pressuresdecreased plasma oncotic pressure and/or increase in plasma hydrostatic pressurewhich force fluid out of the vascular compartment into the interstitial space and body cavities. The cardiac output is decreased in these conditions, because fluid is trapped in the interstitial space and body cavities by the Starling pressure abnormalities. Decreased renal blood flow causes the kidney to reabsorb a slightly hypotonic saltcontaining fluid, which produces hyponatremia ( serum Na+ = TBNa+/TBW). The fluid is redirected into the interstitial space (decreased plasma oncotic pressure) and body cavities (decreased plasma oncotic pressure and increased plasma hydrostatic pressure). Regarding the fluid compartments, the ECF compartment is expanded (addition of fluid) and the ICF compartment is expanded (osmosis effect of hyponatremia). Option D (serum Na+ increased, TBNa+ decreased) is incorrect. This occurs when there is a hypotonic loss of sodiumcontaining fluid ( serum Na+ = TBNa+/TBW)). Examples include excessive sweating and osmotic diuresis (e.g., glucosuria). The patient has signs of volume depletion (TBNa+) from the loss of sodium, mainly dry mucous membranes and a positive tilt test (blood pressure dropped and pulse rate increased when sitting up). Regarding the fluid compartments, the ECF compartment is contracted (loss of fluid) and the ICF compartment is contracted (osmosis effect of hypernatremia). Option E (serum Na + normal, TBNa+ decreased) is incorrect. This occurs when there is an isotonic loss of fluid (serum Na+ = TBNa+/TBW). Excellent examples include travelers diarrhea, which is most often due to enterotoxigenic Escherichia coli and cholera. Loss of isotonic fluid does not alter the serum Na+ concentration (serum Na+ = TBNa+/TBW); however, the decrease in TBNa+ causes signs of volume depletion (dry mucous membranes, poor skin turgor, hypotension). Regarding the fluid compartments, the ECF compartment is contracted (loss of fluid) and the ICF compartment is unchanged (no osmotic effect).

The photograph shows a nonpruritic lesion on the forearm of a 53-year-old farmer. Which of the following lifestyle modifications would reduce the risk of developing this lesion?

A. Adopt a high-fiber, lowsaturated fat diet B. Apply sunscreen before going outdoors C. Lose weight D. Stop drinking alcohol E. Stop smoking cigarettes
Option B (Apply sunscreen before going outdoors) is correct. The photograph shows a black lesion with an irregular border, consistent with a superficial, spreading malignant melanoma. Using sunscreen is the best way to prevent skin cancers. Malignant melanomas derive from nevus cells (modified melanocytes) in the skin. Farmers are at high risk for malignant melanoma, because their skin is exposed to ultraviolet B (UVB) light for prolonged periods. UVB light damages DNA by producing cross-linked pyrimidine dimers in epidermal cells. Option A (Adopt a high-fiber, lowsaturated fat diet) is incorrect. Dietary alterations do not reduce the risk for developing any of the primary skin cancers related to UV light. Option C (Lose weight) is incorrect. Losing weight does not alter the risk for developing a malignant melanoma. Option D (Stop drinking alcohol) is incorrect. Alcohol consumption is not a risk factor for malignant melanoma. Option E (Stop smoking cigarettes) is incorrect. Polycyclic hydrocarbons in cigarette smoke are not risk factors for malignant melanoma.

A 42-year-old man with chronic liver disease, pigment discoloration in the limbus of both eyes, and a movement disorder died of complications related to liver disease. Other members of his family have had similar clinical findings. The photograph shows a section of brain removed at autopsy. Which of the following laboratory findings would most likely have been reported?

A. Decreased serum ammonia B. Decreased serum ceruloplasmin C. Increased serum blood urea nitrogen D. Increased serum copper E. Increased serum iron
Option B (Decreased serum ceruloplasmin) is correct. Wilson's disease is an autosomal recessive disorder with a defect in the secretion of copper into bile and in the incorporation of copper into ceruloplasmin in the liver. Increased free copper in the liver parenchymal cells eventually leads to cirrhosis of the liver. Increased free (unbound) copper in the blood deposits in Descemets membrane in the limbus of the eyes (Kayser-Fleischer rings; see Fig. 18-12 in Rapid Review Pathology, 3rd edition) and in the basal ganglia, particularly the putamen. The photograph shows cavitation in the putamen on both sides of the brain. Option A (Decreased serum ammonia) is incorrect. In chronic liver disease, the urea cycle in hepatocytes is dysfunctional. This results in an increase (not decrease) in serum ammonia (it cannot be converted to urea in the urea cycle) and a decrease in the serum blood urea nitrogen. Option C (Increased serum blood urea nitrogen) is incorrect. In chronic liver disease, the urea cycle in hepatocytes is dysfunctional causing a decreased (not increased) serum blood urea nitrogen. Option D (Increased serum copper) is incorrect. Approximately 95% of the total serum copper represents the copper that is bound to ceruloplasmin. Because ceruloplasmin concentration is decreased in Wilsons disease, the total serum copper level is also decreased. However, free (unbound) copper levels are increased. Option E (Increased serum iron) is incorrect. Iron overload diseases (e.g., hemochromatosis) are characterized by an increase in serum iron. These disorders primarily target the liver, where the excess iron damages hepatocytes, resulting in pigment cirrhosis. There is no deposition of iron in the eye or basal ganglia of the brain.

The rash in this 1-year-old boy developed 3 days after a high fever. Erythematous macules were present on the soft palate 48 hours before the skin rash. What is the most likely diagnosis?

A. Erythema infectiosum B. Molluscum contagiosum C. Roseola D. Rubella E. Rubeola

Option C (Roseola) is correct. The patient has roseola, which is caused by human herpesvirus 6 (DNA virus). It is the most common viral exanthem in children < 2 years old. Erythematous macules develop on soft palate 48 hours before the rash. The maculopapular rash occurs abruptly after 3 to 7 days of high fever and is characterized by blanching with pressure and the presence of subtle peripheral halos around the lesions. High fever may precipitate a febrile convulsion. Option A (Erythema infectiosum) is incorrect. Erythema infectiosum, or fifths disease, is caused by parvovirus B19 (DNA virus). It most often occurs in school-aged children and often occurs in epidemics. It produces a confluent netlike erythema type of rash that begins on the cheeks (slapped face appearance) and then extends to the trunk and proximal extremities. Option B (Molluscum contagiosum) is incorrect. Molluscum contagiosum is caused by a poxvirus (DNA virus). It is characterized by bowl-shaped lesions with central depression filled with keratin. The depression contains viral particles called molluscum bodies. Self-inoculation may occur by scratching the infective viral particles out of the crater. It is usually self-limited once immunity is established against the virus. Option D (Rubella) is incorrect. Rubella, or German measles, is an RNA togavirus that produces 3-day measles. Dusky red spots called Forchheimers spots appear on the posterior soft/hard palate and develop at the beginning of the rash. The rash is a maculopapular rash and lasts 3 days. It is a pinkish, red maculopapular eruption, begins first at the hairline and rapidly spreads cephalocaudally, lasts for 3 days, and fades in 3 days.

Painful postauricular lymphadenopathy is a characteristic finding. Option E (Rubeola) is incorrect. Rubeola, or regular measles, initially has a prodrome of fever, cough, coryza (runny nose), and conjunctivitis. Koplik spots develop on the buccal mucosa and a maculopapular rash develops after the Koplik spots disappear. The rash typically begins on the head and then moves to the trunk and extremities. It tends to become confluent on face and trunk but discrete on the extremities.

A 29-year-old woman complains of morning stiffness in her hands and photophobia. Physical examination shows diastolic hypertension, swelling of the metacarpophalangeal joints, and an erythematous rash on the malar eminence of both sides of the face. Urinalysis shows hematuria, mild proteinuria, and RBC casts. The serum antinuclear antibody test is strongly positive. An H&E stain of a renal biopsy shows diffuse proliferative glomerulonephritis. The photograph shows an electron micrograph of a glomerulus. Which of the following antibodies is most likely to be present in the serum?

A. AntiDNAase B antibodies B. Antidouble-stranded DNA antibodies C. Antihistone antibodies D. Antistreptolysin O antibodies E. Antitopoisomerase antibodies
Option B (Antidouble-stranded DNA antibodies) is correct. The patient has signs and symptoms (positive serum antinuclear antibodies, joint pains, photophobia, hypertension, malar rash) of systemic lupus erythematosus (SLE). The urinalysis findings show a nephritic type of glomerulonephritis with hematuria, RBC casts, and mild proteinuria. The renal biopsy demonstrates diffuse proliferative glomerulonephritis, the most common type of glomerulonephritis in patients with SLE. The electron micrograph shows a band of electron-dense material (DNAanti-DNA immunocomplexes) directly below the endothelial surface of the glomerular basement membrane (subendothelial deposit). Antidouble-stranded DNA antibodies are present in almost all patients with SLE who have the nephritic type of glomerulonephritis related to diffuse proliferative glomerulonephritis (type IV subtype). The serum antinuclear antibody test usually shows a rim pattern of immunofluorescence.

Option A (AntiDNAase B antibodies) is incorrect. AntiDNAase B antibodies are present in poststreptococcal glomerulonephritis, which is usually secondary to a skin infection by a nephritogenic strain of group A streptococcus (e.g., scarlet fever). Option C (Antihistone antibodies) is incorrect. Antihistone antibodies are present in patients with SLE due to drugs such as procainamide. Renal disease does not occur in drug-induced lupus. Option D (Antistreptolysin O antibodies) is incorrect. Antistreptolysin O antibodies are present in patients who have had a recent pharyngeal infection due to a group A streptococcus. Option E (Antitopoisomerase antibodies) is incorrect. Antitopoisomerase antibodies are present in progressive systemic sclerosis, which is not associated with morning stiffness, photophobia, or diffuse proliferative glomerulonephritis. Renal disease presents with a vasculitis (hyperplastic arteriolosclerosis) leading to malignant hypertension and infarctions.

A newborn male infant with Down syndrome fails to pass meconium within the first 24 hours after birth. A rectal examination shows a narrow anal canal and absence of stool in the rectal vault on rectal examination. An abdominal radiograph shows distended loops of colon. Which of the following is the most likely diagnosis?

A. Anorectal malformation B. Enterocolitis C. Functional constipation D. Hirschsprungs disease E. Malrotation


Option D (Hirschsprungs disease) is correct. This patient has Hirschsprungs disease (congenital megacolon), which is the most common cause of lower intestinal obstruction in neonates. The most common manifestation is failure to pass meconium within the first 24 hours after birth. There are no ganglion cells in both the Meissner submucosal and the Auerbach myenteric plexuses. In 75% of cases, the rectosigmoid is aganglionic. Characteristic findings include a narrow anal canal, absence of stool in the rectal vault in rectal examination, and an abdominal radiograph showing distended loops of colon. Peristalsis occurs in segments of colon that do contain ganglion cells. In 3% of cases, there is an association with Down syndrome. Option A (Anorectal malformation) is incorrect. Anorectal malformations are uncommon in newborns. Malformations include imperforate anal membrane, anal stenosis, anal agenesis, rectal agenesis, and rectal atresia. There is a high association with vertebral and genitourinary abnormalities and no association with Down syndrome. Option B (Enterocolitis) is incorrect.Enterocolitis, which is the most common complication of Hirschsprungs disease, is

a type of ischemic necrosis related to an increase in intraluminal pressure and decreased intramural capillary blood flow. Presenting signs include fever and bloody diarrhea. Option C (Functional constipation) is incorrect. Functional constipation is a type of chronic retentive constipation. Meconium passes within the first 24 hours of life, the anal canal is dilated, and palpable stool is present in the rectal vault. Option E (Malrotation) is incorrect. The majority of congenital malrotations involve the small intestine and present with bloody stools.

A 62-year-old woman complains that her tongue is sore. Physical examination shows atrophy of the tongue papillae, decreased vibratory sensation in both lower extremities, and instability when she stands and closes her eyes. An endoscopic examination reveals chronic atrophic gastritis involving the body and fundus of the stomach. Laboratory studies show a severe macrocytic anemia with pancytopenia. The photograph shows the peripheral blood smear and a leukocyte abnormality (arrow). Which of the following laboratory test findings would most likely be reported?

A. Decreased serum folate B. Decreased serum gastrin C. Decreased urine methylmalonic acid D. Increased antigliadin antibodies E. Increased vitamin B12 absorption after addition of intrinsic factor (IF)
Option E (Increased vitamin B12 absorption after addition of intrinsic factor (IF)) is correct. The patient has pernicious anemia (PA), an autoimmune disease in which impaired intestinal absorption of vitamin B 12 is caused by a lack of IF. Antibodies directed against parietal cells in the body and fundus (type II hypersensitivity reaction) cause mucosal damage (chronic atrophic gastritis), achlorhydria (loss of acid production by parietal cells), and a decrease in synthesis of IF. IF normally forms a complex with vitamin B12 in the duodenum which is then reabsorbed in the terminal ileum after the complex attaches to IF receptors. The peripheral smear shows large, egg-shaped macro-ovalocytes. The arrow points to a hypersegmented neutrophil (more than five nuclear lobes), a valuable and early marker of vitamin B12

deficiency (also folate deficiency). Pancytopenia is the rule in PA, because deficiency of vitamin B 12 causes decreased production of DNA leading to nuclear enlargement of hematopoietic cells in the bone marrow (megaloblasts). These cells or derivatives from these cells (e.g., mature RBCs, platelets, neutrophils) are often phagocytosed and destroyed by bone marrow macrophages or are destroyed by apoptosis before they enter the peripheral blood. Reabsorption of orally administered vitamin B12 after the addition of IF confirms the diagnosis of PA. This is called the Schilling test. Option A (Decreased serum folate) is incorrect. Because of the patients neurologic deficits (see below) and the presence of chronic atrophic gastritis of the body and fundus, folic acid deficiency is excluded. Option B (Decreased serum gastrin) is incorrect. The patient has achlorhydria (absence of hydrochloric acid), which causes an increase (not a decrease) in serum gastrin levels. Option C (Decreased urine methylmalonic acid) is incorrect. Vitamin B12, unlike folic acid, is involved in propionate fatty acid metabolism (odd-chain fatty acids). Propionyl CoA is converted to methylmalonic CoA, and methylmalonic CoA is converted to succinyl CoA using an enzyme reaction that requires vitamin B12 as a cofactor. Deficiency of vitamin B12 causes an increase (not a decrease) in methylmalonic acid levels (derives from methylmalonyl CoA) in the urine. It also causes an increase in propionyl CoA (also propionic acid) in the urine. Propionyl CoA replaces acetyl CoA in neuronal membranes resulting in demyelination of the posterior columns and lateral corticospinal tract of the spinal cord (subacute combined degeneration) and peripheral nerves (peripheral neuropathy). The patient has decreased vibratory sensation and joint dysequilibrium (cannot stand up with her eyes closed), both of which are signs of posterior column disease. Option D (Increased antigliadin antibodies) is incorrect. Antigliadin antibodies are diagnostic of celiac disease, which may cause malabsorption of vitamin B12. However, in this patient, PA is the cause of the vitamin B12 deficiency.

A 55-year-old man has an anterior myocardial infarction. He is placed on aspirin, heparin, and warfarin. Which of the following sets of hemostasis studies is most likely to be present in this patient?

Platelet CountBleeding TimePT PTT A.Normal Normal B. Normal Normal Normal C. D.Normal E. Normal Normal PT, prothrombin time; PTT, partial thromboplastin time.

A. platelet count normal, bleeding time increased, PT normal, PTT increased B. platelet count normal, bleeding time normal, PT normal, PTT increased C. platelet count decreased, bleeding time increased, PT increased, PTT increased

D. platelet count normal, bleeding time increased, PT increased, PTT increased E. platelet count normal, bleeding time normal, PT increased, PTT increased

Option D (platelet count normal, bleeding time increased, PT increased, PTT increased) is correct. In an acute anterior myocardial infarction, aspirin is used to prevent thrombus formation either in the coronary arteries or the damaged endothelium in the left ventricle. Aspirin prevents platelet aggregation causing prolongation of the bleeding time without affecting the platelet count. Patients are frequently anticoagulated with heparin and warfarin. Heparin enhances antithrombin III activity leading to neutralization of many of the coagulation factors including thrombin and X in the final common pathway. This prolongs the PT and the PTT, although the latter test is a better test to follow heparin therapy. Warfarin inhibits further activation of the vitamin Kdependent coagulation factors (II, VII, IX, and X). Since factors II and X are in the final common pathway, the PT and PTT are both prolonged; however, the former test is a better test to follow warfarin therapy when it is converted into the international normalized ratio. Option A (platelet count normal, bleeding time increased, PT normal, PTT increased) is incorrect. This set of studies is most often associated with classic von Willebrand disease. In this autosomal dominant disorder, there is a deficiency of von Willebrand factor (vWF), which is necessary for platelet adhesion to areas of endothelial injury. This prolongs the bleeding time without affecting the platelet count. In the circulation, vWF also complexes with factor VIII coagulant (VIII:C), which prevents degradation of VIII:C. Therefore, deficiency of vWF automatically leads to decreased factor VIII:C activity and prolongation of the PTT. The PT is normal because it does not evaluate VIII:C in the intrinsic pathway. Option B (platelet count normal, bleeding time normal, PT normal, PTT increased) is incorrect. PT normal, PTT increased) is incorrect. This set of studies is consistent with a coagulation factor deficiency in the intrinsic coagulation pathway: XII, XI, IX, or VIII. Of the four choices, factor VIII would be the most common deficiency in a patient with hemophilia A. Option C (platelet count decreased, bleeding time increased, PT increased, PTT increased) is incorrect. This set of data in which all the tests are abnormal is most often seen in disseminated intravascular coagulation (DIC). Tissue thromboplastin activates the extrinsic coagulation system causing the formation of fibrin thrombi within the microcirculation. Factors I, II, V, and VIII are consumed in the fibrin clots, which results in anticoagulation (PT and PTT are both increased). Fibrin thrombi also trap platelets causing thrombocytopenia, which, in turn, is responsible for producing a prolonged bleeding time. Option E (platelet count normal, bleeding time normal, PT increased, PTT increased) is incorrect. This set of data is most consistent with a patient that is taking warfarin and/or heparin. Rat poison contains warfarin, which blocks epoxide reductase rendering vitamin K inactive. This prevents further -carboxylation of the vitamin Kdependent coagulation factors: factors II (prothrombin), VII, IX, and X. Since factors X and II are in the final common pathway, both the PT and PTT are prolonged. The PT evaluates factors VII, X, V, II, and I (fibrinogen), while the PTT evaluates factors XII, XI, IX, VIII, X, V, II, and I. The platelet count and bleeding time are not affected by warfarin. Heparin enhances antithrombin III activity, which neutralizes activated serine proteases.

A 5-year-old child has mental retardation, chronic liver disease, and fasting hypoglycemia caused by a deficiency of aldolase B. Which of the following should be eliminated from the diet?

A. Dairy products B. Gluten C. Phenylalanine D. Table sugar E. Tyrosine


Option D (Table sugar) is correct. The patient has hereditary fructose intolerance, an autosomal recessive disorder that is characterized by a deficiency of aldolase B. This enzyme catalyzes a reaction that converts fructose 1-phosphate to glyceraldehyde 3-phosphate and dihydroxyacetone phosphate. Sucrase converts sucrose to glucose and fructose. Deficiency of aldolase B causes accumulation of fructose 1-phosphate, which is toxic to the liver (chronic liver disease, cirrhosis) and to the brain (mental retardation). The end-products of fructose metabolism are used as substrates for gluconeogenesis; therefore, fasting hypoglycemia also may occur. All fructose-containing products should be eliminated from the diet. Table sugar and honey contain sucrose. Option A (Dairy products) is incorrect. Dairy products should be avoided in lactase deficiency and galactosemia. Lactase, a brush border disaccharidase, catalyzes the conversion of lactose to glucose and galactose. Option B (Gluten) is incorrect. Gluten is present in wheat products. Patients who have antibodies against gliadin (alcohol extract of gluten) develop celiac disease, which is characterized by immunologic destruction of the villi in the small intestine, causing malabsorption of fat, carbohydrates, and proteins. Option C (Phenylalanine) is incorrect. Phenylalanine is increased in phenylketonuria, an autosomal recessive disease characterized by a deficiency of phenylalanine hydroxylase. This enzyme converts phenylalanine to tyrosine. Phenylalanine-containing products must be eliminated (e.g., certain sweeteners) to prevent mental retardation and tyrosine must be added to the diet. Option E (Tyrosine) is incorrect. Tyrosine should be eliminated from the diet in patients with tyrosinosis, an autosomal recessive disease caused by a deficiency of fumarylacetoacetate hydrolase, to prevent chronic liver disease, causing hepatocellular carcinoma. It should be added to the diet in phenylketonuria.

A 49-year-old man has alcoholic cirrhosis, ascites, and dependent pitting edema. Which of the following serum Na +, total body sodium (TBNa+) is most likely present?

Serum Na+TBNa+ A.Decreased Decreased B. Decreased Increased C. Decreased Normal D.Increased Decreased

E. Normal

Decreased

A. serum Na+ decreased, TBNa+ decreased B. serum Na+ decreased, TBNa+ increased C. serum Na+ decreased, TBNa+ normal D. serum Na+ increased, TBNa+ decreased E. serum Na+ normal, TBNa+ decreased

Option B (serum Na+ decreased, TBNa+ increased) is correct. Patients with cirrhosis have alterations in Starling pressures. Decreased liver synthesis of albumin reduces the plasma oncotic pressure, which normally retains fluid in the vascular compartment. Decreased plasma oncotic pressure contributes to the development of ascites and is the primary cause of the patients dependent pitting edema. Portal hypertension secondary to cirrhosis is associated with an increase in plasma hydrostatic pressure, which is a force that drives fluid out of the vascular compartment into the interstitial space and body cavities. An increase in plasma hydrostatic pressure along with hypoalbuminemia, contributes to the development of ascites. The cardiac output is decreased in cirrhosis, because fluid is trapped in the interstitial space and body cavities by the Starling pressure abnormalities. Decreased renal blood flow causes the kidney to reabsorb a slightly hypotonic salt-containing fluid, which produces hyponatremia (serum Na+ = TBNa+/TBW). The reabsorbed fluid is redirected into the interstitial space (decreased plasma oncotic pressure) and body cavities (decreased plasma oncotic pressure and increased plasma hydrostatic pressure) rather than returning to the heart to correct the decreased cardiac output. Regarding the fluid compartments, the extracellular fluid (ECF) compartment is expanded (addition of fluid) and the intracellular fluid (ICF) compartment is expanded (osmosis effect of hyponatremia). Option A (serum Na+ decreased, TBNa+ decreased) is incorrect. This occurs when there is a hypertonic loss of fluid (serum Na+ = TBNa+/TBW), which often occurs with a loop diuretic (inhibition of the Na+, K+, 2Cl cotransporter), Addisons disease (loss of mineralocorticoids), or 21-hydroxylase deficiency (loss of mineralocorticoids). Regarding the fluid compartments, the ECF compartment is contracted (loss of fluid) and the ICF compartment is expanded (osmosis effect of hyponatremia). Option C (serum Na+ decreased, TBNa+ normal) is incorrect. This occurs when there is a hypotonic gain of pure water, which is present in the syndrome of inappropriate antidiuretic hormone (ADH) due to excessive stimulation of ADH release (e.g., ectopic secretion of ADH by a small-cell carcinoma of lung). ADH reabsorbs electrolyte-free water from the collecting tubules causing a dilutional hyponatremia (serum Na+ = TBNa+/TBW). Since Na+ is not reabsorbed by ADH, the TBNa+ is normal; therefore, there are no signs of pitting edema. Regarding the fluid compartments, the ECF compartment is expanded (addition of fluid) and the ICF compartment is expanded (osmosis effect of hyponatremia). Option D (serum Na+ increased, TBNa+ decreased) is incorrect. This occurs when there is a hypotonic loss of sodium containing fluid (serum Na+ = TBNa+/TBW)). Examples include excessive sweating and osmotic diuresis (e.g., glucosuria). The patient has signs of volume depletion (TBNa+) from the loss of sodium, mainly dry mucous membranes and a positive tilt test (blood pressure dropped and pulse rate increased when sitting up). Regarding the

fluid compartments, the ECF compartment is contracted (loss of fluid) and the ICF compartment is contracted (osmosis effect of hypernatremia). Option E (serum Na + normal, TBNa+ decreased) is incorrect. This occurs when there is an isotonic loss of fluid (serum Na+ = TBNa+/TBW). Excellent examples include travelers diarrhea, which is most often due to enterotoxigenic Escherichia coli and cholera. Loss of isotonic fluid does not alter the serum Na+ concentration (serum Na+ = TBNa+/TBW); however, the decrease in TBNa+ causes signs of volume depletion (dry mucous membranes, poor skin turgor, hypotension). Regarding the fluid compartments, the ECF compartment is contracted (loss of fluid) and the ICF compartment is unchanged (no osmotic effect).

In which of the following clinical scenarios would the patient most likely have an increase in the leukocyte shown in the photograph?

A. A 4-year-old child has whooping cough. B. A 24-year-old man, who raises hogs, has a habit of eating raw bacon. C. A 28-year-old man has a perforated acute appendicitis. D. A 45-year-old woman has severe rheumatoid arthritis. E. A 56-year-old man with polycythemia vera has flushing of his face.
Option D (A 45-year-old woman has severe rheumatoid arthritis.) is correct. Monocytosis is the primary leukocyte alteration in chronic inflammation (e.g., rheumatoid arthritis). The photograph shows a monocyte with grayish-blue cytoplasm that contains many fine azurophilic granules and nucleus that is horse-shoe shaped (this monocyte), round, or kidney shaped. Option A (A 4-year-old child has whooping cough.) is incorrect. Bordetella pertussis is the cause of whooping cough. The lymphotoxin in this bacteria inhibits signal transduction by chemokine receptors which prevents lymphocytes from entering lymph nodes leading to lymphocytosis. Lymphocyte counts are often >50,000/mm 3 (lymphoid leukemoid reaction). The photograph shows a small lymphocyte with scant cytoplasm surrounding a dark nucleus. Option B (A 24-year-old man, who raises hogs, has a habit of eating raw bacon.) is incorrect. This patient would most likely develop trichinosis, due to Trichinella spiralis. It is contracted by eating raw or undercooked pork. The larvae penetrate muscle producing muscle pain and tenderness. Invasive helminths produce eosinophilia (type I

hypersensitivity). The photograph shows an eosinophil with cytoplasm packed with reddish-orange granules that do not cover the nucleus. Option C (A 28-year-old man has a perforated acute appendicitis.) is incorrect. A perforated acute appendicitis produces neutrophilic leukocytosis with left shift (e.g., band neutrophils) and toxic granulation (prominent azurophilic granules). Option E (A 56-year-old man with polycythemia vera has flushing of his face.) is incorrect. In polycythemia, all cell lines except lymphocytes are increased. An increase in basophils and mast cells causes the release of histamine, which produces flushing of the face (called plethora), headaches, and pruritus after bathing. All of the myeloproliferative diseases have basophilia.

A 45-year-old woman with systemic lupus erythematosus is being treated with aspirin and corticosteroids. Aspirin and corticosteroids both block the production of which of the following compounds?

A. Arachidonic acid B. Bradykinin C. Histamine D. Leukotrienes E. Prostaglandins


Option E (Prostaglandins) is correct. Aspirin interferes with the production of prostaglandins by inhibiting cyclooxygenase, which converts arachidonic acid to prostaglandin. Corticosteroids interfere with the production of prostaglandins by inhibiting phospholipase A2 in the cell membrane, which prevents the release of arachidonic acid, causing decreased synthesis of prostaglandins and leukotrienes. Option A (Arachidonic acid) is incorrect. Corticosteroids block the release of arachidonic acid, aspirin does not. Option B (Bradykinin) is incorrect. The production of bradykinin involves activation of the kininogen system by activated coagulation factor XII. Neither aspirin nor corticosteroids inhibit the production of bradykinin. Option C (Histamine) is incorrect. Histamine is a preformed chemical that is released from mast cells. Aspirin does not inhibit the release of histamine. Corticosteroids stabilize the mast cell membrane, preventing release of histamine. Option D (Leukotrienes) is incorrect. Corticosteroids decrease leukotrienes by preventing the release of arachidonic acid. Aspirin only inhibits the synthesis of prostaglandins.

A 55-year-old man, who has a 50-pack-year history of smoking cigarettes has a moon face, truncal obesity with purple stria, and thin extremities. Which of the following sites is the primary tumor most likely located to produce these findings?

A. Kidney B. Placenta C. Liver D. Lung E. Testicle F. Thyroid


Option D (Lung) is correct. A small-cell carcinoma of the lung can ectopically produce adrenocorticotropic hormone producing Cushing syndrome (moon facies, truncal obesity with purple stria, thin extremities. It also produces ectopic secretion of antidiuretic hormone (ADH) resulting in the syndrome of inappropriate ADH. Option A (Kidney) is incorrect. A renal-cell carcinoma can ectopically secrete parathyroid hormone-related protein producing hypercalcemia and erythropoietin (EPO) producing secondary polycythemia. Option C (Liver) is incorrect. A hepatocellular carcinoma can produce an insulin-like factor producing hypoglycemia or EPO producing secondary polycythemia. Option E (Testicle) is incorrect. A yolk sac tumor of the testicle can produce an increase in -fetoprotein and a choriocarcinoma of the testicle containing trophoblastic tissue can produce an increase in human chorionic gonadotropin leading to gynecomastia. Option F (Thyroid) is incorrect. A medullary carcinoma of the thyroid derives from C cells that synthesize calcitonin. Calcitonin is a hormone that inhibits osteoclasts causing hypocalcemia.

Which of the following best explains why infection is the usual triggering event for a hemolytic anemia characterized by the presence of RBCs similar to the one depicted in the photograph and those with a special stain in the inset?

A. Dysfunction of O2-dependent myeloperoxidase system B. Leukocyte adhesion defect C. Lysosomal defect D. Opsonization defect E. Phagolysosome formation defect
Option A (Dysfunction of O2-dependent myeloperoxidase system) is correct. The photographs show a bite cell and Heinz bodies (denatured hemoglobin) in glucose-6-phosphate dehydrogenase (G6PD) deficiency. G6PD is the ratelimiting enzyme in the pentose phosphate shunt. Recall that deficiency of the enzyme leads to a decrease in glutathione and nicotinamide adenosine dinucleotide phosphate (NADPH). Glutathione is necessary to neutralize hydrogen peroxide normally produced by RBCs. Unneutralized peroxide damages RBC membranes and denatures hemoglobin in the RBCs resulting in a predominantly intravascular hemolytic anemia. Splenic macrophage removal of a portion of damaged RBC membrane produces bite cells. Infection (most common) and drugs (e.g., primaquine) are the usual triggering events for the hemolytic anemia. Infection is the most common initiating event, because the synthesis of NADPH is decreased in G6PD deficiency and NADPH is required as a cofactor for NADPH oxidase in the O2-dependent MPO system, the most effective microbicidal system available to neutrophils and monocytes. Option B (Leukocyte adhesion defect) is incorrect. NADPH is not required for activation of adhesion molecules (e.g., selectins and -integrins) on neutrophils allowing them to adhere to venular endothelium. Option C (Lysosomal defect) is incorrect. NADPH is not required for the formation of lysosomes, the synthesis of lysosomal enzymes, or the delivery of lysosomal enzymes to the lysosomes. Option D (Opsonization defect) is incorrect. NADPH is not required for the proper functioning of the opsonins IgG or C3b in attaching to bacteria or to their receptors located on neutrophils, monocytes, or macrophages. Option E (Phagolysosome formation defect) is incorrect. NADPH is not involved in the fusion of lysosomes containing hydrolytic enzymes with phagosomes to form phagolysosomes.

A 28-year-old man has fever, fatigue, difficulty breathing, and substernal chest pain while walking or at rest. Physical examination shows bibasilar inspiratory crackles, distention of the jugular neck veins, hepatomegaly, and dependent pitting edema. Pansystolic murmurs along the left parasternal border and apex are present as well as S3 and S4 heart sounds. A chest radiograph shows generalized enlargement of all chambers and alveolar infiltrates in the lungs. The ejection fraction was 10&percnt; (normal 55&percnt;). Laboratory studies reveal an increase in cardiac-specific troponins and creatine kinase (CK)-MB. The photograph shows a histologic section of myocardial tissue from a subendocardial biopsy. Which of the following is the most likely cause of the heart disease?

A. Acute rheumatic fever B. Coronary artery thrombosis C. Ischemic heart disease D. Toxin-induced myocarditis E. Viral myocarditis
Option E (Viral myocarditis) is correct. The histologic section shows an extensive lymphocytic infiltrate (round nuclei) and dissolution of myocardial fibers, which are characteristic of a viral-induced acute myocarditis. Clinical findings include left-sided heart failure (dyspnea, bibasilar inspiratory crackles, alveolar infiltrates); right-sided heart failure (neck vein distention, hepatomegaly, dependent pitting edema); tricuspid and mitral valve regurgitation (pansystolic murmur due to dilated valve rings) with S3 and S4 heart sounds related to volume increases in both ventricles; a decreased ejection fraction (systolic dysfunction due to decreased contractility); and myocardial damage (increased cardiac-specific troponin levels and CK-MB). Adenovirus is the most common viral cause of myocarditis. In this patient, the myocarditis has produced dilated (congestive) cardiomyopathy. This patient will likely require a cardiac transplantation. Option A (Acute rheumatic fever) is incorrect. A patient with rheumatic fever would have a history of group A streptococcal infection (usually pharyngitis) and other features of acute rheumatic fever including polyarthritis, subcutaneous nodules, and erythema marginatum. Option B (Coronary artery thrombosis) is incorrect. Histologic features of thrombosis leading to a myocardial infarction include coagulation necrosis (loss of nuclei and cross-striations) and a neutrophilic infiltrate, neither of which are

present in the histologic section. Option C (Ischemic heart disease) is incorrect. Chronic ischemic heart disease is associated with patchy replacement of myocardial tissue by collagen. Option D (Toxin-induced myocarditis) is incorrect. There is nothing in the history to suggest toxin exposure (e.g., diphtheria toxin, alcohol, drugs); hence, this is an unlikely cause of the dilated cardiomyopathy.

A 75-year-old man has acute urinary retention. A serum blood urea nitrogen (BUN) is 40 mg/dL and serum creatinine is 2 mg/dL. Which of the following additional laboratory test findings is most likely present? A. fractional excretion of sodium (FENa+) < 1% B. Random urine sodium > 40 mEq/L C. UOsm < 350 mOsm/kg D. Urine sediment with renal tubular cell casts E. Urine sediment with waxy casts

Option A (fractional excretion of sodium (FENa +) < 1%) is correct. The patient has acute urinary retention most likely due to benign prostate hyperplasia causing obstruction of urine outflow through the urethra. The serum BUN:creatinine ratio is 20 (40:2), which is consistent with a postrenal azotemia. Postrenal azotemia is most often due to urinary tract obstruction, which, in this case, is prostate hyperplasia. In the initial stages of urinary tract obstruction, tubular function is intact and there is a decrease in the glomerular filtration rate and back diffusion of urea into the blood causing the BUN:creatinine ratio to be >15. The FENa+ is a sensitive indicator of tubular function. An FENa+ < 1% is compatible with intact tubular function. If urinary retention is not relieved, the patient will progress into acute renal failure, where the FENa+ > 2% and the serum BUN:creatinine ratio < 15. Option B (Random urine sodium > 40 mEq/L) is incorrect. Since the serum BUN:creatinine ratio is 20, indicating intact tubular function, reabsorption of sodium in the proximal and distal tubules is normal and the random urine sodium should be <20 mEq/L. A random urine sodium > 40 mEq/L indicates tubular dysfunction. Option C (UOsm < 350 mOsm/kg) is incorrect. Since the serum BUN:creatinine ratio is 20, indicating intact tubular function, renal concentration is normal and the UOsm should be >500 mOsm/kg. A UOsm < 350 mOsm/kg indicates a loss of urine concentration, which is the first sign of tubular dysfunction. Option D (Urine sediment with renal tubular cell casts) is incorrect. Since the serum BUN:creatinine ratio is 20, indicating intact tubular function, there should be no casts in the urine. Renal tubular cell casts are present in renal azotemia. Option E (Urine sediment with waxy casts) is incorrect. Waxy casts are present in chronic renal failure, where the FENa+ would be >2.

A normotensive 8-year-old boy has generalized edema and ascites. The photograph shows an electron micrograph of a glomerulus from a kidney biopsy. Which of the following casts are most likely present in the urine?

A. Fatty casts B. Hyaline casts C. RBC casts D. Renal tubular cell casts E. Waxy casts
Option A (Fatty casts) is correct. The patient has minimal change disease (lipoid nephrosis), which is the most common cause of the nephrotic syndrome in children. The nephrotic syndrome is characterized by proteinuria > 3.5 g/24 hr. Minimal change disease is due to T-cell production of cytokines that causes the loss of the negative charge of the glomerular basement membrane. This produces a selective loss of albumin, resulting in a decrease in plasma oncotic pressure with development of generalized edema with ascites. Hypoalbuminemia is a stimulus for increased cholesterol synthesis in the liver. Cholesterol is lost in the urine, resulting in the formation of fatty casts, which when polarized are shaped like Maltese crosses (see Fig. 19-2C in Rapid Review Pathology, 3rd edition). The electron micrograph shows a light gray basement membrane. The podocytes on the epithelial side of the membrane are fused, which is a characteristic finding in all cases of nephrotic syndrome, regardless of the cause. Option B (Hyaline casts) is incorrect. Hyaline casts are acellular casts that occur in any proteinuric state. In the absence of proteinuria, they have no clinical significance. They do not define any particular type of glomerular disease. Option C (RBC casts) is incorrect. RBC casts are present in nephritic types of glomerulonephritis. Additional findings include mild proteinuria and hematuria. Option D (Renal tubular cell casts) is incorrect. Renal tubular cell casts are present in acute tubular necrosis, the most common cause of acute renal failure. Renal failure rarely occurs in minimal change disease; therefore, renal tubular cell

casts are not likely to be present. Option E (Waxy casts) is incorrect. Waxy casts are present in chronic renal failure, which rarely occurs in minimal change disease; therefore, they are not likely to be present.

The photograph shows a surgically removed section of distal esophagus and proximal stomach in a 52-year-old woman who has a 20-year history of severe heartburn. Which of the following interventions could have prevented this patients disease?

A. Cessation of drinking alcohol B. Cessation of cigarette smoking C. Treatment for achalasia D. Treatment for gastroesophageal reflux disease (GERD) E. Treatment for Helicobacter pylori infection
Option D (Treatment for gastroesophageal reflux disease (GERD)) is correct. The patient has a distal adenocarcinoma of the esophagus secondary to Barretts esophagus, which, in turn, is secondary to GERD. The photograph shows a raised lesion at the junction of the distal esophagus and proximal stomach. GERD is associated with reflux of gastric acid and bile into the distal esophagus due to relaxation of the lower esophageal sphincter (LES). It is the most common causes of Barretts esophagus. Squamous epithelium reacts to acid injury by replacing the epithelium with mucussecreting cells (glandular metaplasia). Esophageal ulceration, stricture, and adenocarcinoma are complications of Barretts esophagus. Treatment of GERD is important, because it decreases the risk for developing distal adenocarcinoma of the esophagus, which is the most common primary cancer of the esophagus in the United States. Treatment includes histamine (H2) blockers, proton inhibitors, and prokinetic agents.

Option A (Cessation of drinking alcohol) is incorrect. Although alcohol is a cause of GERD and should be avoided if there is a history of GERD, treatment of GERD to prevent a Barretts esophagus is the most important prevention for distal esophageal adenocarcinoma. Option B (Cessation of cigarette smoking) is incorrect. Although cigarette smoking is a cause of GERD and should be avoided if there is a history of GERD, treatment of GERD to prevent a Barretts esophagus is the most important prevention for distal esophageal adenocarcinoma. Option C (Treatment for achalasia) is incorrect. Achalasia refers to failure of relaxation of the LES due to destruction of the ganglion cells in the myenteric plexus. This decreases production of nitric oxide synthase, which produces nitric oxide, a relaxing agent of the LES. Option E (Treatment for Helicobacter pylori infection) is incorrect. Peptic ulceration due to H. pylori primarily involves the pylorus and antrum of the stomach. Cytokines incite intestinal metaplasia of the stomach mucosa (goblet cells and Paneth cells), leading to chronic atrophic gastritis, peptic ulcers of the stomach and duodenum, and possible gastric adenocarcinoma. H. pylori does not produce a Barretts esophagus; therefore, it is not a risk factor for developing distal adenocarcinoma of the esophagus.

A 40-year-old pedestrian is hit by an automobile and is brought to the emergency department by ambulance. Laboratory studies completed immediately on arrival include normal complete blood count, serum glucose, and electrolytes. An intravenous line is inserted, and 5% dextrose and normal saline is administered. The patient has the smell of alcohol on his breath. While in the emergency department, he becomes confused and develops eye muscle weakness and horizontal nystagmus. The patient most likely has a deficiency of which of the following vitamins?

A. Folic acid B. Niacin C. Thiamine D. Vitamin B6 E. Vitamin B12


Option C (Thiamine) is correct. The patient has Wernickes encephalopathy (confusion, nystagmus, ophthalmoplegia) caused by acute thiamine deficiency. Thiamine is a cofactor for pyruvate dehydrogenase, which converts pyruvate to acetyl CoA. When patients with borderline thiamine deficiency (common in alcoholics) are infused with glucose intravenously, glucose is oxidized to pyruvate (glycolysis), and pyruvate is converted to acetyl CoA. This last reaction depletes the remaining stores of thiamine and precipitates acute Wernickes encephalopathy. This underscores the importance of giving intravenous thiamine before infusing glucose containing fluids into alcoholics. Option A (Folic acid) is incorrect. Folic acid deficiency, which is common in alcoholics, is associated with a macrocytic anemia and does not produce neurologic dysfunction, as in this patient.

Option B (Niacin) is incorrect. Niacin deficiency produces pellagra. Clinical findings include dementia, diarrhea, and dermatitis (hyperpigmentation in sun-exposed areas). None of these findings is present in this patient. Option D (Vitamin B6) is incorrect. Vitamin B6 (pyridoxine) deficiency is most often associated with isoniazid therapy for tuberculosis. Clinical findings include sideroblastic anemia, convulsions, and peripheral neuropathies. These findings are not present in this patient. Option E (Vitamin B12) is incorrect. Vitamin B12 deficiency in addition to producing a macrocytic anemia is associated with neurologic dysfunction, which affects the posterior columns (loss of proprioception) and lateral corticospinal tract (upper motor neuron signs). These neurologic findings are not present in this patient.

A 40-year-old man has dyspnea and cyanosis. He is currently breathing room air. Arterial blood gas shows a pH of 7.20, Pco2 72 mm Hg, HCO3 28 mEq/L, and Po2 50 mm Hg. Which of the following conditions is most compatible with this arterial blood gas?

A. Acute pulmonary edema B. Acute respiratory distress syndrome C. Barbiturate overdose D. Chronic bronchitis E. Pulmonary infarction
Option C (Barbiturate overdose) is correct. The arterial blood gas shows an acute, uncompensated respiratory acidosis with hypoxemia: pH 7.20 (acidemia), Pco2 72 mm Hg (respiratory acidosis), HCO3 28 mEq/L (normal), Po2 50 mm Hg (hypoxemia). To determine whether the hypoxemia is pulmonary or extrapulmonary in origin, it is necessary to determine the alveolar-arterial (A-a) gradient. The alveolar Po2 is calculated as follows: 0.21 (room air) 713 mm Hg (atmospheric pressure water vapor pressure) 72 mm Hg (arterial Pco2)/0.8 (respiratory quotient) = 60 mm Hg. The arterial Po2 is 50 mm Hg. The A-a gradient is 60 50 = 10 mm Hg. An A-a gradient > 30 mm Hg (set for 100% specificity) indicates that the cause of hypoxemia is insufficient O2 exchange in the lungs (e.g., atelectasis, obstructive lung disease). An A-a gradient < 30 mm Hg is caused by an extrapulmonary cause of hypoxemia. Barbiturates suppress the respiratory center, causing acute respiratory acidosis. Option A (Acute pulmonary edema) is incorrect. Acute pulmonary edema produces acute respiratory acidosis with hypoxemia. The hypoxemia is due to decreased diffusion of oxygen from the fluid-filled alveoli into the pulmonary capillaries. The A-a gradient > 30 mm Hg. Option B (Acute respiratory distress syndrome) is incorrect. Acute respiratory distress syndrome is associated with acute respiratory acidosis with hypoxemia. This is due to collapse of the alveoli (ventilation defect) and increased fluid in the alveoli (diffusion defect). The A-a gradient > 30 mm Hg.

Option D (Chronic bronchitis) is incorrect. Chronic bronchitis is an obstructive lung disease associated with chronic respiratory acidosis and hypoxemia. Inability to clear CO2 from the lungs because of inflammation of the segmental bronchi and terminal bronchioles decreases alveolar oxygen, causing hypoxemia and an A-a gradient > 30 mm Hg. Option E (Pulmonary infarction) is incorrect. Pulmonary infarction is associated with an acute respiratory alkalosis (alkaline pH, decreased Pco2) and hypoxemia. Thromboembolism originating from the deep veins of the leg is the most common cause of pulmonary infarction. It produces a perfusion defect in the lungs, causing hypoxemia and an A-a gradient > 30 mm Hg.

A 48-year-old man has a positive fecal occult blood test. The photographs show the gross and microscopic finding of a lesion in the sigmoid colon. Which of the following best characterizes this lesion?

A. Carcinoid tumor B. Complication of chronic constipation C. Hamartomatous polyp D. Premalignant dysplastic polyp E. Villous adenoma
Option D (Premalignant dysplastic polyp) is correct. The photograph shows a tubular adenoma (adenomatous polyp), which is a precursor lesion for colorectal cancer. Grossly, it is a stalked polyp that has a head with a lobulated, raspberry-like appearance. Microscopically, the fibrovascular stalk is lined by normal colonic mucosa and the branching

head is surfaced by dysplastic (blue-staining) epithelial glands. Risk factors for colorectal cancer are polyp size > 2 cm and the presence of multiple polyps. The most common location for polyps is the sigmoid colon, which is also the most common site for colorectal cancer. Option A (Carcinoid tumor) is incorrect. Carcinoid tumors are neuroendocrine tumors that infiltrate the bowel wall and do not have a polypoid appearance. They are most commonly located in the tip of the appendix followed by the terminal ileum. Option B (Complication of chronic constipation) is incorrect. Chronic constipation most often produces sigmoid diverticulosis due to increased intraluminal pressure. Tubular adenomas are not a complication of chronic constipation. Option C (Hamartomatous polyp) is incorrect. Hamartomatous polyps in the gastrointestinal tract include hyperplastic polyps, Peutz-Jeghers polyps, and juvenile polyps. None of these polyps has a stalk with a raspberry-like head. Option E (Villous adenoma) is incorrect. A villous adenoma is a sessile polyp with finger-like projections (similar to villi in the small intestine) and is most commonly located in the rectosigmoid. It has a greater risk for developing colorectal cancer than a tubular adenoma. The polyp shown in the photograph is stalked and does not have finger-like projections.

The photograph shows a surgical specimen removed from the abdomen of a 12-year-old boy, who had fever and severe right lower quadrant pain. Which of the following is the most common complication of this lesion?

A. Fistula formation B. Localized abscess formation C. Metastasis to liver D. Pylephlebitis E. Subdiaphragmatic abscess
Option B (Localized abscess formation) is correct. The photograph shows an inflamed appendix with congestion and dilation of serosal vessels and swollen mesenteric fat. Localized perforation with periappendiceal abscess formation is

the most common complication of appendicitis. Escherichia coli is the most common pathogen. In children, appendicitis is most often a complication of lymphoid hyperplasia induced by a previous viral infection (e.g., adenovirus, measles). In adults, it usually is due to impaction of stool in the lumen (fecalith) and with ischemia and mucosal injury. Option A (Fistula formation) is incorrect. Fistulas (communication between two hollow structures) are not a common complication of appendicitis. They commonly occur in sigmoid diverticulitis and Crohns disease. Option C (Metastasis to liver) is incorrect. Carcinoid tumors are the most common cancer of the appendix and usually occur at the tip of the appendix. They infiltrate the wall of the appendix but rarely reach the size that is required to metastasize to the liver. They do not cause an inflammatory reaction in the appendix. Option D (Pylephlebitis) is incorrect. Pylephlebitis, or inflammation of the portal vein, is most commonly caused by acute appendicitis; however, it is a rare complication. Portal vein thrombosis secondary to pylephlebitis may result in portal hypertension and ascites. Option E (Subdiaphragmatic abscess) is incorrect. A subdiaphragmatic abscess is most commonly caused by appendicitis; however, it is an uncommon complication.

A 68-year-old woman died of a nosocomial pneumonia. She had severe diarrhea prior to her death. The photograph shows a section of colon removed at autopsy. Which of the following is the best screening test for this disease?

A. Blood culture B. Colon biopsy C. Culture of stool D. Gram stain of stool

E. Toxin assay of stool


Option E (Toxin assay of stool) is correct. The patient has pseudomembranous colitis caused by Clostridium difficile. The photograph shows patches of yellow plaque covering the mucosa of the colon. The patients history of nosocomial pneumonia suggests that the pseudomembranous colitis developed after antibiotic therapy for the pneumonia. Antibiotics destroy the normal colonic bacteria and allow C. difficile to proliferate. C. difficile produces toxins A and B, which damage the mucosa and submucosa. It is a noninvasive pathogen. A cytotoxin assay of stool is the best screening test. Option A (Blood culture) is incorrect. C. difficile is noninvasive; therefore, it does not cause septicemia. Option B (Colon biopsy) is incorrect. A colon biopsy is an invasive and expensive test; therefore, it is not recommended as a screening test. Option C (Culture of stool) is incorrect. C. difficile may normally be present in the colon; therefore, a stool culture positive for C. difficile is not confirmation of pseudomembranous colitis. Option D (Gram stain of stool) is incorrect. C. difficile may normally be present in the colon; therefore, a Gram stain of stool showing gram-positive rods does not prove that pseudomembranous colitis is present.

A 2-year-old boy accidentally ingests rat poison. Physical examination shows bleeding from the mouth and gastrointestinal tract. Which of the following sets of laboratory test results is most likely to be reported?

Platelet countBleeding timePT aPTT A.Decreased Prolonged Normal Normal B. Normal Normal Normal Prolonged C. Normal Normal ProlongedProlonged D.Normal Prolonged Normal Normal E. Normal Prolonged Normal Prolonged PT, prothrombin time; aPTT, activated partial thromboplastin time.

A. platelet count decreased, bleeding time prolonged, PT normal, aPTT normal B. platelet count normal, bleeding time normal, PT normal, aPTT prolonged C. platelet count normal, bleeding time normal, PT prolonged, aPTT prolonged D. platelet count normal, bleeding time prolonged, PT normal, aPTT normal E. platelet count normal, bleeding time prolonged, PT normal, aPTT prolonged

Option C (platelet count normal, bleeding time normal, PT prolonged, aPTT prolonged) is correct. Rat poison contains warfarin, an anticoagulant that inhibits epoxide reductase, which normally converts inactive vitamin K to active vitamin K in the liver. Lack of active vitamin K renders vitamin Kdependent coagulation factors, such as prothrombin, factor VII, factor IX, and factor X, nonfunctional. Bleeding from the mouth and gastrointestinal tract is a sign of overanticoagulation. Warfarin does not affect platelet production or function; therefore, the platelet count and bleeding time (test of platelet function) are normal. The PT evaluates the activity of coagulation factors in the extrinsic system (factor VII) to the formation of a fibrin clot in the final common pathway (factor X, factor V, prothrombin, fibrinogen). The aPTT evaluates the activity of the coagulation factors in the intrinsic system (factor XII, factor XI, factor IX, factor VIII) to the formation of a fibrin clot. Both the PT and aPTT are prolonged, because factor X and prothrombin are present in the final common pathway. The best treatment for this patient is infusion of fresh frozen plasma, which contains functional vitamin Kdependent factors (i.e., factors that have been -carboxylated by vitamin K). Option A (platelet count decreased, bleeding time prolonged, PT normal, aPTT normal) is incorrect. A decreased platelet count (thrombocytopenia) prolongs the bleeding time, because the end of the bleeding time is marked by the formation of a temporary platelet plug consisting of platelets held together by fibrinogen. Signs of thrombocytopenia include petechiae, bleeding from superficial scratches, and easy bruising. Both PT and aPTT are normal in thrombocytopenia, because coagulation factors are normal. Warfarin does not affect the platelet count or platelet function. Option B (platelet count normal, bleeding time normal, PT normal, aPTT prolonged) is incorrect. A normal platelet count, normal bleeding time, normal PT, and a prolonged aPTT indicate a factor deficiency in the intrinsic coagulation system (e.g., factor VIII deficiency). A normal PT indicates that there are no coagulation factor deficiencies in the final common pathway. Warfarin prolongs both the PT and aPTT. Option D (platelet count normal, bleeding time prolonged, PT normal, aPTT normal) is incorrect. A prolonged bleeding time associated with a normal platelet count, normal PT, and normal aPTT indicates a defect in platelet function. The most common cause of a prolonged bleeding time is aspirin or other nonsteroidal anti-inflammatory drugs. Warfarin does not affect platelet production or function. Option E (platelet count normal, bleeding time prolonged, PT normal, aPTT prolonged) is incorrect. The most common cause of a prolonged bleeding time and prolonged aPTT is von Willebrands disease. Warfarin does not affect the bleeding time, which evaluates platelet function.

In which of the following clinical scenarios would the patient most likely have an increase in the leukocyte shown in the photograph?

A. A 4-year-old child has whooping cough. B. A 24-year-old man, who raises hogs, has a habit of eating raw bacon. C. A 28-year-old man has a perforated acute appendicitis. D. A 45-year-old woman has severe rheumatoid arthritis. E. A 56-year-old man with polycythemia vera has flushing of his face.
Option E (A 56-year-old man with polycythemia vera has flushing of his face.) is correct. In polycythemia, all cell lines except lymphocytes are increased. An increase in basophils and mast cells causes the release of histamine, which produces flushing of the face (called plethora), headaches, and pruritus after bathing. All of the myeloproliferative diseases have basophilia. The basophil in the photograph shows the cytoplasm packed with large purple-black granules that cover the nucleus. Option A (A 4-year-old child has whooping cough.) is incorrect. Bordetella pertussis is the cause of whooping cough. The lymphotoxin in this bacteria inhibits signal transduction by chemokine receptors which prevents lymphocytes from entering lymph nodes leading to lymphocytosis. Lymphocyte counts are often >50,000/mm 3 (lymphoid leukemoid reaction). Option B (A 24-year-old man, who raises hogs, has a habit of eating raw bacon.) is incorrect. This patient would most likely develop trichinosis, due to Trichinella spiralis. It is contracted by eating raw or undercooked pork. The larvae penetrate muscle producing muscle pain and tenderness. Invasive helminths produce eosinophilia (type I hypersensitivity). The photograph shows an eosinophil with cytoplasm packed with reddish-orange granules that do not cover the nucleus. Option C (A 28-year-old man has a perforated acute appendicitis.) is incorrect. A perforated acute appendicitis produces neutrophilic leukocytosis with left shift (e.g., band neutrophils) and toxic granulation (prominent azurophilic granules). Option E (A 45-year-old woman has severe rheumatoid arthritis.) is incorrect. Monocytosis is the primary leukocyte alteration in chronic inflammation (e.g., rheumatoid arthritis). In which of the following clinical scenarios would the patient most likely have an increase in the leukocyte shown in the photograph?

A. A 4-year-old child has whooping cough. B. A 24-year-old man, who raises hogs, has a habit of eating raw bacon. C. A 28-year-old man has a perforated acute appendicitis. D. A 45-year-old woman has severe rheumatoid arthritis. E. A 56-year-old man with polycythemia vera has flushing of his face.
Option B (A 24-year-old man, who raises hogs, has a habit of eating raw bacon.) is correct. This patient would most likely develop trichinosis, due to Trichinella spiralis. It is contracted by eating raw or undercooked pork. The larvae penetrate muscle producing muscle pain and tenderness. Invasive helminths produce eosinophilia (type I hypersensitivity). The photograph shows an eosinophil with cytoplasm packed with reddish-orange granules that do not cover the nucleus. Option A (A 4-year-old child has whooping cough.) is incorrect. Bordetella pertussis is the cause of whooping cough. The lymphotoxin in this bacteria inhibits signal transduction by chemokine receptors which prevents lymphocytes from entering lymph nodes leading to lymphocytosis. Lymphocyte counts are often >50,000/mm 3 (lymphoid leukemoid reaction). Option C (A 28-year-old man has a perforated acute appendicitis.) is incorrect. A perforated acute appendicitis produces neutrophilic leukocytosis with left shift (e.g., band neutrophils) and toxic granulation (prominent azurophilic granules). Option D (A 45-year-old woman has severe rheumatoid arthritis.) is incorrect. Monocytosis is the primary leukocyte alteration in chronic inflammation (e.g., rheumatoid arthritis). Option E (A 56-year-old man with polycythemia vera has flushing of his face.) is incorrect. In polycythemia, all cell lines except lymphocytes are increased. An increase in basophils and mast cells causes the release of histamine, which produces flushing of the face (called plethora), headaches, and pruritus after bathing. All of the myeloproliferative diseases have basophilia.

A 55-year-old woman is diagnosed with vitamin B 12 deficiency caused by pernicious anemia. Which of the following laboratory findings is used to distinguish pernicious anemia from other causes of deficiency of vitamin B12 or folic acid?

A. Hypersegmented neutrophils B. Increased plasma homocysteine

C. Increased urine methylmalonic acid D. Pancytopenia with megaloblastic bone marrow E. Reabsorption of vitamin B12 after addition of intrinsic factor
Option E (Reabsorption of vitamin B12 after addition of intrinsic factor) is correct. Vitamin B12 requires intrinsic factor synthesized by parietal cells in the stomach for reabsorption into the terminal ileum. Pernicious anemia is an autoimmune disease with antibodies directed against parietal cells and intrinsic factor (type II hypersensitivity reaction), causing a decrease in intrinsic factor and malabsorption of vitamin B12. In the Schilling test, oral administration of intrinsic factor allows vitamin B12 to be reabsorbed, confirming the diagnosis of pernicious anemia. Option A (Hypersegmented neutrophils) is incorrect. Hypersegmented neutrophils (more than five nuclear lobes) in the peripheral blood are indicators of a deficiency of vitamin B12 or folic acid. Option B (Increased plasma homocysteine) is incorrect. Vitamin B12 removes the methyl group from methyltetrahydrofolate (circulating form of folic acid), producing methylvitamin B12 and tetrahydrofolate. The methyl group is then transferred to homocysteine, which is converted to methionine. Therefore, increased plasma homocysteine is caused by both vitamin B12 and folic acid deficiency. Option C (Increased urine methylmalonic acid) is incorrect. Increased urine methylmalonic acid is caused by a deficiency of vitamin B12. Vitamin B12, unlike folic acid, is involved in propionate metabolism (odd-chain fatty acids). Propionyl CoA is converted to methylmalonyl CoA, which is converted to succinyl CoA, using vitamin B 12 as a cofactor. Deficiency of vitamin B12 causes proximal increase in methylmalonyl CoA, which is then converted into methylmalonic acid. Option D (Pancytopenia with megaloblastic bone marrow) is incorrect. Pancytopenia and a megaloblastic bone marrow are caused by a deficiency of vitamin B12 or folic acid, because both vitamin B12 and folic acid are required for DNA synthesis. Delayed nuclear maturation results in large nucleated hematopoietic cells (megaloblastic cells) that are destroyed in the bone marrow by macrophages or by apoptosis, causing pancytopenia (anemia, neutropenia, thrombocytopenia).

A 59-year-old man had an acute anterior myocardial infarction (MI). Six weeks later, he saw his physician because of fever and precordial chest pain that was less severe when he leaned forward. On physical examination, a friction rub was heard over the precordium. Which of the following mechanisms is most likely involved in the pathogenesis of abnormality in the heart? A. Alteration in Starlings pressure B. Immunologic reaction C. Rupture of the anterior wall

D. Ventricular aneurysm E. Viral infection


Option B (Immunologic reaction) is correct. The patient has fibrinous pericarditis producing a pericardial friction rub (see Fig. 2-7 in Rapid Review Pathology, 3rd edition). Because of the MI 6 weeks earlier, an immunologic reaction is the likely cause. In Dresslers syndrome (postMI syndrome), the patient develops antibodies against the pericardial tissue, which is frequently damaged in a transmural (Q wave) infarction. Antibodies are directed against pericardial antigens (type II hypersensitivity reaction) leading to acute inflammation with a subsequent increase in vessel permeability and production of a fibrinous exudate that covers the surface of the heart. Clinical manifestations of Dresslers syndrome include fever, precordial friction rub, and pain that increases on inspiration but lessens when the patient leans forward. Fibrinous pericarditis also may occur in the first week of an acute transmural infarction. However, the pericarditis in this time frame is due to increased vessel permeability not related to immunologic damage of the pericardium. Option A (Alteration in Starlings pressure) is incorrect. An alteration in Starlings pressure refers to increased hydrostatic pressure or decreased oncotic pressure within the vascular compartment. The transudate produced by this change is poor in proteins and cells, unlike the fibrinous exudate that is associated with a fibrinous pericarditis. Option C (Rupture of the anterior wall) is incorrect. A rupture of the anterior wall occurs 3 to 7 days after an acute MI, causing cardiac tamponade and death. Option D (Ventricular aneurysm) is incorrect. A ventricular aneurysm is clinically recognized within 4 to 8 weeks after a transmural infarction. It presents with a precordial bulge during systole as blood enters the aneurysm causing anterior chest wall movement. The most common complications are heart failure and embolization of clot material. There is no association with fibrinous pericarditis. Option E (Viral infection) is incorrect. Coxsackievirus is the most common cause of pericarditis. This patients history of a previous MI indicates that an immunologic cause for the pericarditis is more likely than an infection.

An afebrile sexually active 23-year-old woman develops a swollen right knee, pain in the left wrist, and pustular lesions on the palm of her left hand. One of the pustules is aspirated. Which of the following is most likely found on Gram stain of the pustular fluid?

A. Gram-negative diplococci B. Gram-negative rods C. Gram-positive cocci D. Gram-positive diplococci

E. Gram-positive rods
Option A (Gram-negative diplococci) is correct. The patient has disseminated gonococcemia caused by Neisseria gonorrhoeae, a gram-negative diplococcus. N. gonorrhoeae is the most common cause of septic arthritis in sexually active individuals younger than 40 years of age. Most patients are afebrile and appear healthy. A characteristic pattern of disseminated gonococcemia is the presence of a purulent monoarthritis (usually the knee); tenosynovitis involving the wrist and/or ankles; and pustules on the palms, soles, or trunk. Cultures of blood, joint fluid, and material aspirated from pustules are positive in less than 40% of cases. In those individuals who are deficient in C6-C9, infection by N. gonorrhoeae is always disseminated, because the membrane attack complex is important in the phagocytosis of the pathogen. Option B (Gram-negative rods) is incorrect. Septic arthritis caused by gram-negative organisms (e.g., Escherichia coli, Pseudomonas aeruginosa) primarily occurs in intravenous drug abusers and in immunocompromised patients. Affected patients are usually febrile and do not have pustular lesions on the skin. Option C (Gram-positive cocci) is incorrect. Staphylococcus aureus (gram-positive coccus) is the most common nongonococcal cause of septic arthritis. Affected patients are usually febrile and do not have pustular lesions on the skin. Option D (Gram-positive diplococci) is incorrect. Gram-positive diplococci (e.g., Streptococcus pneumoniae) do not commonly cause septic arthritis. Option E (Gram-positive rods) is incorrect. Gram-positive rods (e.g., Bacillus species) are very uncommon causes of septic arthritis.

A 42-year-old woman had choreiform movements, muscle rigidity, and dementia that began to develop when she was 30 years of age. Other members of her family have had similar abnormalities. The photograph shows a section of the brain at autopsy (left) and a normal section (right). Which of the following gene abnormalities is responsible for this neurologic disorder?

A. Frameshift mutation B. Missense mutation C. Nonsense mutation D. Silent mutation E. Trinucleotide repeats

Option E (Trinucleotide repeats) is correct. Huntingtons disease is an autosomal disorder characterized by choreiform movements, dementia, and muscle rigidity. It is due to a gene defect on the short arm of chromosome 4, where errors in DNA replication lead to an abnormal repetition of trinucleotides (CAG). The disease becomes progressively worse in future generations due to an increase in the number of trinucleotides (anticipation). The photograph on the right shows a normal caudate nucleus, putamen, and globus pallidus compared with the photograph on the left, the diseased brain, which shows atrophy of all these structures and a dilated ventricle. Atrophy is due to loss of neurons in each of these areas of the brain. Option A (Frameshift mutation) is incorrect. A frameshift mutation is caused by insertion or deletion of nucleotides into a gene that shifts the reading frame during translation of messenger RNA, causing randomly incorrect amino acid sequences. This mutation occurs in Tay-Sachs disease. Option B (Missense mutation) is incorrect. A missense mutation is a point mutation resulting in an altered codon that specifies a different amino acid, leading to variable phenotypic effects. This type of mutation (valine replaces glutamic) occurs in sickle cell disease. Option C (Nonsense mutation) is incorrect. A nonsense mutation is a point mutation resulting in an altered codon that is a stop codon (e.g., UAA), which causes premature termination of DNA transcription. This type of mutation (absent to minimal synthesis of -globin chains) occurs in patients with severe -thalassemia.

Option D (Silent mutation) is incorrect. A silent mutation is a point mutation that results in an altered codon that specifies the same amino acid without altering the phenotypic effect.

A 9-year-old boy complained of headaches and had an ataxic gait. An MRI showed a mass lesion in the posterior fossa and enlarged ventricles. Examination of the cerebrospinal fluid (CSF) showed an increase in protein, decrease in glucose, and hyperchromatic cells with elongated nuclei. Three months later, the patient died of a transtentorial herniation. The photograph shows a section of brain removed at autopsy. What is the most likely diagnosis?

A. Cystic cerebellar astrocytoma B. Ependymoma C. Glioblastoma multiforme D. Medulloblastoma E. Meningioma


Option D (Medulloblastoma) is correct. Medulloblastoma is a common primary malignancy of the brain in children. This primitive neuroepithelial tumor arises from the external granular cell layer in the vermis of the cerebellum. The photograph shows a gray-white tumor with multifocal areas of hemorrhage and necrosis. The tumor has invaded the fourth ventricle (bottom of the photograph), where it has obstructed the flow of CSF, leading to a noncommunicating (obstructive) hydrocephalus (cause of the patients headaches). Medulloblastomas often seed the neuraxis via the CSF, which explains the presence of malignant cells in the CSF. CSF studies also show an increase in protein and a decrease in glucose, the latter due to metabolism of glucose by the malignant cells. The majority of primary brain tumors in children arise in the posterior fossa, particularly in the cerebellum.

Option A (Cystic cerebellar astrocytoma) is incorrect. Cystic cerebellar astrocytomas are the most common primary brain tumors in children. They do not invade the fourth ventricle and in most cases, can be surgically excised. Option B (Ependymoma) is incorrect. Ependymomas in children most commonly arise in the fourth ventricle, where they cause obstructive hydrocephalus. In adults, ependymomas usually are located in the cauda equina. Ependymomas do not seed the neuraxis via the CSF. Option C (Glioblastoma multiforme) is incorrect. Glioblastoma multiforme is the most common primary brain cancer in adults. It is a high-grade astrocytoma that primarily occurs in the cerebral cortex rather than in the posterior fossa. They can seed the neuraxis but do not develop in the cerebellum and are not common in children. Option E (Meningioma) is incorrect. Meningiomas are benign tumors arising from the arachnoid granulations. They are uncommon in children and do not invade the brain.

A 24-year-old black man recently returned from a trip to Nigeria and now has a headache, fever with temperature spikes of 40.6C (105F), jaundice, and painful splenomegaly. He reports that his urine is a red color. A complete blood cell count shows a normocytic anemia. A corrected reticulocyte count shows an increase in peripheral blood reticulocytes. A direct Coombs test is negative. A urine dipstick test is positive for blood. The photograph shows a representative section of the peripheral blood smear. Which of the following is the most likely diagnosis?

A. Autoimmune hemolytic anemia B. Glucose-6-phosphate dehydrogenase (G6PD) deficiency C. Hemolytic anemia caused by pyruvate kinase deficiency D. Hereditary spherocytosis E. Plasmodium falciparum malaria
Option E (Plasmodium falciparum malaria) is correct. The patient has P. falciparum malaria, which is endemic in Nigeria. The peripheral blood shows an RBC with multiple ring forms but no other mature parasitic forms (schizonts, merozoites, gametocytes). These findings are diagnostic of P. falciparum. Parasitic hemolysis correlates with the quotidian fever pattern (recurring daily, with occasional fever spikes). Intravascular hemolysis causes hemoglobinuria

(red urine), and extravascular hemolysis leads to jaundice, as splenic macrophages remove parasitized RBCs and degrade the hemoglobin into unconjugated bilirubin. Hemolytic anemias are all associated with an increase in the corrected reticulocyte count (reticulocyte count that is corrected for the degree of anemia). Option A (Autoimmune hemolytic anemia) is incorrect. When autoimmune hemolytic anemia is suspected, a direct Coombs test, which detects IgG and/or C3b on the surface of the RBCs, is used. The negative Coombs test essentially rules out an autoimmune hemolytic anemia. Option B (Glucose-6-phosphate dehydrogenase (G6PD) deficiency) is incorrect. G6PD deficiency is an X-linked recessive disorder that causes hemolytic anemia. Although G6PD deficiency is common in the black population, there is no history of drug exposure (e.g., receiving primaquine, or certain types of antibiotics) and no bite cells (RBCs with missing membrane) in the peripheral blood. Furthermore, a reticulocyte stain would have picked up Heinz bodies in the RBCs (clumped up hemoglobin), which would clearly have been present in this patient if G6PD was the cause of the hemolysis. Option C (Hemolytic anemia caused by pyruvate kinase deficiency) is incorrect. Pyruvate kinase deficiency is an autosomal recessive disorder associated with a hemolytic anemia. The peripheral blood shows shrunken and spiculated RBCs, with no intraerythrocytic inclusions. Option D (Hereditary spherocytosis) is incorrect. Hereditary spherocytosis is an autosomal dominant disorder that causes hemolytic anemia with jaundice and splenomegaly. It is not associated with a spiking fever. Furthermore, there are no spherocytes (dense, round RBCs) in the peripheral blood smear.

An afebrile, nonsexually active 20-year-old woman complains of burning urine and increased frequency of urination. Which of the following urine laboratory findings is most likely present?

A. Dysmorphic RBCs B. Negative reagent strip for nitrites C. Negative routine urine culture D. Positive reagent strip for leukocyte esterase E. Urine sediment with WBC casts

Option D (Positive reagent strip for leukocyte esterase) is correct. The patient has a lower urinary tract infection (dysuria, increased urine frequency) most likely due to Escherichia coli. Neutrophils are invariably present in the urine with bacterial infections; therefore, the reagent strip test for leukocyte esterase is positive. Option A (Dysmorphic RBCs) is incorrect. Dysmorphic RBCs are RBCs that have protrusions from the cell membrane. They are best identified with phase contrast microscopy. Their presence indicates hematuria of glomerular origin (e.g., nephritic type of glomerulonephritis) and essentially excludes any lower urinary tract

cause of hematuria, including a urinary tract infection. Option B (Negative reagent strip for nitrites) is incorrect. The majority of uropathogens are bacteria that are nitrate reducers; therefore, the reagent strip is usually positive for nitrites. Option C (Negative routine urine culture) is incorrect. The majority of lower urinary tract infections are due to E. coli, which is easily identified with routine urine cultures. Option E (Urine sediment with WBC casts) is incorrect. The presence of WBC casts accompanied by lower urinary tract signs of infection indicates acute pyelonephritis, which is the most common upper urinary tract infection. An upper urinary tract infection has fever, flank pain, and WBC casts.

A 62-year-old man sees his physician because of chronic constipation. The photograph shows a double-contrast barium enema. Which of the following is the most common complication of the colon disorder in this patient?

A. Colorectal cancer B. Fistula formation C. Hematochezia D. Inflammation E. Perforation


Option D (Inflammation) is correct. The radiograph shows multiple diverticula sacs in the sigmoid colon. Sigmoid diverticulosis is most often caused by increased intraluminal pressure secondary to chronic constipation. The weakness occurs in the area where the vessels penetrate the bowel wall. The most common complication is diverticulitis due to impaction of stool (fecalith) in the diverticula sac, leading to ischemia and mucosal injury (same pathogenesis as acute appendicitis in adults). Patients with diverticulitis have fever, left lower quadrant pain, and rebound tenderness (leftsided acute appendicitis). A CT scan is the best diagnostic tool for diagnosing diverticulitis. Increased fiber in the diet is important in preventing sigmoid diverticulosis, because it prevents constipation.

Option A (Colorectal cancer) is incorrect. Left-sided obstructive colorectal cancers can cause diverticulosis; however, diverticula do not increase the risk for developing colorectal cancer. Option B (Fistula formation) is incorrect. Sigmoid diverticulosis is the most common cause of fistula formation in the gastrointestinal tract. However, diverticulitis is the most common predisposing cause of fistula formation, because it causes portions of bowel to adhere to each other, leading to communications between the bowel lumens. Option C (Hematochezia) is incorrect. Hematochezia is passage of bloody (bright red) stools. It is most commonly caused by diverticulosis because of the close proximity of a penetrating blood vessel to the thin wall of the diverticular sac. However, it is not as common a complication as diverticulitis. Option E (Perforation) is incorrect. Perforation is a potential complication of diverticulitis; however, it is not as common as diverticulitis.

An obese 55-year-old woman has postmenopausal bleeding. Menopause occurred at age 51. A hysterectomy and bilateral salpingo-oophorectomy is performed (see photograph). Which of the following is the most likely pathogenesis of the lesion in the endometrial cavity?

A. Adenomyosis

B. Herpes simplex virus type 2 (HSV-2) infection C. Human papillomavirus (HPV) infection D. Multiparity E. Unopposed estrogen exposure

Option E (Unopposed estrogen exposure) is correct. The photograph shows a hemorrhagic and necrotic tumor filling the endometrial cavity and extending through the wall of the uterus. This patient has endometrial carcinoma, which is most often due to unopposed estrogen exposure. Excessive estrogen stimulation of the endometrial mucosa initially causes endometrial hyperplasia, which may become atypical and progress to cancer. Early menarche and late menopause, obesity (patient is obese), taking estrogen without progesterone, and nulliparity are potential risk factors for endometrial adenocarcinoma. They are the most common overall gynecologic tumor (excluding breast carcinoma). The majority present with postmenopausal bleeding. Obesity is a risk factor in the menopausal period, because of increased aromatization of adrenal cortexderived androstenedione to estrone in the adipose. Oral contraceptives decrease the risk for developing these cancers. Option A (Adenomyosis) is incorrect. Adenomyosis is the presence of normal endometrial glands and stroma within the myometrium. It is due to invagination of the stratum basalis into the myometrial tissue. It causes thickening of the myometrial tissue, leading to menstrual irregularities. However, it is not a risk factor for endometrial carcinoma. Option B (Herpes simplex virus type 2 (HSV-2) infection) is incorrect. HSV-2 is not an oncogenic virus. Option C (Human papillomavirus (HPV) infection) is incorrect. High-risk types of HPV (e.g., types 16 and 18) predispose women to squamous cell carcinoma (not adenocarcinoma) of the vulva, vagina, and cervix. Option D (Multiparity) is incorrect. Multiparity reduces the risk of cancer because progesterone opposes the effect of estrogen on glandular tissue.

A 48-year-old man with chronic urinary tract infections and poorly controlled type 2 diabetes mellitus has ultrasound evidence of a large stone in the left renal pelvis producing hydronephrosis (see photograph). The urine has an ammonia smell. A CBC is normal. Which of the following additional urine laboratory findings is expected?

A. Alkaline urine pH B. Calcium oxalate crystals C. Hemoglobinuria D. Ketonuria E. RBC casts


Option A (Alkaline urine pH) is correct. The photograph shows a staghorn calculus (struvite stone) in the renal calyces and renal pelvis. The patients urine has an ammonia smell, which is consistent with a urinary tract infection due to urease producers (e.g., Proteus species). Ammonia produces a urine with an alkaline pH. Staghorn calculi are composed of magnesium, ammonia, and phosphate (staghorn calculus). Option B (Calcium oxalate crystals) is incorrect. Calcium oxalate crystals and stones develop in an acid pH urine. Option C (Hemoglobinuria) is incorrect. Hemoglobinuria is most commonly due to an intravascular type of hemolytic anemia (e.g., glucose 6-phosphate dehydrogenase deficiency). Hemoglobinuria does not produce an alkaline urine or an ammonia smell to urine. Furthermore, the patient does not have anemia. Option D (Ketonuria) is incorrect. Ketonuria occurs in type 1 diabetes mellitus and not type 2 diabetes mellitus. Furthermore, ketone bodies do not alkalinize the urine or produce an ammonia smell. Option E (RBC casts) is incorrect. Casts of any type develop within renal tubules and imply intrinsic renal disease. A renal stone is likely to produce hematuria; however, RBC casts are not present.

The photograph shows a lesion on the face of a 64-year-old man. Exposure to which of the following is the most likely cause of this lesion?

A. Arsenic B. Benzene C. Ethanol D. Polycyclic hydrocarbons E. Ultraviolet (UV) light

Option E (Ultraviolet (UV) light) is correct. The lesion is a basal cell carcinoma on the inner aspect of the nose, which is usually associated with prolonged exposure to UV light (UVB > UVA). The nodular mass has a central depression filled with keratin debris. UV light initiates the development of cross-linked pyrimidine dimers that inhibit DNA replication. Crateriform lesions on the upper lip and the face are usually basal cell carcinomas, whereas those on the lower lip are usually squamous cell carcinomas. Using sunscreen is the best way to prevent UV lightinduced cancers. Option A (Arsenic) is incorrect. A crateriform lesion on the face (similar to that shown in the photograph) is usually a basal cell carcinoma, which is not associated with exposure to arsenic. Most skin cancers associated with arsenic exposure are squamous cell carcinomas, and they are not limited to areas of light exposure. Arsenic exposure is also a risk factor for carcinoma of the lung and angiosarcoma of the liver. Option B (Benzene) is incorrect. Exposure to benzene, a solvent, is a risk factor for acute leukemia and aplastic anemia, not for facial lesions. Option C (Ethanol) is incorrect. Ethanol is associated with squamous cell carcinoma in the oropharynx and upper to midesophagus and with adenocarcinoma of the pancreas, not with lesions in the upper facial area. Option D (Polycyclic hydrocarbons) is incorrect. Polycyclic hydrocarbons are the primary carcinogens in cigarette smoke. Exposure to polycyclic hydrocarbons is not a risk factor for basal cell carcinoma.

The photograph shows a 17-year-old boy, who is concerned about his complexion. Which of the following organisms has a pivotal role in the pathogenesis of the facial lesions?

A. Herpes simplex virus type 1 B. Propionibacterium acnes C. Staphylococcus aureus D. Streptococcus pyogenes E. Trichophyton rubrum
Option B (Propionibacterium acnes) is correct. The patient has acne vulgaris, a condition in which numerous papules and pustules develop on the face and other areas of the body. Acne vulgaris is caused by a combination of androgen stimulation of sebaceous glands and bacterial infection in the hair follicle. The sebaceous glands have androgen receptors with increased sensitivity to androgen, leading to follicular hyperkeratosis in the follicle and increased sebum secretion. P. acnes has lipases that break down fat in the sebum to fatty acids, which irritate the follicle, causing an inflammatory reaction. Option A (Herpes simplex virus type 1) is incorrect. Herpes simplex virus type 1 causes vesicles and pustules that usually are located on the vermilion border of the lip. The virus is located in the sensory ganglia; thus, lesions appear periodically and recede. Option C (Staphylococcus aureus) is incorrect. S. aureus causes folliculitis and skin abscesses. It is not involved in the pathogenesis of acne vulgaris. Option D (Streptococcus pyogenes) is incorrect. S. pyogenes is a cause of impetigo, a superficial infection of skin. It is not involved in the pathogenesis of acne vulgaris. Option E (Trichophyton rubrum) is incorrect. T. rubrum is a superficial dermatophyte that causes skin infections limited to the stratum corneum. It is the most common cause of tinea corporis (body), tinea cruris (groin), and tinea pedis (feet). It is not involved in the pathogenesis of acne vulgaris.

A 28-year-old man develops increased thirst and frequency of urination 3 weeks after hitting his head on the windshield in an automobile accident. He states that he carries water with him at all times to satisfy his thirst. Which of the following laboratory findings is expected if the patient is deprived of water?

POsm Serum ADH A.DecreasedDecreased B. DecreasedIncreased C. Increased Decreased D.Increased Increased E. Normal Normal ADH, antidiuretic hormone; POsm, plasma osmolality.

A. POsm decreased, serum ADH decreased B. POsm decreased, serum ADH increased C. POsm increased, serum ADH decreased D. POsm increased, serum ADH increased E. POsm normal, serum ADH normal
Option C (POsm increased, serum ADH decreased) is correct. The patient has central diabetes insipidus (absence of ADH), most likely caused by transection of the pituitary stalk after hitting his head in the automobile accident. ADH is synthesized in the hypothalamus and travels down through the pituitary stalk to the posterior pituitary, where it is secreted. ADH is required to concentrate urine by reabsorbing electrolyte-free water from the collecting tubules of the kidneys; therefore, absence of ADH causes polyuria (urinary frequency), due to a loss of free water in the urine. When the patient is deprived of water, the urine osmolality (UOsm) decreases and the POsm increases, because of increased serum sodium (hypernatremia). An increase in POsm stimulates thirst. Option A (POsm decreased, serum ADH decreased) is incorrect. A decrease in POsm and a decrease in serum ADH occurs in normal dilution of urine. A decrease in POsm inhibits the release of ADH, causing a loss of free water in the urine and a decrease in UOsm. Option B (POsm decreased, serum ADH increased) is incorrect. A decrease in POsm and an increase in serum ADH occurs when a patient has syndrome of inappropriate ADH (SIADH), which is most often caused by ectopic secretion of ADH by a primary small-cell carcinoma of the lung. An excess of ADH causes increased reabsorption of free water, causing a dilutional hyponatremia (decreased POsm) and increased UOsm (concentrated urine), because the presence of ADH normally concentrates urine. Option D (POsm increased, serum ADH increased) is incorrect. An increase in POsm and an increase in ADH occurs in normal concentration. An increase in POsm stimulates the release of ADH, which increases reabsorption of free water, causing an increase in UOsm.

Option E (POsm normal, serum ADH normal) is incorrect. The history of head trauma followed by polyuria and increased thirst is highly predictive for central diabetes inspidus and is not a normal finding.

The photograph shows the cut surface of the liver removed at autopsy from a 65-year-old woman with hepatomegaly. Which of the following is the most likely cause of this disorder?

A. Alcohol abuse B. Biventricular heart failure C. Metastatic disease D. Portal vein thrombosis E. Viral hepatitis

Option B (Biventricular heart failure) is correct. The photograph shows a mottled, cut surface of the liver. The dark areas represent congested central veins. These findings are indicative of centrilobular necrosis (nutmeg liver), which is most often caused by left-sided heart failure (LHF) and right-sided heart failure (RHF). LHF decreases cardiac output causing hypoperfusion of the liver. This results in ischemic necrosis of hepatocytes located around the central vein. RHF causes a backup of systemic venous blood into the central veins and sinusoids, which produces congestion of central veins and sinusoids and necrosis of hepatocytes around the central vein. Clinical findings include painful hepatomegaly with or without jaundice. Increased transaminases are caused by ischemic necrosis. Left untreated, it may progress to cardiac cirrhosis with fibrosis around the central veins. Option A (Alcohol abuse) is incorrect. Alcohol abuse most often causes a fatty liver, which has a fatty, yellow appearance (not a mottled, cut surface). Option C (Metastatic disease) is incorrect. Liver metastasis is associated with nodular masses surrounded by a normal-appearing liver. Option D (Portal vein thrombosis) is incorrect. Portal vein thrombosis does not cause liver congestion, because the portal vein normally empties blood into the liver. However, patients develop portal hypertension,

ascites, and splenomegaly. Option E (Viral hepatitis) is incorrect. Viral hepatitis causes generalized hepatomegaly; however, it is not associated with vessel congestion.

A 38-year-old man has a family history of colectomies between 35 and 40 years of age. The photograph shows a portion of a total colectomy specimen from this patient. Which of the following best characterizes this disorder?

A. Complication of Crohns disease B. Complication of ulcerative colitis C. Inactivation of a suppressor gene D. Oral mucosal pigmentation E. X-linked recessive inheritance pattern

Option C (Inactivation of a suppressor gene) is correct. The photograph shows the mucosal surface of the colon covered by numerous, sessile adenomas. These findings, plus the family history of colectomies between 35 and 40 years of age, are compatible with familial polyposis, the most common inherited polyposis syndrome. Familial polyposis is an autosomal dominant disorder characterized by inactivation of the adenomatous polyposis coli suppressor gene located on chromosome 5. In some cases, it is associated with congenital hypertrophy of retinal pigment epithelium. Option A (Complication of Crohns disease) is incorrect. Crohns disease is a chronic granulomatous ulceroconstrictive disease that involves only the colon in 20% of cases. There is an increased incidence of the disease in first-degree relatives. The inflammation is transmural and produces skip lesions throughout the gastrointestinal tract. Therefore, a total colectomy at an early age is not a treatment option in Crohns disease, although there is minimal risk for developing colorectal cancer. Furthermore, the nodules on the mucosal surface in the photograph are true polyps and do not represent cobblestoning (mucosa surrounded by areas of ulceration).

Option B (Complication of ulcerative colitis) is incorrect. Ulcerative colitis is a chronic ulceroinflammatory disease that begins in the rectum and may spread continuously to involve the entire colon up to the ileocecal valve. There is an increased incidence of the disease in first-degree relatives. Ulcerative colitis is characterized by extensive areas of mucosal and submucosal ulceration. Islands of inflamed and hemorrhagic mucosa representing pseudopolyps are interspersed between the areas of ulceration. These findings are not present in the colectomy specimen. Option D (Oral mucosal pigmentation) is incorrect. Peutz-Jeghers syndrome is an autosomal dominant polyposis with hamartomatous polyps located primarily in the small bowel and in the colon and stomach to a lesser extent. It is characterized by increased melanin pigmentation of the lips and buccal mucosa. There is a greater than 50% risk for developing colorectal cancer. Option E (X-linked recessive inheritance pattern) is incorrect. Familial polyposis is an autosomal dominant disorder. There are no X-linked polyposis syndromes.

Which of the following clinical disorders is most compatible with the distribution of affected patients shown in this pedigree?

A. Adult polycystic kidney disease B. Cleft lip and palate C. Glucose-6-phosphate dehydrogenase (G6PD) deficiency D. Huntingtons disease E. Sickle cell disease
Option C (Glucose-6-phosphate dehydrogenase (G6PD) deficiency) is correct. Deficiency of G6PD is an X-linked recessive disorder associated with a hemolytic anemia induced by infection or oxidant drugs (e.g., dapsone, primaquine). As shown in the pedigree, X-linked recessive disorders are expressed only in males, whereas females with the abnormal allele are usually asymptomatic carriers. The male transmits the abnormal allele to all of his daughters

and none of his sons. The female carrier transmits the abnormal allele to 50% of her sons, which is evident in the pedigree. Option A (Adult polycystic kidney disease) is incorrect. Adult polycystic kidney disease is an autosomal dominant disorder characterized by cystic changes in the kidneys that result in hypertension and renal failure. Autosomal dominant disorders are characterized by a dominant allele that expresses itself in either the homozygous or the heterozygous state. Only one parent must have the abnormal allele for the disease to be transmitted to the children. A heterozygous parent with the disease transmits the disease to 50% of the children. Option B (Cleft lip and palate) is incorrect. Cleft lip and palate is a disorder with multifactorial (polygenic) inheritance. Multifactorial inheritance involves the additive effect of two or more gene mutations of small effect conditioned by environmental and other nongenetic factors. Option D (Huntingtons disease) is incorrect. Huntingtons disease is an autosomal dominant disorder in which atrophy of the caudate nucleus leads to dementia and a movement disorder. Option E (Sickle cell disease) is incorrect. Sickle cell disease is an autosomal recessive disease associated with sickling of RBCs and hemolytic anemia. In autosomal recessive disorders, disease is present only in patients who are homozygous for the abnormal allele (e.g., aa). In most cases, the parents are asymptomatic heterozygous carriers (e.g., Aa). These parents have a 25% chance of having a normal child (AA), a 50% chance of having children who are asymptomatic carriers (Aa), and a 25% chance of having a child with the disease (aa).

A 72-year-old woman developed a lesion on her breast and similar lesions in other locations over a few months period (see photograph). She has epigastric distress and weight loss. Which of the following is the most likely cause of her stomach pain and weight loss?

A. Gastric adenocarcinoma B. Gastroesophageal reflux disease

C. Metastatic malignant melanoma D. Pancreatic carcinoma E. Peptic ulcer disease


Option A (Gastric adenocarcinoma) is correct. The photograph shows a rough pigmented lesion that has the stuckon, warty appearance of seborrheic keratosis. This common epidermal tumor shows a proliferation of pigmented basal cells on histologic examination. When such lesions develop suddenly, especially in the setting of epigastric pain and weight loss, they indicate the presence of an underlying gastric adenocarcinoma (Leser-Trlat sign). Option B (Gastroesophageal reflux disease) is incorrect. Gastroesophageal reflux disease is not associated with seborrheic keratosis. Option C (Metastatic malignant melanoma) is incorrect. Although seborrheic keratoses are pigmented lesions, they should not be confused with malignant melanoma. Lesions in malignant melanoma do not have a stuck-on appearance. Option D (Pancreatic carcinoma) is incorrect. Pancreatic carcinoma is not associated with seborrheic keratosis. Option E (Peptic ulcer disease) is incorrect. Peptic ulcer disease is not associated with seborrheic keratosis.

A 55-year-old woman with a history of chronic rheumatic heart disease suddenly developed weakness and sensory changes in the right side of the face and right upper extremity. On auscultation, an opening snap and mid-diastolic rumble located at the apex of the chest was heard. Pulse was irregular, and there was a loss of a wave in the jugular venous pulse. On day 3 of hospitalization, the patient had cardiorespiratory arrest and died. The photograph shows the gross appearance of the brain at autopsy. Which of the following underlying conditions best explains the cause of the lesion?

A. Cerebral atherosclerosis B. Embolism from the heart C. Metastatic carcinoma D. Ruptured berry aneurysm E. Systemic hypertension
Option B (Embolism from the heart) is correct. The patients mitral stenosis (opening snap followed by a mid-diastolic rumble) with atrial fibrillation (irregularly irregular pulse) secondary to left atrial dilation. A thrombus most likely developed in the left atrium and embolized to the brain, causing an infarction in the left cerebral cortex in the distribution of the middle cerebral artery. The photograph shows a wedge-shaped hemorrhagic infarction that extends to the surface of the brain. It is hemorrhagic because the embolus was broken down by the fibrinolytic system, causing reperfusion of the infarcted area of the brain. Option A (Cerebral atherosclerosis) is incorrect. The majority of cerebral infarctions are due to thrombosis overlying an atheromatous plaque located in the middle cerebral artery. This produces a pale (not hemorrhagic) infarction that extends up to the surface of the brain. Option C (Metastatic carcinoma) is incorrect. Metastases to the brain usually are multiple, nonhemorrhagic, and peripherally located at the junction of the white and gray matter. The history does not suggest the presence of an underlying cancer in this patient.

Option D (Ruptured berry aneurysm) is incorrect. Most ruptured berry (saccular) aneurysms are located at the junction of the anterior cerebral artery and anterior communicating artery. Rupture usually is into the subarachnoid space rather than the brain parenchyma. Option E (Systemic hypertension) is incorrect. Hypertension causes vascular changes in small vessels, causing rupture of a vessel into the brain parenchyma. Most intracerebral hemorrhages are located in the basal ganglia, particularly the putamen and external capsule rather than the periphery of the brain.

In which of the following clinical scenarios would the patient most likely have an increase in the leukocyte shown in the photograph?

A. A 4-year-old child has whooping cough. B. A 24-year-old man, who raises hogs, has a habit of eating raw bacon. C. A 28-year-old man has a perforated acute appendicitis. D. A 45-year-old woman has severe rheumatoid arthritis. E. A 56-year-old man with polycythemia vera has flushing of his face.
Option C (A 28-year-old man has a perforated acute appendicitis.) is correct. A perforated acute appendicitis produces neutrophilic leukocytosis with left shift (e.g., band neutrophils) and toxic granulation (prominent azurophilic granules), both of which are present in the photograph.

Option A (A 4-year-old child has whooping cough.) is incorrect. Bordetella pertussis is the cause of whooping cough. The lymphotoxin in this bacteria inhibits signal transduction by chemokine receptors which prevents lymphocytes from entering lymph nodes leading to lymphocytosis. Lymphocyte counts are often greater than 50,000/mm3 (lymphoid leukemoid reaction). Option B (A 24-year-old man, who raises hogs, has a habit of eating raw bacon.) is incorrect. This patient would most likely develop trichinosis, due to Trichinella spiralis. It is contracted by eating raw or undercooked pork. The larvae penetrate muscle producing muscle pain and tenderness. Invasive helminths produce eosinophilia (type I hypersensitivity). Option D (A 45-year-old woman has severe rheumatoid arthritis.) is incorrect. Monocytosis is the primary leukocyte alteration in chronic inflammation (e.g., rheumatoid arthritis). Option E (A 56-year-old man with polycythemia vera has flushing of his face.) is incorrect. In polycythemia, all cell lines except lymphocytes are increased. An increase in basophils and mast cells causes the release of histamine, which produces flushing of the face (called plethora), headaches, and pruritus after bathing. All of the myeloproliferative diseases have basophilia.

A febrile 8-year-old child complains of chest pain. Physical examination shows painful cervical adenopathy, dry cracked lips, oral erythema, and erythema and swelling of the hands and feet. The photograph shows one of the feet. Which of the following is a potential complication of the patients disease?

A. Acute myocardial infarction B. Aortic arch aneurysm C. Aortic dissection

D. Infective endocarditis E. Mitral stenosis


Option A (Acute myocardial infarction) is correct. The patient has Kawasaki disease (mucocutaneous lymph node syndrome), which is the most common acquired heart disease in children. Characteristic findings include fever, painful cervical lymph nodes, cracked lips, oral erythema, swelling of the hands and feet, a desquamating rash involving the fingers and toes (see photograph), and vasculitis of the coronary arteries. Vasculitis often leads to coronary artery thrombosis and acute myocardial infarction. The treatment is intravenous gamma globulin. Corticosteroids are contraindicated, because they increase the risk for aneurysms in the coronary arteries. Option B (Aortic arch aneurysm) is incorrect. Thoracic aneurysms are most often caused by atherosclerosis. Kawasaki disease is not associated with inflammation of the aortic arch or the aortic arch vessels. Option C (Aortic dissection) is incorrect. Kawasaki disease is not associated with cystic medial degeneration of the aorta (elastic tissue fragmentation; mucin deposition), which is the precursor lesion for aortic dissection. Option D (Infective endocarditis) is incorrect. Infective endocarditis is most often caused by an infectious organism such as Staphylococcus aureus or Streptococcus viridans. Infection of the heart valves or vessels does not occur in Kawasaki disease. Option E (Mitral stenosis) is incorrect. Mitral stenosis is most often caused by chronic rheumatic mitral valvulitis. Infection of the heart valves does not occur in Kawasaki disease.

A 28-year-old man, who delivers mail for a living, is applying for health insurance. A urine sample has a positive reagent strip and sulfosalicylic acid test for protein. All the other urinalysis reagent strip tests are negative and the sediment examination is normal. A repeat test is requested. The patient is specifically asked to submit the first morning urine for repeat analysis. On the repeat test, the reagent strip is negative for protein. Which of the following types of proteinuria is present in this patient?

A. Functional B. Glomerular C. Overflow D. Tubular


Option A (Functional) is correct. The patient has orthostatic proteinuria, which is a type of functional proteinuria that is not associated with an underlying renal disease. In orthostatic proteinuria, proteinuria occurs with standing and is absent in the recumbent state (e.g., first morning void). It occurs in 15% to 20% of healthy young male adults. Option B (Glomerular) is incorrect. Glomerular proteinuria is associated with a loss of protein ranging from 150 mg/24

hours to >3 g/24 hours. It is subdivided into selective and nonselective proteinuria. Selective proteinuria refers to a loss of albumin and not globulins. It is due to loss of the negative charge in the glomerular basement membrane (e.g., nephrotic syndrome caused by minimal change disease). Nonselective proteinuria refers to the loss of plasma proteins (e.g., albumin and globulins) in urine. It is due to damage of the glomerular basement membrane (e.g., poststreptococcal glomerulonephritis). Glomerular proteinuria is unlikely in this patient, because proteinuria disappeared after reclining and no abnormal casts were present in the urine (e.g., RBC or fatty casts). Option C (Overflow) is incorrect. In overflow proteinuria, the protein loss is variable (0.2 to >10g/24 hours). It is a low molecular weight proteinuria in which the amount filtered exceeds the tubular capacity to reabsorb it (e.g., Bence-Jones proteinuria, hemoglobinuria, myoglobinuria). Overflow proteinuria is unlikely in this patient, because the proteinuria disappeared after reclining. Option D (Tubular) is incorrect. Tubular proteinuria is associated with a protein loss of <2 g/24 hours. It is due to a defect in proximal tubule reabsorption of low molecular weight proteins (e.g., 2-microglobulin, amino acids) at normal filtered loads (e.g., heavy metal poisoning, Hartnups disease). Tubular proteinuria is unlikely in this patient, because proteinuria disappeared after reclining.

A 25-year-old woman has a history of menorrhagia since menarche at age 12. She has been diagnosed with anovulatory dysfunctional uterine bleeding. Additional complaints are easy bruisability, excessive bleeding from superficial cuts from shaving her legs, and frequent nosebleeds. Her bleeding problems improve significantly when she is taking an oral contraceptive and recur to the same level of severity when she discontinues the medication. Which of the following sets of hemostasis studies is most likely to be present in this patient?

Platelet CountBleeding TimePT PTT A.Normal Normal B. Normal Normal Normal C. D.Normal E. Normal Normal PT, prothrombin time; PTT, partial thromboplastin time.

A. platelet count normal, bleeding time increased, PT normal, PTT increased B. platelet count normal, bleeding time normal, PT normal, PTT increased C. platelet count decreased, bleeding time increased, PT increased, PTT increased D. platelet count normal, bleeding time increased, PT increased, PTT increased E. platelet count normal, bleeding time normal, PT increased, PTT increased
Option A (platelet count normal, bleeding time increased, PT normal, PTT increased) is correct. The history of menorrhagia, easy bruisability, bleeding from superficial scratches, and epistaxis all abating with oral contraceptives and

recurring again when off of them is a classic scenario for classic von Willebrand disease. In this autosomal dominant disorder, there is a deficiency of von Willebrand factor (vWF), which is necessary for platelet adhesion to areas of endothelial injury. This prolongs the bleeding time without affecting the platelet count. In the circulation, vWF also complexes with factor VIII coagulant (VIII:C), which prevents degradation of VIII:C. Therefore, deficiency of vWF automatically leads to decreased factor VIII:C activity (factor in the intrinsic pathway) and prolongation of the PTT. The PT is normal because it does not evaluate VIII:C activity in the intrinsic pathway. The estrogen in oral contraceptives increases the release of vWF and VIII:C from endothelial cells, which reverses the platelet and coagulation factor defects. Option B (platelet count normal, bleeding time normal, PT normal, PTT increased) is incorrect. This set of studies is consistent with a coagulation factor deficiency in the intrinsic coagulation pathway: XII, XI, IX, or VIII. Of the four choices, factor VIII would be the most common deficiency in a patient with hemophilia A. Option C (platelet count decreased, bleeding time increased, PT increased, PTT increased) is incorrect. This set of data in which all the tests are abnormal is most often seen in disseminated intravascular coagulation (DIC). Tissue thromboplastin activates the extrinsic coagulation system causing the formation of fibrin thrombi within the microcirculation. Factors I, II, V, and VIII are consumed in the fibrin clots, which results in anticoagulation (PT and PTT are both increased). Fibrin thrombi also trap platelets causing thrombocytopenia, which, in turn, is responsible for producing a prolonged bleeding time. Option E (platelet count normal, bleeding time normal, PT increased, PTT increased) is incorrect. This set of data is most consistent with a patient that is taking warfarin and/or heparin. Rat poison contains warfarin, which blocks epoxide reductase rendering vitamin K inactive. This prevents further -carboxylation of the vitamin Kdependent coagulation factors: factors II (prothrombin), VII, IX, and X. Since factors X and II are in the final common pathway, both the PT and PTT are prolonged. The PT evaluates factors VII, X, V, II, and I (fibrinogen), while the PTT evaluates factors XII, XI, IX, VIII, X, V, II, and I. The platelet count and bleeding time are not affected by warfarin. Heparin enhances antithrombin III activity, which neutralizes activated serine proteases.

A 50-year-old black man with diastolic hypertension develops fever, jaundice, and a hemolytic anemia. The hematocrit is 20% and the corrected reticulocyte count is 18%. The mean corpuscular volume (MCV) is normal. Spherocytes and polychromasia are present in the peripheral smear. The direct Coombs test is positive. An indirect Coombs test reveals antibodies with specificity against the Rh antigens. What is the most likely diagnosis?

A. Cold-autoimmune hemolytic anemia B. Drug-induced hemolytic anemia C. Glucose 6-phosphate dehydrogenase (G6PD) deficiency D. Hereditary spherocytosis E. Sickle cell disease

Option B (Drug-induced hemolytic anemia) is correct. The patient has diastolic hypertension that is complicated by a drug-induced hemolytic anemia (reticulocyte index > 3%). Methyldopa is the most likely drug he is taking, because it is commonly associated with a positive direct Coombs test and with a hemolytic anemia in a small percentage of cases. The drug alters Rh antigens on the surface of RBCs. IgG autoantibodies develop against the altered Rh antigens, attach to RBCs, and are phagocytosed by splenic macrophages (extravascular hemolysis). Patients develop fever, an unconjugated hyperbilirubinemia, spherocytes in the peripheral blood (membrane removed by macrophages), and polychromasia (blue-tinted young reticulocytes). Option A (Cold-autoimmune hemolytic anemia) is incorrect. This type of anemia is IgM-mediated and, in most cases, produces an intravascular hemolysis (not extravascular hemolysis). Most cases are idiopathic. Known causes include infection (Mycoplasma, infectious mononucleosis) and cancer (chronic lymphocytic leukemia). Drugs are not a common cause of drug-induced hemolytic anemias. The majority of immune hemolytic anemias have a positive direct Coombs test. Option C (Glucose 6-phosphate dehydrogenase (G6PD) deficiency) is incorrect. G6PD deficiency is an X-linked recessive disease. Due to the lack of G6PD, there is a corresponding deficiency of glutathione (GSH), which is necessary to neutralize oxidants like hydrogen peroxide. Drugs are a common cause for precipitating a hemolytic episode; however, methyldopa is not one of them. Furthermore, the direct Coombs test is negative in G6PD deficiency. Option D (Hereditary spherocytosis) is incorrect. Hereditary spherocytosis is an autosomal dominant disorder with a deficiency of ankyrin in the RBC membrane. Key findings include an extravascular hemolytic anemia (unconjugated hyperbilirubinemia), splenomegaly, calcium bilirubinate gallstones (black stones), an increase in the mean corpuscular hemoglobin concentration (MCHC), and the presence of spherocytes. The direct Coombs test is negative in hereditary spherocytosis. Option E (Sickle cell disease) is incorrect. Sickle cell anemia has an abnormal cell morphology in the peripheral blood (sickle cells, target cells), unlike the cell morphology in this patient. In addition, the direct Coombs test is negative in sickle cell disease.

The heart shown in the photograph is from a 22-year-old man, who died suddenly while playing softball. Physical examination prior to death revealed a systolic ejection type murmur that decreased in intensity when the patient was supine and increased in intensity when he stood up. What is the most likely cause diagnosis?

A. Aortic regurgitation B. Aortic stenosis C. Hypertrophic cardiomyopathy D. Mitral stenosis E. Mitral valve prolapse (MVP)

Option C (Hypertrophic cardiomyopathy) is correct. The patient had hypertrophic cardiomyopathy, the most common cause of sudden cardiac death in young people. The familial form (most common) is autosomal dominant with nearly complete penetrance. Abnormal genes are mapped to chromosome 14, where there is usually a mutation in the myosin heavy chain gene. There is hypertrophy of the myocardium with disproportionately greater hypertrophy of the interventricular septum (IVS) than the free left ventricular wall, as evident in the photograph. IVS hypertrophy may obstruct blood flow through the outflow tract; however, most patients do not have severe obstruction of the outflow tract. As blood exits the left ventricle, the anterior leaflet of the mitral valve is drawn against the asymmetrically hypertrophied IVS causing the obstruction. Aberrant myofibers are present in the conduction system. Because the left ventricle is hypertrophied, it is noncompliant and restricts filling. A harsh systolic ejection murmur is best heard along the left sternal border and has a palpable double apical impulse. Murmur intensity increases (obstruction worsens) with decreased preload (e.g., standing up) and decreases (obstruction lessens) with increased preload (e.g., lying supine). Increasing preload opens the outflow track. Sudden death is due to ventricular tachycardia/fibrillation, most likely related to the aberrant myofibers in the conduction system. -Blockers are the mainstay of therapy, because they prolong diastole allowing for more blood to enter the left ventricle. Option A (Aortic regurgitation) is incorrect. Aortic regurgitation is characterized by an early diastolic murmur directly after the second heart sound. The left ventricle would be dilated and hypertrophied. Option B (Aortic stenosis) is incorrect. Aortic stenosis is characterized by a systolic ejection murmur due to a stenotic aortic valve (normal in this patient). Murmur intensity increases with an increase in preload (decreases in hypertrophic cardiomyopathy) and decreases in intensity with a decrease in preload (increases in hypertrophic cardiomyopathy). Aortic stenosis is associated with syncope but not sudden cardiac death. Option D (Mitral stenosis) is incorrect. Mitral stenosis is characterized by the presence of an opening snap followed by a diastolic rumble. The mitral valve is normal in this patient. Option E (Mitral valve prolapse (MVP)) is incorrect. MVP produces a midsystolic ejection click followed by a murmur. There is no redundancy of the mitral valve leaflets in the photograph. MVP does not cause sudden cardiac death except when associated with Marfan syndrome, where conduction defects are often present.

A 19-year-old man with type 1 diabetes mellitus develops diabetic ketoacidosis (DKA). Physical examination shows dry mucous membranes and poor skin turgor. The blood pressure lying down and sitting up is decreased and the pulse rate

is increased. The serum glucose is 800 mg/dL and ketone bodies are increased in plasma and urine. Which of the following serum Na+, total body sodium (TBNa+) is most likely present?

Serum Na+TBNa+ A.Decreased Decreased B. Decreased Increased C. Decreased Normal D.Increased Decreased E. Normal Decreased A. serum Na+ decreased, TBNa+ decreased B. serum Na+ decreased, TBNa+ increased C. serum Na+ decreased, TBNa+ normal D. serum Na+ increased, TBNa+ decreased E. serum Na+ normal, TBNa+ decreased
Option A (serum Na+ decreased, TBNa+ decreased) is correct. In DKA, hyperglycemia is the major osmotically active solute in the vascular compartment (not sodium) and causes water to move out of the intracellular fluid (ICF) compartment into the extracellular fluid (ECF) compartment producing a dilutional hyponatremia. However, glucosuria is also present in DKA, which produces an osmotic diuresis causing loss of a hypotonic salt-containing fluid in the urine. The loss of Na+ is responsible for the signs of volume depletion (dry mucous membranes, poor skin turgor, hypotension). Regarding the fluid compartments, the ECF compartment is contracted (loss of fluid by osmotic diuresis) and the ICF compartment is contracted (osmosis effect of hyperglycemia). Option B (serum Na+ decreased, TBNa+ increased) is incorrect. This is an example of a hypotonic gain of sodium containing fluid. Examples include the edema states (e.g., cirrhosis, right-sided heart failure). In these conditions there are alterations in Starling pressuresdecreased plasma oncotic pressure and/or increase in plasma hydrostatic pressurewhich force fluid out of the vascular compartment into the interstitial space and body cavities. The cardiac output is decreased in these conditions, because fluid is trapped in the interstitial space and body cavities by the Starling pressure abnormalities. Decreased renal blood flow causes the kidney to reabsorb a slightly hypotonic saltcontaining fluid, which produces hyponatremia ( serum Na+ = TBNa+/TBW). The fluid is redirected into the interstitial space (decreased plasma oncotic pressure) and body cavities (decreased plasma oncotic pressure and increased plasma hydrostatic pressure). Regarding the fluid compartments, the ECF compartment is expanded (addition of fluid) and the ICF compartment is expanded (osmosis effect of hyponatremia). Option C (serum Na+ decreased, TBNa+ normal) is incorrect. This occurs when there is a hypotonic gain of pure water, which is present in the syndrome of inappropriate antidiuretic hormone (ADH) due to excessive stimulation of ADH release (e.g., ectopic secretion of ADH by a small-cell carcinoma of lung). ADH reabsorbs electrolyte-free water from the collecting tubules causing a dilutional hyponatremia (serum Na+ = TBNa+/TBW). Since Na+ is not reabsorbed by

ADH, the TBNa+ is normal; therefore, there are no signs of pitting edema. Regarding the fluid compartments, the ECF compartment is expanded (addition of fluid) and the ICF compartment is expanded (osmosis effect of hyponatremia). Option D (serum Na+ increased, TBNa+ decreased) is incorrect. This occurs when there is a hypotonic loss of sodiumcontaining fluid ( serum Na+ = TBNa+/TBW)). Examples include excessive sweating and osmotic diuresis (e.g., glucosuria). The patient has signs of volume depletion (TBNa+) from the loss of sodium, mainly dry mucous membranes and a positive tilt test (blood pressure dropped and pulse rate increased when sitting up). Regarding the fluid compartments, the ECF compartment is contracted (loss of fluid) and the ICF compartment is contracted (osmosis effect of hypernatremia). Option E (serum Na + normal, TBNa+ decreased) is incorrect. This occurs when there is an isotonic loss of fluid (serum Na+ = TBNa+/TBW). Excellent examples include travelers diarrhea, which is most often due to enterotoxigenic Escherichia coli and cholera. Loss of isotonic fluid does not alter the serum Na+ concentration (serum Na+ = TBNa+/TBW); however, the decrease in TBNa+ causes signs of volume depletion (dry mucous membranes, poor skin turgor, hypotension). Regarding the fluid compartments, the ECF compartment is contracted (loss of fluid) and the ICF compartment is unchanged (no osmotic effect).

The photograph shows a peripheral blood smear from a 2-year-old black child with sickle cell anemia and sepsis. A bone scan is normal. Which of the following pathogens is the most likely cause of the sepsis?

A. Escherichia coli B. Neisseria meningitidis C. Salmonella paratyphi D. Staphylococcus aureus E. Streptococcus pneumoniae

Option E (Streptococcus pneumoniae) is correct. The peripheral blood smear shows a few boat-shaped sickle cells and two RBCs with black dots in the cytosol, which are remnants of nuclear material (Howell-Jolly bodies). The presence of these bodies indicates splenic dysfunction, because a properly functioning spleen normally has macrophages in the cords of Billroth that will remove nuclear remnants from RBCs without destroying the integrity of the cell membrane. Patients with splenic dysfunction and sickle cell anemia are very susceptible to sepsis from S. pneumoniae (encapsulated bacteria); hence, the importance of Pneumovax at an early age. Option A (Escherichia coli) is incorrect. E. coli is most likely to produce sepsis and meningitis in newborns and sepsis in adults with indwelling urinary catheters. A functioning spleen is not necessary to clear E. coli from the blood. Option B (Neisseria meningitidis) is incorrect. N. meningitidis is the most common cause of sepsis leading to meningitis in patients between 1 and 18 years of age. A functioning spleen is not necessary to clear N. meningitidis from the blood. Option C (Salmonella paratyphi) is incorrect. Sepsis due to S. paratyphi is common in sickle cell disease, because Salmonella species require a functioning spleen to be cleared from the blood. However, S. paratyphi is most often associated with sepsis leading to osteomyelitis, which is not present in this patient (normal bone scan). Option D (Staphylococcus aureus) is incorrect. S. aureus is usually associated with sepsis in patients with indwelling arterial or venous catheters. A functioning spleen is not required to clear S. aureus from the blood.

A 62-year-old man complains of fatigue. Physical examination shows a harsh systolic ejection murmur, grade 4/6, that radiates into the carotid arteries. Laboratory studies show a mild microcytic anemia. A urine dipstick test is positive for blood. The photograph shows a representative section of the peripheral blood smear. Which of the following laboratory studies would most likely identify the cause of the microcytic anemia?

A. Direct Coombs test B. Enzyme assay for pyruvate kinase C. Hemoglobin electrophoresis

D. Osmotic fragility test E. Serum ferritin test


Option E (Serum ferritin test) is correct. The systolic ejection murmur indicates aortic stenosis, and the fragmented RBCs (schistocytes) in the peripheral blood indicate the presence of a microangiopathic hemolytic anemia due to intravascular destruction of the RBCs as they hit the stenotic and dystrophically calcified valve. The damaged cells release hemoglobin directly into the blood. Haptoglobin, a protein synthesized in the liver, combines with the free hemoglobin to form complexes that are phagocytosed by macrophages in the spleen and completely degraded causing very low to absent serum haptoglobin levels. The excess hemoglobin in the plasma is then filtered into the urine producing a red color and a positive dipstick for blood. Chronic hemoglobinuria eventually causes iron deficiency and a microcytic anemia. Serum ferritin is the best screening test for iron deficiency. Since ferritin is a soluble iron-binding protein, some ferritin leaks out of bone marrow macrophages and directly reflects the ferritin stores in the bone marrow. In iron deficiency, ferritin stores are decreased; therefore, serum ferritin is also decreased. Aortic stenosis is the most common cause of a hemolytic anemia associated with schistocytes. Option A (Direct Coombs test) is incorrect. The direct Coombs test detects the presence of IgG and/or C3b on the surface of RBCs. It is useful when autoimmune hemolytic anemia is suspected. Autoimmune hemolytic anemia is usually normocytic; however, it would not explain the presence of schistocytes in the peripheral blood and aortic stenosis. Option B (Enzyme assay for pyruvate kinase) is incorrect. When anemia is caused by pyruvate kinase deficiency, the peripheral blood smear shows dehydrated RBCs with thorny projections, unlike the cells shown in the photograph. In addition, pyruvate kinase deficiency produces a normocytic (not microcytic) anemia. Option C (Hemoglobin electrophoresis) is incorrect. Hemoglobin electrophoresis detects changes in the concentration of normal and abnormal forms of hemoglobin, such as hemoglobin S in sickle cell disease. Option D (Osmotic fragility test) is incorrect. The osmotic fragility test is used to confirm a diagnosis of hereditary spherocytosis, in which the osmotic fragility of RBCs is increased. Hereditary spherocytosis is a normocytic anemia and spherocytes are present in the peripheral blood not schistocytes.

A 29-year-old man sustains bilateral femoral bone fractures and multiple pelvic fractures after crashing his mountain bike into a tree. About 48 hours after the crash, he suddenly develops dyspnea. There are petechial lesions in the upper thorax area and neurologic abnormalities. Laboratory studies show hypoxemia and thrombocytopenia. Which of the following is the most likely diagnosis?

A. Air embolism B. Cardiogenic shock C. Fat embolism

D. Pulmonary thromboembolism E. Septic shock

Option C (Fat embolism) is correct. Fat embolization occurs following traumatic fracture of long bones (e.g., femur) and pelvic bones. Microglobules of fat from the bone marrow of the fractured bones and surrounding adipose lodge in the microvasculature throughout the body where they produce ischemia and hemorrhage. In the lungs, they block perfusion of the capillaries leading to hypoxemia. Fatty acids released from the microglobules of fat damage vessel endothelium, causing formation of platelet thrombi to develop at the sites of injury. Symptoms are usually delayed until 24 to 72 hours after the initial injury. Dyspnea (hypoxemia due to fat microglobules in the pulmonary capillaries) and petechial hemorrhages (due to thrombocytopenia) in the upper chest suddenly develop. Neurologic symptoms include restlessness and mental status changes with progression to coma. Diagnosis of fat embolism is usually a clinical diagnosis based on the above findings. Search for fat globules in urine, pulmonary alveolar lavage, and spinal fluid is sometimes helpful. Treatment is supportive with an emphasis on maintaining good arterial oxygenation. Mortality < 10%. Option A (Air embolism) is incorrect. Air embolism occurs during obstetric procedures (e.g., uterine evacuation in abortion), head and neck surgery, or as a consequence of chest trauma. Suction of air into the venous system mixes with blood in the right side of the heart, causing a frothy substance to block blood flow into the pulmonary artery. Option B (Cardiogenic shock) is incorrect. Cardiogenic shock is most commonly caused by acute myocardial infarction. Cardiac output is decreased, and left ventricular end-diastolic pressure and pulmonary capillary hydrostatic pressure are increased, causing pulmonary edema (left-sided heart failure). It is not associated with thrombocytopenia and petechial lesions in the upper chest. Option D (Pulmonary thromboembolism) is incorrect. Pulmonary thromboemboli originate mainly in the femoral vein. The most common cause of deep venous thrombosis in the calf vessels is stasis of blood flow in the lower extremities due to prolonged bed rest. Clots propagate toward the heart and may break off and form an embolism once they enter the femoral vein. It is not associated with mental status changes, thrombocytopenia, and petechial lesions in the upper chest. Option E (Septic shock) is incorrect. Septic (endotoxic) shock most often occurs in patients with indwelling urinary catheters. Sepsis due to Escherichia coli causes the release of endotoxins, which activate the complement system (releasing anaphylatoxins) and damage endothelial cells (releasing vasodilators such as prostacyclin and nitric oxide). Vasodilation of the peripheral resistance arterioles produces warm skin and increases venous return to the heart, causing high-output cardiac failure. These findings are not present in this patient.

The patient is a 51-year-old woman who states that she feels a lump in the right upper outer quadrant of her breast. The photograph shows an excisional biopsy of the lump. Which of the following is the greatest risk factor for this lesion?

A. BRCA1 suppressor gene B. Cigarette smoking C. Silicone breast implant D. Traumatic fat necrosis E. Unopposed estrogen exposure
Option E (Unopposed estrogen exposure) is correct. The patient has infiltrating ductal carcinoma. The biopsy shows a scirrhous tumor with a stellate appearance in the fat tissue of breast. Estrogen exposure without progesterone increases the risk for developing hyperplasia of the ductal epithelium, which may develop into atypical ductal hyperplasia, dysplasia, and cancer. Risk factors associated with unopposed estrogen exposure include early menarche and late menopause, nulliparity, postmenopausal obesity, and a history of endometrial carcinoma. Option A (BRCA1 suppressor gene) is incorrect. Inactivation of the BRCA1 suppressor gene has a genetic basis (autosomal dominant) and accounts for fewer than 10% of cases of breast cancer. Option B (Cigarette smoking) is incorrect. Cigarette smoking is a risk factor for breast cancer; however, it poses less of a risk than unopposed estrogen stimulation. Option C (Silicone breast implant) is incorrect. There is no proven relationship between silicone breast implants and breast cancer. Silicone gel seeping through the shell results in a granulomatous reaction with foreign body giant cells. Option D (Traumatic fat necrosis) is incorrect. Traumatic fat necrosis causes scarring and skin retraction that simulates infiltrating ductal carcinoma; however, there is no causal relationship with breast cancer.

A 48-year-old man with alcoholic cirrhosis has ascites and dependent pitting edema in the lower legs. Fluid accumulation in the peritoneal cavity and legs occurs by which of the following mechanisms?

A. Decreased plasma oncotic pressure B. Increased plasma hydrostatic pressure C. Increased vessel permeability due to histamine D. Lymphatic obstruction with lymphedema E. Movement of water into the intracellular compartment
Option A (Decreased plasma oncotic pressure) is correct. Edema is the accumulation of fluid in body cavities (e.g., ascites) and in the interstitial space (e.g., peripheral edema). Edema caused by cirrhosis of the liver involves alterations in vascular hydrostatic pressure and in oncotic pressure. An increase in hydrostatic pressure and/or a decrease in plasma oncotic pressure (hypoalbuminemia) cause outflow of a protein-poor (<3 g/dL) and cell poor fluid into body cavities and interstitial spaces. This defines a transudate. In cirrhosis, the portal vein encounters increased resistance to emptying blood into the liver sinusoids (intrasinusoidal hypertension) due to compression of the sinusoids by regenerative nodules and fibrosis. This causes increased hydrostatic pressure (portal hypertension) that contributes to ascites formation. The synthetic function of the liver is compromised in cirrhosis; therefore, hypoalbuminemia occurs, which decreases the plasma oncotic pressure, further contributing to ascites and peripheral edema (dependent pitting edema). Because transudates have decreased protein and cells, they obey the law of gravity and percolate through the interstitial tissue and settle in the most dependent portions of the body (e.g., feet). Option B (Increased plasma hydrostatic pressure) is incorrect. Increased hydrostatic pressure is involved only in ascites formation. It is not involved in dependent pitting edema in the legs. Option C (Increased vessel permeability due to histamine) is incorrect. Increased vessel permeability due to histamine, which is a marker of acute inflammation, causes a nonpitting type of peripheral edema. The edema fluid is a proteinrich exudate (>3 g/dL) that contains polymorphonuclear leukocytes. Exudates also accumulate in body cavities (e.g., pleural effusion in pneumonia). Option D (Lymphatic obstruction with lymphedema) is incorrect. Obstruction of lymphatic channels causes leakage of lymphatic fluid into the interstitial space (e.g., filariasis), producing a nonpitting lymphedema. Lymphatic fluid accumulates in body cavities (e.g., chylous effusions in the pleural cavities caused by a tear in the thoracic duct). Option E (Movement of water into the intracellular compartment) is incorrect. Movement of water between the extracellular fluid compartment (ECF) and the intracellular fluid compartment (ICF) is called osmosis. Alterations in the serum Na+ concentration in the ECF compartment is the primary cause of water movement between the compartments. In hyponatremia, water moves from the ECF into the ICF compartment, whereas in hypernatremia, water moves from the ICF into the ECF compartment.

A 62-year-old man, who has smoked cigarettes for the past 40 years, has a headache accompanied by projectile vomiting. The photograph shows the retina in the patient. Which of the following disorders is most likely responsible for this lesion?

A. Alzheimers disease B. Exophthalmos C. Glaucoma D. Macular degeneration E. Metastatic cancer to the brain
Option E (Metastatic cancer to the brain) is correct. The retina shows swelling of the optic disk with a loss of the disk margins and hard exudates (white streaks). This may be due to inflammation of the optic nerve (e.g., optic neuritis), severe hypertension, intraocular disease (e.g., central retinal vein occlusion), optic nerve lesions (e.g., optic neuritis), or an increase in intracerebral pressure (papilledema; e.g., metastatic cancer). Since the patient is a smoker, the most likely cause is metastasis to the brain from a primary lung cancer. Option A (Alzheimers disease) is incorrect. Alzheimers disease is associated with dementia and brain atrophy without an increase in intracranial pressure. Option B (Exophthalmos) is incorrect. Graves disease, the most common cause of hyperthyroidism, is associated with exophthalmos (proptosis) of the eyes due to increased adipose and glycosaminoglycans in the orbital tissue. It is not associated with swelling of the optic disk. Option C (Glaucoma) is incorrect. Glaucoma is associated with an increase in intraocular pressure due to a narrow anterior chamber (acute angle-closure glaucoma) or decreased aqueous drainage through the trabecular meshwork (open-angle glaucoma). The optic disk usually shows cupping and pallor without evidence of swelling. Option D (Macular degeneration) is incorrect. Macular degeneration is the leading cause of permanent visual loss in the elderly. Yellow deposits are seen in the macular region (called drusen) followed by degeneration of the retina. There is no swelling of the optic disk.

A 25-year-old man develops fever, vomiting, and severe, steady right lower quadrant pain. Physical examination shows rebound tenderness in the right lower quadrant. Laboratory studies reveal a neutrophilic leukocytosis and left shift. What is the most likely diagnosis?

A. Acute cholecystitis B. Acute Meckels diverticulitis C. Acute sigmoid diverticulitis D. Crohns disease E. Ischemic colitis
Option B (Acute Meckels diverticulitis) is correct. A maxim in surgery is that if a patient has what looks like acute appendicitis (fever, right lower quadrant pain with rebound tenderness, neutrophilic leukocytosis), consider acute Meckels diverticulitis. Clinically, it cannot be distinguished from acute appendicitis. A Meckel diverticulum is a vitelline (omphalomesenteric) duct remnant. It is a true diverticulum, because it has all layers from mucosa to serosa. The mnemonic is: 2 inches long, 2 feet from ileocecal valve, 2% of population, and 2% are symptomatic. They contain pancreatic rests and heterotopic gastric mucosa, which increase the risk for bleeding. In newborns, they may present with fecal material in the umbilical area due to persistence of the vitelline duct. Bleeding is the most common complication of a Meckel diverticulum and is a common cause of iron deficiency in newborns and young children (less common in adults). Diverticulitis is the second most common complication. The diagnosis is confirmed with a 99mTc nuclear scan, which identifies parietal cells in ectopic gastric mucosa. The treatment is surgery. Option A (Acute cholecystitis) is incorrect. Acute cholecystitis presents with fever; right upper quadrant (not lower quadrant) constant, dull pain 15 to 30 minutes after eating; vomiting; radiation of pain to the right scapula/shoulder; and jaundice (25%). None of these findings is in the patient. Option C (Acute sigmoid diverticulitis) is incorrect. Sigmoid diverticulitis presents like acute appendicitis and Meckels diverticulitis, except the pain and rebound tenderness is the left lower quadrant (not the right lower quadrant). Option D (Crohns disease) is incorrect. Crohns disease presents with recurrent right lower quadrant colicky pain (obstruction) with diarrhea. Option E (Ischemic colitis) is incorrect. Ischemic colitis presents with severe pain in the splenic flexure area (left upper quadrant) shortly after eating. It is associated with bloody stools in some cases.

A 28-year-old man develops a rash on the abdomen (photograph A). Examination of the dark-colored areas shows profuse fine scaling. Light scraping of the scales from the dark-colored area is treated with potassium hydroxide (KOH) and examined under the microscope (photograph B). Which of the following organisms is the cause of the rash?

A. Malassezia furfur B. Microsporum audouinii C. Microsporum canis D. Trichophyton rubrum E. Trichophyton tonsurans
Option A (Malassezia furfur) is correct. The patient has tinea versicolor, a common superficial dermatophyte infection caused by M. furfur. This skin disease primarily occurs during the summer months. M. furfur produces an acid, which inhibits melanin transfer to keratinocytes, thereby producing dark and light areas of skin (part A). Part B shows the classic spaghetti and meatballs appearance of the hyphae (spaghetti) and yeasts (meatballs). Option B (Microsporum audouinii) is incorrect. M. audouinii is a superficial dermatophyte and is a common cause of tinea capitis. It does not infect skin and produce the spaghetti and meatball appearance in a KOH preparation. Option C (Microsporum canis) is incorrect. M. canis is a superficial dermatophyte and is a common cause of tinea capitis. It does not infect skin and produce the spaghetti and meatball appearance in a KOH preparation. Option D (Trichophyton rubrum) is incorrect. T. rubrum is a superficial dermatophyte that is most often responsible for fungal infections involving the skin (tinea corporis), feet (tinea pedis), and groin (tinea cruris). It does not produce light and dark areas in the skin, and the KOH preparation has predominantly fungal hyphae. Option E (Trichophyton tonsurans) is incorrect. T. tonsurans is a superficial dermatophyte and is a common cause of tinea capitis. It does not infect skin and produce the spaghetti and meatball appearance in a KOH preparation.

The photograph is of a pruritic skin rash in a 35-year-old man who went camping in a densely populated woods during the summer. Which of the following conditions has a similar hypersensitivity reaction?

A. Acute rejection of a donor kidney B. Hemolytic anemia after receiving penicillin C. Myasthenia gravis D. Rh hemolytic disease of the newborn E. Skin rash caused by penicillin

Option A (Acute rejection of a donor kidney) is correct. The patient has poison ivy, which is a woody plant that produces urushiol, a skin irritant that causes a pruritic rash with vesicle formation. It is an example of an allergic contact dermatitis, which is a type IV hypersensitivity reaction. Specifically, it is an example of a cellmediated cytotoxicity reaction where CD8 T cells interact with altered class I antigens on neoplastic, virusinfected, or donor graft cells, causing cell lysis or, as in this case, a CD8 T-cell reaction against antigens in skin. Cell-mediated cytotoxic reactions also play the key role in acute rejection of organ transplants, where CD4 T cells release cytokines, resulting in activation of host macrophages, proliferation of CD8 T cells, and destruction of donor graft cells. Option B (Hemolytic anemia after receiving penicillin) is incorrect. Hemolytic anemia caused by penicillin is a type II hypersensitivity reaction. IgG antibodies are directed against penicillin attached to the RBC membrane. Macrophages with Fc receptors for IgG phagocytose and destroy the sensitized RBCs. Option C (Myasthenia gravis) is incorrect. Myasthenia gravis is a type II hypersensitivity reaction. IgG antibodies are directed against acetylcholine receptors, causing muscle weakness. Option D (Rh hemolytic disease of the newborn) is incorrect. Rh hemolytic disease of the newborn is a type II hypersensitivity reaction. IgG antibodies against D antigen in the mother cross the placenta and attach to fetal RBCs containing D antigen. Fetal macrophages phagocytose and destroy the sensitized RBCs, causing severe anemia.

Option E (Skin rash caused by penicillin) is incorrect. Penicillin is the most common drug that produces allergic skin reactions caused by the release of histamine from previously sensitized mast cells. Allergic skin reactions to penicillin are type I hypersensitivity reactions involving IgE antibodies.

A 28-year-old man with a history of intravenous drug abuse died of complications related to septicemia. He had a history of fever and a pansystolic murmur at the apex that did not increase in intensity on deep, held inspiration. The photograph shows a valvular lesion that was present at autopsy. Which of the following valvular disorders and pathogen is the most likely cause of death?

A. Aortic regurgitation and Staphylococcus aureus B. Mitral regurgitation and S. aureus C. Mitral stenosis and Streptococcus viridans D. Tricuspid regurgitation and S. aureus E. Tricuspid regurgitation and Staphylococcus epidermidis
Option B (Mitral regurgitation and S. aureus) is correct. The patient had acute bacterial endocarditis (ABE; fever, septicemia, heart murmur) involving the mitral valve. Mitral valve regurgitation produces a pansystolic murmur that does not increase in intensity with deep, held inspiration. The mitral valve in the photograph shows bulky vegetations (see arrow) located along the margin or line of closure of the valve. S. aureus is the most common pathogen in acute infective endocarditis associated with intravenous drug abuse. The tricuspid valve is the most common valve involved followed by the aortic and mitral valve. Option A (Aortic regurgitation and Staphylococcus aureus) is incorrect. Aortic regurgitation is associated with a highpitched diastolic blowing murmur that occurs immediately after S2. As with all left-sided valvular murmurs and abnormal heart sounds, it does not increase in intensity with deep, held inspiration. Option C (Mitral stenosis and Streptococcus viridans) is incorrect. The heart sounds associated with mitral stenosis begin with an opening snap in early of mid diastole followed by a mid-diastolic rumbling murmur. Bacterial endocarditis that is not associated with intravenous drug abuse is most often due to S. viridans, which is a pathogen that can only seed a previously damaged valve.

Option D (Tricuspid regurgitation and S. aureus) is incorrect. Although the tricuspid valve is the most common valve involved in ABE due to intravenous drug abuse, the pansystolic murmur most often located along the left parasternal border increases on deep, held inspiration, which distinguishes it from mitral regurgitation.(The negative intrathoracic pressure increases on deep inspiration and draws blood into the right side of the heart causing all heart murmurs and abnormal heart sounds to increase in intensity.) Option E (Tricuspid regurgitation and Staphylococcus epidermidis) is incorrect. S. epidermidis is the most common cause of ABE associated with prosthetic heart valves.

A 36-year-old woman diagnosed with chronic hepatitis B has hypertension and dependent pitting edema in the lower extremities. A urine dipstick test is strongly positive for albumin. Urinalysis shows numerous casts that contain particles that are in the shape of Maltese crosses when the cast is polarized. The photograph shows an immunofluorescence study of a glomerulus from a kidney biopsy. Which of the following renal disorders is most likely present?

A. Diffuse membranous glomerulopathy B. Focal segmental glomerulosclerosis C. Goodpasture syndrome D. IgA glomerulopathy E. Minimal change disease

Option A (Diffuse membranous glomerulopathy) is correct. The photograph shows granular immunofluorescence with irregular deposits located in the glomerular capillaries. Granular immunofluorescence is due to the deposition of immunocomplexes in the glomeruli. The chronic hepatitis B infection, hypertension, dependent pitting edema, heavy proteinuria, and fatty casts with Maltese crosses is compatible with a diagnosis of nephrotic syndrome due to diffuse membranous glomerulopathy. This type of glomerulonephritis may be associated with hepatitis B and is the second most common overall cause of the nephrotic syndrome in adults. The glomerular injury is due to immunocomplex deposition in a subepithelial location (type III hypersensitivity reaction).

Option B (Focal segmental glomerulosclerosis) is incorrect. Focal segmental glomerulosclerosis is most often associated with AIDS in young black males and with intravenous heroin addiction. It is now considered the most common adult cause of nephrotic syndrome. Glomerular injury is due to cytokine damage of the visceral epithelial cells. It is not an immunocomplex disease; therefore, the immunofluorescent study is negative. It is a very common cause of end stage renal disease. Option C (Goodpasture syndrome) is incorrect. Goodpasture syndrome is associated with antibasement membrane antibodies directed against pulmonary capillary and glomerular capillary basement membranes (type II hypersensitivity). It produces linear immunofluorescence (as opposed to granular) and is associated with a nephritic type of glomerulonephritis (RBC casts, mild proteinuria). It is more common in men. Option D (IgA glomerulopathy) is incorrect. IgA glomerulopathy is an immunocomplex type of glomerulonephritis associated with episodic bouts of microscopic or macroscopic hematuria. The immunocomplexes deposit in the mesangium rather than in the glomerular capillaries. Most patients are normotensive. There is no association with hepatitis B. Option E (Minimal change disease) is incorrect. Minimal change disease (lipoid nephrosis) is the most common cause of the nephrotic syndrome in children. Cytokine damage to the basement membrane causes a loss of the negative charge, resulting in a selective loss of albumin in the urine. Patients usually are normotensive. It is not an immunocomplex disease; therefore, the immunofluorescent study is negative.

You are advising a 30-year-old woman about preventive screening for diseases. Which of the following screening tests would you tell her has most reduced the incidence of cancer in women in the United States?

A. Cervical Pap smear B. Clinical breast examination C. Colonoscopy D. Mammography E. Stool guaiac
Option A (Cervical Pap smear) is correct. The incidence of cervical carcinoma has markedly decreased because of the increased use of the cervical Pap smear. This explains why cervical cancer is the least common of the gynecologic cancers in the United States. This screening test is able to detect squamous dysplasia, the precursor lesion for squamous cell carcinoma. Colposcopy is used to identify the dysplastic site, which is then surgically removed. Option B (Clinical breast examination) is incorrect. There has been a slight decline in the incidence of breast cancer because of annual clinical breast examinations. However, breast cancer still remains the most common cancer in women. Option C (Colonoscopy) is incorrect. There has been a slight decline in the incidence of colorectal cancer because of colonoscopy screening. However, colorectal carcinoma remains the second most common cancer in adults.

Option D (Mammography) is incorrect. There has been a slight decline in the incidence of breast cancer because of mammography. However, breast cancer remains the most common cancer in women. Option E (Stool guaiac) is incorrect. There has been a slight decline in the incidence of colorectal cancer because of screening with stool guaiac (detects blood in stool). However, colorectal carcinoma still remains the third most common cancer in women.

A 26-year-old woman has retro-orbital pain and blurry vision in the right eye. Physical examination shows flame hemorrhages around the disk vessels and a swollen optic disk. After treatment with systemic corticosteroids, the patients vision is restored to normal. A few months later, the patient has slurry speech, an ataxic gait, and weakness and paresthesias in the arms and legs that eventually remit without sequelae. Which of the following findings is most likely present in the cerebrospinal fluid (CSF)?
Option D (Oligoclonal bands) is correct. The patient has multiple sclerosis. It is an autoimmune disease initiated by genetic factors (e.g., HLA-DR2) and environmental triggers, the latter including microbial pathogens (e.g., Epstein Barr virus, human herpes virus 6, Chlamydophilia pneumoniae), vitamin D, and sun exposure. Environmental triggers activate helper T-cells whose antigen-specific receptors recognize CNS myelin basic protein (other antigens as well) as an antigen. T cells release cytokines that activate macrophages, which also release cytokines (e.g., tumor necrosis factor-) that destroy the myelin sheath as well as oligodendrocytes that synthesize myelin (type IV hypersensitivity). Antibodies directed against the myelin sheath and oligodendrocytes may also be involved. The episodic course of acute relapses with optic neuritis (blurry vision), scanning speech, cerebellar ataxia, and sensory and motor dysfunction, followed by remissions, are characteristic of this disease. The demyelinating plaques in multiple sclerosis occur in the white matter of the cerebral cortex (see Fig. 25-27B in Rapid Review Pathology, 3rd edition). The plaques usually have a perivenular distribution and are accompanied by a perivascular lymphoid and plasma cell infiltrate with microglial cells containing phagocytosed myelin. CSF shows an increase in CSF protein. High-resolution electrophoresis of CSF shows discrete bands of immunoglobulins in the -globulin region called oligoclonal bands (see Fig. 25-27D in Rapid Review Pathology, 3rd edition). They are indicative of a demyelinating process. Option A (Decreased glucose) is incorrect. The CSF glucose is normal in multiple sclerosis. Option B (Increased neutrophils) is incorrect. T lymphocytes (not neutrophils) are increased in the CSF in multiple sclerosis. Option C (Normal protein) is incorrect. The CSF protein is increased (not normal) in multiple sclerosis. Option E (Positive Gram stain) is incorrect. The Gram stain is negative (not positive) for microbial pathogens in multiple sclerosis.

An afebrile blood group O, Rh negative (O) 75-year-old man has a massive lower gastrointestinal bleed from sigmoid diverticulosis. He has to be transfused with blood group O, Rh positive (O+) blood, because no group O, Rh negative (O) blood is currently available in the blood banks in the area. He states that he has been transfused once in the past without any problems. In the pretransfusion workup, the patient has a negative antibody screen and a compatible major crossmatch with 4 units of group O, Rh positive (O+) blood. Midway through infusion of the third unit of blood he

develops fever, headache, and tachycardia. The transfusion is stopped and a transfusion workup in the blood bank exhibits the following on a posttransfusion specimen of patient blood:

Patient temperature: 103o F (39.4 C) Patient blood pressure: 130/86 mm Hg Patient pulse: 130 beats/minute Patient plasma: clear Patient antibody screen: negative Patient direct Coombs: negative Patient urine: negative dipstick for blood

Which of the following best explains the mechanism for the transfusion reaction? A. Delayed hemolytic transfusion reaction B. Error in the major crossmatch C. Hemolytic transfusion reaction related to receiving Rh positive blood D. Histamine-related transfusion reaction E. Patient anti-HLA antibodies are directed against donor leukocytes
Option E (Patient anti-HLA antibodies are directed against donor leukocytes) is correct. The patient has a febrile transfusion reaction. In these reactions, the recipient has anti-human leukocyte antigen (HLA) antibodies directed against foreign HLA antigens on donor leukocytes (type II hypersensitivity reaction). Destruction of the donor leukocytes releases pyrogens causing fever as well as other findings such as chills, headache, and flushing. Option A (Delayed hemolytic transfusion reaction) is incorrect. The antibody screen and direct Coombs test are both negative. This excludes the presence of IgG antibodies in the serum attaching to foreign antigens on donor RBCs. This would lead to phagocytosis and destruction of the RBCs by splenic macrophages (extravascular hemolysis) and unconjugated hyperbilirubinemia, with a possibility of developing jaundice. Option B (Error in the major crossmatch) is incorrect. Although errors in crossmatching blood do occur, they are extremely rare. Furthermore, an error would not explain the correlation of fever with infusion of the blood. Option C (Hemolytic transfusion reaction related to receiving Rh positive blood) is incorrect. The patient had a negative antibody screen prior to his transfusions, which excludes the presence of preexisting anti-D antibodies. Receiving D antigen positive blood would not result in the immediate development of antibodies leading to a hemolytic anemia. However, it is likely, that in the future, he would have to receive D antigen negative blood, because of anti-D antibodies from D antigen positive transfusions. Option D (Histamine-related transfusion reaction) is incorrect. The patient does not have an allergic type of transfusion reaction, which involves the release of histamine from mast cells leading to itching, flushing, and hives (type I hypersensitivity reaction).

A 46-year-old man died soon after complaining of a severe headache at the back of his head. His medical history indicated that family members developed chronic renal failure between 50 and 70 years of age. The photograph shows both kidneys removed at autopsy. Which of the following is the most likely cause of the patient s death?

A. Atherosclerotic stroke B. Bacterial meningitis C. Cerebral abscess D. Embolic stroke E. Ruptured berry aneurysm
Option E (Ruptured berry aneurysm) is correct. The photograph shows complete effacement of the normal kidney architecture in both kidneys by cysts within the cortex and medulla. Because of the patients family history of chronic renal failure, he most likely had adult polycystic kidney disease (APKD). The patient had a classic history of a subarachnoid hemorrhage (severe occipital headache), which is most often due to a rupture of a congenital berry aneurysm. Berry aneurysms occur in 10% to 30% of patients with APKD. Hypertension is the most important risk factor for developing berry aneurysms, because cerebral vessels normally have an area of weakness at the junction of the communicating branches with the main cerebral artery (absent smooth muscle and internal elastic lamina in the media). Hypertension occurs in the majority of patients with APKD. Option A (Atherosclerotic stroke) is incorrect. Atherosclerotic strokes are not associated with the type of headache described in this patient. Generally, patients with atherosclerotic stroke develop contralateral sensory and motor dysfunction. Option B (Bacterial meningitis) is incorrect. Bacterial meningitis does not present with a sudden headache. Patients usually complain of pain in the neck on movement of the head (nuchal rigidity). Option C (Cerebral abscess) is incorrect. Patients with a cerebral abscess do not have a sudden onset of headache. They are more likely to have neurologic deficits corresponding to the location of the abscess.

Option D (Embolic stroke) is incorrect. Embolic strokes usually occur in the distribution of the middle cerebral artery; therefore, occipital headaches are unlikely.

A 2-year-old boy with Bruton's agammaglobulinemia has recurrent pneumonia caused by Streptococcus pneumoniae. Which of the following defects is the most likely cause of increased susceptibility to bacterial infections?

A. Leukocyte adhesion molecule defect B. Neutrophil chemotactic defect C. Neutrophil lysosomal protein defect D. Neutrophil microbicidal defect E. Neutrophil opsonization defect
Option E (Neutrophil opsonization defect) is correct. Brutons agammaglobulinemia is an X-linked disorder characterized by failure of pre-B cells to develop into B cells. This produces hypogammaglobulinemia, because there are not sufficient numbers of B cells to be antigenically stimulated to become plasma cells. Deficiency of IgG produces an opsonizing defect. The antigen recognition site of IgG attaches to the bacteria, and the Fc portion of the immunoglobulin attaches to receptors in the plasma membrane of phagocytic leukocytes (neutrophils, monocytes, macrophages). This facilitates the phagocytosis of bacteria by triggering engulfment of bacteria by pseudopods and eventual formation of a phagocytic vacuole. Option A (Leukocyte adhesion molecule defect) is incorrect. A defect in neutrophil adhesion molecules (i.e., selectins and integrins) prevents neutrophils from adhering to endothelial cells and transmigrating into tissue. Immunoglobulins are not involved in the activation of leukocyte adhesion molecules or in their function. Option B (Neutrophil chemotactic defect) is incorrect. Movement of neutrophils toward the site of acute inflammation is chemotactic. Chemical mediators (e.g., C5a, leukotriene B4) bind to neutrophil receptors causing release of calcium, which increases neutrophil motility. Immunoglobulins are not involved in neutrophil chemotaxis. Option C (Neutrophil lysosomal protein defect) is incorrect. Neutrophil lysosomal protein defects occur in ChdiakHigashi syndrome. This defect interferes with the fusion of lysosomes that contain hydrolytic enzymes with phagosomes in the cytosol of phagocytic leukocytes, producing phagolysosomes. Immunoglobulins are not involved in membrane fusion and the formation of phagolysosomes. Option D (Neutrophil microbicidal defect) is incorrect. A leukocyte defect in killing bacteria usually involves a defect in the O2-dependent myeloperoxidase (MPO) system (e.g., deficiency of nicotinamide adenosine dinucleotide phosphate (NADPH) oxidase, NADPH, or MPO deficiency). Activation of this system eventually results in the formation of

hypochlorous free radicals in phagolysosomes that kill bacteria. Immunoglobulins are not involved in the proper function of the O2-dependent MPO system.

A 52-year-old woman with a 30-pack-year history of smoking cigarettes has had a dragging sensation in the right upper quadrant and a history of weight loss for the past several months. A CT scan of the liver shows multiple mass lesions consistent with metastasis. The photograph shows a microscopic section of a biopsy of one of the mass lesions in the liver. Which of the following is the most likely primary site of origin of this cancer?

A. Bone marrow B. Lower lip C. Lung D. Midesophagus E. Sigmoid colon


Option C (Lung) is correct. The photograph shows multiple areas of stromal invasion by a well-differentiated adenocarcinoma. Notice the formation of glands lined by cells with enlarged, hyperchromatic nuclei. The gland lumens contain a scant amount of mucous secretions. Of the choices listed, the lung and the sigmoid colon are the most likely primary sites of origin for an adenocarcinoma. However, because the patient is a smoker, the most likely primary site of origin of the cancer is the lung. Primary lung cancer is overall the most common cancer that metastasizes to the liver. If the patient was a nonsmoker, colorectal cancer would be the most common site. Option A (Bone marrow) is incorrect. The most common cancer arising from the bone marrow is leukemia, which arises from bone marrow stem cells. Option B (Lower lip) is incorrect. Most cancers in the mouth area are well-differentiated squamous cell carcinomas most often related to cigarette smoking. Option D (Midesophagus) is incorrect. The most common primary cancer in the midesophagus is a squamous cell

carcinoma most often related to smoking. Option E (Sigmoid colon) is incorrect. The most common cancer of the sigmoid colon is an adenocarcinoma. Colorectal cancer also most commonly metastasizes to the liver; however, smoking is not as great a risk factor for developing this cancer as it is for cancers in the lung.

A 36-year-old woman with focal segmental glomerulosclerosis secondary to intravenous heroin abuse has chronic renal failure. The serum blood urea nitrogen (BUN) is 60 mg/dL (718) and serum creatinine is 6 mg/dL (0.61.2). An ECG shows peaked T waves. Which one of the following electrolyte profiles is most likely to be present in this woman?

Serum Na+ Serum K+ Serum Cl Serum HCO3 (135147 mEq/L) (3.55.0 mEq/L) (95105 mEq/L) (2228 mEq/L) A.128 5.9 96 20 B. 146 5.5 104 18 C. 138 2.2 114 14 D.130 2.9 80 36 E. 152 2.8 110 33 A. serum Na+ 128, serum K+ 5.9, serum Cl 96, serum HCO3 20 B. serum Na+ 146, serum K+ 5.5, serum Cl 104, serum HCO3 18 C. serum Na+ 138, serum K+ 2.2, serum Cl 114, serum HCO3 14 D. serum Na+ 130, serum K+ 2.9, serum Cl 80, serum HCO3 36 E. serum Na+ 152, serum K+ 2.8, serum Cl 110, serum HCO3 33

Option B (serum Na+ 146, serum K+ 5.5, serum Cl 104, serum HCO318) is correct. These findings are most compatible with chronic renal failure. In chronic renal failure there is tubular cell dysfunction resulting in retention of K+ (hyperkalemia with peaked T waves on an ECG) and an increased anion gap type of metabolic acidosis due to retention of organic acids like sulfuric and phosphaturic acid. The anion gap in this case is calculated as follows: anion gap = serum Na+ 146 (serum Cl 104 + serum HCO318) = 24 mEq/L (12 mEq/L + 2). Option A (serum Na+ 128, serum K+ 5.9, serum Cl 96, serum HCO3 20) is incorrect. This electrolyte profile is most compatible with mineralocorticoid deficiency (e.g., Addisons disease, 11-hydroxylase deficiency). There is inhibition of Na+ reabsorption and K+ secretion by the aldosterone-dependent Na+ and K+ channels located in the distal tubule and collecting ducts causing a hypertonic loss of Na+ in the urine (hyponatremia) and retention of K+ (hyperkalemia with peaked T waves). Due to dysfunction of the aldosterone-dependent H+/K+ ATPase pump in the collecting tubules, there is retention of H+ ions and a subsequent lack of synthesis of HCO3, which produces metabolic acidosis. The excess H+ ions combine with Cl anions producing a normal anion gap type of metabolic acidosis, calculated as follows: anion gap = serum Na+ 128 ( (serum Cl 96 + serum HCO320) = 12 mEq/L.

Option C (serum Na+ 138, serum K+ 2.2, serum Cl 114, serum HCO3 14) is incorrect. These findings are most compatible with a secretory type of diarrhea (e.g., travelers diarrhea due to enterogenic Escherichia coli, cholera). In these conditions a toxin activates adenylate cyclase in enterocytes causing the ion pumps in the small intestine to secrete isotonic fluid. Loss of isotonic fluid does not alter the serum Na+ concentration. Diarrheal fluid is rich in K+ and HCO3, the former resulting in hypokalemia and the latter a normal anion gap type of metabolic acidosis. The anion gap calculation is as follows: anion gap = serum Na+ 138 (serum Cl 114 + serum HCO314) = 10 mEq/L (12 mEq/L 2). The serum Cl is increased, because it replaces the HCO3 anions that are lost in the diarrheal fluid in order to maintain electroneutrality. Option D (serum Na+ 130, serum K+ 2.9, serum Cl 80, serum HCO3 36) is incorrect. This profile is compatible with metabolic alkalosis due to loop diuretics, thiazide diuretics, or vomiting. Thiazide and loop diuretics inhibit channels in the nephron causing decreased reabsorption of Na+. This causes a hypertonic loss of Na+ (hyponatremia) and Cl (hypochloremia). Increased delivery of Na+ to the late distal and collecting tubules results in augmented exchange of Na+ for K+ in the aldosterone-dependent Na+ channels resulting in increased urinary loss of K+ (hypokalemia with U waves on an ECG). When K+ is depleted, Na+ exchanges with H+ ions causing increased synthesis and reabsorption of HCO3 causing metabolic alkalosis. In vomiting, hydrochloric acid is lost. For every H + ion lost in the vomitus, there is a corresponding HCO3 left unneutralized in the blood causing metabolic alkalosis (HCO3> 28 mEq/L). Option E (serum Na + 152, serum K+ 2.8, serum Cl 110, serum HCO3 33) is incorrect. This profile is compatible with mineralocorticoid excess (e.g., primary aldosteronism; 11-hydroxylase deficiency). Enhanced function of aldosteronemediated Na+-H+ channels in the late distal and collecting ducts increases the synthesis of HCO3 leading hypernatremia, hypokalemia (initially exchanged with Na+ until depleted), and metabolic alkalosis (Na+ exchanges with H+; for every H+ lost in the urine there is a corresponding HCO3 entering the blood).

A young couple with fertility problems adopted a 1-year-old boy from Southeast Asia. Approximately 30 days after obtaining the child, the mother developed jaundice. Physical examination reveals tender hepatomegaly. Serum transaminases are markedly increased with serum alanine aminotransferase (ALT) higher than serum aspartate aminotransferase (AST). Fractionation of the bilirubin reveals a mixed type of hyperbilirubinemia. A urine dipstick is positive for increased bilirubin and urobilinogen. Which of the following serum serologic tests is most likely to be positive in the mother? (HAV, hepatitis A; HCV, hepatitis C; HDV, hepatitis D; HBeAg, hepatitis B e antigen; HBsAg, hepatitis B surface antigen)

A. Anti-HAV-IgM B. Anti-HCV-IgG C. Anti-HDV-IgM D. HBeAg E. HBsAg

Option A (Anti-HAV-IgM) is correct. Hepatitis A is very common in Southeast Asia and many children adopted from that country have hepatitis A who have stools containing the virus. The risk for developing hepatitis A is very high, since it is a fecal-oral transmission. It is recommended that parents receive hepatitis A vaccine before bringing their adopted child home. One would expect the presence of anti-HAV-IgM in active hepatitis A. Option B (Anti-HCV-IgG) is incorrect. Hepatitis C though present in Southeast Asia is not transmitted by contact with stool. Furthermore, anti-HCV-IgG indicates active infection or recovery from the infection. Option C (Anti-HDV-IgM) is incorrect. Hepatitis D is an incomplete virus and requires hepatitis B surface antigen to be infective. Although hepatitis B is common in Southeast Asia, it is not transmitted by contact with stool. Option D (HBeAg) is incorrect. HBeAg is the infective particle of hepatitis B. As stated above, hepatitis B is not transmitted by contact with stool. Option E (HBsAg) is incorrect. HBsAg is the first antigenic marker of a hepatitis B infection and is not infective. As stated above, hepatitis B is not transmitted by contact with stool.

A 25-year-old woman whose menstrual period has just ended complains of fever and severe lower abdominal pain. A pelvic examination shows adnexal tenderness on movement of the cervix. The photograph shows a Gram stain preparation of a creamy exudate in the exocervix. Which of the following pathogens is the most likely cause of the infection?

A. Candida albicans B. Chlamydia trachomatis C. Gardnerella vaginalis D. Neisseria gonorrhoeae E. Trichomonas vaginalis

Option D (Neisseria gonorrhoeae) is correct. The Gram stain shows neutrophils with phagocytosed gram-negative diplococci consistent with N. gonorrhoeae. The presence of fever, abdominal pain, and adnexal tenderness with

movement of the cervix (chandelier sign) suggests pelvic inflammatory disease (PID). N. gonorrhoeae is the most common cause of PID in the United States. Treatment is ceftriaxone. Option A (Candida albicans) is incorrect. C. albicans produces a white discharge containing yeast and pseudohyphae. Infection with C. albicans does not cause PID. Option B (Chlamydia trachomatis) is incorrect. C. trachomatis infects metaplastic squamous cells. It is not phagocytosed by neutrophils. However, infection with C. trachomatis does cause PID and often coexists with N. gonorrhoeae. Option C (Gardnerella vaginalis) is incorrect. G. vaginalis is a gram-negative rod that produces bacterial vaginosis where there is an alteration in the vaginal pH greater than 4.5. Option E (Trichomonas vaginalis) is incorrect. T. vaginalis is a flagellated protozoan that produces cervicitis and vaginitis.

A 50-year-old man who has a 35-pack-year history for smoking cigarettes, develops a cough, a headache, and blurry vision. Physical examination shows papilledema and coarse inspiratory crackles in both lungs. A chest radiograph shows a centrally located mass in the left hilum. An MRI shows cerebral edema but no localizing masses. A sputum cytology shows small, hyperchromatic cells and necrotic debris. Which of the following laboratory results is most likely expected?

POsm Serum ADH A.DecreasedDecreased B. DecreasedIncreased C. Increased Decreased D.Increased Increased E. Normal Normal ADH, antidiuretic hormone; POsm, plasma osmolality. A. POsm decreased, serum ADH decreased B. POsm decreased, serum ADH increased C. POsm increased, serum ADH decreased D. POsm increased, serum ADH increased E. POsm normal, serum ADH normal

Option B (POsm decreased, serum ADH increased) is correct. A decrease in POsm and an increase in serum ADH occurs in the syndrome of inappropriate ADH (SIADH), which is most often caused by ectopic secretion of ADH by a primary smallcell carcinoma of the lung (left hilar mass; small, hyperchromatic cells in the sputum). The most common cause of this

lung carcinoma is cigarette smoking. Excess ADH causes increased reabsorption of free water and a dilutional hyponatremia (decreased POsm). Water moves by osmosis from the extracellular fluid compartment into the intracellular fluid compartment, causing cerebral edema and an increase in intracranial pressure, which is manifested by papilledema (swelling of the head of the optic nerve). The coarse inspiratory crackles are caused by increased mucus in the segmental bronchi and terminal bronchioles (chronic bronchitis). Option A (POsm decreased, serum ADH decreased) is incorrect. A decrease in POsm and a decrease in serum ADH occurs in normal dilution of urine. A decrease in POsm inhibits the release of ADH, causing a loss of free water in the urine and a decrease in UOsm. Option C (POsm increased, serum ADH decreased) is incorrect. An increase in POsm and a decrease in UOsm occurs in diabetes insipidus. ADH is required to concentrate urine by reabsorbing free water from the collecting tubules of the kidneys. Therefore, absence of ADH (central diabetes insipidus) or refractoriness to ADH (nephrogenic diabetes insipidus) causes polyuria (urinary frequency) due to a loss of free water in the urine. Both types of diabetes insipidus decrease the urine osmolality and increase the POsm due to an increase in the serum sodium (hypernatremia). Option D (POsm increased, serum ADH increased) is incorrect. An increase in POsm and an increase in ADH occurs in normal concentration. An increase in POsm stimulates the release of ADH, which increases reabsorption of free water, causing an increase in UOsm. Option E (POsm normal, serum ADH normal) is incorrect. The history of cigarette smoking, mass lesion in the lung, and a cytology showing small, hyperchromatic cells is pathognomonic of a small-cell carcinoma, which commonly secretes ADH producing a dilutional hyponatremia.

A 24-year-old hiker has had diarrhea since returning from a camping trip in the Rocky Mountains 3 weeks ago. He states that his stool has a greasy consistency. The photograph shows a trichrome-stained slide of diarrheal fluid. Which of the following is the most likely diagnosis?

A. Amebiasis B. Ascariasis C. Balantidiasis D. Cryptosporidiosis

E. Giardiasis

Option E (Giardiasis) is correct. The photograph shows a trophozoite of Giardia lamblia with its prominent nuclei, median bodies, flagellae, and tapered posterior.Giardia is the most common protozoal cause of diarrhea in the United States. It is commonly contracted by drinking contaminated water from streams and lakes. Giardiasis is a common cause of chronic diarrhea and often produces fat malabsorption in association with greasy stools. The stool antigen test is the best screen for giardiasis. The treatment is metronidazole. Option A (Amebiasis) is incorrect. Amebiasis due to Entamoeba histolytica causes bloody diarrhea. The trophozoites phagocytose RBCs and do not have flagellae. Option B (Ascariasis) is incorrect. Ascariasis is caused by the nematode Ascaris lumbricoides. Adult worms cause intestinal obstruction and diarrhea. Option C (Balantidiasis) is incorrect. Balantidiasis is caused by Balantidium coli, which causes a bloody diarrhea. B. coli does not have flagellae. Option D (Cryptosporidiosis) is incorrect. Cryptosporidiosis is caused by Cryptosporidium parvum. Cryptosporidiosis is a common cause of diarrhea in patients with AIDS. The oocysts are partially acid-fast and do not have flagellae.

A 65-year-old man has fever, fatigue, hepatosplenomegaly, and generalized nontender lymphadenopathy. A complete blood cell count shows a mild normocytic anemia and thrombocytopenia, and the total WBC count is 60,000/mm3. Serum protein electrophoresis shows a flat -globulin peak. The photograph shows a representative section of the peripheral blood smear. Which of the following is the most likely diagnosis?

A. Acute myeloblastic leukemia (AML) B. Chronic lymphocytic leukemia (CLL) C. Chronic myelogenous leukemia (CML)

D. Hairy cell leukemia E. Leukemoid reaction


Option B (Chronic lymphocytic leukemia (CLL)) is correct. The clinical and laboratory findings (age > 60 yr, fatigue, hepatosplenomegaly, generalized nontender lymphadenopathy, anemia, thrombocytopenia, and an increased leukocyte count) confirm the presence of leukemia. The peripheral blood shows numerous lymphocytes and occasional smudge cells (nuclear debris) and the absence of neutrophils, findings consistent with CLL. The lymphocytes are neoplastic virgin B cells that are unable to mature into plasma cells. This explains the hypogammaglobulinemia (flat globulin peak on serum protein electrophoresis), which imposes an increased risk of infections. CLL is the most common leukemia and the most common cause of generalized lymphadenopathy in patients > 60 years of age. Option A (Acute myeloblastic leukemia (AML)) is incorrect. AML occurs between 15 and 60 years of age. A peripheral blood smear of patients with AML shows myeloblasts containing Auer rods (red, splinter to rod-shaped inclusions in the cytosol). Option C (Chronic myelogenous leukemia (CML)) is incorrect. CML usually occurs between 40 and 60 years of age. The peripheral smear shows numerous myeloid cells at different stages of development (e.g., myeloblasts, myelocytes, segmented neutrophils). Option D (Hairy cell leukemia) is incorrect. Hairy cell leukemia occurs in middle-aged men. It is an uncommon chronic B-cell leukemia in which lymphadenopathy rarely occurs. In a patient with hairy cell leukemia, a peripheral blood smear shows neoplastic cells with cytoplasmic projections, which stain positive for tartrate-resistant acid phosphatase. Option E (Leukemoid reaction) is incorrect. A leukemoid reaction is a benign increase in leukocytes (e.g., neutrophils, lymphocytes, eosinophils), often >50,000/mm3. It usually occurs in response to infection (e.g., tuberculosis). Anemia and thrombocytopenia usually are not present.

Which of the following best characterizes a major crossmatch for a red blood cell transfusion? A. Guarantees survival of infused donor RBCs B. Negative patient antibody screen usually correlates with a compatible crossmatch C. Prevents febrile transfusion reactions D. Prevents formation of antibodies against donor blood group antigens E. Prevents transmission of infectious diseases
Option B (Negative patient antibody screen usually correlates with a compatible crossmatch) is correct. An antibody screen (indirect Coombs test) detects atypical antibodies that could potentially attack donor RBC antigens. If the antibody screen is negative, then the major crossmatch (recipient serum + donor RBCs) should be compatible.

Option A (Guarantees survival of infused donor RBCs) is incorrect. All packed RBC transfusions contain a small amount of plasma containing the natural antibodies of the recipient. Therefore if packed O RBCs are given to an A, B, or AB blood group patient, donor anti A-IgM, anti B-IgM, and anti A and/or B-IgG antibodies can attach to recipient RBCs leading to a minimal degree of macrophage destruction of the recipient RBCs. Option C (Prevents febrile transfusion reactions) is incorrect. Febrile transfusion reactions are due to recipient antihuman leukocyte antigen antibodies reacting against donor leukocytes (type II hypersensitivity reaction). Destruction of donor leukocytes releases pyrogens causing fever. The antibody screen does not detect these types of antibodies; therefore, there is no protection against developing a febrile transfusion reaction. Option D (Prevents formation of antibodies against donor blood group antigens) is incorrect. With the possible exception of identical twins, no two individuals have exactly the same RBC antigen profile. Therefore, a recipient is exposed to foreign antigens from donor RBCs and can develop antibodies against one or more of them. Option E (Prevents transmission of infectious diseases) is incorrect. Although donor blood is screened for most of the important infectious diseases, some donors may be in the incubation phase of an infection and not have antibodies present in the blood that can be detected.

A 52-year-old woman had a 10-year history of chronic left-sided and right-sided heart failure and, most recently, pulmonary hypertension. As a child and young adult, she had numerous episodes of pharyngitis. At autopsy, the heart showed thickening of the mitral valve leaflets and fusion of the commissures. There was thickening of the right ventricular wall and atherosclerosis of the pulmonary artery. The pulmonary vessels prominently protruded from the cut surface of the lung parenchyma. Which of the following is the most likely cause of the valvular disease?

A. Ischemic heart disease B. Libman-Sacks endocarditis C. Mitral valve prolapse (MVP) D. Recurrent bacterial endocarditis E. Recurrent rheumatic fever
Option E (Recurrent rheumatic fever) is correct. The patient had chronic recurrent rheumatic fever and developed mitral stenosis. Recall that in rheumatic fever antibodies directed against the M proteins of certain strains of group A streptococci cross-react with antigens present in cardiac valves (most commonly the mitral valve) and other tissues. Repeated valve inflammation leads to repair by fibrosis, dystrophic calcification, and eventual stenosis of the valve. This leads to atrial dilation and hypertrophy and an increase in pulmonary venous pressure resulting in pulmonary edema and pulmonary venous hypertension. Increased pulmonary vein pressure caused right ventricular hypertrophy and right-sided heart failure. The combination of pulmonary hypertension and right ventricular hypertrophy is called cor pulmonale. Gross signs of pulmonary hypertension are atherosclerosis of the pulmonary artery and prominent vessels

on cut sections of the lungs. Option A (Ischemic heart disease) is incorrect. Ischemic heart disease produces heart failure with dilation of the mitral and tricuspid valve rings leading to regurgitation murmurs; however, there is no damage to the valves leading to mitral or tricuspid stenosis. Option B (Libman-Sacks endocarditis) is incorrect. Libman-Sacks endocarditis is the most common valvular disease in systemic lupus erythematosus. Sterile vegetations occur on the mitral valve and chordae leading to valve dysfunction characterized by mitral regurgitation rather than mitral stenosis. Option C (Mitral valve prolapse (MVP)) is incorrect. MVP, where there is redundancy of the valve tissue due to myxomatous degeneration, is more likely to be associated with mitral regurgitation than mitral stenosis. Option D (Recurrent bacterial endocarditis) is incorrect. Recurrent bacterial endocarditis is more likely to produce mitral regurgitation rather than mitral stenosis, because the bacteria destroy the valve leaflet and often cause rupture of the chordae tendineae.

A 42-year-old man has hypertension and dependent pitting edema. Urinalysis shows fatty casts with Maltese crosses visible with polarization. Proteinuria > 3.5 g/24 hr. The photograph shows an H&E stain of a representative glomerulus in a kidney biopsy. What is the most likely diagnosis?

A. Diffuse membranous glomerulopathy B. Focal segmental glomerulosclerosis C. Goodpasture syndrome D. IgA glomerulopathy

E. Minimal change disease

Option A (Diffuse membranous glomerulopathy) is correct. The biopsy shows uniformly thickened glomerular basement membranes with no increase in the number of nuclei. The patient has diffuse membranous glomerulopathy, the second most common cause of the nephrotic syndrome in adults. Signs of the nephrotic syndrome in this patient include dependent pitting edema (due to hypoalbuminemia), proteinuria > 3.5 g/24 hr, and fatty casts in the urine. Membranous glomerulopathy is an immunocomplex disorder with deposition of immunocomplexes in a subepithelial location. These are often visible as epimembranous spikes with silver stains of a glomerulus. Option B (Focal segmental glomerulosclerosis) is incorrect. Focal segmental glomerulosclerosis is most often associated with AIDS and with intravenous heroin addiction. It causes the nephrotic syndrome. There is focal sclerosis within the glomerulus. Membrane thickening is not present. Option C (Goodpasture syndrome) is incorrect. Goodpasture syndrome is associated with antibasement membrane antibodies directed against the pulmonary capillary and glomerular capillary basement membranes. It is associated with a nephritic presentation (not nephrotic), which includes hematuria, RBC casts, and mild proteinuria. Option D (IgA glomerulopathy) is incorrect. IgA glomerulopathy is an immunocomplex type of glomerulopathy associated with episodic bouts of microscopic or macroscopic hematuria. Histologic sections show a mesangioproliferative process (not thickened basement membranes). The glomerulopathy is most often nephritic and less commonly nephrotic in its presentation. Option E (Minimal change disease) is incorrect. Minimal change disease (lipoid nephrosis) is the most common cause of the nephrotic syndrome in children. It is due to a loss of negative charges in the glomerular basement membrane. There is a selective loss of albumin in the urine. Glomerular histologic features are normal.

A 55-year-old woman with a 5-year history of alcohol abuse has epigastric pain that radiates into the back. A left-sided pleural effusion is present on a chest x-ray. Which of the following laboratory test results is most likely to be present?

A. Decreased serum immunoreactive trypsin B. Decreased serum glucose C. Decreased total WBC count D. Increased serum calcium E. Increased serum lipase

Option E (Increased serum lipase) is correct. The patient has acute pancreatitis and a pleural effusion on the left. Serum lipase is a more specific test for acute pancreatitis than serum amylase, which may be normal in the serum after 2 to 4 days. Lipase initially increases in 3 to 6 hours; peaks in 12 to 30 hours, and returns to normal in 7 to 14 days. Regarding the left-sided pleural effusion, it is also a potential complication of acute pancreatitis. It is an exudate with increased protein with a pleural fluid (PF) protein/serum protein > 0.5, PF LDH/serum lactate dehydrogenase (LDH) > 0.6, and a PF LDH > 200 U/L. PF amylase is also increased. Option A (Decreased serum immunoreactive trypsin) is incorrect. Serum immunoreactive trypsin is increased (not decreased) in 95% to 100% of cases of acute pancreatitis. It is decreased in chronic pancreatitis. Option B (Decreased serum glucose) is incorrect. In severe acute pancreatitis, as in this case, serum glucose is frequently increased (not decreased) due to destruction of -islet cells in the pancreas. Option C (Decreased total WBC count) is incorrect. In acute pancreatitis, the total WBC count is usually increased (neutrophilic leukocytosis) and a left-shift is present (increased number of band neutrophils). Option D (Increased serum calcium) is incorrect. In severe acute pancreatitis, as in this case, the serum calcium is frequently decreased due to enzymatic fat necrosis in the pancreas (calcium binds to fatty acids).

A 7-year-old boy has recurrent infections caused by Staphylococcus aureus. Blood studies show that the peripheral blood leukocytes are normal. A leukocyte nitroblue tetrazolium (NBT) test is abnormal. Which of the following best describes the pathogenesis of this disease?

A. Defect in phagolysosome formation B. Defect in producing hypochlorous acid C. Deficiency of complement C3 D. Deficiency of immunoglobulins E. Deficiency of myeloperoxidase (MPO)
Option B (Defect in producing hypochlorous acid) is correct. The patient has chronic granulomatous disease (CGD). The most common variant of CGD is X-linked recessive and is caused by a deficiency of nicotinamide adenosine dinucleotide phosphate (NADPH) oxidase. This is the key enzyme in the O2-dependent MPO system, which is the most effective microbicidal system in neutrophils and monocytes. In this system, NADPH oxidase, using NADPH as a cofactor, converts molecular O2 to superoxide free radicals. This reaction releases energy called the respiratory or oxidative burst. Superoxide free radicals are normally converted to peroxide, which is then combined with chloride ions by MPO to produce hypochlorous acid to kill bacteria. A small amount is converted by iron into hydroxyl free radicals. The NBT test detects whether the respiratory burst is functioning. A colorless dye is added to a test tube filled with neutrophils. The dye is phagocytosed and converted to a blue color if the respiratory burst is intact. This test has largely been replaced by one involving the oxidation of dihydrorhodamine to fluorescent rhodamine. In CGD, the respiratory burst is absent;

hence, the abnormal leukocyte NBT test. Peroxide is also absent; therefore, the cells cannot synthesize hypochlorous acid to kill bacteria. S. aureus is the primary cause of death in CGD. S. aureus is catalase-positive, and catalase degrades H2O2. Therefore, any H2O2 produced by the bacteria in the phagolysosome that could potentially be used to synthesize hypochlorous free radicals (HOCl) is neutralized and the bacteria cannot be killed. Bacteria such as Streptococcus pyogenes are catalase negative; hence, when they supply the missing peroxide in the phagolysosome, hypochlorous acid is generated and the bacteria are killed. Option A (Defect in phagolysosome formation) is incorrect. Phagolysosomes are formed when lysosomes merge with phagosomes. In Chdiak-Higashi syndrome, a lysosomal protein defect prevents the formation of phagolysosomes. Affected individuals have leukocytes containing large azurophilic granules (lysosomes) in the cytosol. Option C (Deficiency of complement C3) is incorrect. Complement C3b is an opsonizing agent that binds to the surface of bacteria and assists leukocytes in phagocytosis and the formation of phagosomes. Option D (Deficiency of immunoglobulins) is incorrect. IgG is an opsonizing agent that binds to the surface of bacteria and assists leukocytes in phagocytosis and the formation of phagosomes. Option E (Deficiency of myeloperoxidase (MPO)) is incorrect. Deficiency of MPO causes decreased synthesis of HOCl, which causes a defect in microbicidal activity. However, the respiratory (oxidative) burst is intact, the NBT test is normal, and H2O2 is produced in phagolysosomes.

An Rh-positive 25-year-old woman with blood group O delivers an Rh-negative infant with blood group A. The infant develops jaundice 8 hours after birth. An antibody screen of the mother is negative. Laboratory studies show that the infant has a mild normocytic anemia and an unconjugated hyperbilirubinemia. A direct Coombs test of the cord blood is positive. A peripheral blood smear shows spherocytes. Which of the following is the most likely diagnosis?

A. ABO hemolytic disease of the newborn B. Glucose-6-phosphate dehydrogenase (G6PD) deficiency C. Hereditary spherocytosis D. Pyruvate kinase deficiency E. Rh hemolytic disease of the newborn

Option A (ABO hemolytic disease of the newborn) is correct. The most common cause of jaundice within the first 24 hours after birth is ABO hemolytic disease of the newborn. Individuals with blood group O normally have anti-A and anti-B antibodies of the IgM type as well as anti-A,B antibodies of the IgG type. Therefore, ABO incompatibility occurs in pregnant women who are blood group O and carrying fetuses with blood group A or blood group B. Maternal anti-A,B-IgG antibodies cross the placenta and attach to the A or B

antigens of the fetal RBCs. Fetal macrophages in the spleen phagocytose and destroy the IgG-coated fetal RBCs, causing the release of unconjugated bilirubin (macrophage end-produce of bilirubin degradation) into the fetal blood and a mild hemolytic anemia. The unconjugated bilirubin is removed by the mothers liver. In this neonate, the liver is unable to metabolize the increased unconjugated bilirubin, causing jaundice within the first 24 hours. A direct Coombs test of the RBCs is positive, because IgG antibodies coat the surface of the RBCs. The peripheral blood contains numerous spherocytes, which are produced when splenic macrophages remove part of the RBC membrane coated by the IgG antibodies rather than the entire RBC. In most cases, an exchange transfusion is unnecessary in treating ABO hemolytic disease of the newborn, because the degree of anemia is not very severe and the level of unconjugated bilirubin is not high enough to produce kernicterus. Option B (Glucose-6-phosphate dehydrogenase (G6PD) deficiency) is incorrect. G6PD deficiency is an Xlinked recessive disorder that produces an intravascular hemolysis caused by hydrogen peroxideinduced damage of RBC membranes and hemoglobin. G6PD deficiency is not an antibody-mediated hemolysis; therefore, the direct Coombs test is negative, not positive. Option C (Hereditary spherocytosis) is incorrect. Hereditary spherocytosis is an autosomal dominant disorder caused by a defect in ankyrin in the cell membrane. Hereditary spherocytosis is not an antibodymediated hemolysis; therefore, the direct Coombs test is negative, not positive. Option D (Pyruvate kinase deficiency) is incorrect. Pyruvate kinase deficiency is an autosomal recessive hemolytic anemia. In pyruvate kinase deficiency, RBCs have no adenosine triphosphate, causing dehydration of the RBCs due to damage to the RBC membrane. The RBCs have spiny protuberances from the surface. Splenic macrophages remove the damaged RBCs causing a hemolytic anemia with jaundice. Pyruvate kinase deficiency is not an antibody-mediated hemolysis; therefore, the direct Coombs test is negative, not positive. Option E (Rh hemolytic disease of the newborn) is incorrect. Rh hemolytic disease of the newborn occurs when Rh-positive infants are born to Rh-negative (D-antigen negative) mothers. The neonate, in this case, is Rh negative.

A 35-year-old has a 3-month history of epigastric pain without radiation into the back. The stool is black colored and is positive for blood. He has a past history of gastric ulcer disease due to Helicobacter pylori. Which of the following tests will identify the cause of his pain and also provide an indicator on whether treatment is effective?

A. Radiolabeled urea breath test B. Serologic test C. Stool antigen test D. Urease test on a gastric biopsy
Option C (Stool antigen test) is correct. The stool antigen test detects pyloroantigens. It is the least expensive test and helps support a diagnosis of infection and determine whether treatment is effective. The sensitivity is 92% and specificity 90100%.

Option A (Radiolabeled urea breath test) is incorrect. H. pylori converts orally administered labeled urea to labeled CO2, which is detected in the breath. It is an expensive test, when compared to the stool antigen test. It has a sensitivity > 97% and a specificity > 90%. Option B (Serologic test) is incorrect. This does not differentiate current or past infection. The patient has a history of H. pyloriinduced peptic ulcer disease. The test also does not determine whether treatment has been effective. Option D (Urease test on a gastric biopsy) is incorrect. This is an expensive test that has excellent sensitivity (90%) and specificity (99100%); however, it would have to be repeated to see if treatment was effective.

A 22-year-old woman has not menstruated for the past 6 months. Physical examination is unremarkable except for the finding noted in the photograph. A pregnancy test is negative. Serum thyroid-stimulating hormone (TSH) is normal. Which of the following is the most likely diagnosis?

A. Cushing syndrome B. Graves disease C. Hypopituitarism D. Primary hypothyroidism E. Prolactinoma


Option E (Prolactinoma) is correct. The patient has a prolactinoma, the most common functioning tumor of the pituitary gland. Excess prolactin suppresses the release of gonadotropin-releasing hormone from the hypothalamus. This, in turn, causes secondary amenorrhea (absent menses) due to decreased release of follicle-stimulating hormone (FSH) and luteinizing hormone (LH) by the anterior pituitary gland. Prolactin also stimulates milk production in the breasts, causing galactorrhea (see photograph). Prolactinomas are treated either by surgery or by using dopamine analogues that inhibit prolactin secretion by the tumor (e.g., bromocriptine, cabergoline). Option A (Cushing syndrome) is incorrect. Cushing syndrome is characterized by signs of hypercortisolism (e.g., truncal obesity with purple stria, hypertension), which are not present in this patient. Furthermore, hypercortisolism does not cause the typical findings (amenorrhea and galactorrhea) that are present in this patient.

Option B (Graves disease) is incorrect. Graves disease is characterized by exophthalmos and thyromegaly, which are not present in this patient. Excess thyroid hormone causes menstrual irregularities (usually oligomenorrhea) but not galactorrhea. Option C (Hypopituitarism) is incorrect. Hypopituitarism is associated with amenorrhea (decreased FSH and LH). However, other expected findings of anterior pituitary hypofunction (e.g., decreased serum TSH producing secondary hypothyroidism) are not present in this patient. Option E (Primary hypothyroidism) is incorrect. Primary hypothyroidism usually causes menorrhagia (not amenorrhea). It may also cause galactorrhea, because a decrease in serum thyroxine increases both serum TSH and thyrotropinreleasing hormone. Thyrotropin-releasing hormone is a potent stimulator of prolactin release. However, the serum TSH is normal in this patient and should be increased in primary hypothyroidism.

The photograph shows a child, who has primary amenorrhea. Physical examination shows discrete masses in both inguinal canals. Speculum examination of the vagina shows that it ends as a blind pouch. Which of the following laboratory findings is most likely present?

A. Buccal smear with one Barr body B. Chromosome study with XO genotype C. Chromosome study with XY genotype D. Decreased serum testosterone E. Increased serum gonadotropins
Option C (Chromosome study with XY genotype) is correct. The patient has testicular feminization syndrome (androgen insensitivity syndrome), which is an X-linked recessive disorder with absence of the androgen receptors. Because the patient is genotypically a male (XY), but phenotypically looks female (see photograph), this is an example of male pseudohermaphroditism. The testes are present and either remain in the abdominal cavity or present as masses in

the inguinal canal. Concentrations of testosterone and dihydrotestosterone (DHT) are normal, but the lack of androgen receptors prevents the development of male secondary sex characteristics and external genitalia. Testosterone effects normal development of the epididymis, seminal vesicles, and vas deferens from the mesonephric duct. DHT converts female-appearing external genitalia into a penis and scrotal sac and develops the prostate gland. The Sertoli cells in the seminiferous tubules produce mllerian inhibitory factor, which causes mllerian structures (fallopian tubes, uterus, cervix, upper one-third of vagina) to undergo apoptosis. In testicular feminization, due to a lack of testosterone effect, there is no epididymis, seminal vesicles, and vas deferens. Due to a lack of DHT effect, there is no prostate gland; the vagina ends as a blind pouch (lower two-thirds of the vagina develops from the urogenital sinus), and the external genitalia remains female. Breast development and female secondary sex characteristics are normal, because estrogen receptors are present. Patients are usually reared as females and both testes are removed to reduce the risk for developing a seminoma. Option A (Buccal smear with one Barr body) is incorrect. One Barr body (inactivated X chromosome) is present in a normal female. The patient is a genotypic male (male pseudohermaphrodite); therefore, there are no Barr bodies. Option B (Chromosome study with XO genotype) is incorrect. An XO genotype occurs in Turners syndrome. The ovaries lack germinal follicles; therefore, estrogen is not produced, resulting in primary amenorrhea and undeveloped female sex characteristics. Patients are short statured and usually have a webbed neck from a lymphatic cyst. Option D (Decreased serum testosterone) is incorrect. In this patient, testosterone levels are normal (negative feedback with luteinizing hormone [LH]). Option E (Increased serum gonadotropins) is incorrect. In this patient, inhibin levels from the Sertoli cells are normal (negative feedback with follicle-stimulating hormone [FSH]), therefore, FSH levels are normal. Testosterone is normal; however, serum LH is slightly increased.

The photograph shows an aorta that was removed at surgery from a 69-year-old man with a long history of peripheral vascular disease and ischemic heart disease. Which of the following mechanisms is most likely involved in the pathogenesis of the aortic lesion?

A. Atherosclerosis B. Cystic medial degeneration C. Defect in collagen D. Defect in fibrillin E. Infection


Option A (Atherosclerosis) is correct. The photograph shows a large abdominal aortic aneurysm just above the aortic bifurcation. The aorta is dilated, and fissured atheromatous plaques are located above and below the aneurysm. Most of the aneurysmal sac is filled with a laminated thrombus. The probe marks the site of rupture of the aneurysm. The

pathogenesis of abdominal aortic aneurysms is multifactorial, but atherosclerosis is the primary mechanism causing weakness of the vessel wall and its eventual aneurysmal dilation. Atherosclerosis involving the peripheral arteries (e.g., femoral artery) is invariably present. Rupture is the most common complication, as noted in this picture. The classic rupture triad is a sudden onset of left flank pain (blood in the retroperitoneal space), followed by hypotension (due to hypovolemic shock), and a pulsatile mass that is palpated during abdominal examination. Option B (Cystic medial degeneration) is incorrect. Cystic medial degeneration refers to elastic tissue fragmentation with subsequent formation of cystic spaces filled with glycosaminoglycans. It causes weakening of the media of elastic arteries and is the primary mechanism for aortic dissection rather than an abdominal aortic aneurysm. Option C (Defect in collagen) is incorrect. Although defective connective tissue may play a role in the pathogenesis of an abdominal aortic aneurysm, it is not the primary mechanism for weakening of the abdominal aorta resulting in the formation of an aneurysm. Option D (Defect in fibrillin) is incorrect. A defect in fibrillin synthesis is present in Marfan syndrome. Fibrillin is the primary component of elastic tissue. Weakening of elastic tissue in the aorta predisposes to cystic medial degeneration, which, in turn, produces an aortic dissection rather than an abdominal aortic aneurysm. Option E (Infection) is incorrect. Vessel wall weakening with aneurysm formation that is due to an infection is called a mycotic aneurysm. Although the term implies infection by a vessel-invading fungi (e.g., Aspergillus, Candida, Mucor species), it can also be vessel invasion secondary to bacteria, such as Bacteroides fragilis, Pseudomonas aeruginosa, and Salmonella species. Clinical findings include thrombosis with or without infarction and rupture. It does not produce an abdominal aortic aneurysm.

A 42-year-old man has a positive PPD test and is currently being treated prophylactically with isoniazid. Laboratory studies show a severe hypochromic and microcytic anemia. The photograph shows a bone marrow aspirate stained with Prussian blue for iron. Which of the following laboratory findings is most likely to be reported?

A. Decreased serum ferritin B. Decreased serum iron

C. Increased serum free erythrocytic protoporphyrin D. Increased serum total iron-binding capacity E. Increased serum transferrin saturation
Option E (Increased serum transferrin saturation) is correct. This patient has a sideroblastic anemia caused by a deficiency of pyridoxine (vitamin B6). The most common cause of pyridoxine deficiency is the use of isoniazid (INH) in the treatment of tuberculosis. INH produces the deficiency by forming a complex with pyridoxine. In general, sideroblastic anemias are caused by defects in heme synthesis within the mitochondria of developing normoblasts in the bone marrow. Pyridoxine is a cofactor with -aminolevulinic acid (ALA) synthase, which combines succinyl CoA with glycine to form -ALA, the first step in heme synthesis in the mitochondria. Therefore, the synthesis of heme (iron + protoporphyrin) is decreased causing a corresponding decrease in hemoglobin synthesis resulting in a microcytic hypochromic anemia. Since iron entering the developing normoblasts is not being used to synthesize heme, it accumulates in the mitochondria to produce ringed sideroblasts, the hallmark of a sideroblastic anemia. The bone marrow aspirate shows normoblasts with dark blue granules of iron within mitochondria arranged around the nuclei. Many of these iron-laden cells die within the bone marrow, leading to excess iron stores in the bone marrow macrophages and a corresponding increase in serum iron and the percentage saturation of transferrin, the binding protein of iron. Option A (Decreased serum ferritin) is incorrect. Excess iron delivered to bone marrow macrophages in the sideroblastic anemias causes increased binding of the iron to ferritin, a soluble iron-binding protein synthesized by the macrophages. An increase in ferritin stores in the macrophages automatically leads to an increase (not a decrease) in serum ferritin. Option B (Decreased serum iron) is incorrect. Excess iron stores in the sideroblastic anemias leads to a greater percentage of iron transferred from macrophages to transferrin. The iron that is bound to transferrin is measured as the serum iron, which is increased (not decreased) in the sideroblastic anemias. Option C (Increased serum free erythrocytic protoporphyrin) is incorrect. In this patient, the sideroblastic anemia is caused by a deficiency of pyridoxine (vitamin B6), which causes a decrease in RBC protoporphyrin (not increase) and the other intermediates required to synthesize heme. Option D (Increased serum total iron-binding capacity) is incorrect. Excess iron stores in the bone marrow macrophages (i.e., ferritin stores) is a signal for the liver to decrease the synthesis of transferrin, the binding protein of iron. Since transferrin directly correlates with total iron-binding capacity (TIBC), the TIBC is decreased (not increased) in all iron overload diseases. This should not be confused with percent saturation of transferrin (number of binding sites occupied by iron), which is increased in all iron overload diseases. Hence, in iron overload diseases, serum iron is increased, serum TIBC is decreased, serum percent saturation of transferrin is increased, and serum ferritin is increased.

The photograph shows painless and nonpruritic lesions on the left side of the neck of a 25-year-old man with AIDS. Similar lesions are also present on the hard palate. Which of the following is the most likely causal organism?

A. Bartonella henselae B. Cytomegalovirus (CMV) C. Epstein-Barr virus D. Human herpesvirus 8 E. HIV


Option D (Human herpesvirus 8) is correct. The lesions shown on the neck are typical of Kaposi sarcoma, a vascular malignancy closely associated with human herpesvirus 8. Kaposis sarcoma is the most common malignancy in patients with AIDS and is an AIDS-defining lesion. Lesions appear most often on the skin but may also occur in the intestinal tract, particularly on the hard palate. Option A (Bartonella henselae) is incorrect. B. henselae is a gram-negative bacterium that causes bacillary angiomatosis, a disease that occurs almost exclusively in patients with AIDS. It produces highly vascular skin lesions that can mimic the lesions of Kaposis sarcoma. Systemic signs of the infection include fever, lymphadenopathy, and hepatomegaly. Option B (Cytomegalovirus (CMV)) is incorrect. CMV is not an oncogenic virus and does not produce vascular lesions on the skin or in the gastrointestinal tract. In patients with AIDS, CMV is the most common cause of blindness, biliary tract disease, and pancreatitis. Option C (Epstein-Barr virus) is incorrect. In patients with AIDS, Epstein-Barr virus does not cause vascular skin lesions, although it may cause hairy leukoplakia (glossitis), primary central nervous system lymphoma, and Burkitts lymphoma. Option E (HIV) is incorrect. HIV is not oncogenic and does not produce vascular skin lesions. In patients with AIDS, it is associated with generalized lymphadenopathy, destruction of CD4 helper T cells, and many central nervous system findings (e.g., AIDS dementia).

A 48-year-old man has fever, weight loss, sweating, and a dragging sensation in the abdomen. Physical examination shows generalized lymphadenopathy and massive hepatosplenomegaly. Laboratory studies reveal a moderately severe normocytic anemia, thrombocytopenia, and a WBC count of 110,000/mm3. A bone marrow biopsy shows hypercellularity and the presence of neutrophils at all stages of development. Less than

2&percnt; of the WBCs in the bone marrow are myeloblasts. The photograph shows a representative section of the bone marrow smear. Which of the following laboratory test findings is most likely to be present?

A. Increased leukocyte alkaline phosphatase score B. Leukocytes positive for tartrate-resistant acid phosphatase C. Leukocytes positive for Philadelphia chromosome D. Lymphocytes positive for CD10 antigen E. Lymphocytes positive for terminal deoxynucleotidyl transferase
Option C (Leukocytes positive for Philadelphia chromosome) is correct. This patient has chronic myelogenous leukemia (CML). CML occurs in patients between 40 and 60 years of age. It is caused by translocation of the ABL proto-oncogene on chromosome 9 to chromosome 22 (Philadelphia chromosome), where it forms a fusion gene with the break cluster region. The Philadelphia chromosome is present in >95% of patients with CML; however, detection of the fusion gene has much greater specificity for confirming the diagnosis. The presence of neutrophils in all stages of development and a myeloblast count of <10% in the bone marrow indicate a chronic, not an acute leukemia. The myeloblasts in CML do not contain Auer rods (red splinter to rod-shaped inclusion) even when CML transforms into an acute leukemia. The bone marrow smear shows a marked increase in the number of leukocytes and basophils (cells with dark granules). Segmented and band neutrophils are prominent, as are myelocytes and metamyelocytes. Normocytic anemia and thrombocytopenia (sometimes thrombocytosis) commonly occur in CML. CML commonly metastasizes to the lymph nodes (lymphadenopathy), liver, and spleen (hepatosplenomegaly). Option A (Increased leukocyte alkaline phosphatase score) is incorrect. Benign neutrophils contain alkaline phosphatase in the cytoplasmic granules; however, neoplastic neutrophils do not contain this enzyme. Therefore, the leukocyte alkaline phosphatase score is usually zero in CML. Option B (Leukocytes positive for tartrate-resistant acid phosphatase) is incorrect. Leukemia cells positive for tartrateresistant acid phosphatase are present in hairy cell leukemia, which is a B-cell leukemia. The leukemia cells have hairlike cytoplasmic projections in the peripheral blood and do not contain Auer rods. Option D (Lymphocytes positive for CD10 antigen) is incorrect. CD10 in lymphocytes is the marker for the common acute lymphoblastic leukemia antigen, which is present in early preB-cell types of acute lymphoblastic leukemia.

Option E (Lymphocytes positive for terminal deoxynucleotidyl transferase) is incorrect. Terminal deoxynucleotidyl transferase is present in early preB-cell and T-cell types of acute lymphoblastic leukemia.

A febrile 30-year-old woman with systemic lupus erythematosus (SLE) has a 4-day history of precordial chest pain that increased when she leaned back and decreased when she leans forward. She now complains of progressively worsening dyspnea. Physical examination shows muffled heart sounds and neck vein distention during inspiration. Blood pressure and pulse amplitude both decrease during inspiration. What is the most likely diagnosis?

A. Aortic dissection B. Constrictive pericarditis C. Dilated cardiomyopathy D. Pericarditis with effusion E. Restrictive cardiomyopathy
Option D (Pericarditis with effusion) is correct. Pericarditis with effusion is the most common cardiac manifestation of SLE. Inflammation of serosal epithelium (e.g., pericardium, pleura) is a characteristic finding in SLE. Immunocomplexes directed against the pericardium cause increased vessel permeability and leakage of a fibrinous exudate onto the serosal surface, producing a friction rub heard over the precordium during filling and emptying of the heart (see Fig. 2-7 in Rapid Review Pathology, 3rd edition). Fluid accumulates in the pericardial sac producing a water bottle configuration on a chest x-ray (see Fig. 10-24 in Rapid Review Pathology, 3rd edition). This prevents filling of the right side of the heart during inspiration (neck vein distention), when venous blood is normally drawn into the right side of the heart by the increasing negative intrathoracic pressure. Decreased right-sided filling causes a decrease in cardiac output, resulting in decreased pulse amplitude and blood pressure during inspiration as well; this is called pulsus paradoxus. The pain caused by inflammation of the pericardium is aggravated by leaning back and is relieved by leaning forward. Fluid must be removed from the pericardial sac by paracentesis or death will occur. Option A (Aortic dissection) is incorrect. Aortic dissection occurs in elderly men with hypertension or young individuals with defects in collagen (Ehlers-Danlos syndrome) or elastic tissue (Marfan syndrome). Blood enters an intimal tear and dissects proximally and/or distally through the weakened media of the aorta. Pain radiates into the back and is often associated with absent upper extremity pulses. Option B (Constrictive pericarditis) is incorrect. Constrictive pericarditis is most often idiopathic or a complication of tuberculosis. Incomplete filling of the cardiac chambers is caused by thickening of the parietal pericardium. Constrictive pericarditis is not a complication of the pericarditis associated with SLE. Option C (Dilated cardiomyopathy) is incorrect. Dilated cardiomyopathy is associated with an enlarged heart with dilation of atria and ventricles as well as biventricular congestive heart failure. Signs of right-sided heart failure (e.g.,

dependent pitting edema) or left-sided heart failure (e.g., pulmonary edema) are not present in this patient. Option E (Restrictive cardiomyopathy) is incorrect. Restrictive cardiomyopathy is characterized by decreased ventricular compliance (decreased filling of the heart) caused by an infiltrative disease of the myocardium (e.g., amyloid). Pericardial effusions are not usually present.

A 35-year-old man has an intermittent 12-year history of crampy pain in the left lower quadrant and bloody diarrhea. The photograph shows a portion of large bowel. Which of the following best characterizes this disease?

A. Increased incidence of colorectal cancer B. Noncaseating granulomas C. Obstruction D. Terminal ileal involvement E. Transmural inflammation
Option A (Increased incidence of colorectal cancer) is correct. This patient has ulcerative colitis. The colectomy specimen shows diffuse ulceration of the mucosal surface and residual islands of inflamed, hemorrhagic mucosa (pseudopolyps). These gross findings, plus the history of recurrent bloody diarrhea, are associated with ulcerative colitis. Ulcerative colitis is a chronic ulceroinflammatory disease that targets individuals between 20 and 25 years of age. An increased incidence of colorectal cancer correlates with long duration of the disease (>10 years), early onset (e.g., <20 years of age), and extent of the disease (pancolitis has the greatest risk). Option B (Noncaseating granulomas) is incorrect. Crohns disease is an inflammatory bowel disease characterized by chronic granulomatous ulceroconstrictive disease that mainly targets the terminal ileum. It produces inflammation involving the entire thickness of the bowel. Bloody diarrhea is uncommon. Option C (Obstruction) is incorrect. Crohns disease produces transmural inflammation that leads to obstruction. Ulcerative colitis produces ulceration of the mucosa and submucosa; therefore, obstruction is unlikely.

Option D (Terminal ileal involvement) is incorrect. Terminal ileal involvement is characteristic of Crohns disease. Ulcerative colitis is limited to the large bowel. Option E (Transmural inflammation) is incorrect. Transmural inflammation involving the terminal ileum causes narrowing of the lumen and signs of obstruction (colicky pain) in the right lower quadrant. This is a characteristic finding in Crohns disease. Ulcerative colitis has mucosal and submucosal ulceration.

A 20-year-old man requires an extraction of an impacted wisdom tooth. He is not taking any prescription or over-thecounter-drugs. He has no previous history of surgery. There is a history of bleeding problems in his mothers family, but he is not sure which individuals had the problems. Shortly after the tooth extraction, he rinses his mouth out with lukewarm water and develops severe bleeding from the tooth socket that cannot be controlled. Which of the following sets of hemostasis studies is most likely to be present in this patient?

Platelet CountBleeding TimePT PTT A.Normal Normal B. Normal Normal Normal C. D.Normal E. Normal Normal PT, prothrombin time; PTT, partial thromboplastin time.

A. platelet count normal, bleeding time increased, PT normal, PTT increased B. platelet count normal, bleeding time normal, PT normal, PTT increased C. platelet count decreased, bleeding time increased, PT increased, PTT increased D. platelet count normal, bleeding time increased, PT increased, PTT increased E. platelet count normal, bleeding time normal, PT increased, PTT increased
Option B (platelet count normal, bleeding time normal, PT normal, PTT increased) is correct. The patient has mild hemophilia A, and X-linked recessive disorder with deficiency of VIII:C activity in the intrinsic coagulation system. Deficiency of VIII:C causes an increase in the PTT, but not in the PT. (It does not evaluate intrinsic pathway factors.) The patient experienced bleeding from the extraction site, because the damaged small vessel lumens only had temporary platelet plugs held together by fibrinogen, preventing the vessels from bleeding. The lack of thrombin from the VIII:C deficiency prevented conversion of the fibrinogen to fibrin, producing a stable platelet thrombus held together by fibrin. When the patient rinsed his mouth out with lukewarm water, the temporary platelet plugs were dislodged causing serious bleeding. Mild hemophilia A, as in this patient, can be treated with desmopressin acetate, which increases the release of VIII:C from endothelial cells. Severe disease requires infusion of recombinant factor VIII.

Option A (platelet count normal, bleeding time increased, PT normal, PTT increased) is incorrect. This set of studies is most often associated with classic von Willebrand disease. In this autosomal dominant disorder, there is a deficiency of von Willebrand factor (vWF), which is necessary for platelet adhesion to areas of endothelial injury. This prolongs the bleeding time without affecting the platelet count. In the circulation, vWF also complexes with factor VIII coagulant (VIII:C), which prevents degradation of VIII:C. Therefore, deficiency of vWF automatically leads to decreased factor VIII:C activity and prolongation of the PTT. The PT is normal because it does not evaluate VIII:C activity in the intrinsic pathway. Option C (platelet count decreased, bleeding time increased, PT increased, PTT increased) is incorrect. This set of data in which all the tests are abnormal is most often seen in disseminated intravascular coagulation. Tissue thromboplastin activates the extrinsic coagulation system causing the formation of fibrin thrombi within the microcirculation. Factors I, II, V, and VIII are consumed in the fibrin clots, which results in anticoagulation (PT and PTT are both increased). Fibrin thrombi also trap platelets causing thrombocytopenia, which, in turn, is responsible for producing a prolonged bleeding time. Option D (platelet count normal, bleeding time increased, PT increased, PTT increased) is incorrect. This set of studies is uncommon, but would most commonly be seen in primary fibrinolysis, which may occur in radical prostate surgery. In this setting, urokinase is released from the tissue causing activation of plasminogen and the release of plasmin. Plasmin degrades multiple coagulation factors (e.g., V, VIII, fibrinogen). This increases the PT and PTT, because factors V and VIII are in the final common pathway. The degradation products of fibrinogen interfere with platelet aggregation causing an increase in the bleeding time. The platelet count is not affected. The test for fibrinogen degradation products is positive; however, d-dimers are negative because fibrin clots are not present in primary fibrinolysis. Option E (platelet count normal, bleeding time normal, PT increased, PTT increased) is incorrect. This set of data is most consistent with a patient that is taking warfarin and/or heparin. Rat poison contains warfarin, which blocks epoxide reductase rendering vitamin K inactive. This prevents further -carboxylation of the vitamin Kdependent coagulation factors: factors II (prothrombin), VII, IX, and X. Since factors X and II are in the final common pathway, both the PT and PTT are prolonged. The PT evaluates factors VII, X, V, II, and I (fibrinogen), while the PTT evaluates factors XII, XI, IX, VIII, X, V, II, and I. The platelet count and bleeding time are not affected by warfarin. Heparin enhances antithrombin III activity, which neutralizes activated serine proteases.

A 58-year-old woman with a history of chronic alcoholism had memory loss, confusion, ataxia, and diplopia due to lateral rectus muscle weakness. Laboratory studies showed a macrocytic anemia with pancytopenia and hypersegmented neutrophils in the peripheral blood. The photograph shows a section of brain removed at autopsy. Which of the following deficiencies is most likely responsible for these lesions?

A. Folic acid B. Thiamine C. Vitamin B6 (pyridoxine) D. Vitamin B12 (cyanocobalamin) E. Zinc

Option B (Thiamine) is correct. Thiamine deficiency results in a deficiency of adenosine triphosphate, which underlies the development of dry beriberi (peripheral neuropathies, Wernicke-Korsakoff syndrome) and wet beriberi (dilated cardiomyopathy). The photograph shows changes associated with Wernickes encephalopathymainly hemorrhages in the periventricular gray matter and mamillary bodies, due to capillary proliferation in these areas of the brain. Red blood cells break down, causing the release of iron, which produces a rusty brown discoloration of brain tissue. The classic triad of Wernickes syndrome is confusion, ataxia, and ophthalmoplegia (which is most often due to lateral rectus muscle weakness). Korsakoffs syndrome is associated with anterograde and retrograde memory loss. The patient also has a macrocytic anemia due to folate deficiency, which is most often caused by alcohol abuse. Option A (Folic acid) is incorrect. Folic acid deficiency results in a macrocytic anemia with no neurologic deficits. Alcoholism is the most common cause of folic acid deficiency. Option C (Vitamin B6 (pyridoxine)) is incorrect. Vitamin B6 (pyridoxine) deficiency causes a microcytic sideroblastic anemia with ringed sideroblasts. Seizures and peripheral neuropathies also may occur. Deficiency is most often due to interactions with medications (e.g., isoniazid), or it may be caused by alcoholism. Option D (Vitamin B12 (cyanocobalamin)) is incorrect. Vitamin B12 (cyanocobalamin) deficiency causes a macrocytic anemia with neurologic abnormalities primarily involving the spinal cord (posterior columns and lateral corticospinal tracts). It does not produce the Wernicke-Korsakoff syndrome.

Option E (Zinc) is incorrect. Zinc deficiency causes skin rashes, improper wound healing, and inability to taste and smell.

The ECG shows lead V3 from a 61-year-old man who has muscle weakness. Which of the following conditions could potentially produce this clinical finding?

A. Chronic renal failure B. Diarrhea C. Primary aldosteronism D. Type 1 renal tubular acidosis E. Vomiting
Option A (Chronic renal failure) is correct. In chronic renal failure, the aldosterone-mediated Na+, K+-ATPase pump in the renal tubules is dysfunctional. This leads to increased urinary loss of Na+ and retention of K+ (hyperkalemia). The ECG shows a peaked T wave, indicating hyperkalemia. Any increase or decrease in K+ concentration produces conduction disturbances in muscle that may lead to weakness. Option B (Diarrhea) is incorrect. Diarrhea in adults is associated with an isotonic loss of fluid (normal serum sodium), loss of HCO3- (normal anion gap metabolic acidosis), and loss of K+ (hypokalemia). On an ECG, hypokalemia is indicated by a prominent U wave following the T wave (not a peaked T wave). Option C (Primary aldosteronism) is incorrect. An increase in aldosterone enhances the Na+-K+ pump in the late distal and collecting tubules. This increases the exchange of Na+ for K+, causing loss of K+ in the urine (hypokalemia) and retention of Na+ (hypernatremia).

Option D (Type 1 renal tubular acidosis) is incorrect. Type 1 (distal) renal tubular acidosis results from dysfunction of the aldosterone-mediated H+, K+-ATPase pump in the collecting tubule. This is associated with retention of H + (normal anion gap metabolic acidosis) and loss of K+ in the urine, leading to hypokalemia. Option E (Vomiting) is incorrect. Vomiting causes metabolic alkalosis due to the loss of acid and retention of Hco3-. Alkalosis causes a shift of H+ out of the cells in exchange for K+, causing hypokalemia.

A 17-year-old boy complains of pain in his legs when he runs more than 400 meters. Physical examination shows +1 dorsalis pedis pulses and +4 radial pulses. Blood pressure is 160/96 mm Hg in the upper extremity and 130 mm Hg in the lower extremity. A high-pitched, blowing early diastolic murmur is heard in the right second intercostal space, and it does not increase in intensity with deep, held inspiration. An arterial blood gas shows a normal Pao2 and oxygen saturation. Which of the following lesions is most likely present?

A. Aortic dissection B. Aortic valve stenosis C. Patent ductus arteriosus D. Postductal coarctation of the aorta E. Takayasus arteritis
Option D (Postductal coarctation of the aorta) is correct. Strong pulses in the upper extremities and diminished pulses in the lower extremities associated with pain in the legs with running (claudication) are indicative of postductal coarctation of the aorta. Proximally there may be dilation of the aorta leading to aortic regurgitation (early diastolic murmur). The blood pressure and pulse amplitude is increased in the upper extremities from increased blood flow into the upper extremity vessels. There is at least a 10 mm Hg drop in blood pressure in the lower extremities. Systemic hypertension is due to reduced renal blood flow leading to activation of the renin-angiotensin-aldosterone system. Aldosterone causes renal retention of sodium, and angiotensin II constricts the peripheral resistance arterioles. There is no shunting of blood between chambers of the heart and arterial blood gases are normal. Collateral vessels develop between the anterior and posterior intercostal arteries and between the superior epigastric artery and inferior epigastric arteries to bypass the area of constriction in aorta. The collateral circulation between the anterior and posterior intercostal arteries leads to rib notching, which is visible on chest radiographs (see Fig. 10-12F in Rapid Review Pathology, 3rd edition). Option A (Aortic dissection) is incorrect. Aortic dissection occurs in elderly men with hypertension or in young individuals with defects in collagen (e.g., Ehlers-Danlos syndrome) or elastic tissue (Marfan syndrome). Blood enters an intimal tear and dissects proximally and/or distally through the weakened media of the aorta. Aortic dissections produce pain that radiates to the back and are often associated with absent (not increased) upper extremity pulses. Option B (Aortic valve stenosis) is incorrect. Aortic valve stenosis causes a systolic ejection murmur (not diastolic murmur). The pulse amplitude in the upper extremities is the same as that in the lower extremities.

Option C (Patent ductus arteriosus) is incorrect. Patent ductus arteriosus causes a continuous machinery-like murmur. The pulse amplitude in the upper extremities is the same as that in the lower extremities. Option E (Takayasus arteritis) is incorrect. Takayasus arteritis is an elastic artery vasculitis that occurs in young Asian women. Pulses in the upper extremities are absent (not increased).

The photograph shows a physical diagnostic finding in a 23-year-old gymnast. Examination of the eyes is normal; however, the patient has a systolic click followed by a murmur at the apex. Which of the following best describes the connective tissue defect in this patient?

A. Acquired defect in cross-bridging of collagen B. Acquired defect in hydroxylation of lysyl residues in collagen C. Genetic defect in synthesis of fibrillin in elastic tissue D. Genetic defect in synthesis of type 1 collagen E. Genetic defect in synthesis or structure of collagen
Option E (Genetic defect in synthesis or structure of collagen) is correct. The photograph shows hyperelastic skin, which is a characteristic finding in Ehlers-Danlos syndrome (EDS). EDS is a Mendelian connective tissue disorder with defects in collagen synthesis and/or structure. Other clinical features that may be present include hypermobility of joints, aortic dissection, mitral valve prolapse (present in this patient), and rupture of the colon. Option A (Acquired defect in hydroxylation of lysyl residues in collagen) is incorrect. An acquired defect in crossbridging of collagen fibers is associated with scurvy, which is caused by deficiency of vitamin C (ascorbic acid). The weak collagen (decreased tensile strength) produces vascular instability of small vessels, leading to skin and joint hemorrhages and bleeding gums with the loss of teeth. Option B (Acquired defect in hydroxylation of lysyl residues in collagen) is incorrect.An acquired defect in the

hydroxylation of lysyl residues in collagen is present in scurvy. The hydroxylation sites become the anchors for crosslinking collagen, which explains why the collagen has decreased tensile strength. Option C (Genetic defect in synthesis of fibrillin in elastic tissue) is incorrect. A genetic defect in the synthesis of fibrillin in elastic tissue defines Marfan syndrome, an autosomal dominant disorder. Clinical findings in Marfan syndrome include mitral valve prolapse, aortic dissection, arachnodactyly, and lens dislocation. Although there is some overlap in clinical finding with EDS, hyperelastic skin is not a feature of Marfan syndrome. Option D (Genetic defect in synthesis of type 1 collagen) is incorrect. A defect in the synthesis of type 1 collagen is present in osteogenesis imperfecta, an autosomal dominant disorder. Clinical findings include pathologic fractures beginning at birth and blue sclera.

An afebrile 50-year-old man complains of watery diarrhea and weight loss over the past 6 months. He states that his face often becomes flushed. Physical examination shows an enlarged, nodular liver. A pansystolic murmur along the parasternal border that increases in intensity with deep, held inspiration is heard. A fecal smear of stool for leukocytes is negative. Which of the following laboratory studies would be most useful for confirming the diagnosis?

A. Blood cultures B. Liver function tests C. Serum electrolytes D. Stool cultures E. Urine test for 5-hydroxyindoleacetic acid (5-HIAA)
Option E (Urine test for 5-hydroxyindoleacetic acid (5-HIAA)) is correct. The patient has the signs and symptoms of carcinoid syndrome (facial flushing, diarrhea, tricuspid regurgitation). In carcinoid syndrome, a carcinoid tumor that secretes serotonin must metastasize to the liver. Although the appendix is the most common site for carcinoid tumors (see Fig. 17-36 in Rapid Review Pathology, 3rd edition), they do not metastasize to the liver to produce the carcinoid syndrome. Most tumors causing the carcinoid syndrome arise in the terminal ileum. When these tumors metastasize to the liver, tumor nodules release serotonin directly into tributaries of the hepatic vein, which allows serotonin to enter the systemic circulation. Serotonin causes vasodilation of arterioles (causes facial flushing) and increases bowel motility (causes diarrhea). Serotonin is also fibrogenic causing the tricuspid valve leaflets to fibrose producing tricuspid regurgitation (pansystolic murmur that increases with deep, held inspiration). The metabolic end-product of serotonin is 5-HIAA. Option A (Blood cultures) is incorrect. Infective endocarditis involving valves on the right side of the heart usually occurs in intravenous drug abusers who have dirty needles. The majority of patients with endocarditis are febrile (not afebrile) and flushing and diarrhea are not usually present nor are nodular livers.

Option B (Liver function tests) is incorrect. Liver function tests are usually normal in the presence of liver metastasis caused by focal, rather than diffuse, involvement of the liver parenchyma. Furthermore, liver function tests cannot specifically diagnose the carcinoid syndrome. Option C (Serum electrolytes) is incorrect. The diarrhea in carcinoid syndrome is a secretory type (isotonic loss of fluid). Serum electrolytes in secretory diarrheas show hypokalemia and metabolic acidosis, due to loss of potassium and bicarbonate in diarrheal fluid. Serum electrolytes are not useful in confirming the diagnosis of carcinoid syndrome. Option D (Stool cultures) is incorrect. Stool cultures to rule out an invasive type of diarrhea are not warranted, because a fecal smear for leukocytes is negative.

A 68-year-old man suddenly develops severe, diffuse abdominal pain and bloody diarrhea. He has a history of severe peripheral vascular disease. Physical findings include a sinus tachycardia, diffuse abdominal distention, absence of rebound tenderness, and absence of bowel sounds. A plain abdominal film shows air-fluid levels in the affected bowel. The photograph shows a section of small bowel removed at surgery. Which of the following is the most likely cause of the small bowel disease?

A. Embolus to the superior mesenteric artery B. Proximal thrombosis of the superior mesenteric artery C. Strangulation of bowel in a direct inguinal hernia sac D. Strangulation of bowel in a femoral hernia sac E. Strangulation of bowel in an indirect hernia sac

Option B (Proximal thrombosis of the superior mesenteric artery) is correct. The photograph shows a hemorrhagic infarction of small bowel. The thrombosed superior mesenteric artery is attached to the aorta (arrow), the latter showing severe fissuring from atherosclerotic plaques. A red thrombus is noted in the proximal portion of the superior mesenteric artery and most likely represents a platelet thrombus overlying atherosclerotic plaques similar to the type of thrombus in coronary artery thrombosis. This is the most common

cause of small bowel infarction. Small bowel infarction causes diffuse abdominal distention, absence of rebound tenderness, and absence of bowel sounds (called ileus). Abdominal radiographs show air-fluid levels in the infarcted bowel due to the lack of peristalsis. Other causes of small bowel infarction include embolism, usually in a patient with atrial fibrillation, and thrombosis of the mesenteric vein. Option A (Embolus to the superior mesenteric artery) is incorrect. Embolization to the superior mesenteric artery is most often caused by chronic atrial fibrillation. The patient has a sinus tachycardia and not the classic irregularly irregular pulse of atrial fibrillation. Option C (Strangulation of bowel in a direct inguinal hernia sac) is incorrect. Direct inguinal hernias do not entrap the small bowel; therefore, strangulation and infarction are unlikely. Option D (Strangulation of bowel in a femoral hernia sac) is incorrect. Femoral hernias are uncommon; however, they are associated with entrapment of bowel and bowel strangulation. Femoral hernias produce a bulge below the inguinal ligament. Option E (Strangulation of bowel in an indirect hernia sac) is incorrect. Indirect inguinal hernias are common and are often associated with strangulation of a segment of small bowel. Entrapped bowel in an indirect hernial sac produces localized pain over the hernial sac, not diffuse abdominal pain.

A 35-year-old man has hepatosplenomegaly and pancytopenia. The arrow in the photograph points to one of many similar cells present in a bone marrow aspirate biopsy. Which of the following enzymes is most likely to be deficient in this patient?

A. Arylsulfatase B. Glucocerebrosidase C. Hexosaminidase D. Iduronidase E. Sphingomyelinase

Option B (Glucocerebrosidase) is correct. The patient has type 1 (adult type) Gauchers disease, an autosomal recessive lysosomal storage disease. It is the most common lysosomal storage disease in the United States. It is caused by a deficiency of glucocerebrosidase, which leads to accumulation of glucocerebroside within the lysosomes of macrophages in the bone marrow, liver, and spleen. The arrow points to a Gaucher cell, whose cytoplasm has a fibrillary, wrinkled appearance. Replacement of the bone marrow by these macrophages causes pancytopenia. There is no central nervous system involvement in adult type Gauchers disease, unlike in the childhood variants. Option A (Arylsulfatase) is incorrect. Arylsulfatase is deficient in metachromatic leukodystrophy, a rare autosomal recessive lysosomal storage disease associated with the synthesis of abnormal myelin and the accumulation of sulfatides in lysosomes. Leukodystrophy causes mental retardation and peripheral neuropathies. Option C (Hexosaminidase) is incorrect. Hexosaminidase is deficient in Tay-Sachs disease, an autosomal recessive lysosomal storage disease that occurs primarily in Jews of Eastern European (Ashkenazi) descent. Tay-Sachs disease is associated with accumulation of GM2 gangliosides in lysosomes. It is associated with severe mental retardation, motor weakness, and a cherry-red macula. Option D (Iduronidase) is incorrect. Iduronidase is deficient in Hurlers syndrome, an autosomal recessive lysosomal storage disease associated with the accumulation of dermatan sulfate and heparan sulfate in the lysosomes. Hurlers syndrome causes severe mental retardation, a coarse facies, corneal clouding, and coronary artery disease. Option E (Sphingomyelinase) is incorrect. Sphingomyelinase is deficient in Niemann-Pick disease, an autosomal recessive lysosomal storage disease associated with the accumulation of sphingomyelin in the lysosomes of macrophages. This gives the macrophages a foamy or soap-bubble appearance. Niemann-Pick disease causes severe mental retardation, hepatosplenomegaly, and psychomotor retardation in children.

A 30-year-old man is hospitalized after an automobile accident and has comminuted fractures of the right humerus and left femur and multiple rib fractures in the left lower chest. After being hospitalized for 24 hours, the patients hemoglobin is 7 g/dL. He receives three units of packed RBCs, and on the following morning, his hemoglobin is 7.5 g/dL. A direct Coombs test is negative. A urine dipstick for blood is negative, and a stool guaiac is negative. Which of the following is the most likely cause of the decreased hemoglobin level following the transfusion?

A. Autoimmune hemolytic anemia B. Gastrointestinal bleed C. Hemolytic transfusion reaction D. Injury to the kidneys or urinary bladder E. Intra-abdominal bleed
Option E (Intra-abdominal bleed) is correct. If a patient is no longer bleeding, then each unit of packed RBCs should increase the hemoglobin by 1 gram. However, this does not occur in this patient; therefore, continued blood loss must

be present. The patient has multiple rib fractures in the left lower chest. Therefore, the most likely cause of bleeding is a ruptured spleen with loss of blood into the peritoneum. Option A (Autoimmune hemolytic anemia) is incorrect. It is highly unlikely that the patient has an autoimmune hemolytic anemia, because the direct Coombs test is negative. This test detects the presence of IgG antibodies and/or C3b on the surface of RBCs. Option B (Gastrointestinal bleed) is incorrect. The negative stool guaiac rules out gastrointestinal bleeding. Option C (Hemolytic transfusion reaction) is incorrect. It is highly unlikely that the patient has a hemolytic transfusion reaction, because the direct Coombs test is negative. This test detects the presence of IgG antibodies and/or C3b on the surface of RBCs. Option D (Injury to the kidneys or urinary bladder) is incorrect. The negative urine dipstick for blood rules out injury to the kidneys or urinary bladder as the cause of the bleeding.

A 60-year-old man with angina pectoris and a 35-pack-year history of smoking cigarettes suddenly clutches his chest and dies. He had no previous history of a heart murmur. Which of the following findings is most likely present at autopsy?

A. Aortic dissection B. Coronary artery thrombosis C. Hypertrophic cardiomyopathy D. Rupture of the anterior wall of the left ventricle E. Severe coronary artery atherosclerosis
Option E (Severe coronary artery atherosclerosis) is correct. Sudden cardiac death is defined as death within 1 hour after the onset of symptoms (e.g., chest pain). Severe coronary artery atherosclerosis, often involving multiple vessels, is present at autopsy. Coronary artery thrombosis is present in <20% of cases. Smoking is one of the most common environmental contributors to sudden cardiac death. It often precipitates ventricular arrhythmias. Option A (Aortic dissection) is incorrect. Aortic dissections cause pain radiating into the back and are not a common cause of sudden death. Option B (Coronary artery thrombosis) is incorrect. Severe coronary artery atherosclerosis, often involving multiple vessels, is present at autopsy in sudden cardiac death. Coronary artery thrombosis occurs in <20% of cases. Option C (Hypertrophic cardiomyopathy) is incorrect. Hypertrophic cardiomyopathy is characterized by asymmetric hypertrophy of the interventricular septum. The conduction system in the septum is abnormal, which may precipitate

fatal arrhythmias. Although there is a variant that occurs in the elderly, it would not be the most common cause of sudden cardiac death. Option D (Rupture of the anterior wall of the left ventricle) is incorrect. Rupture of the anterior wall leading to cardiac tamponade and death usually occurs 3 to 7 days following an acute anterior myocardial infarction, not immediately after the onset of symptoms of chest pain.

In which one of the following clinical scenarios does the patient most likely have a false positive (FP) laboratory test result?

A. A 7-year-old child, with a healing Colles fracture, has an increased serum alkaline phosphatase that is beyond the age-adjusted normal range. B. A 35-year-old man has an increased serum cholesterol over previous baseline levels about 6 hours after eating bacon and eggs for breakfast. C. A 45-year-old man, who is an alcoholic, has an increased anion gap metabolic acidosis. The physician suspects lactic acidosis and ketoacidosis. Lactic acid levels are increased; however, the serum and urine test for ketone bodies is negative. D. A 70-year-old man with a normal serum blood urea nitrogen and creatinine level, has a creatinine clearance of 65 mL/min. E. A 20-year-old woman with systemic lupus erythematosus, who is not sexually active, has a positive rapid plasma reagin (RPR) test. The physician orders a fluorescent treponeme antibody absorption test (FTA-ABS), which returns negative.
Option E (a 20-yr-old woman has a positive RPR test and a negative FTA-ABS) is correct. The antigen used in the RPR test system is beef cardiolipin, which reacts with the reagin antibodies of syphilis as well as anticardiolipin antibodies, which are most often present in patients with systemic lupus erythematosus. Since the FTA-ABS test has high specificity for diagnosing syphilis, it should be positive if the patient has syphilis. However, the test is negative, which means that the RPR test is most likely a FP for syphilis and suggests the presence of anticardiolipin antibodies. These antibodies are present in the antiphospholipid syndrome. Option A (A 7-year-old child, with a healing Colles fracture, has an increased serum alkaline phosphatase that is beyond the age-adjusted reference interval) is incorrect. The serum alkaline phosphatase (ALP) is normally increased in children due to increased bone growth corresponding with an increase in osteoblastic activity, which is the source of the ALP. The patient has a Colles fracture. In the normal repair of a fracture, there is increased osteoblastic activity, which explains the increase in ALP. This is a true positive. Option B (A 35-year-old man has an increased serum cholesterol over previous baseline levels about 6 hours after eating bacon and eggs for breakfast) is incorrect. Cholesterol that is derived from food is packaged in chylomicrons, which circulate in the bloodstream. The percentage of cholesterol in chylomicrons is less than 3%; therefore, fasting is not required for obtaining an accurate serum cholesterol. In this patient, the increased serum cholesterol is a true positive.

Option C (A 45-year-old man, who is an alcoholic, has an increased anion gap metabolic acidosis. The physician suspects lactic acidosis and ketoacidosis. Lactic acid levels are increased; however, the serum and urine test for ketone bodies is negative.) is incorrect. The physician suspects lactic acidosis and ketoacidosis. Lactic acid levels are increased, however, the serum and urine test for ketone bodies is negative) is incorrect. An increased anion gap metabolic acidosis in an alcoholic is due to lactic acidosis and ketoacidosis, the latter due to an increase in hydroxybutyric (-OHB) acid. The increase in lactic acid relates to the conversion of pyruvate to lactate by an increase in nicotinamide adenine dinucleotide (NADH) that occurs in alcohol metabolism. The increase in ketoacids is due to the conversion of acetyl CoA, the metabolic end-product of alcohol metabolism, into ketone bodies. Acetoacetic acid is converted to -OHB acid due the presence of NADH as a cofactor. The standard serum and urine tests for ketone bodies detect acetone and acetoacetic acid; however, they do not detect -OHB acid. Therefore, the negative test result for ketone bodies is a false negative. Option D (A 70-year-old man with a normal serum blood urea nitrogen and creatinine level, has a creatinine clearance of 65 mL/min.) is incorrect. The creatinine clearance normally decreases with advancing age. The presence of a normal blood urea nitrogen and creatinine level indicates that the decrease in the creatinine clearance is normal and represents a true negative.

A 42-year-old woman has severe iron deficiency anemia. Physical examination reveals an enlarged uterus. The photograph shows the hysterectomy specimen. Which of the following is a complication associated with the lesions depicted in the photograph?

A. Ectopic pregnancy B. Menorrhagia C. Metastasis D. Renal failure E. Sarcomatous transformation


Option B (Menorrhagia) is correct. The uterus has multiple, well-circumscribed, gray-white nodules (leiomyomas) that distort the uterus. Leiomyomas are benign smooth-muscle tumors. Those that are submucosally located are associated

with excessive menstrual bleeding (menorrhagia) and often require removal by hysterectomy, as in this patient. Menorrhagia is the most common cause of iron deficiency in women < 50 years of age. Option A (Ectopic pregnancy) is incorrect. Scarring of the fallopian tubes from previous pelvic inflammatory disease is the most common cause of ectopic implantation. There is no increased risk for ectopic pregnancies with leiomyomas of the uterus. Option C (Metastasis) is incorrect. Uterine leiomyomas rarely undergo sarcomatous transformation to leiomyosarcomas; therefore, this complication is unlikely. Option D (Renal failure) is incorrect. Renal failure is not a complication of leiomyomas. Option E (Sarcomatous transformation) is incorrect. Uterine leiomyomas rarely undergo sarcomatous transformation to leiomyosarcomas, which are the most common sarcomas of the uterus albeit a very uncommon complication of a leiomyoma.

A 23-year-old woman complains of a malodorous vaginal discharge, particularly after sexual intercourse. The photograph shows squamous cells in her cervical Pap smear. Which of the following is the causal pathogen of the discharge?

A. Candida albicans B. Chlamydia trachomatis C. Gardnerella vaginalis D. Neisseria gonorrhoeae E. Trichomonas vaginalis
Option C (Gardnerella vaginalis) is correct. The cervical Pap smear shows squamous cells covered by G. vaginalis (clue cells) giving them a granular appearance. G. vaginalis is the causative agent of bacterial vaginosis, the most common cause of vaginal discharge in women. In this disorder, the vaginal pH > 4.5 due to a decrease in lactobacilli (acidproducing bacteria) in the vaginal flora. This allows Gardnerella to proliferate, producing a vaginal discharge with a

strong fishy odor. The treatment is metronidazole. Option A (Candida albicans) is incorrect. C. albicans causes intense erythematous mucosal inflammation and a white discharge that contain yeasts and pseudohyphae. Option B (Chlamydia trachomatis) is incorrect. C. trachomatis causes cervicitis associated with a vaginal discharge. C. trachomatis infects metaplastic squamous cells and produces reticulate bodies within vacuoles in the cells. Option D (Neisseria gonorrhoeae) is incorrect. N. gonorrhoeae causes a cervicitis associated with a vaginal discharge. The gram-negative diplococci are phagocytosed by neutrophils. Option E (Trichomonas vaginalis) is incorrect. Trichomonas cervicitis causes a frothy vaginal discharge. The pearshaped flagellate organisms are not seen in the photograph.

The photograph shows the legs of a 50-year-old man, who complains of chronic diarrhea. His diet consists mainly of corn products. A similar rash is present around his neck and on the dorsum of both hands and forearms. The patient most likely has a deficiency of which of the following vitamins?

A. Niacin B. Thiamine C. Vitamin A D. Vitamin B6 E. Vitamin C

Option A (Niacin) is correct. The patient has pellagra due to a deficiency of niacin. Niacin is primarily synthesized by colonic bacteria and is also present in meats, fish, vegetables, and nuts. Some niacin is also synthesized from tryptophan. Patients who have corn-based diets commonly develop niacin deficiency (pellagra) because the niacin in corn is in a bound form that cannot be reabsorbed, and corn is deficient in tryptophan. The patient has diarrhea and dermatitis, two of the three major symptoms of pellagra. Dementia also may occur.

Option B (Thiamine) is incorrect. Thiamine is present in most foods. Thiamine deficiency, which is most commonly caused by alcoholism, produces dry and wet beriberi. Dry beriberi is manifested as Wernickes encephalopathy (confusion, ataxia, nystagmus, ophthalmoplegia), Korsakoffs psychosis (antegrade and retrograde memory deficits), and peripheral neuropathy. Wet beriberi is manifested as dilated cardiomyopathy with biventricular heart failure. Option C (Vitamin A) is incorrect. Vitamin A is present in liver and green and yellow vegetables. Vitamin A deficiency is associated with night blindness, blindness caused by squamous metaplasia of conjunctival epithelium, respiratory infections, and dry skin (follicular hyperkeratosis). These findings are not present in this patient. Option D (Vitamin B6) is incorrect. Vitamin B6 (pyridoxine) is present in most foods. Vitamin B6 deficiency is associated with defects in heme synthesis (sideroblastic anemia), peripheral neuropathy, and convulsions. Treatment of tuberculosis with isoniazid is the most common cause of pyridoxine deficiency. Option E (Vitamin C) is incorrect. Vitamin C is present in fruits and vegetables. Vitamin C deficiency (scurvy) is associated with small-vessel instability (ecchymoses), poor wound healing, glossitis, hemarthroses, perifollicular hemorrhage, and loose teeth. These findings are not present in this patient.

A 56-year-old farmer states that when he scrapes off hyperkeratotic raised lesions from the back of his hand (see photograph), they grow back within a few months. Which of the following is the pathogenesis of these lesions?

A. Excessive exposure to sunlight B. Neoplastic transformation of nevus cells C. Normal condition related to aging D. Superficial dermatophyte infection E. Viral infection of squamous cells

Option A (Excessive exposure to sunlight) is correct. The patient has actinic (solar) keratoses. The photograph shows rough, scaling papules and plaques on sun-damaged skin. Actinic keratoses are premalignant lesions that progress to squamous cell carcinomas in 2% to 5% of cases. Biopsy specimens of the lesions show dysplastic squamous cells beneath a hyperkeratotic layer of cells. Actinic keratoses tend to recur if scraped off. Protective clothing and sunscreen prevent the lesions from occurring. Option B (Neoplastic transformation of nevus cells) is incorrect. Nevus cells are modified melanocytes. Benign nevocellular nevi develop in early childhood and undergo progressive morphologic changes with age ranging from flat pigmented lesions to nodular lesions. Malignant melanomas also derive from nevus cells. The lesions on the hand are not pigmented. Option C (Normal condition related to aging) is incorrect. Although cumulative exposure to sunlight is more likely to result in actinic keratoses in elderly individuals, actinic keratoses are not a normal age-related finding. Option D (Superficial dermatophyte infection) is incorrect. Superficial dermatophytes (e.g., Trichophyton rubrum) are confined to the stratum corneum (anucleate cells surfacing the skin). Lesions involving the skin (tinea corporis) are usually ring-shaped patches with erythema and scaling on the outer margin and clearing in the center. Option E (Viral infection of squamous cells) is incorrect. Verruca vulgaris (due to human papillomavirus) and molluscum contagiosum (due to a poxvirus) produce discrete, raised lesions on the skin. Verrucae are papular lesions with a rough surface, whereas the lesions of molluscum contagiosum are bowl-shaped and have a central depression filled with keratin.

The photograph shows a 25-year-old man. The initial lesion developed prior to development of a pruritic rash that developed 2 weeks later over his chest and back. What is the most likely diagnosis?

A. Eczema B. Pityriasis rosea C. Secondary syphilis D. Tinea corporis E. Tinea versicolor


Option B (Pityriasis rosea) is correct. The patient has pityriasis rosea, a dermatitis of unknown etiology. This eruptive dermatitis begins with an oval-shaped lesion (herald patch) with an erythematous margin. The central area has fine white scales. Within 1 to 2 weeks, a more widespread eruption follows the lines of cleavage of the skin (Christmas tree distribution), as evident in this patient. The lesions tend to be pruritic and remit spontaneously in 2 to 10 weeks. Antihistamines control pruritus and ultraviolet light therapy hastens resolution. Option A (Eczema) is incorrect. Eczema does not have a herald patch or a rash with a Christmas tree distribution. Option C (Secondary syphilis) is incorrect. The palms, soles, and mucous membranes (condyloma latum) are affected in secondary syphilis. Option D (Tinea corporis) is incorrect. Tinea corporis is caused by Trichophyton rubrum, which is a superficial dermatophytes. The potassium hydroxide (KOH) examination is negative; therefore, the diagnosis of tinea corporis is excluded. Option E (Tinea versicolor) is incorrect. Tinea versicolor is caused by Malassezia furfur, which is a superficial dermatophyte. The KOH examination is negative; therefore, the diagnosis of tinea corporis is excluded.

A 52-year-old man with a 30-pack-year history of smoking cigarettes develops a painless mass in his left scrotal sac that was not present 3 months ago (see photograph). Testicular volume on the left side is below normal. Laboratory studies show microscopic hematuria and an absence of serum human chorionic gonadotropin (hCG) and -fetoprotein (AFP). Which of the following is the most likely cause of the scrotal mass?

A. Epididymitis B. Hydrocele C. Renal cell carcinoma D. Testicular cancer E. Torsion of spermatic cord
Option C (Renal cell carcinoma) is correct. The patient has a varicocele in the left scrotal sac caused by engorgement of the internal spermatic veins above the testis forming a tangle of vessels (bag full of worms appearance; see photograph). Varicoceles almost always occur on the left side because the left spermatic vein empties into the left renal vein, which has increased resistance to blood flow. The right spermatic vein empties into the vena cava, which has less resistance to blood flow. Sudden development of a left-sided varicocele is indicative of a lesion blocking blood flow into the left renal vein, such as a renal cell carcinoma associated with invasion of the left renal vein. Cigarette smoking is the greatest risk factor for renal cell carcinoma. Hematuria is the most common presenting symptom. Option A (Epididymitis) is incorrect. Epididymitis is associated with scrotal pain with radiation of the pain to the spermatic cord or flank. Option B (Hydrocele) is incorrect. A hydrocele is a collection of fluid between the two layers of the tunica vaginalis producing enlargement of the scrotal sac that transilluminates. Option D (Testicular cancer) is incorrect. Testicular cancer presents with a painless mass in the testes. In this patient, testicular volume is reduced, and tumor markers for testicular cancer (hCG and AFP) are absent. Option E (Torsion of spermatic cord) is incorrect. Torsion of the spermatic cord produces a sudden onset of testicular pain. The testicle draws up into the inguinal canal and there is loss of the cremasteric reflex.

A 62-year-old man with urinary retention caused by prostate hyperplasia develops fever, warm skin, and bounding pulses. Blood culture is positive for Escherichia coli. Which of the following chemical mediators is most likely responsible for the warm skin? A. Complement C3b B. Leukotriene B4 C. Leukotrienes C4, D4, and E4 D. Nitric oxide (NO) E. Thromboxane A2
Option D (Nitric oxide (NO)) is correct. The patient has endotoxic shock caused by sepsis due to E. coli. Endotoxins released during sepsis associated with E. coli directly damage endothelial cells, causing the release of NO and prostaglandin I2 (prostacyclin), both of which are vasodilators. NO is a free radical gas that is produced mainly by macrophages and endothelial cells. Arteriolar vasodilation is responsible for the warm skin, which characterizes septic shock versus other types of shock (e.g., hypovolemic and cardiogenic shock), where there is vasoconstriction of arterioles and cold, clammy skin. Option A (Complement C3b) is incorrect. Complement C3b is an opsonizing agent that facilitates phagocytosis by leukocytes. Option B (Leukotriene B4) is incorrect. Leukotriene B4 activates neutrophil adhesion molecules and is a chemotactic agent for leukocytes. Option C (Leukotrienes C4, D4, and E4) is incorrect. Leukotrienes C4, D4, and E4 cause vasoconstriction, bronchospasm, and increased vessel permeability. Option E (Thromboxane A2) is incorrect. Thromboxane A2 causes vasoconstriction and inhibits platelet aggregation.

A 25-year-old woman who is a pure vegan is breastfeeding her infant. Which of the following vitamin supplements should she be taking to prevent anemia in her infant?

A. Folic acid B. Vitamin B6 C. Vitamin B12

D. Vitamin C E. Vitamin D

Option C (Vitamin B12) is correct. The patient is a pure vegan; therefore, her diet lacks vitamin B 12, which is mainly obtained by eating animal products. Lack of vitamin B12 in breast milk predisposes the infant to a macrocytic anemia and the potential for neurologic dysfunction involving the posterior columns and lateral corticospinal tract. Option A (Folic acid) is incorrect. Folic acid is adequate in the diet of pure vegans. Therefore, a folic acid deficiency is unlikely to cause a macrocytic anemia in the infant. Option B (Vitamin B6) is incorrect. Vitamin B6 (pyridoxine) is adequate in the breast milk of pure vegans. Therefore, a vitamin B6 deficiency is unlikely to cause a sideroblastic anemia (defect in heme synthesis) in the infant. Option D (Vitamin C) is incorrect. Vitamin C is important for reabsorption of iron in the duodenum. Breast milk is rich in vitamin C. Therefore, a vitamin C deficiency is unlikely to cause iron deficiency anemia in the infant. Option E (Vitamin D) is incorrect. Vitamin D is deficient in breast milk. However, vitamin D deficiency causes rickets (defect in mineralization of bone) and does not cause anemia.

A 78-yr-old woman with chronic ischemic heart disease develops both left and right-sided heart failure. She is not on any medications. Which of the following volume (X axis) and plasma osmolality changes (Y axis) in the extracellular (small compartment) and intracellular fluid compartments (large compartment) is most likely present?

A. A B. B C. C D. D E. E F. F

Option D is correct. In congestive heart failure, the cardiac output is decreased and the kidney reabsorbs a slightly hypotonic fluid producing hyponatremia (? serum Na+ = ?TBNa+/??TBW). An increase in total body sodium (TBNa+) is clinically manifested by weight gain, dependent pitting edema, and body effusions. Since POsm and serum Na+ parallel each other, hyponatremia produces a decreased P Osm; therefore, the overall height of the interrupted squares is decreased. Since excess fluid is being added to the ECF compartment (primarily the interstitial space), the ECF compartment is expanded. Hyponatremia establishes an osmotic gradient causing water to shift from the ECF into the ICF compartment causing expansion of that compartment as well. This emphasizes the importance of restricting water and sodium in heart failure and using diuretics to eliminate excess sodium and water. Other edema states that can be associated with schematic D are cirrhosis and the nephrotic syndrome. Option A is incorrect. A represents an isotonic loss of fluid (e.g., secretory diarrhea in cholera and travelers diarrhea; loss of whole blood). Serum sodium remains normal (also P Osm) when equal amounts of water and sodium are lost (?serum Na+ = ?TBNa+/?TBW). Only the ECF compartment is contracted. A decrease in TBNa+ produces signs of volume depletion (hypotension, dry mucous membranes, tenting of the skin). There is no osmotic gradient; therefore, the ICF compartment is normal. Option B is incorrect. Schematic B occurs when there is a hypertonic loss of fluid (e.g., diuretics, Addisons disease, 21-hydroxylase deficiency). Loss of hypertonic fluid produces a hyponatremia (?serum Na + = ??TBNa+/?TBW). Since fluid is lost, the ECF compartment is contracted. Hyponatremia establishes an osmotic gradient; therefore, water shifts into the ICF compartment. Option C is incorrect. Schematic C occurs when there is an isotonic gain of fluid (e.g., excessive infusion of normal (0.9%) saline. The serum sodium remains normal (?serum Na + = ?TBNa+/?TBW); therefore, the POsm is normal. Since fluid is being added to the ECF, it is expanded. There is no osmotic gradient; therefore, the ICF remains normal. The treatment is diuretics. Option E is incorrect. Schematic E occurs when there is a hypotonic loss of sodium (e.g., sweating, osmotic diuresis, glucosuria). The serum sodium is increased (hypernatremia; ? serum Na + = ?TBNa+/??TBW), causing an increase in POsm and an increase in the height of the interrupted squares. Since fluid is being lost, the ECF compartment is contracted. Hypernatremia establishes an osmotic shift; therefore, water moves out of the ICF compartment.

Option F is incorrect. Schematic F occurs when there is a hypertonic gain of sodium (e.g., excessive infusion of sodium bicarbonate; infusion of a sodium containing antibiotic). The serum sodium is increased (hypernatremia; ? serum Na+ = ??TBNa+/?TBW), causing an increase in POsm and an increase in the height of the interrupted squares. Since fluid is being added, the ECF compartment is expanded (pitting edema; body effusions). Hypernatremia establishes an osmotic shift; therefore, water moves out of the ICF compartment.

A 35-year-old man has recurrent peptic ulcer disease. Treatment of the underlying cause of peptic ulcers will most likely decrease the incidence of which of the following cancers?

A. Adenocarcinoma of distal esophagus B. Leiomyosarcoma of the stomach C. Malignant lymphoma of the stomach D. Pancreatic adenocarcinoma E. Squamous cell carcinoma of esophagus
Option C (Malignant lymphoma of the stomach) is correct. Helicobacter pylori is the most common cause of peptic ulcer disease, which includes gastric and duodenal ulcers. Production of urease by the bacteria produces ammonia, which damages the protective mucus layer covering the pylorus and antrum of the stomach and the duodenum, sites colonized by H. pylori. Cytokines released by the bacteria cause chronic atrophic gastritis of the body and fundus with intestinal metaplasia (presence of Paneth cells and goblet cells), which is the precursor lesion for gastric adenocarcinoma at this location. Cytokines also elicit a marked lymphoid inflammatory response in the mucosa predisposing to a low-grade malignant lymphoma. Therefore, treatment of H. pylori not only prevents peptic ulcer disease, but also decreases the risk for malignant lymphoma of the stomach and gastric adenocarcinoma in the body and fundus. Option A (Adenocarcinoma of distal esophagus) is incorrect. Adenocarcinoma of the distal esophagus arises from intestinal metaplasia of the distal esophagus (Barretts esophagus) due to gastroesophageal reflux disease (GERD). H. pylori does not cause GERD; therefore, treatment of H. pylori does not decrease the risk for developing distal adenocarcinoma of the esophagus. However, treatment of Barretts esophagus and GERD with histamine and proton blockers would decrease the risk for this type of cancer. Option B (Leiomyosarcoma of the stomach) is incorrect. Leiomyosarcoma is the most common sarcoma of the stomach; however, H. pylori does not increase the risk for developing this cancer. Option D (Pancreatic adenocarcinoma) is incorrect. H. pylori does not increase the risk for pancreatic carcinoma. Cessation of smoking and alcohol would have a greater effect on reducing the risk for this type of cancer. Option E (Squamous cell carcinoma of esophagus) is incorrect. Squamous carcinoma of the esophagus is most often

caused by cigarette smoking and alcohol abuse to a lesser extent. H. pylori does not increase the risk for developing squamous cell carcinoma of the esophagus.

A 40-year-old black man with dyspnea is diagnosed with sarcoidosis. A CBC shows a mild microcytic anemia with hypochromic RBCs. The WBC count and platelet count are normal. The corrected reticulocyte count is 1%. A hemoglobin electrophoresis is normal. Serum ferritin is increased. What is the most likely diagnosis? A. -Thalassemia trait B. Anemia of chronic disease (ACD) C. -Thalassemia minor D. Iron deficiency anemia E. Sideroblastic anemia
Option B (Anemia of chronic disease (ACD)) is correct. The patient has sarcoidosis, which is a noninfectious, chronic granulomatous disease that targets the lungs and other organ systems. It is common in the black population. Chronic inflammation is associated with the release of hepcidin, an acute phase reactant synthesized and released by the liver. It is the key factor involved in the pathogenesis of ACD. Hepcidin enters bone marrow macrophages and prevents the release of iron to transferrin (binding protein of iron). Decreased delivery of iron to developing normoblasts in the bone marrow leads to a decrease in hemoglobin synthesis, which, in turn, is a stimulus for extra cell divisions and a microcytic anemia. Iron continues to enter the macrophages; however, this is not balanced by iron moving out to bind to transferrin. This results in an increase in iron stores. Ferritin, a soluble iron-binding protein, is synthesized by macrophages and small fraction leaks into the blood hence providing an indirect measurement of macrophage iron stores. In this case, it is increased. The amount of ferritin in macrophages has an inverse relationship with transferrin synthesis in the liver. Therefore, increased ferritin stores in macrophages leads to decreased transferrin synthesis in the liver and decreased transferrin, or total iron-binding capacity (TIBC) in the blood. In summary, the laboratory findings include a decreased MCV (sometimes normocytic), decreased serum iron, decreased TIBC, decreased percent saturation of transferrin, and an increase in serum ferritin Option A (-Thalassemia trait) is incorrect. -Thalassemia trait is an autosomal recessive disease with deletions of 1 or more of the 4 genes involved in -globin chain synthesis. Deletion of 1 gene is not associated with anemia. A 2 gene deletion produces a mild microcytic anemia called -thalassemia trait. Since HbA (2, 2), HbA2 (2, 2), and HbF (2, 2) all require -globin chains, they are all decreased on a Hb electrophoresis. However, the relative proportions of the normal Hbs remains the same; therefore, the Hb electrophoresis is normal. An increase in the RBC count is a common finding in all of the thalassemias (not increased in this patient) and is a useful marker for thalassemia. Serum ferritin is normal (increased in this patient). Option C (-Thalassemia minor) is incorrect. -Thalassemia minor is an autosomal recessive disease with a DNA splicing defect causing a slight decrease in -globin chain synthesis with subsequent development of a microcytic anemia. A decrease in -globin chain synthesis decreases the synthesis of HbA (2, 2) and increases in the synthesis of HbA2 (2,

2), and HbF (2, 2), which do not contain -globin chains. The hemoglobin electrophoresis in this patient is normal. Since the defect in thalassemia involves globin chain synthesis, serum ferritin is normal. Option D (Iron deficiency anemia) is incorrect. Iron deficiency is the most common cause of microcytic anemia. As anticipated, serum iron is decreased, TIBC is increased (due to decrease in ferritin stores), percent saturation of transferrin is decreased, and serum ferritin is decreased (not increased as in this patient). Option E (Sideroblastic anemia) is incorrect. Sideroblastic anemias are due to defects in heme synthesis in the mitochondria of developing RBC precursors in the bone marrow. Most acquired types of sideroblastic anemia are microcytic. The most common causes are alcohol, pyridoxine, and lead poisoning. Iron accumulates in the mitochondria producing ringed sideroblasts, which are identified with a Prussian blue stain. Iron overload occurs, which causes an increase in serum ferritin, increase in serum iron, decrease in serum TIBC (ferritin stores increased), and increased percent saturation of transferrin. Since the patient has chronic inflammation, the most likely diagnosis is ACD rather than sideroblastic anemia.

A 22-year-old woman with a history of chronic diarrhea describes her stools as greasy. She recently developed a lesion on her elbow. The photograph show the skin rash and an endoscopic biopsy of the jejunum. Which of the following tests is most useful to screen for the cause of her diarrhea?

A. Antinuclear antibodies B. Antitissue transglutaminase IgA antibody C. Fecal smear for leukocytes D. Stool for ova and parasites E. Stool osmotic gap
Option B (Antitissue transglutaminase IgA antibody) is correct. This patient has celiac disease. The photograph of the biopsy specimen shows villous atrophy (flat mucosa) and hyperplastic glands with an increased number of chronic inflammatory cells in the lamina propria. The photograph shows a vesicular lesion on the patients elbow that is

consistent with dermatitis herpetiformis, an autoimmune skin disease that has an almost 100% correlation with underlying celiac disease. Celiac disease is a multiorgan autoimmune disease characterized by an inappropriate T-cell and IgA-mediated response against gluten in genetically predisposed persons (HLA DQ2 and HLA DQ8). Timing and dose when gluten is introduced in the diet is important in the pathogenesis. Tissue transglutaminase (tTG; deamidating enzyme) in the lamina propria has a pivotal role. It deaminates mucosally absorbed gluten to produce deaminated and negatively charged gluten peptides. It also enhances the immunostimulatory effect of the deaminated gluten peptides. These peptides are phagocytosed by antigen-processing cells in the lamina propria and are presented in complex with HLA DQ2 or DQ8 to gluten-specific CD4 T helper cells. CD4 T cells produce cytokines that release matrix proteases causing cell death and degradation in the epithelial cells in the villi. Important diagnostic antibodies include antitissue transglutaminase IgA (most important and an excellent screening test); antiendomysial IgA (excellent screening test); and antigliadin IgA and IgG (moderately good screening test). Treatment of celiac disease is to restrict gluten-containing products. Option A (Antinuclear antibodies) is incorrect. The antibodies in celiac disease are not directed against nuclear proteins. Option C (Fecal smear for leukocytes) is incorrect. A fecal smear for leukocytes is used to evaluate diarrhea that may be caused by invasive microbial pathogens (e.g., Campylobacter jejuni, Shigella sonnei). The presence of leukocytes presumes an invasive enterocolitis (not celiac disease). Option D (Stool for ova and parasites) is incorrect. Testing the stool for ova and parasites (or stool antigens) is recommended in the workup of a patient with chronic diarrhea. Giardiasis is the most common cause of chronic diarrhea associated with malabsorption; however, the characteristic pear-shaped organisms are not present in the biopsy and it is not associated with dermatitis herpetiformis. Option E (Stool osmotic gap) is incorrect. A stool sample to calculate the osmotic gap is used for high-volume diarrheal states to differentiate a secretory type of diarrhea from an osmotic type of diarrhea. Secretory diarrheas are characterized by isotonic diarrheal fluid (e.g., due to certain types of laxatives, enterotoxigenic bacteria), whereas osmotic diarrheas are characterized by hypotonic stool because of the presence of osmotically active solutes (e.g., lactase deficiency with an excess of lactose). The stool osmotic gap is calculated with the following formula: 300 mOsm/kg (value used to represent normal POsm) 2 ( (random stool Na+ + random stool K +). A gap < 50 mOsm/kg from the POsm is a secretory diarrhea, while a gap > 100 mOsm/kg from the Posm is an osmotic diarrhea.

This child has moderately severe mental retardation. Chromosome analysis shows 46 chromosomes. Which of the following types of genetic mutation is most likely responsible for his condition?

A. Balanced translocation B. Frameshift mutation C. Microdeletion D. Nondisjunction E. Point mutation


Option A (Balanced translocation) is correct. The child has Down syndrome (epicanthal folds, flat nasal bridge). The presence of 46 chromosomes in the child indicates that a translocated chromosome, inherited from one of the parents, is responsible. Translocation occurs when one part of a chromosome is transferred to a nonhomologous chromosome. In balanced (robertsonian) translocation, the translocated fragment is functional. In this case, the long arm of chromosome 21 was translocated onto chromosome 14 in the mother, creating one long chromosome (14;21). The mother also has one chromosome 14 and one chromosome 21. The father has the normal 46 chromosomes. The affected child has 46 chromosomes with three functional 21 chromosomes including chromosome (14;21) and chromosome 21 from the mother and chromosome 14 and chromosome 21 from the father. Option B (Frameshift mutation) is incorrect. The childs facial features are characteristic of Down syndrome, which is not caused by a frameshift mutation. In a frameshift mutation, nucleotides are inserted into or deleted from a DNA strand, resulting in synthesis of an abnormal protein product (e.g., Tay-Sachs disease). Option C (Microdeletion) is incorrect. The childs facial features are characteristic of Down syndrome, which is not caused by microdeletion. Microdeletion involves the loss of a small portion of one chromosome, which can be detected only by high-resolution techniques. Option D (Nondisjunction) is incorrect. Nondisjunction refers to unequal separation of chromosomes in the first meiotic phase, resulting in an egg or a sperm with 22 or 24 chromosomes. Nondisjunction is responsible for most numeric chromosome disorders (e.g., trisomy 21), but because this patient has 46 chromosomes, a balanced

translocation is most likely responsible. Option E (Point mutation) is incorrect. A point mutation involves the substitution of a single nucleotide base. If the altered DNA codes for the same amino acid, there is no change in the phenotypic effect (silent mutation). If the altered DNA codes for a different amino acid, there is a change in the phenotypic effect (missense mutation). If the altered DNA codes for a stop codon (e.g., UAA), there is premature termination of protein synthesis (nonsense mutation). These types of mutations are not operative in the pathogenesis of Down syndrome.

A 28-year-old man who had his right cryptorchid testis removed as a child develops painless enlargement of the left testicle, which is now in the left scrotal sac. The enlarged testicle does not transilluminate. Serum human chorionic gonadotropin and -fetoprotein are absent. Which of the following is the most likely diagnosis?

A. Choriocarcinoma B. Hydrocele C. Seminoma D. Varicocele E. Yolk sac tumor


Option C (Seminoma) is correct. A painless testicular mass that does not transilluminate is highly predictive of testicular cancer. Adults with a cryptorchid testis are at risk for seminoma in the undescended testis (if still present) as well as the descended testis (see Figs. 20-6 and 20-7 in Rapid Review Pathology, 3rd edition). Option A (Choriocarcinoma) is incorrect. A choriocarcinoma of the testicle typically is small and does not cause enlargement of the entire testicle. The syncytiotrophoblast component of the tumor produces human chorionic gonadotropin, which is absent in this patient. Option B (Hydrocele) is incorrect. A hydroceles is the most common cause of scrotal enlargement. It is due to a failure of closure of the tunica vaginalis. Fluid accumulates in the serous space between the layers of the tunica vaginalis. The mass of fluid transilluminates, unlike in this patient. It is invariably associated with an indirect inguinal hernia, which is also not present in this patient. Option D (Varicocele) is incorrect. A varicocele is the tortuous dilation of the veins of the spermatic cord that is often described as a bag of worms. A variocele typically occurs on the left side, not on the right side, because the left spermatic vein empties into the left renal vein, where it encounters more resistance to blood flow. The right spermatic vein empties into the inferior vena cava, where there is less resistance to blood flow. Varicoceles do not enlarge the testicle. Option E (Yolk sac tumor) is incorrect. Yolk sac tumors typically occur in infants and children and are not associated with cryptorchidism. They secrete -fetoprotein, which is absent in this patient.

The photograph shows a biopsy of a nonpruritic, red, crusted lesion from the vulva of a 62-year-old woman. A mucin stain is positive for some of the cells in the biopsy. Topical treatment with hydrocortisone cream has had no effect on the lesion. What is the most likely diagnosis?

A. Infiltrating squamous cell carcinoma B. Inflammatory dermatitis C. Malignant melanoma D. Pagets disease E. Squamous dysplasia
Option D (Pagets disease) is correct. The biopsy shows an intraepidermal neoplasm with large, malignant epithelial cells that are disposed singly or in clusters throughout the epidermis. The cells are reported to be mucin positive indicating that they are adenocarcinoma cells that derive from intraepidermal stem cells. The nature of the rash (red and crusted) and the morphology of the cells (Pagets cells) are diagnostic of Pagets disease of the vulva. The cancer tends to recur if not completely removed and, in rare cases, it may invade the dermis. Option A (Infiltrating squamous cell carcinoma) is incorrect. The basement membrane of the biopsy is intact, therefore, infiltrating squamous cell carcinoma is excluded. Furthermore, the epidermis does not show dysplastic squamous cells (hyperchromatic nuclear chromatin, irregular nuclear borders, loss of normal stratification of the epidermis). Option B (Inflammatory dermatitis) is incorrect. Many types of dermatitis (e.g., eczema) can simulate the external appearance Pagets disease of the vulva. Therefore, biopsy is important with eczematous vulvar lesions that do not respond to topical steroid therapy. Option C (Malignant melanoma) is incorrect. The lesion is red and crusted. The gross appearance of malignant melanoma is a localized lesion with a dark brown to black color. However, the histologic appearance of a melanoma is similar to that of the cells in Pagets disease. A key difference is the presence of mucin-positive cells in Pagets cells and the absence of mucin in a malignant melanoma.

Option E (Squamous dysplasia) is incorrect. Dysplastic squamous cells have large, hyperchromatic nuclei and do not develop as isolated cells or in clusters, as in this case.

A 2-year-old child has severe mental retardation, psychomotor retardation, and hepatosplenomegaly. The photograph shows representative cells in a bone marrow aspirate biopsy. Which of the following enzymes is most likely to be deficient in this patient?

A. Arylsulfatase B. Glucocerebrosidase C. Hexosaminidase D. Iduronidase E. Sphingomyelinase


Option E (Sphingomyelinase) is correct. The patient has Niemann-Pick disease, an autosomal recessive lysosomal storage disease associated with a deficiency of sphingomyelinase and the accumulation of sphingomyelin in the lysosomes of macrophages. This gives the macrophages a foamy or soap-bubble appearance, as shown in the histologic section. Niemann-Pick disease causes severe mental retardation, psychomotor retardation, and hepatosplenomegaly in children. Option A (Arylsulfatase) is incorrect. Arylsulfatase is deficient in metachromatic leukodystrophy, a rare autosomal recessive lysosomal storage disease associated with the synthesis of abnormal myelin and the accumulation of sulfatides in lysosomes. Leukodystrophy causes mental retardation and peripheral neuropathies. Option B (Glucocerebrosidase) is incorrect. Glucocerebrosidase is deficient in Gauchers disease, an autosomal recessive lysosomal storage disease associated with the accumulation of glucocerebroside in the lysosomes of macrophages in the bone marrow, liver, and spleen. The macrophages have a fibrillary or wrinkled appearance. Gauchers disease causes pancytopenia and hepatosplenomegaly. Option C (Hexosaminidase) is incorrect. Hexosaminidase is deficient in Tay-Sachs disease, an autosomal recessive lysosomal storage disease that occurs primarily in Jews of Eastern European (Ashkenazi) descent. Tay-Sachs disease is

associated with accumulation of GM2 gangliosides in lysosomes and causes severe mental retardation, motor weakness, and a cherry-red macula. Option D (Iduronidase) is incorrect. Iduronidase is deficient in Hurlers syndrome, an autosomal recessive lysosomal storage disease associated with the accumulation of dermatan sulfate and heparan sulfate in the lysosomes. Hurlers syndrome causes severe mental retardation, a coarse facies, corneal clouding, and coronary artery disease.

A 48-year-old man with hematemesis died of hypovolemic shock. The photograph shows the distal esophagus that was removed at autopsy. Which of the following best characterizes the pathogenesis of this lesion?

A. Gastroesophageal reflux of acid B. Malignant transformation of glandular metaplasia C. Motor disorder of the distal esophagus D. Portal hypertension E. Retching
Option D (Portal hypertension) is correct. The photograph shows dilated and tortuous veins (esophageal varices) in the submucosa of the distal esophagus. Esophageal varices are most often caused by portal hypertension secondary to cirrhosis. The most common cause of cirrhosis is alcohol abuse. The veins in esophageal varices are dilated, left gastric coronary veins, which normally drain venous blood from the distal esophagus and proximal stomach into the portal vein. In cirrhosis, the portal vein pressure increases, due to compression and destruction of the sinusoids in the liver by regenerative nodules. This results in an increase in pressure in the left gastric coronary veins, which causes them to dilate and rupture. Hematemesis with hypovolemic shock is the most common cause of death in patients with cirrhosis. Option A (Gastroesophageal reflux of acid) is incorrect. Relaxation of the lower esophageal sphincter with acid reflux leads to ulceration (not varices) of the squamous epithelium of the distal esophagus. Option B (Malignant transformation of glandular metaplasia) is incorrect. Adenocarcinoma arising from Barretts esophagus (malignant transformation of glandular metaplasia) is a complication of gastroesophageal reflux disease,

which is not a cause of esophageal varices. Option C (Motor disorder of the distal esophagus) is incorrect. Achalasia is a motor disorder of the distal esophagus characterized by failure of relaxation of the lower esophageal sphincter due to loss of ganglion cells in the myenteric plexus. This causes aperistalsis and dilation of the distal esophagus, not varices formation. Option E (Retching) is incorrect. Complications associated with retching are distal esophageal tears (Mallory-Weiss syndrome) and esophageal rupture (Boerhaaves syndrome), both of which can produce hematemesis but not varices formation.

A 62-year-old man awakes one morning with a frontal headache. He experiences dizziness, suddenly loses consciousness, and dies. The photograph shows the gross appearance of the brain at autopsy. Which of the following underlying conditions best explains the pathogenesis of this lesion?

A. Cerebral atherosclerosis B. Embolism from the heart C. Metastatic carcinoma D. Ruptured berry aneurysm E. Systemic hypertension
Option E (Systemic hypertension) is correct. The patients intracerebral hemorrhage is associated with systemic hypertension. Hypertension causes vascular changes in small vessels, such as hyaline arteriolosclerosis or the formation of Charcot-Bouchard microaneurysms in small penetrating branches of the middle cerebral artery (which supplies the basal ganglia and anterior and posterior limbs of the internal capsule). Rupture of the aneurysms caused an intracerebral hematoma, as in this patient (see photograph). The clot extends into the ventricle. Most hypertensive intracerebral bleeds occur in the basal ganglia, particularly the putamen and external capsule. It is a maxim in internal medicine that control of hypertension has its greatest effect on reducing the incidence of strokes, of which this intracerebral hematoma is an example. Option A (Cerebral atherosclerosis) is incorrect. Atherosclerotic strokes are most commonly due to thrombosis

overlying an atheromatous plaque in the middle cerebral artery or internal carotid artery. This produces a wedgeshaped pale infarction that extends up to the surface of the brain. Liquefactive necrosis occurs, leading to the formation of a cystic space within 10 days to 3 weeks. Atherosclerotic strokes are pale and do not commonly occur in the basal ganglia area. Option B (Embolism from the heart) is incorrect. Embolic strokes cause a hemorrhagic infarction, usually within the distribution of the middle cerebral artery. The infarction extends to the surface of the brain. Embolic strokes are unlikely in the basal ganglia area. Option C (Metastatic carcinoma) is incorrect. Metastases to the brain usually are multiple, nonhemorrhagic, and peripherally located at the junction of the white and gray matter. Option D (Ruptured berry aneurysm) is incorrect. Most ruptured berry (saccular) aneurysms are located at the junction of the anterior cerebral artery and anterior communicating artery. Most of the aneurysms are congenital and are located at the bifurcation of the two vessels. Rupture usually is into the subarachnoid space.

A medical student accidentally sticks his finger after drawing blood on a woman, who is having elective surgery for removal of a benign breast mass. As part of the normal routine in handling accidental needle sticks, a baseline liver profile is ordered. The profile consists of a total bilirubin, serum aspartate aminotransferase (AST), serum alanine aminotransferase (ALT), serum alkaline phosphatase, and serum -glutamyltransferase. What is the approximate percent chance of one of these five tests having a value outside the normal reference interval for the test?

A. 10% B. 13% C. 23% D. 35% E. 47%


Option C (23%) is correct. Most normal ranges are established by adding and subtracting 2 standard deviations (SD) from the mean of the test, which encompasses 95% of the normal population. Therefore, 5% of the normal population will be outside out of the normal range (outlier). The likelihood of an outlier increases as the number of tests ordered increases. The likelihood of an outlier = 100 (0.95n100), where n is the number of tests ordered. In this patient, five tests were ordered: 100 (0.955100) = 100 (0.77100) = 23% chance of an outlier in one of the five tests. Option A (10%) is incorrect.

Option B (13%) is incorrect. Option D (35%) is incorrect. Option E (47%) is incorrect.

The photograph shows the L2 dermatome of a 60-year-old man who is currently being treated for a follicular Bcell malignant lymphoma. Which of the following best characterizes the pathogenesis of this lesion?

A. Photosensitive reaction to a drug B. Reactivation of a latent virus in the sensory dorsal root ganglia C. Skin invasion by malignant CD4 helper T cells D. Toxin-producing strain of Staphylococcus aureus E. Toxin-producing strain of Streptococcus pyogenes
Option B (Reactivation of a latent virus in the sensory dorsal root ganglia) is correct. The patient has shingles, caused by the varicella-zoster virus. The photograph shows the band distribution that is characteristic of this vesiculobullous skin disorder. The virus remains latent in the sensory dorsal root ganglia after the primary infection. It occurs in approximately 10% to 20% of people during their lifetime. Incidence increases with age and in patients with cancer and AIDS. There is a prodrome of radicular pain and itching before the rash occurs, which is characterized by groups of vesicles on an erythematous base. The rash follows sensory dermatomes in the distribution of cranial nerves or spinal nerves. Prevention is accomplished with previous immunization for varicella or with zoster vaccine. Immunocompromised patients are often treated with acyclovir, valacyclovir, or famiciclovir, which are best started before the rash has erupted. Option A (Photosensitive reaction to a drug) is incorrect. Photosensitive drug eruptions develop in areas of exposure to sunlight (e.g., face, neck, hands). They do not involve a specific dermatome.

Option C (Skin invasion by malignant CD4 helper T cells) is incorrect. The plaques and nodular lesions of cutaneous Tcell lymphoma (mycosis fungoides) do not follow a dermatome. Option D (Toxin-producing strain of Staphylococcus aureus) is incorrect. S. aureus is a toxin-producing agent that causes toxic shock syndrome. The rash does not involve a dermatome. The toxin (a superantigen that stimulates cytokine production) produces an erythematous rash that desquamates. Option E (Toxin-producing strain of Streptococcus pyogenes) is incorrect. S. pyogenes produces an erythrogenic toxin that causes scarlet fever. The erythematous rash has a sandpaper consistency and is followed by desquamation.

The photograph shows a 16-year-old girl who has primary amenorrhea. Her height is 58 inches, and her breasts are underdeveloped. Which of the following laboratory findings would most likely be reported?

A. Decreased concentration of luteinizing hormone (LH) B. Decreased concentration of serum growth hormone C. Increased concentration of follicle-stimulating hormone (FSH) D. Increased concentration of serum estradiol E. Increased concentration of serum testosterone
Option C (Increased concentration of follicle-stimulating hormone (FSH)) is correct. The patient has Turners syndrome. Characteristic signs of Turners syndrome are primary amenorrhea, short stature, underdeveloped breasts, and a webbed neck (see photograph). Most patients have 45 chromosomes with an XO phenotype. This is due to nondisjunction of the sex chromosomes in the first stage of meiosis. No Barr bodies (randomly inactivated X chromosomes) are present. The ovaries of patients with Turners syndrome lack oocytes, which leads to decreased synthesis of estradiol and a concomitant increase in FSH. Option A (Decreased concentration of luteinizing hormone (LH)) is incorrect. In Turners syndrome, the ovaries lack oocytes; therefore, no progesterone is synthesized and serum LH levels are increased. Option B (Decreased concentration of serum growth hormone) is incorrect. An isolated deficiency of growth hormone produces short stature but does not affect the development of secondary female characteristics or the initiation of

menarche. Option D (Increased concentration of serum estradiol) is incorrect. In Turners syndrome, the ovaries lack oocytes; therefore, no estradiol is synthesized. Recall that estradiol is normally synthesized in the granulosa cells by conversion of testosterone into estradiol by aromatase. Option E (Increased concentration of serum testosterone) is incorrect. In Turners syndrome, the ovaries lack oocytes; therefore, no testosterone synthesis occurs. Recall that the theca interna around the developing follicle is normally responsible for synthesizing testosterone during the proliferative phase of the menstrual cycle and 17hydroxyprogesterone during the secretory phase of the cycle.

Study the following data on a test performed on 100 patients, who are known to have disease X, and a control group containing 100 people who do not have disease X. Which of the following conclusions can be drawn from this study?

Disease XControl Group Positive test 100 20 Negative test0 80

A. A negative test result excludes disease X. B. A positive test result confirms disease X. C. It is a poor screening test for disease X. D. The predictive value of a negative test result is 100%. E. The prevalence of disease is 60%.

Option A (A negative test result excludes disease X.) is correct. People with disease either have a true positive (TP) or a false negative (FN) test result. A TP test result is a positive test result in a person with disease, while a FN test result is a negative test result in a person with disease. People in a control group, who do not have disease, either have a true negative (TN) or a false positive (FP) test result. A TN test result is a negative test in a person without disease, while a FP test result is a positive test in a person without disease. The sensitivity of a test is the likelihood of having positive test results in patients who have a selected disease. Since people with disease either have TP or FN test results, the formula for calculating the sensitivity of a test is TP/ TP+FN. The less the FN rate, the greater the sensitivity of the test. The sensitivity of the test for disease X is 100/100+0=100%. Tests with 100% sensitivity are most often used as screening tests for disease. If the test result is negative, the likelihood (predictive value of a negative test result, PV-) is a TN rather than a FN is 100%, because a test with 100% sensitivity has no FNs. Since tests with 100% sensitivity always have a PV(TN/TN+FN) of 100%, a negative test result excludes disease. A positive test result includes all people with disease but does not confirm disease, because the test may be a FP in a person without disease. Other tests are often necessary to distinguish a TP from a FP test result. Disease XControl Group

Positive test 100 TP Negative test0 FN

20 FP 80 TN

Option B (A positive test result confirms disease X.) is incorrect. The predictive value of a positive test result (PV+) is the likelihood that a positive test is a TP rather than a FP. The formula for PV+is TP/TP+FP. For a test to have a positive predictive value (PV+) of 100%, the test must have no FPs. A test with no FPs has 100% specificity. Specificity of a test is the likelihood of having negative test results in people without disease. Since people without disease have test results that are either a TN or a FP, the formula for calculating the specificity is TN/TN+FP. Tests with 100% specificity always have a PV+ of 100%; therefore, they are most useful for confirming disease. The specificity of this test is 80/80+20=80%, and the PV+is 100/100+20=83%. There is an 83% chance that the test result is a TP and a 17% chance that it is a FP. Option C (It is a poor screening test for disease X) is incorrect. . Since the test has a sensitivity of 100% and a PV- of 100% the test is an excellent screening test. A negative test result (PV-) excludes disease, while a positive test result includes all people with disease, but does not confirm the presence of disease.. Option D (The predictive value of a negative test result is 100%.) is incorrect. The predictive value of a positive test result (PV+) is 83%. Option E (The prevalence of disease is 60%.) is incorrect. Prevalence is the total number of people with disease in the population under study which includes all the people with coronary artery disease and all the people in the control group. The formula is TP+FN/TP+FN+TN+FP. The prevalence of disease in this study is 100+0/100+0+80+20 = 50% (not 60%).

A 26-year-old man complains of dyspnea. He has a family history of lung disease developing at an early age. Physical examination shows inspiratory crackles and diminished breath sounds in the lower lobes of both lungs. A chest radiograph shows enlargement of the air spaces in both lower lobes. A serum protein electrophoresis most likely shows which of the following abnormalities? A. Absent 1-globulin peak B. Absent 2-globulin peak C. Absent -globulin peak D. Flat -globulin peak E. Monoclonal spike
Option A (Absent 1-globulin peak) is correct. The patient has alpha-1-antitrypsin (AAT) deficiency, which is an autosomal codominant disorder. Alleles are inherited codominantly (each allele expresses itself). The normal allele is M (95% frequency in U.S.). MM is the normal genotype with AAT in the normal range. The deficient variant (decreased

AAT) involves the Z allele (1% to 2% frequency). Another deficient variant (decreased AAT) is the S allele (2% to 3% frequency). Severe deficiency most commonly occurs in the homozygous ZZ variant, which is present in this patient. AAT levels are markedly decreased and patients develop panacinar emphysema, where the elastic tissue in the entire respiratory unit is affected. It most commonly occurs in the lower lobes. A serum protein electrophoresis shows absence of the 1-globulin peak, which normally corresponds with the presence of 1-antitrypsin. The treatment involves the use of pooled AAT given intravenously. Lung transplantation may be necessary. Option B (Absent 2-globulin peak) is incorrect. The 2-globulin peak contains haptoglobin. Haptoglobin is decreased in intravascular hemolysis, in which it forms a complex with hemoglobin and is removed from the circulation by macrophages. Option C (Absent -globulin peak) is incorrect. Proteins in the -globulin peak include transferrin and complement. Option D (Flat -globulin peak) is incorrect. The -globulin region of serum protein electrophoresis contains most of the immunoglobulins (e.g., IgG and IgM). IgG is the most abundant immunoglobulin; therefore, the normal -globulin peak reflects IgG. A flat -globulin peak indicates hypogammaglobulinemia. This is most often caused by a genetic disorder involving B cells (e.g., Brutons agammaglobulinemia, common variable immunodeficiency). Option E (Monoclonal spike) is incorrect. A monoclonal spike (needle-shaped spike) in the -globulin region indicates a malignancy of a single clone of plasma cells with production of a single immunoglobulin and its corresponding light chain (e.g., multiple myeloma).

A 65-year-old man on the 5th day of hospitalization for an acute anterior myocardial infarction has recurrence of chest pain and an increase in both creatine kinase (CK)-MB and troponin I and T. Examination of the heart and lungs is normal. Which of the following is most likely responsible for the laboratory test abnormalities?

A. Myocardial rupture B. Papillary muscle dysfunction C. Reinfarction D. Right ventricular infarction E. Ventricular aneurysm
Option C (Reinfarction) is correct. CK-MB isoenzymes, a marker for acute myocardial infarction, are usually gone by 3 days. Therefore, reappearance of CK-MB after 3 days indicates reinfarction or further extension of an existing myocardial infarction. Troponins I and T are the gold standard for diagnosing an acute myocardial infarction. However, troponin I remains increased for a week, while troponin T remains increased for 10 to 14 days; therefore, they cannot be used to diagnose a reinfarction. Option A (Myocardial rupture) is incorrect. Rupture of the myocardium either produces a murmur (e.g., mitral regurgitation from posteromedial papillary muscle infarction) or cardiac tamponade with muffling of the heart sounds

and jugular neck vein distention and muffled heart sounds. Neither of these findings are present in the patient. Option B (Papillary muscle dysfunction) is incorrect. The posteromedial papillary muscle is supplied by the right coronary artery. Dysfunction or infarction due to thrombosis of the artery should produce the pansystolic murmur of mitral valve regurgitation. Cardiac examination is normal in the patient. Option D (Right ventricular infarction) is incorrect. The right coronary artery supplies the right ventricle. Infarction of the right ventricle produces signs of right-sided heart failure, which include neck vein distention and the murmur of tricuspid valve regurgitation. These findings are not present in the patient. Option E (Ventricular aneurysm) is incorrect. Ventricular aneurysms are a late finding in an acute myocardial infarction. They produce a precordial bulge with systole due to blood entering the aneurysm and expanding it out. They do not produce reappearance of CK-MB.

The photograph shows a peripheral blood smear from a 23-year-old man. Which of the following biochemical processes occurs in the cell identified by the arrows?

A. Anaerobic glycolysis B. Citric acid cycle C. Fatty acid synthesis D. Gluconeogenesis E. Oxidative phosphorylation

Option A (Anaerobic glycolysis) is correct. The arrow in the peripheral blood smear points to a mature RBC. Since mature RBCs lack mitochondria, anaerobic glycolysis is the primary source of energy adenosine-5'-triphosphate (ATP) in RBCs. Lactic acid is the end-product of anaerobic glycolysis. Lactic acid is taken up by the liver, converted into pyruvate, which is then used to synthesize glucose by gluconeogenesis in the fasting state. The glucose that is produced is used by RBCs for fuel. This is called the Cori cycle. Several platelets (small cells with dark centers) are also present in the peripheral blood. Option B (Citric acid cycle) is incorrect. The citric acid cycle is present in the mitochondrial matrix. Mature RBCs do not have mitochondria. Option C (Fatty acid synthesis) is incorrect. RBCs do not have the enzymes necessary to synthesize fatty acids in the cytosol. Option D (Gluconeogenesis) is incorrect. RBCs lack gluconeogenic enzymes. The first step in gluconeogenesis (pyruvate conversion to oxaloacetate) occurs in the mitochondria and mitochondria are not present in mature RBCs. Option E (Oxidative phosphorylation) is incorrect. Oxidative phosphorylation occurs in the inner membrane of the mitochondria. Mature RBCs do not have mitochondria; therefore, they cannot use oxidative phosphorylation to supply ATP.

A 14-year-old boy has pain in both knees, a color change in his urine, and bloody stools that occurred a few weeks following an upper respiratory infection. Palpable and painful lesions are present on the ankles and feet (see the photograph) and buttocks. Both knees are swollen, warm, and painful. Cardiorespiratory examination is unremarkable. Urinalysis shows hematuria with dysmorphic RBCs, RBC casts, and proteinuria > 3.5 g/24 hours. Which of the following immune responses is the most likely pathogenesis of this disorder?

A. Anti-basement membrane antibodies B. Anti-neutrophil cytoplasmic antibodies C. Anti-nuclear antibodies D. IgA antibodies complexed with IgA

E. IgM antibodies complexed with IgG


Option D (IgA antibodies complexed with IgA) is correct. The distinctive skin rash on the lower extremities and buttocks, hematuria with RBC casts, gastrointestinal bleeding, and joint inflammation characterize Henoch-Schnlein purpura. It is a type III hypersensitivity reaction involving the deposition of IgAanti-IgA immunocomplexes in small vessels, glomeruli, joints, and gastrointestinal tract. A key feature of small vasculitis is palpable purpura, which is evident in the photograph. It is raised and painful, because it is an example of the tumor and dolor of acute inflammation. In the glomerulus, immunocomplexes deposited in the mesangium cause an acute, nephritic type of glomerulonephritis, which explains the presence of hematuria, dysmorphic RBCs (damaged by inflammation), RBC casts, and proteinuria. Gastrointestinal bleeding and polyarthritis are also common features. Previous upper respiratory infections or group A streptococcal pharyngitis frequently act as a trigger leading to antibody formation and eventual formation of immunocomplexes. Deposition of immunocomplexes in tissue activates the complement system, and chemotactic agents act as signals for neutrophils to enter the tissue producing acute inflammation. Option A (Anti-basement membrane antibodies) is incorrect. Antibodies directed against basement membrane antigens characterize Goodpasture syndrome. In this syndrome, IgG antibasement-membrane antibodies react against antigens in type IV collagen in the basement membranes of pulmonary and glomerular capillaries (type II hypersensitivity reaction). This causes pulmonary hemorrhage with hemoptysis and acute glomerulonephritis, respectively. Small vessel vasculitis producing palpable purpura, polyarthritis, and gastrointestinal bleeding are not present in Goodpasture syndrome. Option B (Anti-neutrophil cytoplasmic antibodies) is incorrect. Anti-neutrophil cytoplasmic antibodies (ANCA) are present in microscopic polyangiitis and Wegeners granulomatosis. Microscopic polyangiitis does produce palpable purpura and acute glomerulonephritis; however, it is not associated with polyarthritis and gastrointestinal bleeding. Wegeners granulomatosis is associated with necrotizing granulomas in the skin and upper and lower respiratory tract as well as a necrotizing vasculitis in the lungs and kidneys. Option C (Anti-nuclear antibodies) is incorrect. Anti-nuclear antibodies against DNA, histones, acidic nuclear proteins, and nucleolar antigens are commonly seen in autoimmune diseases such as systemic lupus erythematosus and systemic sclerosis. Option E (IgM antibodies complexed with IgG) is incorrect. Rheumatoid factor (RF) is an IgM that is directed against the Fc portion of IgG. When RF forms immunocomplexes with IgG in the joints (type III hypersensitivity reaction), it initiates the destruction of articular cartilage by pannus formation (granulation tissue) that characterizes rheumatoid arthritis. Small vessel vasculitis can occur in rheumatoid arthritis; however, acute glomerulonephritis and gastrointestinal bleeding are not features of the disease.

A 59-year-old woman has chronic renal failure. Urinalysis shows proteinuria > 3.5 g/24 hr and fatty casts. The photograph shows an H&E stain of a representative glomerulus in a kidney biopsy. Which of the following is the most likely diagnosis?

A. Crescentic glomerulonephritis B. Diffuse membranous glomerulopathy C. Minimal change disease D. Nodular glomerulosclerosis E. Type I membranoproliferative glomerulonephritis

Option D (Nodular glomerulosclerosis) is correct. The biopsy shows an arteriole that has an increase in proteinaceous material in the vessel wall (i.e., hyaline arteriolosclerosis). A mesangial nodule within the glomerulus contains type IV collagen and trapped protein. These findings are characteristic of diabetic nephropathy, specifically, nodular glomerulosclerosis. Proteinuria is due to nonenzymatic glycosylation (glucose attached to amino acids) of the glomerular basement membrane. Advanced glycosylation products render the membrane permeable to protein, resulting in proteinuria, often in the nephrotic range, as in this patient. Option A (Crescentic glomerulonephritis) is incorrect. Crescents are a proliferation of epithelial cells in Bowmans capsule. They are seen in rapidly progressive crescentic glomerulonephritis, which presents as a nephritic type of glomerulonephritis (RBC casts, mild to moderate proteinuria). Option B (Diffuse membranous glomerulopathy) is incorrect. Diffuse membranous glomerulopathy presents as a nephrotic syndrome. It is associated with thickening of the glomerular basement membrane; however, it does not show mesangial nodules or hyaline arteriolosclerosis of the arterioles. Option C (Minimal change disease) is incorrect. Minimal change disease (lipoid nephrosis) is associated with normal-appearing glomeruli. It is the most common cause of the nephrotic syndrome in children. Option E (Type I membranoproliferative glomerulonephritis) is incorrect. Type I membranoproliferative glomerulonephritis is associated with thickening of the glomerular basement membranes and an increase in glomerular cellularity. Hyaline arteriolosclerosis and mesangial nodules are not present. It usually presents as a nephrotic syndrome and less commonly has a nephritic presentation. It has a strong association with hepatitis C.

A 22-year-old man has blood oozing from his nose and mouth. Physical examination shows petechiae and ecchymoses distributed over most of his body. There is generalized lymphadenopathy and hepatosplenomegaly. Laboratory studies show a moderately severe normocytic anemia, thrombocytopenia, and a WBC count of 32,000/mm3. The d-dimer blood test is increased and both the prothrombin and partial thromboplastin times are increased. The photograph shows the peripheral blood smear. What chromosome translocation is most likely to be present in this patient?

A. t(8;14) B. t(9;22) C. t(12;21) D. t(14;18) E. t(15;17)

Option E (t(15;17)) is correct. The patient has acute promyelocytic leukemia (APL) complicated by disseminated intravascular coagulation (DIC). The peripheral blood smear shows a hypergranular promyelocyte filled with multiple Auer rods. Auer rods are present only in variants of acute myeloblastic leukemia. Patients with APL have a characteristic t(15;17) translocation, which causes an abnormality in retinoic acid metabolism. DIC is invariably present in APL, because the release of procoagulants from the granules of the promyelocytes activates the coagulation system cascade. DIC causes multiple coagulation factor deficiencies (prolonged prothrombin time and partial thromboplastin time), thrombocytopenia (which causes petechiae and ecchymoses), and activation of the fibrinolytic system (increased d-dimers). Leukemias commonly metastasize to the lymph nodes (generalized lymphadenopathy), liver, and spleen (hepatosplenomegaly). Treatment of APL with retinoic acid causes maturation of the promyelocytes; however, relapses invariable occur leading to death. Option A (t(8;14)) is incorrect. A t(8;14) translocation is associated with Burkitts lymphoma, which is a malignancy involving B cells. Option B (t(9;22)) is incorrect. A t(9;22) translocation is associated with chronic myelogenous leukemia

(CML). In CML, Auer rods are not present in the myeloblasts, which account for <10% of the myeloid cells. The ABL-proto-oncogene on chromosome 9 is translocated to chromosome 22 where it fuses with the break cluster region to form a fusion gene. Chromosome 22 is called the Philadelphia chromosome. Option C (t(12;21)) is incorrect. A t(12;21) translocation has prognostic significance in acute lymphoblastic leukemia. When present, the prognosis is favorable. Option D (t(14;18)) is incorrect. A t(14;18) translocation is associated with a follicular B-cell lymphoma. The translocation causes overexpression (increased activity) of the BCL2 family of genes. The protein product of the gene prevents apoptosis of the cell by preventing cytochrome c from leaving the mitochondria. Therefore, the translocation ensures immortality of the B cells in the germinal follicles of lymph nodes.

A 28-year-old man with AIDS has chronic nonbloody, watery diarrhea. The photograph shows a biopsy of the small intestine. Which of the following is the most likely pathogen to be identified?

A. Balantidium coli B. Cryptosporidium parvum C. Entamoeba histolytica D. Giardia lamblia E. Strongyloides stercoralis
Option B (Cryptosporidium parvum) is correct. The pathogen identified is C. parvum, a sporozoan that is contracted by ingesting the oocysts. The photograph shows small, round oocysts lining the luminal surface of the small intestine. The oocysts are partially acid fast. C. parvum is one of the most common pathogens causing diarrhea and biliary tract disease in AIDS, particularly when the CD4 helper T-cell count < 100/mm3. The treatment is nitazoxanide. Option A (Balantidium coli) is incorrect. B. coli is a ciliate protozoan that produces colonic ulcers, causing bloody diarrhea. Option C (Entamoeba histolytica) is incorrect. E. histolytica is an ameba with trophozoites that produce flask-shaped ulcers in the cecum, causing bloody diarrhea. Trophozoites common phagocytose RBCs.

Option D (Giardia lamblia) is incorrect. G. lamblia is a flagellate protozoan with cysts that attach to the small intestine mucosa, causing acute and chronic diarrhea with malabsorption. Option E (Strongyloides stercoralis) is incorrect. S. stercoralis is the most common helminthic infection causing diarrhea in AIDS patients. Adult worms lay eggs in the bowel mucosa that develop into rhabditiform larvae, which are excreted in the feces. It is a common parasitic infection in AIDS; however, filariform larvae would be seen in the bowel lumen.

A 48-year-old man complains of fever and several fainting spells over the past few months. He states that he faints when he stands up and not when he is lying down. He also complains of pain in the left upper quadrant that is aggravated by inspiration as well as pain in the right flank. Physical examination shows a normal blood pressure when lying down and sitting up. A late diastolic murmur is heard. The spleen is enlarged and tender, and a splenic friction rub is present. There is right flank pain on percussion. A urine dipstick is positive for blood, and RBCs are present in the urine sediment. What is the most likely diagnosis?

A. Calcific aortic stenosis B. Hypertrophic cardiomyopathy C. Left atrial myxoma D. Mitral stenosis E. Pericardial effusion
Option C (Left atrial myxoma) is correct. Cardiac myxomas are the most common primary cardiac tumors in adults (see Fig. 10-28 in Rapid Review Pathology, 3rd edition). Symptoms include nonspecific complaints such as fever and malaise. The tumor has a ball-valve effect that causes sudden blockage of blood flow through the mitral valve, resulting in episodic fainting spells. A diastolic murmur similar to that of mitral stenosis is also present. Peripheral embolization of tumor also occurs. Infarctions of the spleen cause pain in the left upper quadrant and friction rubs. Infarctions of the kidneys cause flank pain and hematuria. Option A (Calcific aortic stenosis) is incorrect. Calcific aortic stenosis is associated with a systolic ejection murmur. Angina with exercise occurs because of ischemia of the subendocardium in the concentrically hypertrophied left ventricle. The decreased cardiac output through the stenotic valve causes syncope with exercise. Peripheral embolization does not occur. Option B (Hypertrophic cardiomyopathy) is incorrect. Hypertrophic cardiomyopathy is characterized by asymmetric hypertrophy of the interventricular septum causing findings similar to those described for calcific aortic stenosis. Peripheral embolization does not occur. Option D (Mitral stenosis) is incorrect. Mitral stenosis is a complication of chronic rheumatic fever. Left atrial dilation and thrombus formation with embolization is a common finding. The patient has no history of recurrent rheumatic fever and physical exam does not reveal an opening snap in early diastole in this patient.

Option E (Pericardial effusion) is incorrect. A pericardial effusion produces muffled heart sounds and is not associated with syncope or with peripheral embolization.

A 63-year-old man has lesions on his tongue (see photograph). Which of the following is the greatest risk factor for developing this type of lesion?

A. Excessive consumption of alcohol B. Glossitis due to Candida albicans C. Glossitis due to Epstein-Barr virus D. Glossitis due to vitamin B12 deficiency E. Smoking cigarettes
Option E (Smoking cigarettes) is correct. The photograph shows discrete raised white patches (leukoplakia) on both sides of the tongue. The lesions on the left side of the tongue appear to be displacing the midline of the tongue to the right and are due to an invasive squamous cell carcinoma. The right side of the tongue shows a leukoplakic patch on the right border of the tongue posteriorly. A biopsy of the lesion showed squamous dysplasia, a precursor lesion for squamous cell cancer. This observation underscores the necessity for performing a biopsy of leukoplakic lesions in the oral cavity to distinguish benign epithelial lesions from precancerous or cancerous lesions. Cigarette smoking is the greatest risk factor for squamous cell cancer in the mouth and oropharynx. Pipe smoking and smokeless tobacco are also risk factors. Option A (Excessive consumption of alcohol) is incorrect. Alcohol excess is a risk factor for squamous cell cancer in the

mouth, but it is not as strong a risk factor as smoking. If the patient is a smoker and abuses alcohol, the risk for squamous cancer is markedly increased. Option B (Glossitis due to Candida albicans) is incorrect. Oral candidiasis is characterized by the presence of a white pseudomembrane overlying the surface of the tongue. It does not distort the architecture of the tongue. Option C (Glossitis due to Epstein-Barr virus) is incorrect. Hairy leukoplakia is a glossitis caused by the Epstein-Barr virus that is commonly seen in the setting of HIV infection. It is not a precancerous lesion. White patches usually are located on the lateral border of the tongue. Hairy leukoplakia frequently occurs when the CD4 helper T-cell count is 200 to 500 cells/mm3. Option D (Glossitis due to vitamin B12 deficiency) is incorrect. The glossitis in vitamin B12 deficiency is characterized by atrophy of the papillae, resulting in a smooth-surfaced tongue without leukoplakic lesions. It is not a precancerous lesion.

A 25-year-old woman with a previous history of head trauma now has polyuria and increased thirst. A water deprivation test is abnormal. Which of the following letters in the schematic best represents this patients findings after the water deprivation test? The square represents the normal values for POsm and UOsm.

A. decreased POsm, decreased UOsm B. increased POsm, decreased UOsm C. increased POsm, increased UOsm

D. decreased POsm, increased UOsm

Option B (increased POsm, decreased UOsm) is correct. The patient has central diabetes insipidus due to transection of the pituitary stalk from head trauma. Due to a lack of antidiuretic hormone (ADH), the patient is always diluting her urine and never concentrating urine. Therefore, in a water deprivation test, the P Osm will increase (stimulates thirst), due to excess loss of free water in the urine, and the UOsm will decrease, due to excess free water in the urine (dilution) because ADH is absent. The treatment is desmopressin acetate. Option A (decreased POsm, decreased UOsm) is incorrect. This occurs in normal dilution of urine. As P Osm decreases, ADH release is suppressed and free water is lost in the urine to produce a decreased U Osm. Option C (increased POsm, increased UOsm) is incorrect. This occurs in normal concentration of urine. Increased POsm stimulates the release of ADH, which reabsorbs free water out of the collecting tubule to concentrate urine. Option D (decreased POsm, increased UOsm) is incorrect. This occurs with ectopic secretion of ADH by a small cell carcinoma of the lung causing the syndrome of inappropriate ADH. With constant stimulation by ADH, the kidneys are always concentrating urine (reabsorbing free water) and never diluting. Adding the free water to plasma produces a dilutional hyponatremia.

The photograph shows the distal femur of a 21-year-old woman. When the patient was 2 years old, a cancerous lesion was surgically removed from the right eye. Which of the following regulatory genes is most likely responsible for these eye and bone lesions?

A. BRCA1 suppressor gene B. MYC proto-oncogene C. RAS proto-oncogene D. RB suppressor gene

E. TP53 suppressor gene


Option D (RB suppressor gene) is correct. The patient has osteosarcoma and a history of retinoblastoma, which together indicate inactivation of the RB suppressor gene on chromosome 13. Osteosarcoma is a tumor of the connective tissue that arises from osteocytes. They most commonly occur in the metaphysis of the distal femur or proximal tibia. In the figure, the osteosarcoma appears as a gray-white necrotic tumor in the metaphysis that extends through the cortex into the surrounding connective tissue. Inactivation of the RB suppressor gene may be sporadic or inherited as an autosomal dominant trait. In the latter type, one of the alleles on chromosome 13 is inactivated in utero. After birth, only one additional mutation must occur on the remaining allele to produce cancer (one hit theory). The patient then is at risk for developing a retinoblastoma, the most common malignancy of the eye in children. However, there is also an increased risk for developing an osteosarcoma when the patient is between 10 and 25 years of age. Option A (BRCA1 suppressor gene) is incorrect. The BRCA1 suppressor gene is involved in DNA repair. Inactivation of the gene is associated with breast cancer in women and prostate cancer in men. Option B (MYC proto-oncogene) is incorrect. The c-MYC and n-MYC proto-oncogenes are involved in nuclear transcription. Activation of the c-MYC proto-oncogene by a t(8;14) translocation produces Burkitts lymphoma, whereas activation of the n-MYC proto-oncogene produces a neuroblastoma. Option C (RAS proto-oncogene) is incorrect. The RAS proto-oncogene is a signal transducer that generates second messengers. RAS is activated by a point mutation and accounts for 30% of human cancers (e.g., cancers of the lung, colon, and pancreas; leukemias). It is not associated with retinoblastoma and osteosarcoma. Option E (TP53 suppressor gene) is incorrect. The TP53 suppressor gene is involved in DNA repair. Inactivation of the gene by a point mutation allows unregulated proliferation of the cell. Although TP53 accounts for 70% of all cancers (e.g., cancers of the colon, breast, lung, and brain), it is not associated with retinoblastoma and osteosarcoma.

The umbilical cord of a 6-week-old infant boy has not sloughed off after 3 weeks. The cord is removed surgically and histologic sections show an absence of neutrophil margination along the umbilical vessels and absence of neutrophil transmigration into interstitial tissue. Which of the following defects of neutrophil function is most likely responsible?

A. Absent respiratory burst B. Leukocyte adhesion molecule defect C. Myeloperoxidase (MPO) deficiency D. Opsonization defect E. Phagocytosis defect

Option B (Leukocyte adhesion molecule defect) is correct. A congenital leukocyte adhesion molecule defect prevents separation of the umbilical cord. Adhesion molecules activated on neutrophils include selectins and -integrins (CD11 CD18 positive). Selectins are responsible for rolling of the neutrophils in the venules, while -integrins cause neutrophils to adhere to venules (margination). Neutrophils then release collagenase, dissolve basement membranes between contracted endothelial cells in venules, and transmigrate into the interstitial tissue. Deficiency of either type of adhesion molecule causes an absence of neutrophil adhesion (margination) and an absence of neutrophils in the interstitial tissue, because neutrophils must adhere to endothelium before they can transmigrate. Other findings in leukocyte adhesion defects include an increase in the absolute neutrophil count, problems with wound healing, and severe gingivitis. Option A (Absent respiratory burst) is incorrect. The respiratory (oxidative) burst in neutrophils and monocytes is part of the O2-dependent MPO system for killing bacteria. Activated nicotinamide adenosine dinucleotide phosphate (NADPH) oxidase in the cell membrane oxidizes reduced NADPH, converting molecular O2 to superoxide free radicals (O2-). The respiratory burst is the energy released in this reaction. In chronic granulomatous disease, an X-linked recessive disease caused by absence of NADPH oxidase, the respiratory burst is absent, resulting in a defect in microbicidal activity. A functioning O2-dependent MPO system is not required for cord separation, because it does not affect neutrophil adhesion or transmigration. Option C (Myeloperoxidase (MPO) deficiency) is incorrect. In the O2-dependent MPO system, O2 is converted to peroxide (H2O2) by superoxide dismutase in the phagolysosomes of neutrophils and monocytes. MPO catalyzes a reaction that combines H2O2 with Cl to form hypochlorous free radicals that kill the phagocytosed bacteria. Deficiency of MPO results in a defect in microbicidal activity; however, it does not affect neutrophil adhesion and transmigration. Option D (Opsonization defect) is incorrect. IgG and C3b are opsonizing agents that bind to the surface of bacteria. Receptors for IgG and C3b are located on the plasma membranes of phagocytic leukocytes. Binding of the opsonized bacteria to leukocytes facilitates phagocytosis of the bacteria. A deficiency of IgG or C3 produces a defect in phagocytosis; however, it has no effect on neutrophil adhesion or transmigration. Option E (Phagocytosis defect) is incorrect. Leukocyte phagocytic defects include defects in opsonization and defects in the formation of phagolysosomes. Phagolysosomes are produced by fusion of lysosomes containing hydrolytic enzymes with phagosomes. In Chdiak-Higashi syndrome, a defect in lysosomal degranulation into phagosomes is present. Leukocytes contain large azurophilic granules (lysosomes) in the cytosol and there are defects in opsonization and in the formation of phagolysosomes; however, there are no defects in neutrophil adhesion or transmigration.

A 50-year-old man had an acute anterior myocardial infarction (MI) 4 days ago, and he now complains of substernal chest pain with radiation into the left arm and jaw. Physical examination shows no cardiac or pulmonary abnormalities. Laboratory studies show increases in serum creatine kinase MB (CK-MB), serum troponin-I (cTnI), and serum troponin-T (cTnT). Which of the following is the most likely cause of the chest pain?

A. Angina pectoris B. Pericarditis

C. Reinfarction D. Right ventricular infarction E. Rupture of the anterior wall


Option C (Reinfarction) is correct. After an uncomplicated acute MI, serum CK-MB peaks in 24 hours and returns to normal within 72 hours. Therefore, the presence of CK-MB on day 4, after an MI, indicates reinfarction. Both cTnI and cTnT peak in 24 hours and return to normal within 7 to 10 days following an uncomplicated MI. The presence of cTnI and cTnT on day 4 is expected. Option A (Angina pectoris) is incorrect. Angina pectoris causes myocardial ischemia without causing myocardial cell injury. Angina pectoris does not lead to an increase in CK-MB, troponin-I, and troponin-T. Option B (Pericarditis) is incorrect. Pericarditis is inflammation of the surface lining of the heart, causing precordial chest pain and a pericardial friction rub. Pericarditis does not damage the myocardial tissue and lead to an increase in the cardiac enzymes. Option C (Right ventricular infarction) is incorrect. Isolated right ventricular infarction is extremely rare, because the blood vessels perfusing the right ventricle are usually too small to develop occluding atheromas. Right ventricular infarction produces signs of right-sided heart failure (neck vein distention, dependent pitting edema), which are not present in the patient. Option D (Rupture of the anterior wall) is incorrect. Rupture caused by necrosis of the myocardial wall usually occurs on days 3 to 7 following an MI. Rupture of the anterior wall produces cardiac tamponade with muffled heart sounds and neck vein distention followed by rapid death. The patient has a normal cardiac exam.

A 2-year-old boy has a history of recurrent abdominal colic and diarrhea. The child lives in an old apartment complex and is repeatedly putting things in his mouth. His conjunctiva are pale, and his height and weight are below normal for his age. Laboratory studies show a moderately severe microcytic anemia. The photographs show a representative section of the peripheral blood smear (and an abdominal radiograph. Which of the following is the most appropriate diagnostic test for this child?

A. Direct Coombs test B. Hemoglobin electrophoresis C. Osmotic fragility test D. Serum ferritin test E. Serum lead test
Option E (Serum lead test) is correct. The child has lead poisoning, most likely caused by ingestion of lead-based paint. The peripheral smear shows hypochromic RBCs (increased central areas of pallor) and coarse basophilic stippling are present in one RBC, indicating the persistence of ribosomes. Lead, a heavy metal, denatures enzymes, deposits in the epiphyses of bone, and causes encephalopathy and peripheral neuropathy. The microcytic anemia is due to denaturation of ferrochelatase, which normally combines iron with protoporphyrin to form heme in the mitochondria. A decrease in heme causes a decrease in hemoglobin synthesis resulting in a microcytic hypochromic anemia. Lead also denatures ribonuclease, which normally degrades ribosomes. This explains the presence of coarse basophilic stippling. Lead deposits in the epiphyses of bone explain the patients growth retardation. Lead is also easily identified in an abdominal radiograph as metallic densities, which is evident in this patient. Since -aminolevulinic acid is proximal to the enzyme block in heme synthesis, it is increased and is responsible for the encephalopathy (demyelination and cerebral edema) and neuropathy in lead poisoning (abdominal colic, peripheral neuropathies). An increase in serum lead level confirms the diagnosis Option A (Direct Coombs test) is incorrect. A direct Coombs test detects IgG and/or C3b on the surface of the RBCs. It is used when an autoimmune hemolytic anemia is suspected. Option B (Hemoglobin electrophoresis) is incorrect. Hemoglobin electrophoresis detects changes in the concentration

of normal and abnormal forms of hemoglobin (e.g., hemoglobin S in sickle cell disease). Option C (Osmotic fragility test) is incorrect. The osmotic fragility test is used to confirm a diagnosis of hereditary spherocytosis, in which the osmotic fragility of RBCs is increased due to membrane damage from a defect in ankyrin. Hereditary spherocytosis produces a normocytic anemia. Numerous dense-appearing RBCs are present in the peripheral smear and coarse basophilic stippling is not present. Option D (Serum ferritin test) is incorrect. Serum ferritin levels increase in lead poisoning. Iron that enters the nucleated normoblasts in the bone marrow cannot combine with protoporphyrin and accumulates in the mitochondria producing a ringed sideroblast.When these iron-overloaded ringed sideroblasts die, the excess iron is taken up by bone marrow macrophages and converted into ferritin. Since ferritin stores in macrophages increase, the serum ferritin is also increased. Serum ferritin is increased in other disorders (e.g., anemic of chronic disease); therefore, it is not a specific enough test to diagnose lead poisoning.

A 59-year-old man is on hemodialysis for chronic renal failure (CRF). The photograph shows a urine sediment finding. Which of the following additional laboratory findings would most likely be reported?

A. Decreased serum parathyroid hormone B. Decreased serum phosphorus C. Increased serum calcium D. Increased urine-specific gravity > 1.020 E. Serum blood urea nitrogen (BUN):creatinine ratio < 15

Option E (Serum blood urea nitrogen (BUN):creatinine ratio < 15) is correct. The photograph shows an acellular (waxy) cast with sharp, refractile borders. The cast is usually found in the sediment of patients with CRF. Tubular dysfunction is present in CRF, resulting in decreased clearance of both creatinine and urea by the kidneys. Initially, this results in a proportionate increase in both the serum BUN and serum creatinine, which should maintain the normal ratio of BUN:creatinine at 15. However, creatinine is also lost in the skin and gastrointestinal tract (extrarenal sites); hence, the

ratio < 15 (e.g., serum BUN 80 mg/dL and serum creatinine 8 mg/dL; ratio 10), which is a characteristic finding of CRF. Option A (Decreased serum parathyroid hormone) is incorrect. In CRF, the second hydroxylation of vitamin D is impaired due to the loss of 1-hydroxylase in the proximal renal tubules. Hypovitaminosis D occurs with development of hypocalcemia, which is a stimulus for the synthesis and release of parathyroid hormone (secondary hyperparathyroidism). Option B (Decreased serum phosphorus) is incorrect. In CRF, acidification of urine is impaired causing retention of phosphorus in the blood, leading to hyperphosphatemia (not hypophosphatemia). Option C (Increased serum calcium) is incorrect. In CRF, the second hydroxylation of vitamin D is impaired due to the loss of 1-hydroxylase in the renal tubules. Hypovitaminosis D occurs with development of hypocalcemia (not hypercalcemia). Option D (Increased urine-specific gravity > 1.020) is incorrect. An increase in urine-specific gravity (urine density) corresponds with an increase in urine osmolality. Both measurements reflect the ability of the renal tubules to concentrate and dilute urine. In CRF, neither concentration nor dilution is normal. Therefore, the urine-specific gravity is essentially the same as the glomerular filtrate, which is approximately 1.010. The specific gravity remains unchanged regardless of time of day and fluid intake; hence, the term fixed specific gravity, a sign of CRF.

A 3-year-old boy ingests an unknown quantity of rat poison. He is bleeding from the nose, mouth, and gastrointestinal tract. Multiple ecchymoses are also present. Which of the following sets of hemostasis studies is most likely to be present in this patient?

Platelet CountBleeding TimePT PTT A.Normal Normal B. Normal Normal Normal C. D.Normal E. Normal Normal PT, prothrombin time; PTT, partial thromboplastin time. A. platelet count normal, bleeding time increased, PT normal, PTT increased B. platelet count normal, bleeding time normal, PT normal, PTT increased C. platelet count decreased, bleeding time increased, PT increased, PTT increased D. platelet count normal, bleeding time increased, PT increased, PTT increased E. platelet count normal, bleeding time normal, PT increased, PTT increased

Option E (platelet count normal, bleeding time normal, PT increased, PTT increased) is correct. Rat poison contains warfarin, which blocks epoxide reductase, rendering vitamin K inactive. This prevents further -carboxylation of the vitamin Kdependent coagulation factors: factors II (prothrombin), VII, IX, and X. Since factors X and II are in the final common pathway, both the PT and PTT are prolonged. The PT evaluates factors VII, X, V, II, and I (fibrinogen), while the PTT evaluates factors XII, XI, IX, VIII, X, V, II, and I. The platelet count and bleeding time are not affected by warfarin. Since the patient has serious bleeding, fresh frozen plasma is the treatment of choice. Option A (platelet count normal, bleeding time increased, PT normal, PTT increased) is incorrect. This set of studies is most often associated with classic von Willebrand disease. In this autosomal dominant disorder, there is a deficiency of von Willebrand factor (vWF), which is necessary for platelet adhesion to areas of endothelial injury. This prolongs the bleeding time without affecting the platelet count. In the circulation, vWF also complexes with factor VIII coagulant (VIII:C), which prevents degradation of VIII:C. Therefore, deficiency of vWF automatically leads to decreased factor VIII:C activity and prolongation of the PTT. The PT is normal because it does not evaluate VIII:C activity. Option B (platelet count normal, bleeding time normal, PT normal, PTT increased) is incorrect. This set of studies is consistent with a coagulation factor deficiency in the intrinsic coagulation pathway: XII, XI, IX, or VIII. Of the four choices, factor VIII would be the most common deficiency in a patient with hemophilia A. Option C (platelet count decreased, bleeding time increased, PT increased, PTT increased) is incorrect. This set of data in which all the tests are abnormal is most often seen in disseminated intravascular coagulation (DIC). Tissue thromboplastin activates the extrinsic coagulation system causing the formation of fibrin thrombi within the microcirculation. Factors I, II, V, and VIII are consumed in the fibrin clots, which results in anticoagulation (PT and PTT are both increased). Fibrin thrombi also trap platelets causing thrombocytopenia, which, in turn, is responsible for producing a prolonged bleeding time. Option D (platelet count normal, bleeding time increased, PT increased, PTT increased) is incorrect. This set of studies is uncommon but would most commonly be seen in primary fibrinolysis, which may occur in radical prostate surgery. In this setting, urokinase is released from the tissue causing activation of plasminogen and the release of plasmin. Plasmin degrades multiple coagulation factors (e.g., V, VIII, fibrinogen). This increases the PT and PTT, because factors V and VIII are in the final common pathway. The degradation products of fibrinogen interfere with platelet aggregation causing an increase in the bleeding time. The platelet count is not affected. The test for fibrinogen degradation products is positive; however, d-dimers are negative because fibrin clots are not present in primary fibrinolysis.

A sexually active 25-year-old woman complains of irritation in the anogenital region. The photograph shows lesions on the vulva. A rapid plasma reagin (RPR) test is negative. Which of the following is the most likely pathogen causing these lesions?

A. Candida albicans B. Chlamydia trachomatis, subtype C. Herpes simplex virus type 2 (HSV-2) D. Human papillomavirus (HPV) E. Treponema pallidum
Option D (Human papillomavirus (HPV)) is correct. The vulva has numerous keratotic papillary processes covering the labia. The patient has condyloma acuminatum (venereal warts), a sexually transmitted disease caused by HPV types 6 or 11. It is the most common sexually transmitted disease in the United States. The lesions develop in moist areas of the anogenital region. Approximately 90% spontaneously clear within 2 years (most within 8 months). Older women will more often have persistent disease. They are usually treated with topical podophyllin. Other treatment modalities include -interferon or imiquimod. Option A (Candida albicans) is incorrect. Candida causes a white vaginal discharge and intense erythema of the skin and vaginal mucosa. Option B (Chlamydia trachomatis, subtype) is incorrect. A subtype of Chlamydia causes lymphogranuloma venereum. This sexually transmitted disease produces papular lesions and granulomatous microabscesses in the inguinal lymph nodes. Option C (Herpes simplex virus type 2 (HSV-2)) is incorrect. HSV-2 produces vesicles, pustules, and ulcers in the vulvar region. Option E (Treponema pallidum) is incorrect. Condyloma latum is a lesion found in secondary syphilis that is often confused with condyloma acuminatum. Condyloma latum occurs in moist areas in the anogenital region, where the lesions tend to be papular and smooth-surfaced. Because the patients RPR test is negative, secondary syphilis is excluded.

A 34-year-old missionary in India develops cholera. Physical examination shows dry mucous membranes, poor skin turgor, and hypotension. Which one of the following electrolyte profiles is most likely to be present in this man?

Serum Na+ Serum K+ Serum Cl Serum HCO3 (135147 mEq/L) (3.55.0 mEq/L) (95105 mEq/L) (2228 mEq/L) A.128 5.9 96 20 B. 146 5.5 104 18 C. 138 2.2 114 14 D.130 2.9 80 36 E. 152 2.8 110 33

A. serum Na+ 128, serum K+ 5.9, serum Cl 96, serum HCO3 20 B. serum Na+ 146, serum K+ 5.5, serum Cl 104, serum HCO3 18 C. serum Na+ 138, serum K+ 2.2, serum Cl 114, serum HCO3 14 D. serum Na+ 130, serum K+ 2.9, serum Cl 80, serum HCO3 36 E. serum Na+ 152, serum K+ 2.8, serum Cl 110, serum HCO3 33
Option C (serum Na+ 138, serum K+ 2.2, serum Cl 114, serum HCO3 14) is correct. These findings are most compatible with a secretory type of diarrhea, in this case, cholera caused by Vibrio cholerae. The bacteria produces a toxin that activates adenylate cyclase in enterocytes causing a secretory diarrhea with the loss of isotonic fluid. Loss of isotonic fluid does not alter the serum Na+ concentration. Diarrheal fluid is rich in K+ and HCO3, the former resulting in hypokalemia and the latter a normal anion gap type of metabolic acidosis. The anion gap calculation is as follows: anion gap = serum Na+ 138 ( (serum Cl 114 + serum HCO314) = 10 mEq/L (12 mEq/L ( 2). The serum Cl is increased, because it replaces the HCO3 anions that are lost in the diarrheal fluid to maintain electroneutrality. Option A (serum Na+ 128, serum K+ 5.9, serum Cl 96, serum HCO3 20) is incorrect. This electrolyte profile is most compatible with mineralocorticoid deficiency (e.g., Addisons disease, 11-hydroxylase deficiency). There is inhibition of Na+ reabsorption and K+ secretion by the aldosterone-dependent Na+ and K+ channels located in the distal tubule and collecting ducts causing a hypertonic loss of Na+ in the urine (hyponatremia) and retention of K+ (hyperkalemia with peaked T waves). Due to dysfunction of the aldosterone-dependent H+/K+ ATPase pump in the collecting tubules, there is retention of H+ ions and a subsequent lack of synthesis of HCO3, which produces metabolic acidosis. The excess H+ ions combine with Cl anions producing a normal anion gap type of metabolic acidosis, calculated as follows: anion gap = serum Na+ 128 (serum Cl 96 + serum HCO3 20) = 12 mEq/L. Option B (serum Na+ 146, serum K+ 5.5, serum Cl 104, serum HCO3 18) is incorrect. These findings are most compatible with chronic renal failure. In chronic renal failure there is tubular cell dysfunction resulting in retention of K + (hyperkalemia with peaked T waves on an ECG) and an increased anion gap type of metabolic acidosis due to retention of organic acids like sulfuric and phosphaturic acid. The anion gap in this case is calculated as follows: anion gap = serum Na+ 146 (serum Cl 104 + serum HCO318) = 24 mEq/L (12 mEq/L 2). Option D (serum Na+ 130, serum K+ 2.9, serum Cl 80, serum HCO3 36) is incorrect. This profile is compatible with metabolic alkalosis due to loop diuretics, thiazide diuretics, or vomiting. Thiazide and loop diuretics inhibit channels in the nephron causing decreased reabsorption of Na+. This causes a hypertonic loss of Na+ (hyponatremia) and Cl (hypochloremia). Increased delivery of Na+ to the late distal and collecting tubules results in augmented exchange of Na + for K+ in the aldosterone-dependent Na+ channels resulting in increased urinary loss of K+ (hypokalemia with U waves on an ECG). When K+ is depleted, Na+ exchanges with H+ ions causing increased synthesis and reabsorption of HCO3 causing metabolic alkalosis. In vomiting, hydrochloric acid is lost. For every H+ ion lost in the vomitus, there is a corresponding HCO3 left unneutralized in the blood causing metabolic alkalosis (HCO3> 28 mEq/L). Option E (serum Na + 152, serum K+ 2.8, serum Cl 110, serum HCO3 33) is incorrect. This profile is compatible with mineralocorticoid excess (e.g., primary aldosteronism; 11-hydroxylase deficiency). Enhanced function of aldosterone-

mediated Na+-H+ channels in the late distal and collecting ducts increases the synthesis of HCO3 leading hypernatremia, hypokalemia (initially exchanged with Na+ until depleted), and metabolic alkalosis (Na+ exchanges with H+; for every H+ lost in the urine there is a corresponding HCO3 entering the blood).

A 31-year-old man has pruritic vesicles on the extensor surface of the elbow. The patient has a 10-year history of chronic diarrhea characterized by excessive flatulence and greasy stools. Which of the following laboratory findings is most likely to be reported?

A. Antidouble-stranded DNA antibodies B. Antigliadin antibodies C. Antimicrosomal antibodies D. Antimitochondrial antibodies E. Antinuclear antibodies
Option B (Antigliadin antibodies) is correct. The location of the painful vesicular lesions on the extensor surface of the elbow and the history of chronic diarrhea with greasy stools suggest dermatitis herpetiformis associated with glutensensitive enteropathy (celiac disease). The skin lesion is due to the deposition of IgA along the tips of the dermal papillae, resulting in an immune reaction that produces subepidermal vesicles. IgA and IgG antibodies formed against the gliadin protein fraction in gluten (component of wheat) cross-react with reticulin, which normally anchors the epidermal basement membrane to the superficial dermis. Most patients with dermatitis herpetiformis have flattening of the small bowel villi and inflammation of the lamina propria indicative of celiac disease. Antigliadin and antireticulin antibodies are commonly found in these patients. Antiendomysial antibodies directed against an enzyme that degrades gliadin peptides are also present. Option A (Antidouble-stranded DNA antibodies) is incorrect. Antidouble-stranded DNA antibodies are present in systemic lupus erythematosus (SLE). SLE is not associated with celiac disease. Option C (Antimicrosomal antibodies) is incorrect. Antimicrosomal antibodies are present in Hashimotos thyroiditis, which has no association with dermatitis herpetiformis or celiac disease. Option D (Antimitochondrial antibodies) is incorrect. Antimitochondrial antibodies are present in primary biliary cirrhosis, which is characterized by autoimmune destruction of bile ducts in the portal triads. There is no association with dermatitis herpetiformis and celiac disease. Option E (Antinuclear antibodies) is incorrect. Antinuclear antibodies are present in SLE. SLE is not associated with celiac disease.

A febrile 65-year-old man with prostate hyperplasia and urinary retention develops endotoxic shock. Within 24 hours, he has oozing of blood from all needle puncture sites, extensive ecchymoses and petechiae, and gastrointestinal bleeding. Laboratory studies show a hemoglobin of 9 g/dL, platelet count 75,000/mm3, prothrombin time (PT) 20 seconds, partial thromboplastin time, activated partial thromboplastin time (aPTT) 50 sec, and a positive d-dimer assay. What is the most likely diagnosis?

A. Autoimmune thrombocytopenia B. Circulating anticoagulant C. Disseminated intravascular coagulation (DIC) D. Primary fibrinolysis E. Thrombotic thrombocytopenic purpura (TTP)
Option C (Disseminated intravascular coagulation (DIC)) is correct. The endotoxins in endotoxic shock (most often due to Escherichia coli sepsis) damage tissue, causing the release of tissue thromboplastin. This activates the extrinsic coagulation system, causing DIC. Septic shock is the most common cause of DIC. In DIC, fibrin clots are produced in the microcirculation that obstruct blood flow and consume coagulation factors, causing bleeding from needle puncture sites and the gastrointestinal tract. Factors that are normally consumed in a fibrin clot are fibrinogen (factor I), prothrombin (factor II), and factors V and VIII. Since coagulating factors present in the final common pathway are consumed (fibrinogen, prothrombin, factor V), there is prolongation of the PT and aPTT. Platelets are trapped in the fibrin clots leading to thrombocytopenia, which produces petechiae and ecchymoses. The fibrinolytic system is activated (secondary fibrinolysis), and plasmin cleaves the fibrin strands holding the fibrin clots together. Fibrin strands are held together by cross-links, and cleaved fragments with cross-links are detected in the d-dimer assay, which is the most sensitive test for diagnosing DIC. Option A (Autoimmune thrombocytopenia) is incorrect. DIC is associated with fibrin clots, multiple coagulation factor deficiencies, activation of the fibrinolytic system, and thrombocytopenia. Therefore, conditions that produce thrombocytopenia, such as autoimmune thrombocytopenia, do not explain all of the clinical and laboratory findings that are present in DIC. Option B (Circulating anticoagulant) is incorrect. Circulating anticoagulants are antibodies that destroy coagulation factors (e.g., factor VIII), causing prolongation of the aPTT and/or PT. However, these antibodies do not destroy platelets or produce fibrin clots that obstruct the microcirculation as in DIC. Option D (Primary fibrinolysis) is incorrect. Primary fibrinolysis is uncommon. It is usually seen in the setting of radical prostatectomy and open heart surgery. Only the fibrinolytic system is activated. Therefore, clinical findings are primarily those related to coagulation factor deficiencies (e.g., fibrinogen, factor V, factor VIII). The platelet count is normal and d-dimers are not present, because there are no fibrin clots. Option E (Thrombotic thrombocytopenic purpura (TTP)) is incorrect. DIC is associated with fibrin clots, multiple coagulation factor deficiencies, activation of the fibrinolytic system, and thrombocytopenia. TTP is a disorder

characterized by endothelial injury with the formation of platelet thrombi that consume platelets, not coagulation factors; therefore, the PT and PTT are normal (not prolonged).

A few weeks after an upper respiratory infection, a 10-year-old boy has several bouts of epistaxis associated with nonpruritic lesions on the skin (see photograph). The skin lesions are nonpalpable and do not blanch under digital pressure. There is no evidence of lymphadenopathy or hepatosplenomegaly on physical examination. Hemoglobin and WBC counts are normal. WBC morphology is normal. The platelet count is 25,000/mm3. Which of the following is the most likely cause of the platelet abnormality?

A. Antibodies directed against a platelet receptor B. Deficiency of von Willebrand factorcleaving protease C. Immunocomplex vasculitis involving small vessels D. Infiltrative bone marrow disease with destruction of megakaryocytes E. Vessel damage due to a toxin produced by Escherichia coli
Option A (Antibodies directed against a platelet receptor) is correct. The patient has idiopathic thrombocytopenic purpura (ITP), which is caused by IgG antibodies that are directed against glycoprotein IIb/IIIa fibrinogen receptors on the surface of platelets (type II hypersensitivity reaction). Splenic macrophages with receptors for IgG phagocytose destroy the platelets leading to thrombocytopenia. The lesions shown in the photograph are petechiae, which are defined as pinpoint areas of hemorrhage into the subcutaneous tissue. Petechiae and epistaxis are common signs of a platelet disorder. ITP is the most common cause of thrombocytopenia in children and is treated with corticosteroids. The prognosis is excellent. Option B (Deficiency of von Willebrand factorcleaving protease) is incorrect. A deficiency of von Willebrand factor cleaving protease in endothelial cells is found in thrombotic thrombocytopenic purpura (TTP), an uncommon cause of thrombocytopenia in women. It is characterized by endothelial damage at the arteriole-capillary junctions throughout the body. This exposes von Willebrand factor (vWF), which is a platelet adhesion factor that is normally synthesized by endothelial cells. Platelets have GpIb receptors that adhere to the exposed vWF. Following adhesion, there is platelet aggregation and the formation of platelet thrombi at all of these injury sites. Consumption of platelets in these thrombi

is sufficient enough to produce thrombocytopenia. TTP produces a classic pentad of fever, thrombocytopenia, microangiopathic hemolytic anemia with schistocytes (platelet thrombi damage RBCs), central nervous system dysfunction, and renal failure. Option C (Immunocomplex vasculitis involving small vessels) is incorrect. Immunocomplex vasculitis involving small vessels damages the vessels and causes multifocal areas of subcutaneous hemorrhage (purpura or ecchymoses). The acute inflammatory reaction causes the affected tissue to swell causing the lesions to be palpable; hence, the term palpable purpura as an identifying feature of small vessel vasculitis. Option D (Infiltrative bone marrow disease with destruction of megakaryocytes) is incorrect. Infiltrative bone marrow disease with destruction of megakaryocytes commonly occurs in leukemia, myelofibrosis, and metastatic disease to the bone marrow. In all these conditions, immature WBCs and nucleated RBCs are present in the peripheral blood. Option E (Vessel damage due to a toxin produced by Escherichia coli) is incorrect. The O157:H7 strain of E. coli produces a toxin that damages small vessels causing hemolytic uremic syndrome. It is usually contracted by eating undercooked beef products (e.g., steak, hamburgers). Findings are very similar to what has been described for TTP; except, the toxin is responsible for injury at the arteriole-capillary junctions that initiates platelet thrombus formation and consumption of platelets.

A febrile 33-year-old woman complains of a headache, blurry vision, and decreased urine output. Physical examination shows bilateral retinal hemorrhages and generalized petechiae and ecchymoses. The peripheral smear shows numerous fragmented RBCs. Laboratory studies show:

Hemoglobin 8 g/dL WBC count 9500/mm3 Platelet count 30,000/mm3 Reticulocyte count, corrected 12% Serum blood urea nitrogen (BUN) 50 mg/dL Serum creatinine 5 mg/dL Partial thromboplastin time (activated, aPTT) 35 sec Prothrombin time (PT) 13 sec Bleeding time >15 min

Which of the following mechanisms is most likely responsible for this disorder?

A. Antiphospholipid antibodies B. Circulating anticoagulant C. Consumption of coagulation factors

D. Consumption of platelets E. Qualitative platelet disorder


Option D (Consumption of platelets) is correct. The patient has thrombotic thrombocytopenic purpura (TTP), which is caused by an acquired or genetic deficiency in von Willebrands factor (vWF)-cleaving protease in endothelial cells. vWF is normally synthesized in endothelial cells and megakaryocytes. In endothelial cells, its function is to adhere to GpIb receptors on platelets if there is endothelial injury (platelet adhesion factor). The vWF in endothelial cells is composed of a series of multimers (dimers held together by sulfide bonds) that vary in molecular weight. When some of these multimers enter the circulation, they are normally acted upon by vWF-cleaving protease and converted into a smaller molecular weight vWF, which combines with factor VIII antigen to prevent it from becoming degraded. In TTP, the circulating vWF is the large molecular weight type of multimeric vWF that is present in endothelial cells, because the vWF-cleaving protease is deficient in the plasma. This large multimer is present in large amounts because of injury to the endothelial cells at arteriole-capillary junctions throughout the body and leakage of the multimer into the circulation. The actual cause of the endothelial injury is multifactorial and includes drugs such as ticlopidine, clopidogrel, cyclosporine, and oral contraceptives; hypertension; and pregnancy and the postpartum state. The exposed large multimeric vWF in the areas of endothelial injury has a very high binding affinity for the GpIb receptor on platelets, causing them to adhere, aggregate, and form platelet thrombi at all these injury sites. Consumption of platelets in these thrombi is sufficient enough to produce thrombocytopenia. The coagulation system is not activated in TTP; hence, coagulation studies are normal. TTP produces a classic pentad of fever, thrombocytopenia (prolongs the bleeding time), microangiopathic hemolytic anemia with schistocytes (platelet thrombi damage RBCs), central nervous system dysfunction, and renal failure (serum BUN:creatinine ratio < 15). Treatment is plasma exchange, corticosteroids, and vincristine if corticosteroids do not work. Mortality is 10% to 20%. Option A (Antiphospholipid antibodies) is incorrect. Antiphospholipid antibodies, such as anticardiolipin antibodies and lupus anticoagulant, are directed against phospholipids bound to plasma proteins. These antibodies are associated with arterial and venous thrombosis syndromes (e.g., strokes). They do not produce thrombocytopenia and a microangiopathic hemolytic anemia. Option B (Circulating anticoagulant) is incorrect. Circulating anticoagulants are antibodies that inhibit specific coagulation factors (e.g., antibodies against factor VIII), hence simulating a coagulation factor deficiency. A normal aPTT and PT excludes circulating anticoagulants from the differential diagnosis. Option C (Consumption of coagulation factors) is incorrect. The thrombi that are formed in the vessel lumen are composed of platelets held together by fibrin. Generalized activation of the coagulation cascade is not present in TTP. Stated another way, TTP is not a variant of disseminated intravascular coagulation. Option E (Qualitative platelet disorder) is incorrect. TTP is characterized by normal platelet adhesion and aggregation, which excludes a qualitative platelet disorder.

This 23-year-old woman complains of fever, increased sensitivity to sunlight, and stiffness in both hands in the morning that resolves in a few hours. Physical examination shows splenomegaly and diffuse swelling of the joints and wrists in both hands. Urinalysis and complete blood cell count are normal. Which of the following laboratory test results would you expect in this patient?

A. Positive anticentromere antibody test B. Positive antimicrosomal antibody test C. Positive antitopoisomerase antibody test D. Positive direct Coombs test E. Positive serum antinuclear antibody (ANA) test
Option E (Positive serum antinuclear antibody (ANA) test) is correct. The serum ANA test is most likely to be positive, because the clinical findings are most compatible with systemic lupus erythematosus (SLE). The serum ANA test is the screening test of choice for SLE, because it is positive in the majority of patients who have the disease. The photograph shows a malar butterfly rash, which is a classic sign of SLE that is present in up to 80% of patients. The patient also has symmetric small joint arthritis in the hands and wrists, morning stiffness, photophobia, and splenomegaly, which are common presenting signs of SLE. Option A (Positive anticentromere antibody test) is incorrect. Anticentromere antibodies are most often seen in progressive systemic sclerosis and a limited variant called CREST syndrome. Neither one of these conditions has photophobia, symmetrical small joint disease, and a malar rash. Option B (Positive antimicrosomal antibody test) is incorrect. Antimicrosomal antibodies occur in Hashimotos thyroiditis and Graves disease. Option C (Positive antitopoisomerase antibody test) is incorrect. Antitopoisomerase (antiScl-70) antibodies are present in progressive systemic sclerosis. Option D (Positive direct Coombs test) is incorrect. The direct Coombs (antiglobulin) test is not likely to be positive in this patient, because the complete blood count is normal. A positive test would indicate the presence of an autoimmune hemolytic anemia caused by IgG antibodies on the surface of the RBCs. SLE is the most common cause of a warm type of autoimmune hemolytic anemia.

A 22-year-old woman has an intermittent history of colicky, right lower quadrant pain and diarrhea. Laboratory studies show an anemia with a mean corpuscular volume of 120 m3, a WBC count of 2000 cells/mm3, and a platelet count of 50,000 cells/mm3. The peripheral smear shows macro-ovalocytes and hypersegmented neutrophils. Serum vitamin B12 level is decreased, and serum folate is normal. Addition of intrinsic factor or administration of broad-spectrum antibiotics does not correct the anemia. Which of the following is the most likely cause of the anemia?

A. Alcohol excess B. Bacterial overgrowth C. Pernicious anemia D. Pregnancy E. Terminal ileal disease

Option E (Terminal ileal disease) is correct. The patient most likely has Crohns disease involving the terminal ileum, which causes colicky, right lower quadrant pain and diarrhea. Crohns disease is complicated by a macrocytic anemia caused by malabsorption of vitamin B12 in the inflamed terminal ileum, where the vitamin B12intrinsic factor complex is normally reabsorbed via a receptor for the complex. Hematologic findings in vitamin B12 deficiency include macro-ovalocytes, hypersegmented neutrophils, and pancytopenia. Macrophage destruction and apoptosis of megaloblastic hematopoietic cells in the bone marrow cause the pancytopenia. Option A (Alcohol excess) is incorrect. Alcohol excess is a common cause of folate deficiency, which, like vitamin B12 deficiency, also produces a macrocytic anemia with pancytopenia. However, serum folate is normal in this patient. Option B (Bacterial overgrowth) is incorrect. Bacterial overgrowth can produce vitamin B12 deficiency. However, bacterial overgrowth can be ruled out, because treatment with broad-spectrum antibiotics does not correct the anemia. Option C (Pernicious anemia) is incorrect. Pernicious anemia is the most common cause of vitamin B12 deficiency. However, lack of correction of the anemia after the addition of intrinsic factor rules out pernicious anemia. Option D (Pregnancy) is incorrect. Pregnancy is a common cause of folate deficiency, which, like vitamin B12 deficiency, also produces a macrocytic anemia with pancytopenia. However, serum folate is normal in this patient.

A 32-year-old woman awakes one morning with a painful lesion on her right forearm, which she attributes to a spider bite (see photograph). Her symptoms include chills, headache, and nausea. Examination of the forearm

shows a 3-inch area of erythema and swelling with a central area of vesiculation. Which of the following is the clinical outcome of this bite?

A. Ascending paralysis and hypertension B. Muscle cramping and severe abdominal pain C. Mydriasis and muscle paralysis D. Resolution of the lesion without scar formation E. Ulceration with extensive necrosis
Option E (Ulceration with extensive necrosis) is correct. The photograph shows a brown recluse spider (Loxosceles reclusa). Its body is yellow-brown, and there is a violin-shaped dark brown spot on the dorsal side of the cephalothorax. The spider produces a powerful necrolytic toxin. Initial symptoms from the bite are usually minimal; within a few hours, severe pain, swelling, and erythema develop at the site of the bite. A central area of vesiculation develops that usually is followed by extensive ulceration and necrosis of the skin and subcutaneous tissue. In some cases, a hemolytic anemia may occur. Option A (Ascending paralysis and hypertension) is incorrect. Ascending paralysis and hypertension are due to a neurotoxin produced by the scorpion (Centruroides species). There is no skin ulceration with scorpion bites. Option B (Muscle cramping and severe abdominal pain) is incorrect. Muscle cramping and abdominal pain are due to a neurotoxin produced by the black widow spider (Latrodectus mactans). There is no skin ulceration with black widow bites. Option C (Mydriasis and muscle paralysis) is incorrect. Mydriasis and muscle paralysis are due to a neurotoxin produced by the coral snake. The neurotoxin binds to presynaptic nerve terminals and acetylcholine receptors. There is no skin ulceration with coral snake bites. Option D (Resolution of the lesion without scar formation) is incorrect. The extent of ulceration and necrosis in brown recluse spider bites often necessitates skin grafting. Healing without scar formation is an unlikely outcome.

The photograph shows a lesion on the face of an 8-yr-old boy. A younger sister has similar lesions on her face. Which of the following is the causal agent?

A. Herpes simplex virus type 1 B. Malassezia furfur C. Propionibacterium acnes D. Staphylococcus aureus E. Trichophyton rubrum
Option D (Staphylococcus aureus) is correct. The child has impetigo. The rash usually begins on the face. Vesicles and pustules rupture to form honey-colored, crusted lesions, which are evident in this patient. Staphylococcus aureus is the most common cause of this superficial skin lesion. Streptococcus pyogenes is the second most common cause of impetigo. Impetigo is highly contagious, which explains why the childs sister developed similar lesions. Treatment is with mupirocin ointment plus dicloxacillin. Option A (Herpes simplex virus type 1) is incorrect. Herpes simplex virus type 1 produces vesicles and pustules on the vermilion border of the lip. It does not produce honey-colored crusted lesions on the face. Option B (Malassezia furfur) is incorrect. M. furfur, a superficial dermatophyte, causes tinea versicolor and seborrheic dermatitis (dandruff). Option C (Propionibacterium acnes) is incorrect. P. acnes is an anaerobe involved in producing the inflammatory reaction associated with acne vulgaris. Option E (Trichophyton rubrum) is incorrect. T. rubrum, a superficial dermatophyte, causes tinea corporis (body), tinea cruris (groin), and tinea pedis (foot). It does not produce honey-colored crusted lesions on the face.

The photograph shows an ova and parasite preparation of stool from an 8-year-old girl. Which of the following clinical findings is most likely present?

A. Anal pruritus B. Bloody diarrhea C. Intestinal obstruction D. Iron deficiency E. Rectal prolapse
Option A (Anal pruritus) is correct. The photograph shows the embryonated eggs of Enterobius vermicularis (pinworm). The eggs are characteristically flattened on one side. The female worms lay their eggs in the anus, which causes irritation leading to anal pruritus. Female children and adults may also develop urinary tract infections. Appendicitis is a rare complication. Treatment is albendazole or mebendazole. Option B (Bloody diarrhea) is incorrect. Parasitic causes of bloody diarrhea include amebiasis and balantidiasis, both of which are protozoal diseases. Option C (Intestinal obstruction) is incorrect. The adult worms of Ascaris lumbricoides cause intestinal obstruction. The eggs are oval-shaped and are not embryonated. Option D (Iron deficiency) is incorrect. Necator americanus (hookworm) is the most common parasite causing iron deficiency. The eggs are oval-shaped and do not have flattened sides. Option E (Rectal prolapse) is incorrect. Trichuris trichiura, or whipworm, is the most common parasite causing rectal prolapse in children. The eggs are barrel-shaped and have plugs at both ends.

The photograph shows a kidney removed at autopsy from a 55-year-old woman. Laboratory studies prior to death showed a hemoglobin level of 20 g/dL and the presence of RBCs in the urine sediment. Which of the following laboratory findings was most likely present prior to death?

A. Decreased arterial oxygen saturation B. Decreased partial pressure of arterial oxygen C. Decreased serum erythropoietin D. Normal plasma volume E. Normal RBC mass
Option D (Normal plasma volume) is correct. The photograph shows a kidney with a large, yellow mass in the upper pole with multifocal areas of hemorrhage and necrosis. The gross findings, plus the history of polycythemia (hemoglobin 20 g/dL) and hematuria, confirm the diagnosis of renal cell carcinoma with ectopic secretion of erythropoietin (EPO). Ectopic EPO production by the tumor causes an increase in bone marrow production of RBCs. Therefore, the number of RBCs/mm3 of blood (RBC count) and the total number of RBCs produced in mL/kg body weight (RBC mass) are both increased. However, the plasma volume remains normal and does not increase in concert with the increase in RBC mass. Option A (Decreased arterial oxygen saturation) is incorrect. Polycythemia due to ectopic production of EPO is not released in response to a hypoxemic stimulus but is produced by the tumor itself. Therefore, the oxygen saturation (percentage of heme groups occupied by O2) is normal (not decreased). Option B (Decreased partial pressure of arterial oxygen) is incorrect. Polycythemia due to ectopic production of EPO is not released in response to a hypoxemic stimulus but is produced by the tumor itself. Therefore, the partial pressure of arterial oxygen is normal (not decreased). Option C (Decreased serum erythropoietin) is incorrect. EPO is increased in renal cell carcinoma (not decreased). Option E (Normal RBC mass) is incorrect. Ectopic EPO causes an increase in bone marrow production of RBCs. Therefore, the total number of RBCs produced in mL/kg body weight (RBC mass) is increased (not decreased).

This 10-year-old girl was asked to close your eyes. There is no history of a previous tick bite. Which type of microbial pathogen has most often been implicated as a cause of her disease?

A. Fungus B. Protozoan C. Rickettsia D. Spirochete E. Virus


Option E (Virus) is correct. The patient has the lower motor neuron (LMN) type of idiopathic Bells palsy, which is an inflammatory reaction of the facial nerve (CN VII) near the stylomastoid foramen or in the bony facial canal. It may be associated with Herpes simplex (most common), HIV, sarcoidosis, Lyme disease (often bilateral), and pregnancy. Clinical findings in LMN disease are ipsilateral upper and lower face involvement, drooping of the corner of the mouth, difficulty speaking, inability to close the eye, and hyperacusis in some cases. In the picture, when asked to close your eyes, she could not close her right eye, because the orbicularis oculi muscle is innervated by the VIIth cranial nerve. There is also drooping of the right side of the mouth. If she was asked to wrinkle your forehead she would not have been able to wrinkle the forehead muscles. Clinical findings in upper motor neuron disease are contralateral lower face involvement with contralateral sparing of the upper face. Option A (Fungus) is incorrect. Fungal infections do not produce Bells palsy. Option B (Protozoan) is incorrect. Protozoal infections do not produce Bells palsy. Option C (Rickettsia) is incorrect. Rickettsial diseases do not produce Bells palsy. Option D (Spirochete) is incorrect. Borrelia burgdorferi in Lyme disease is transmitted by a tick bite and commonly produces Bells palsy. However, there is no history of a tick bite.

Which of the following laboratory findings is present in both mild hemophilia A and classic von Willebrands disease?

A. Decreased factor VIII coagulant B. Decreased von Willebrands factor C. Normal activated partial thromboplastin time (aPTT) D. Prolonged bleeding time E. Prolonged prothrombin time (PT)
Option A (Decreased factor VIII coagulant) is correct. Von Willebrands disease is an autosomal dominant disorder characterized by a deficiency of von Willebrands factor and factor VIII coagulant in the intrinsic coagulation system. Deficiency of von Willebrands factor prevents platelets from adhering to damaged endothelial cells. It also complexes with circulating factor VIII coagulant and prevents degradation of the factor. Therefore, a deficiency of von Willebrands factor automatically causes a corresponding decrease in VIII coagulant, which causes prolongation of the aPTT. The aPTT evaluates the activity of the coagulation factors in the intrinsic system (factor XII, factor XI, factor IX, factor VIII) to the formation of a fibrin clot in the final common pathway (factor X, factor V, prothrombin, and fibrinogen). Hemophilia A is an X-linked disorder characterized by a deficiency of factor VIII coagulant, causing a prolonged aPTT. Normal amounts of von Willebrands factor are produced; therefore, there is no problem with platelet adhesion to injured endothelium and the bleeding time is normal. Option B (Decreased von Willebrands factor) is incorrect. Von Willebrands factor is decreased in von Willebrands disease, but it is normal in hemophilia A. Option C (Normal activated partial thromboplastin time (aPTT)) is incorrect. The aPTT is prolonged (not normal) in von Willebrands disease and in hemophilia A. Option D (Prolonged bleeding time) is incorrect. The bleeding time evaluates the ability of platelets to adhere to injured endothelial cells and to aggregate to form a temporary platelet plug to stop bleeding. The bleeding time is prolonged in von Willebrands disease, because von Willebrands factor is deficient. The bleeding time is normal in hemophilia A, because von Willebrand factor is present. Option E (Prolonged prothrombin time (PT)) is incorrect. The PT evaluates the activity of coagulation factors in the extrinsic system (VII) to the formation of a fibrin clot in the final common pathway (factor X, factor V, prothrombin, and fibrinogen). Factor VIII is not in the extrinsic system. Therefore, the PT is normal in both von Willebrands disease and hemophilia A.

A 62-year-old man is involved in a head-on automobile collision and is examined by paramedics at the scene of the accident. The patient is conscious and responds to questions. There is marked tenderness along the left and right lower rib cage. No open fractures are noted. His heart rate is 160 beats/min and his blood pressure is 70/40 mm Hg. The skin is cold and clammy. Which of the following hemodynamic changes is most likely to be present?

A. Decreased arterial PO2 B. Decreased hemoglobin concentration C. Increased left ventricular end-diastolic pressure D. Increased mixed venous O2 (MVO2) content E. Increased total peripheral arteriolar resistance
Option E (Increased total peripheral arteriolar resistance) is correct. The patient has hypovolemic shock caused by blood loss (tachycardia with weak pulse; cold, clammy skin; decreased blood pressure), most likely a ruptured spleen and/or liver. In the initial phase of acute blood loss, the hemoglobin and RBC count are normal, because whole blood that contains both RBCs and plasma is lost. Within a few hours, plasma begins to be replaced and the RBC count and hemoglobin level drop. A decrease in cardiac output causes underfilling of the aortic arch, which activates the sympathetic nervous system, subsequently releasing catecholamines. Catecholamines cause venoconstriction, increased myocardial contraction, increased heart rate, and vasoconstriction of the smooth muscle cells of the peripheral resistance arterioles. Decreased renal blood flow activates the renin-angiotensin-aldosterone system, causing the release of angiotensin II, which produces peripheral arteriolar vasoconstriction. Antidiuretic hormone is also released, which produces vasoconstriction of peripheral resistance arterioles. Vasoconstriction of arterioles in the skin shunts blood to more important areas of the body, causing cold, clammy skin. Option A (Decreased arterial PO2) is incorrect. In hypovolemic shock due to blood loss, gas exchange in the lung is normal; therefore, the arterial PO2 is normal (not decreased). Option B (Decreased hemoglobin concentration) is incorrect. Hemoglobin concentration is normal in the early phase of hypovolemic shock resulting from blood loss. Option C (Increased left ventricular end-diastolic pressure) is incorrect. Left ventricular end-diastolic pressure is decreased (not increased) in hypovolemic shock, because there is less blood in the circulation and less venous return to the heart. Option D (Increased mixed venous O2 (MVO2) content) is incorrect. The MVO2 is the best indicator of tissue hypoxia. It is measured in the right heart with a Swan-Ganz catheter and indicates the degree of extraction of O2 from the blood delivered to tissue. In hypovolemic shock, decreased blood flow through the microcirculation leads to increased extraction of O2 from the blood and a decreased MVO2 (not increased).

A 25-year-old intravenous drug abuser develops fever, scleral icterus, and right upper quadrant pain. Laboratory studies show an increase in serum aminotransferases and an increase in the serum bilirubin with approximately equal unconjugated and conjugated bilirubin fractions. The suspected diagnosis is acute hepatitis B. Which of the following hepatitis B profiles is expected?

HBsAg HBeAg/HBV DNAAnti-HBc-IgMAnti-HBc-IgGAnti-HBs A.NegativeNegative Negative Negative Positive B. NegativeNegative Negative Positive Positive C. NegativeNegative Positive Negative Negative D.Positive Positive Negative Positive Negative E. Positive Positive Positive Negative Negative HBsAg, hepatitis B surface antigen; HBeAg, hepatitis B e antigen; HBV DNA, hepatitis B DNA; anti-HBcIgM, anti-HBV core antibody IgM; anti-HBc-IgG, anti-HBV core antibody IgG; Anti-HBs, anti-HBV surface antibody. A. A B. B C. C D. D E. E
Option (E) is correct. The presence of a mixed hyperbilirubinemia (increase in unconjugated and conjugated bilirubin) and an increase in the serum transaminases are markers of the icteric phase of acute hepatitis B. HBsAg, HBeAg, HBV DNA, and anti-HBc-IgM are positive. HBsAg first appears in the serum 2 to 8 weeks before symptoms develop and is the last antigen marker to disappear if recovery occurs. HBeAg and HBV DNA are infective particles that appear shortly after HBsAg and disappear before HBsAg. Anti-HBc-IgM is a nonprotective antibody that appears shortly after HBsAg and is completely converted into anti-HBc-IgG by 6 months. Option (A) is incorrect. The presence of anti-HBs and the absence of HBsAg, HBeAg, HBV-DNA, anti-HBc-IgM, and antiHBc-IgG indicate immunization with hepatitis B vaccine. Anti-HBs is a protective antibody. Option (B) is incorrect. The presence of anti-HBs and anti-HBc-IgG and the absence of the other markers indicate recovery from hepatitis B. Anti-HBs is a protective antibody, and anti-HBc-IgG develops after 6 months. Option (C) is incorrect. The presence of anti-HBc-IgM and the absence of all of the other markers indicate that the patient is in the window phase of recovery from hepatitis B when all the antigens have disappeared and their corresponding antibodies have not yet had time to develop. Option (D) is incorrect. The presence of HBsAg, HBeAg, HBV-DNA, and anti-HBc-IgG indicates an infective type of chronic hepatitis B. Note that the key difference between acute HBV and chronic HBV that is infective, is the presence of

anti-HBc-IgG, which only occurs after 6 months. If HBeAg and HBV DNA were absent, that would indicate a chronic hepatitis B, who is a healthy carrier.

An afebrile, sexually active 26-year-old man complains of increased frequency of urination, dysuria, and penile drainage. His last sexual encounter was 10 days ago. The urine reagent strip is positive for leukocyte esterase and negative for nitrite and blood. Sediment examination shows neutrophils in clumps; however, bacteria are not present. A standard urine culture is negative for organisms. What is the most likely diagnosis?

A. Escherichia coli urethritis or cystitis B. Chlamydia trachomatis urethritis C. Renal tuberculosis D. Renal stone E. Viral cystitis
Option B (Chlamydia trachomatis urethritis) is correct. The patient has a sterile pyuria, which refers to the presence of neutrophils in the urine (positive reagent strip for leukocyte esterase), a negative reagent strip for nitrites, and no growth of organisms with a standard urine culture. Due to the close proximity of the patients urinary findings with a previous sexual encounter, the patient most likely has urethritis due to C. trachomatis. Option A (Escherichia coli urethritis or cystitis) is incorrect. In lower urinary tract infections due to E. coli, the reagent strip is positive for nitrites and leukocyte esterase and routine cultures isolate the organism. Option C (Renal tuberculosis) is incorrect. Renal tuberculosis produces a sterile pyuria; however, the history of symptoms within 10 days of a sexual encounter is more likely to represent urethritis due to C. trachomatis than renal tuberculosis. Option D (Renal stone) is incorrect. A renal stone presents with a sudden onset of flank pain with radiation of pain into the ipsilateral groin. Hematuria is invariably present. None of these findings are present in the patient. Option E (Viral cystitis) is incorrect. Adenovirus produces a hemorrhagic cystitis, which is not present in this patient.

A 32-year-old alcoholic has fever, jaundice, and tender hepatomegaly. A liver biopsy exhibits fatty change, Mallory bodies, and a neutrophilic infiltrate. Which of the following serum and urine laboratory test abnormalities would you expect?

% CB Urine BilirubinUrine UBGAST ALT ALP GGT Normal Negative NormalNormalNormalNormal A. <20 Negative NormalNormalNormal B. >50 Negative C. 2050 D. 2050 E. <20 Negative Normal NormalNormalNormalNormal Number of arrows relates to the degree of elevation. ALP, alkaline phosphatase; ALT, alanine aminotransferase; AST, aspartate aminotransferase; CB, conjugated bilirubin; GGT, -glutamyltransferase; UBG, urobilinogen. A. A B. B C. C D. D E. E
Option C is correct. The patient has alcoholic hepatitis, which produces a mixed jaundice (CB 2050%), due to decreased uptake and conjugation of unconjugated bilirubin (UCB) and destruction of bile ductules among liver cells leading to an increase in CB in the blood. Urine UBG is increased, because UBG that is normally recycled back to the liver is redirected into the urine. Serum transaminases are increased with serum AST (mitochondrial enzyme) greater than ALT (cytosol enzyme). Since AST is normally located in the mitochondria of hepatocytes, alcohol, a mitochondrial toxin, causes preferential release of AST into the blood and is higher that ALT. Serum ALP is slightly increased; however, serum GGT is markedly increased, because alcohol induces enzyme synthesis in the hepatocyte cytochrome P450 system in the smooth endoplasmic reticulum (SER). GGT is also located in the SER and is increased as well. Therefore, key markers for alcoholic hepatitis are AST > ALT and marked increase in GGT. Option A is incorrect. The laboratory studies are most consistent with extravascular hemolytic anemias (e.g., hereditary spherocytosis, antibodies blood group hemolytic disease of the newborn), where RBCs are removed extravascularly by macrophages leading to an increase in UCB and a CB < 20%. AST is present in RBCs; therefore, there is a slight increase in serum AST. Urine UBG is increased due to increased production of CB by the liver and increased conversion of CB to UBG in the colon. Fecal UBG gives the color to stool. Normally, a small portion of UBG in the colon is reabsorbed back into the blood (enterohepatic circulation) where most is taken up by the liver and the remainder is filtered into the urine to give the color to urine. Therefore, if there is more UBG in the stool (e.g., extravascular hemolytic anemia), then proportionately more UBG ends up in the urine (darker yellow than normal). Option B is incorrect. The laboratory findings indicate an obstructive type of jaundice, either due to intrahepatic cholestasis (e.g., drug-induced oral contraceptives) or extrahepatic cholestasis. Since bile is predominantly CB, in obstructive jaundice this is the predominant fraction that enters the bloodstream (CB > 50%) through ruptured intrahepatic bile ductules. CB is water soluble; therefore, it enters the urine to produce a positive dipstick test for bilirubin. Since UBG normally present in the urine derives from enterohepatic circulation of UBG initially produced in the liver, absence of UBG in the stool in obstructive liver disease (light-colored stools) leads to an absence of UBG in the urine. Enzyme markers for obstructive liver disease are serum ALP and serum GGT. Serum AST and ALT are also increased but to a lesser degree.

Option D is incorrect. The laboratory findings are most consistent with hepatitis in which there is diffuse necrosis of hepatocytes (e.g., hepatitis A). Hepatitis produces a mixed jaundice (CB 2050%), due to decreased uptake and conjugation of UCB and destruction of bile ductules among liver cells leading to an increase in CB in the blood. Urine UBG is increased, because UBG that is normally recycled back to the liver is redirected into the urine. Urine bilirubin is increased, because CB is water soluble. Serum transaminases are markedly increased because of massive liver cell necrosis; serum ALT is greater than AST. Serum ALP and GGT are only slightly increased. Option E is incorrect. The laboratory findings indicate an increase in UCB. This could be due to hemolytic anemias; however, destruction of RBCs also increases AST and urine UBG is increased. Hence, the most common cause of this profile is Gilberts disease, which is an autosomal dominant disease characterized by decreased uptake and conjugation of bilirubin particularly exacerbated by the fasting state. Baseline UCB levels are increased to over twice normal in the fasting state leading to visible evidence of jaundice (CB < 20%). Next to viral hepatitis, it is the second overall most common cause of jaundice in the United States. All enzyme studies and urine studies are normal. It has no clinical significance.

The photograph shows the scalp of a 12-year-old boy. Woods lamp examination of the lesion is negative for fluorescence. Which of the following is the most likely cause of the hair loss?

A. Malassezia furfur B. Microsporum audouinii C. Microsporum canis D. Trichophyton rubrum E. Trichophyton tonsurans

Option E (Trichophyton tonsurans) is correct. The patient has tinea capitis. Note the area of baldness with black dots representing where the hair shaft broke off. The superficial dermatophyte T. tonsurans is the most likely pathogen, because the Woods lamp examination is negative for fluorescence. This dermatophyte is the most common cause of tinea capitis in the United States. It infects the inner portion of the hair shaft; hence, the fluorescent metabolites produced by the pathogen are not detected by Woods lamp examination.

Option A (Malassezia furfur) is incorrect. M. furfur is a superficial dermatophyte that is the causal agent of tinea versicolor and seborrheic dermatitis. It does not cause hair loss. Option B (Microsporum audouinii) is incorrect. M. audouinii is a superficial dermatophyte that commonly causes tinea capitis. It infects the outer portion of the hair shaft; therefore, its fluorescent metabolites are detected by Woods lamp examination. Option C (Microsporum canis) is incorrect. M. canis is a superficial dermatophyte that commonly causes tinea capitis. It infects the outer portion of the hair shaft; therefore, its fluorescent metabolites are detected by Woods lamp examination. Option D (Trichophyton rubrum) is incorrect. T. rubrum is a superficial dermatophyte that is the most common cause of tinea corporis (body), tinea pedis (feet), and tinea cruris (groin). Its metabolites do fluoresce with Woods lamp. It does not commonly infect hair.

A 49-year-old woman develops generalized pruritus. Physical examination shows nontender hepatomegaly. Initial laboratory studies show an increased serum alkaline phosphatase and -glutamyltransferase (GGT), and a normal total bilirubin. A serum antinuclear antibody (ANA) test is positive. An ultrasound of the gallbladder is negative for gallstones. Which of the following additional laboratory findings is most likely to be reported?

A. Antimitochondrial antibodies B. Antismooth muscle antibodies C. Increased serum -fetoprotein (AFP) D. Increased serum IgG E. Increased triglycerides
Option A (Antimitochondrial antibodies) is correct. The patient has primary biliary cirrhosis (PBC), which is an autoimmune disease associated with granulomatous destruction of the bile ducts in the portal triads. Pathogenesis involves an environmental insult affecting mitochondrial proteins triggering CD8-T cell destruction of intralobular bile duct epithelium. Autoantibodies (antimitochondrial antibodies) develop against the mitochondria. Clinical findings include pruritus (cause unknown; not bile salts deposited in skin) is an early finding well before jaundice appears; painful hepatosplenomegaly; and jaundice, which is a late finding after most of the bile ducts have been destroyed. The key laboratory findings are the presence of antimitochondrial antibodies (>90%), a positive serum ANA (50%), and an increase in serum IgM. Increased alkaline phosphatase and GGT are enzymatic markers of bile duct obstruction (cholestasis). Treatment includes budesonide + ursodeoxycholic acid, cholestyramine for pruritus, and liver transplantation, which improves survival (70% survive 10 years). Option B (Antismooth muscle antibodies) is incorrect. Antismooth muscle antibodies are present in autoimmune

hepatitis, which is commonly associated with other autoimmune diseases (e.g., rheumatoid arthritis, Sjgrens syndrome). In these autoimmune diseases, the serum ANA test is usually positive, and the concentration of serum IgG is increased (not IgM). The resulting hepatic cell necrosis releases aminotransferases (e.g., alanine transferase) rather than alkaline phosphatase and GGT. Option C (Increased serum (-fetoprotein (AFP)) is incorrect. Increased serum -fetoprotein (AFP) occurs in hepatocellular carcinoma, which does not present with signs of obstructive liver disease. Serum AFP is normal in PBC unless it develops into hepatocellular carcinoma. Option D (Increased serum IgG) is incorrect. The main immunoglobulin increased in PBC is IgM, not IgG. Option E (Increased triglycerides) is incorrect. Obstruction of bile flow causes a reflux of bile into the blood. Cholesterol, not triglyceride, is the primary lipid excreted in bile. Patients with PBC very frequently have hypercholesterolemia manifested by xanthelasma.

The schematic shows a normal control population (solid line) and a population of people with a 24-hour-old acute myocardial infarction (AMI; dotted line) who were evaluated with a new serum test to detect myocardial injury. Two normal ranges for the same test are noted, with the first extending from 0 to 2 and the second from 0 to 4. The areas under the two curves are subdivided into different populations of people, which are represented by the letters A through E. Depending on the normal range selected, some of these areas contain people whose test results are true negatives (TN), false negatives (FN), true positives (TP), or false positives (FP). Which of the following correctly describes the test results in the lettered patient subpopulations for the two different normal ranges?

Normal range 0-2Normal range 0-4 A.Area A TN FN B. Area B FP TN C. Area C TN+FN TP+FP D.Area D FP TP E. Area E FN FN

A. Area A B. Area B C. Area C D. Area D E. Area E


Option B (Area B) is correct. Using the schematic A, note that the cutoff point for the normal range 02 is set at the beginning of the disease curve (dotted line). This creates a test with 100% sensitivity. The sensitivity of a test is established by performing the test on people with disease. People with disease either have a true positive (TP) or a false negative (FN) test result; therefore, the formula for the sensitivity of a test is TP/TP + FN. Note in schematic A that there are no FN test results in the interval between 0 and 2; therefore, the test has a sensitivity of 100%. Using schematic B, the cutoff point for the normal range 04 is set at the end of the normal curve (solid line). This creates a test with 100% specificity. The specificity of a test is established by performing the test on people without disease (control group; solid line). People without disease have either a true negative (TN) or a false positive (FP) test result. Note in schematic B that there are no FP test results beyond the 4 cutoff point. Area B has normal people without disease. Using the normal range of 02 (schematic A), these people are correctly classified as having FP test results (beyond the 2 cutoff point). Using the normal range of 04 (schematic B), these people are correctly classified as having TN test results. Option A (Area A) is incorrect. Area A contains normal people without disease. Using the normal range of 02

(schematic A), these people have TN test results. However, using the normal range of 04 (schematic B), these individuals have TN test results, not FN test results. Option C (Area C) is incorrect. Area C is an overlap area that has people who are normal and people who have an acute myocardial infarction (AMI). Using the normal range of 0-2 (schematic A), these people have TP and FP test results (not TN and FN test results). Using the normal range of 0-4 (schematic B), these people have TN and FN test results (not TP and FP test results). Option D (Area D) is incorrect. Area D contains people with an AMI. Using the normal range of 02 (schematic A), these people have TP test results (not FP test results). Using the normal range of 04 (schematic B), these people have FN test results (not TP test results). Option E (Area E) is incorrect. Area E contains people with an AMI. Using the normal range of 02 (schematic A) or the normal range of 04 (schematic B), these people have TP test results (not FN test results).

A 22-year-old woman saw her physician because of a severe sore throat. Three weeks later, she complained of fever, pains in the knees, and a rash on the arms that consisted of a circular ring of erythema around normal skin. Other findings on physical examination included bibasilar inspiratory crackles, an S3 and S4 heart sound, and a pansystolic murmur at the apex that radiated into the axilla. Both murmur and S3 and S4 heart sounds did not increase in intensity on deep, held inspiration. Which of the following test results is most likely present in this patient?

A. Increased anti-streptokinase O (ASO) titer B. Normal serum anti-DNAase B titers C. Positive blood culture D. Positive serum antinuclear antibody test E. Positive throat culture for Staphylococcus aureus
Option A (Increased anti-streptokinase O (ASO) titer) is correct. This patient developed acute rheumatic fever (ARF) several weeks after a group A streptococcal (Streptococcus pyogenes) pharyngitis. Clinical findings in ARF include sore throat, followed within several weeks by fever, joint pain, subcutaneous nodules, and skin changes (erythema marginatum, as in this case). The pansystolic murmur at the apex is caused by mitral regurgitation due to a sterile endocarditis (see Fig. 10-13 in Rapid Review Pathology, 3rd edition), and the bibasilar inspiratory crackles and S3 and S4 heart sounds are caused by left-sided heart failure due to myocarditis, which decreases the force of contraction leading to a systolic dysfunction type of heart failure. Valvular regurgitation and the myocarditis produce left ventricular volume overload, which is responsible for the abnormal heart sounds. The S3 heart sound is due to blood entering a volume overloaded chamber in early diastole, while the S4 heart sound is due to blood entering a noncompliant left ventricle in late diastole. Cardiac damage is caused by antibodies directed against the streptococcal M proteins (virulence factor),

which cross-react with antigens in the heart and other tissues (i.e., type II hypersensitivity reaction). Type IV hypersensitivity has also been implicated in the disease. Blood cultures are negative, because rheumatic fever causes immunologic damage (fibrinoid necrosis) to the heart and tissues. The serum antistreptolysin O titers are increased and are useful in confirming the diagnosis. Option B (Normal serum anti-DNAase B titers) is incorrect. In ARF, anti-DNAase B antibodies are increased, because DNAase B is an important inflammatory enzyme associated with Streptococcus pyogenes infections involving the skin and throat. Option C (Positive blood culture) is incorrect. The blood culture is negative in ARF. It is not an example of infective endocarditis. Option D (Positive serum antinuclear antibody test) is incorrect. In ARF, antibodies are directed against the M protein (virulence factor) of Streptococcus pyogenes and not against nuclear antigens (e.g., DNA, histones, acidic nuclear proteins, nucleolar antigens). Option E (Positive throat culture for Staphylococcus aureus) is incorrect. In ARF, the throat culture is positive for Streptococcus pyogenes, a group A streptococcus.

The photograph shows a 20-year-old woman who weighs 40 kg (88 pounds) and complains of absent menses and back pain. She has fine downy hair on her face and back. When asked how she feels about her appearance, she says that she feels fat. A urine test for pregnancy is negative. Which of the following laboratory findings is most likely present?

A. Decreased concentration of serum adrenocorticotropic hormone (ACTH)

B. Decreased concentration of serum cortisol C. Decreased concentration of serum gonadotropins D. Decreased concentration of serum growth hormone E. Increased concentration of serum gonadotropin-releasing hormone
Option C (Decreased concentration of serum gonadotropins) is correct. The patient has anorexia nervosa (selfstarvation). Features include loss of muscle mass and subcutaneous tissue, fine downy hair (lanugo hair), distorted body image (feel fat), and secondary amenorrhea. Excessive loss of body weight (>25% of ideal weight) leads to a decrease in secretion of gonadotropin-releasing hormone from the hypothalamus and a corresponding decrease in the concentration of serum gonadotropins (follicle-stimulating hormone and luteinizing hormone). Amenorrhea is caused by decreased ovarian synthesis of estradiol, which may lead to osteoporotic changes resulting in compression fractures in the vertebral column, which is most likely causing the patients back pain. All the stress hormones are increased (e.g., ACTH, cortisol, growth hormone, catecholamines). Option A (Decreased concentration of serum adrenocorticotropic hormone (ACTH)) is incorrect. Serum ACTH is increased in anorexia nervosa because it is a stress hormone. Option B (Decreased concentration of serum cortisol) is incorrect. Serum cortisol is increased in anorexia nervosa because it is a stress hormone. Option D (Decreased concentration of serum growth hormone) is incorrect. Growth hormone is increased in anorexia nervosa because it is a stress hormone. Option E (Increased concentration of serum gonadotropin-releasing hormone) is incorrect. In anorexia nervosa, excessive loss of body weight (>25% of ideal weight) leads to a decrease in secretion of gonadotropin-releasing hormone from the hypothalamus.

The photograph shows a facial lesion on a febrile child. A 3-year-old boy develops a maculopapular rash on his face 24 hours after the onset of a mild fever and sore throat. The rash causes prominent erythema of the cheeks. The causal agent of this disorder may also cause which of the following complications?

A. Acute lymphocytic leukemia B. Acute myeloblastic leukemia C. Aplastic anemia D. B-cell malignant lymphoma E. Disseminated intravascular coagulation
Option C (Aplastic anemia) is correct. The child has erythema infectiosum, or fifth disease, which is caused by parvovirus B19. It produces a confluent netlike erythema type of rash that begins on the cheeks (slapped face appearance) and then extends to the trunk and proximal extremities. If a patient has chronic hemolytic anemia (e.g., hereditary spherocytosis), the virus can infect either a trilineage stem cell, resulting in a self-limited aplastic anemia, or an erythroid stem cell, resulting in a pure red blood cell aplasia. Option A (Acute lymphocytic leukemia) is incorrect. Parvovirus B19 has not been associated with the development of any leukemia. Option B (Acute myeloblastic leukemia) is incorrect. Parvovirus B19 has not been associated with the development of any leukemia. Option D (B-cell malignant lymphoma) is incorrect. Parvovirus B19 has not been associated with the development of any type of malignant lymphoma. Option E (Disseminated intravascular coagulation) is incorrect. Activation of the intrinsic or extrinsic coagulation system must occur to produce intravascular clotting. Parvovirus B19 does not cause tissue damage (release of tissue thromboplastin) or endothelial cell damage (activation of factor XII) to initiate disseminated intravascular coagulation.

An enlarged, nontender right supraclavicular lymph node is removed from a 17-year-old boy. The photograph shows an abnormal cell discovered after examining numerous histologic sections of the surgically removed lymph node. Complete blood cell count, urinalysis, and biochemical tests are all normal. Which of the following is the most likely diagnosis?

A. Burkitts lymphoma B. Follicular B-cell lymphoma C. Hodgkins lymphoma D. Metastatic cancer E. Reactive lymphadenitis
Option C (Hodgkins lymphoma) is correct. The patient has lymphocyte-predominant Hodgkins lymphoma. The photograph shows a Reed-Sternberg (RS) cell, the neoplastic cell found in Hodgkins lymphoma. It is a large, multilobed cell with prominent nucleoli surrounded by a halo of clear nucleoplasm. In most cases, it is a transformed germinal center B-cell that is CD15 and CD30 positive. Presence of a classic RS cell or presence of RS-variant cells in a background of reactive cells (e.g., eosinophils, plasma cells, histiocytes) is required to confirm the diagnosis. The patient has stage 1 disease, in which the RS cells are difficult to find. Stage 1 disease generally involves only a single group of lymph nodes and has an excellent prognosis. The most common type of Hodgkins lymphoma is nodular sclerosing Hodgkins lymphoma, which invariably involves anterior mediastinal nodes and one other lymph node group above the diaphragm (stage 2 disease) and is most often seen in young women. Option A (Burkitts lymphoma) is incorrect. Burkitts lymphoma is characterized by a diffuse infiltrate of small, round B lymphocytes interspersed with pale-staining macrophages and apoptotic bodies. Option B (Follicular B-cell lymphoma) is incorrect. Follicular B-cell lymphoma is the second most common nonHodgkins lymphoma in adults. It is characterized by a nodular or diffuse pattern of nodal involvement. The neoplastic cells arise from the germinal follicle. It is associated with a t(14;18) translocation leading to overexpression of the BCL 2 gene (antiapoptosis gene). Option D (Metastatic cancer) is incorrect. Metastatic cancer is the most common malignancy of the lymph nodes. However, it is an uncommon cause of nodal enlargement in patients younger than 30 years.

Option E (Reactive lymphadenitis) is incorrect. Reactive lymphadenitis is associated with painful (not painless) lymphadenopathy and is caused by an inflammatory condition (e.g., tonsillitis or rheumatoid arthritis). Nodal enlargement is caused by hyperplasia of B-cells, T-cells, or histiocytes, or a combination of these.

A 40-year-old man with alcoholism complains of loss of taste and smell. A lipoma was surgically removed from the arm recently, but the wound has not healed properly. Physical examination is unremarkable except for a perioral rash. Which of the following is most likely deficient in this patient?

A. Chromium B. Copper C. Iodide D. Selenium E. Zinc

Option E (Zinc) is correct. Zinc deficiency is common in alcoholics. Patients develop a perioral rash and loss of taste (dysgeusia) and smell (anosmia). Wound healing is impaired, because zinc is a cofactor for collagenase, which is important in remodeling of a wound. In remodeling, type III collagen is replaced by type I collagen, which increases tensile strength in the wound to 80% of the original strength. Option A (Chromium) is incorrect. Chromium is a component of glucose tolerance factor, which maintains a normal glucose. It is also a cofactor for insulin, which facilitates the binding of insulin to receptors in the adipose and muscle. Chromium deficiency is associated with impaired glucose tolerance and peripheral neuropathy, neither of which is present in this patient. Option B (Copper) is incorrect. Copper is a cofactor for lysyl oxidase, which normally places cross-bridges between the tropocollagen molecules. Copper deficiency, which is most often caused by total parenteral deficiency, is associated with poor wound healing. However, there is no rash or loss of taste and smell in copper deficiency. Option C (Iodide) is incorrect. A deficiency of iodide produces goiter, because iodide is necessary to synthesize thyroid hormone. The patient does not have a goiter. Option D (Selenium) is incorrect. Selenium is a cofactor for glutathione peroxidase, which neutralizes peroxide and peroxide free radicals in cells (e.g., RBCs). Selenium deficiency is associated with muscle pain and weakness as well as a cardiomyopathy, none of which is present in this patient.

A 58-year-old man with a 40-pack-year history of cigarette smoking sees his physician because of weight loss, loss of appetite, yellow eyes, and right upper quadrant pain. Physical examination shows scleral icterus, tender hepatomegaly, a palpable gallbladder, and light-colored stools. An ultrasound of the gallbladder shows dilation of the common bile duct. However, no gallstones are present in the gallbladder. Laboratory studies show serum bilirubin total is 12 mg/dL and the direct is 10 mg/dL, alanine aminotransferase (ALT) is 150 U/L, aspartate aminotransferase (AST) is 100 U/L, alkaline phosphatase is 450 U/L, and -glutamyl transferase (GGT) is 330 U/L. What is the most likely diagnosis?

A. Carcinoma of the gallbladder B. Carcinoma of the head of the pancreas C. Gallstone in the common bile duct D. Hepatocellular carcinoma E. Primary sclerosing cholangitis

Option B (Carcinoma of the head of the pancreas) is correct. Cigarette smoking is the most common cause of carcinoma of the pancreas. Most pancreatic carcinomas occur in the head of the pancreas, which results in blockage of the common bile duct and obstructive jaundice. Clinical findings include weight loss, scleral icterus, a palpable gallbladder (Courvoisiers sign), and light-colored stools. Laboratory findings indicative of obstruction of bile flow include increased serum bilirubin, most of which is conjugated (>50% of the total), and a greater increase in alkaline phosphatase and GGT than in ALT and AST. A urine dipstick would show an absence of urobilinogen and increased bilirubin, signs of obstruction. Option A (Carcinoma of the gallbladder) is incorrect. Carcinoma of the gallbladder is most commonly caused by cholelithiasis. Ultrasound does not show gallstones in the gallbladder. Option C (Gallstone in the common bile duct) is incorrect. Gallstones in the common bile duct most often originate in the gallbladder. A stone in the common bile duct would not likely produce a palpable gallbladder, because preexisting inflammation of the gallbladder wall would be associated with scar formation and less distensibility of the gallbladder under increased pressure. Option D (Hepatocellular carcinoma) is incorrect. Hepatocellular carcinoma does not produce obstructive jaundice. Option E (Primary sclerosing cholangitis) is incorrect. Primary sclerosing cholangitis is most often associated with ulcerative colitis. Fibrosis develops around the common bile duct and other bile duct radicles causing obstructive jaundice.
A 59-year-old man complains of painful joints, fullness in the right upper quadrant, and a rash. Physical examination reveals palpable purpura and hepatosplenomegaly. There is no evidence of lymphadenopathy. His CBC shows a hemoglobin of 8 gm/dL, platelet count of 75,000/mm 3, and a WBC count of 3500/mm3. Abnormal cells are present in

the peripheral blood (see photograph). A bone marrow biopsy reveals a monomorphic infiltrate of cells with abundant cytoplasm. The patient is scheduled for a splenectomy. What is the most likely diagnosis?

A. Acute promyelocytic leukemia (APL) B. Adult T-cell leukemia C. Chronic lymphocytic leukemia (CLL) D. Chronic myelogenous leukemia (CML) E. Hairy cell leukemia (HCL)
Option E (Hairy cell leukemia (HCL)) is correct. The patient has HCL. The peripheral smear shows lymphocytes with dense chromatin and cytoplasmic projections. His leukemia is complicated by arthritis, palpable purpura (a small vessel vasculitis), and pancytopenia. Lymphadenopathy is conspicuously absent. Splenomegaly is the most consistent finding. The spleen is the most important site for proliferation of the neoplastic cells, which explains the importance of splenectomy in treatment. The malignant B cells characteristically have a positive tartrate-resistant acid phosphatase (TRAP) stain. Option A (Acute promyelocytic leukemia (APL)) is incorrect. APL is the most common leukemia associated with disseminated intravascular coagulation (DIC). In APL, the neoplastic promyelocytes contain numerous splinter-shaped Auer rods in the cytoplasm (not present in this patient). Option B (Adult T-cell leukemia) is incorrect. This leukemia is associated with the human T-lymphocyte virus (HTLV)-1. This is a common leukemia in Japan and occurs sporadically in the United States. Lymphoblasts secrete osteoclastactivating factor, which produces lytic bone lesions and the potential for hypercalcemia (not present in this patient). Tcell infiltration of the skin is commonly present (not present in this patient). Option C (Chronic lymphocytic leukemia (CLL)) is incorrect. CLL is the most common leukemia in patients older than 60 years of age and is also the most common cause of generalized lymphadenopathy in patients in the same age bracket (this patient lacks lymphadenopathy). The peripheral smear in CLL shows numerous lymphocytes with dense nuclear chromatin and inconspicuous nucleoli; however, they lack the cytoplasmic projections that are present in HCL.

Option D (Chronic myelogenous leukemia (CML)) is incorrect. The peripheral blood smear in CML shows neutrophils at different stages of development and increased numbers of basophils. These cells are not present in the peripheral smear in this patient.

A 32-year-old woman who was stabbed in the left lung is brought to the emergency department in severe respiratory distress. Physical examination of the left chest shows hyper-resonance to percussion and absent breath sounds. Heart sounds are heard. There is no jugular venous distention in the neck. The photograph shows the chest x-ray. Which of the following is the most likely diagnosis?

A. Cardiac tamponade B. Left pleural effusion C. Spontaneous pneumothorax D. Tension pneumothorax E. Transection of the pulmonary artery
Option D (Tension pneumothorax) is correct. A tension pneumothorax is a common complication of a knife wound to the chest. A flaplike pleural tear allows air to enter the pleural cavity during inspiration. The flap closes and prevents the air from leaving the cavity during expiration. Increased intrapleural pressure causes compression of the lung (atelectasis), hyper-resonance to percussion, absent breath sounds, and deviation of the trachea to the contralateral side (right side in this patient). The chest x-ray shows the margin of the compressed lung in the left chest cavity and deviation of the trachea to the right (arrow). Treatment is to relieve pressure by inserting a needle into the second intercostal space on the mid-clavicular line. Afterwards, a chest tube is inserted. Option A (Cardiac tamponade) is incorrect. Penetrating injury to the heart may cause cardiac tamponade. Blood enters the pericardial sac, producing muffled heart sounds and neck vein distention. These findings are not present in this

patient. Option B (Left pleural effusion) is incorrect. Physical findings in a pleural effusion include dullness to percussion (not hyper-resonance), absent breath sounds, and deviation of the trachea to the contralateral side depending on the amount of pleural fluid present. Option C (Spontaneous pneumothorax) is incorrect. Spontaneous pneumothorax is most often caused by rupture of a preexisting intrapleural bleb or a subpleural bleb. However, it also may be caused by penetrating injury to the chest. Characteristic findings include hyper-resonance to percussion, absent breath sounds, and tracheal deviation to the side of the collapsed lung (not the contralateral side), the latter depending upon the amount of lung that is collapsed. Option E (Transection of the pulmonary artery) is incorrect. Transection of a large elastic artery would most likely cause hypovolemic shock of death within minutes of injury. Such transection would most likely produce a pleural effusion.

A 55-year-old woman with hypertension and peptic ulcer disease suddenly develops right flank pain that radiates to the right groin. An electrocardiogram (ECG) shows a short QT interval and a widened T wave. Urinalysis shows a positive dipstick for blood and a negative dipstick for nitrite and leukocyte esterase. Urine sediment contains numerous RBCs and calcium oxalate crystals. Which of the following parathyroid hormone (PTH) and calcium findings is most likely present?

PTH Serum calcium A.DecreasedDecreased B. DecreasedIncreased C. Increased Decreased D.Increased Increased E. Normal Normal

A. PTH decreased, serum calcium decreased B. PTH decreased, serum calcium increased C. PTH increased, serum calcium decreased D. PTH increased, serum calcium increased E. PTH normal, serum calcium normal
Option D (PTH increased, serum calcium increased) is correct. An ECG showing short QT intervals and a widened T wave is highly suggestive of hypercalcemia. The most common cause of hypercalcemia in ambulatory individuals is primary hyperparathyroidism due to a parathyroid adenoma. In this patient, primary hyperparathyroidism is complicated by hypertension, peptic ulcer disease, and a renal calculus composed of calcium oxalate. Hypercalcemia causes contraction of the smooth muscle cells of the peripheral resistance arterioles, which causes hypertension.

Furthermore, hypercalcemia stimulates gastrin release, which causes peptic ulcer disease. Excess calcium in the urine predisposes to formation of calcium-containing calculi. Recall that hypercalcemia should suppress PTH. Therefore, the presence of hypercalcemia and an increase in PTH means that the parathyroid gland is not responding to negative feedback, which is usually a sign of tumor or primary hyperplasia. Option A (PTH decreased, serum calcium decreased) is incorrect. The ECG hallmark of hypocalcemia is prolongation of the QT interval (not shortening) because of lengthening of the ST segment, which is directly proportional to the degree of hypocalcemia. A decrease in both PTH and serum calcium is characteristic of primary hypoparathyroidism, which is most often due to previous thyroid surgery. Primary hypoparathyroidism is not associated with hypertension, peptic ulcers, or renal calculi. Recall that hypocalcemia should cause an increase in serum PTH if negative feedback is operative. Decreased PTH indicates that the parathyroid glands are nonfunctional. Option B (PTH decreased, serum calcium increased) is incorrect. A decrease in PTH and an increase in serum calcium is characteristic of hypercalcemia due to malignancy (most common cause) or other non-parathyroid-gland-related disorders (e.g., sarcoidosis, hypervitaminosis D). Hypercalcemia suppresses the release of PTH by the parathyroid gland. Primary hyperparathyroidism is the most common cause of hypercalcemia in the ambulatory population, while malignancy-induced hypercalcemia is likely to be seen in the hospitalized patient. Option C (PTH increased, serum calcium decreased) is incorrect. An increase in PTH and decrease in serum calcium is characteristic of secondary hyperparathyroidism. Hypocalcemia that is not due to primary hypoparathyroidism is a potent stimulus of PTH release. Hypovitaminosis D is a common pathologic cause of hypocalcemia leading to secondary hyperparathyroidism. Secondary hyperparathyroidism is a compensatory phenomenon that brings the serum calcium to up to or within the normal range. Secondary hyperparathyroidism does not cause hypertension, peptic ulcer disease, or renal stones. Option E (PTH normal, serum calcium normal) is incorrect. The patient has ECG evidence of hypercalcemia plus a constellation of findings that are most compatible with primary hyperparathyroidism.

A sexually active 32-year-old homosexual man has lesions on the palms (see photograph), upper and lower extremities, and trunk. He recalls having rectal bleeding after anal intercourse approximately 2 months ago. Which of the following studies is most likely to be positive?

A. Gram-stain of urethral secretions B. Rapid plasma reagin (RPR) test C. Serologic test for Rickettsia rickettsii D. Serum antinuclear antibody E. Standard urine culture
Option B (Rapid plasma reagin (RPR) test) is correct. The photograph shows the characteristic erythematous palmar papules and plaques of secondary syphilis, caused by Treponema pallidum, a Gram-negative spirochete. It is a maxim in medicine that any sexually active individual with a palmar rash has secondary syphilis until proven otherwise. The patient's history of rectal bleeding 3 months prior to the rash was most likely related to ulceration of a primary chancre. The RPR test is a nontreponemal serologic test that detects the presence of reagin antibodies. A positive test result is confirmed by the fluorescent treponemal antibody absorption (FTA-ABS) test. Option A (Gram-stain of urethral secretions) is incorrect. Treponema pallidum cannot be detected by the standard Gram stain. Silver stains or fluorescent stains are often used to identify the spirochetes in tissue. Since the treponemes invade arterioles and produce endarteritis obliterans, they would not be expected in urethral material. Option C (Serologic test for Rickettsia rickettsii) is incorrect. R. rickettsii is the pathogen responsible for Rocky Mountain spotted fever. Although the rash usually begins on the palms and spreads to the trunk; it is a petechial rash related to a vasculitis. Option D (Serum antinuclear antibody) is incorrect. The serum antinuclear antibody test detects antibodies directed against various nuclear proteins (not reagin antibodies). Option E (Standard urine culture) is incorrect. Treponemes cannot be grown in culture.

A 25-year-old woman has slightly pruritic lesions on the thigh (photograph). A potassium hydroxide (KOH) preparation of skin scrapings taken from the erythematous margin of one of the thigh lesions is positive. Which of the following organisms is the cause of the lesion?

A. Malassezia furfur B. Microsporum audouinii C. Microsporum canis D. Trichophyton rubrum E. Trichophyton tonsurans
Option D (Trichophyton rubrum) is correct. The patient has tinea corporis, an infection caused by T. rubrum, a superficial dermatophyte. The photograph shows a group of annular lesions with scaly erythematous margins and central clearing. A KOH examination of material taken from the leading edge of the lesion showed yeasts and hyphae. T. rubrum is the most common cause of fungal infections involving the skin (tinea corporis), feet (tinea pedis), nails (tinea unguim), and groin (tinea cruris). The yeast and hyphae are limited to the stratum corneum; therefore, they are easily scraped off with a scalpel blade and identified under the microscope after the keratin debris is removed by KOH. Option A (Malassezia furfur) is incorrect. M. furfur is a superficial dermatophyte that is the causal agent of tinea versicolor and seborrheic dermatitis (dandruff). Tinea versicolor is characterized either by areas of hypopigmentation or hyperpigmentation of the skin. Option B (Microsporum audouinii) is incorrect. M. audouinii is a superficial dermatophyte that is a common cause of tinea capitis. Option C (Microsporum canis) is incorrect. M. canis is a superficial dermatophyte that is a common cause of tinea capitis. Option E (Trichophyton tonsurans) is incorrect. T. tonsurans is a superficial dermatophyte that is a common cause of tinea capitis.

A 25-year-old man who lives in Ohio and enjoys cave exploration has a solitary coin lesion in the upper lobe of the right lung. The size of the lesion has not increased in the past 3 years. A chest radiograph shows that the lesion has concentric calcifications. The patient does not smoke cigarettes. Which of the following is the most likely diagnosis?

A. Benign neoplasm B. Granuloma C. Hamartoma

D. Malignant primary neoplasm E. Metastasis from an extrapulmonary site


Option B (Granuloma) is correct. A solitary pulmonary nodule, or coin lesion, is the term applied to a peripheral lung nodule that is smaller than 5 cm. Causes of a solitary pulmonary nodule in descending order include granulomas (e.g., TB, histoplasmosis), malignancy (usually primary cancer), and a bronchial (chondroid) hamartoma. Patients under 35 years of age have a 1% risk of a solitary coin lesion representing a malignancy, but patients older than 50 years have a 50% to 60% risk of malignancy, usually a primary cancer. In evaluating solitary coin lesions, comparing previous chest xrays for changes in size of the nodule is the most important initial step. In this patient, the most likely cause of the lesion is previous infection with Histoplasma capsulatum. The patient lives in Ohio and explores caves. H. capsulatum is endemic in the Ohio River and central Mississippi River valleys. Cave-dwelling bats often eliminate H. capsulatum in their stools and the microconidia can be inhaled causing the infection. Option A (Benign neoplasm) is incorrect. In patients under 35 years of age, the majority of solitary coin lesions in the lung are granulomas, not benign neoplasms. Furthermore, neoplasms do not have concentric calcification. Option C (Hamartoma) is incorrect. Bronchial hamartomas are a non-neoplastic proliferation of cartilage and adipose tissue. The majority are peripherally located. They appear as a solitary coin lesion with popcorn (not concentric) calcifications. Option D (Malignant primary neoplasm) is incorrect. A malignant primary neoplasm in a patient under 35 years old, who does not smoke, is highly unlikely. Option E (Metastasis from an extrapulmonary site) is incorrect. The majority of metastases to the lungs are multifocal rather than solitary.

A 26-year-old man is scuba diving off the coast of Bermuda in water 30 to 60 feet deep when he develops problems with his air tank and must ascend quickly to the surface. One hour later, he has pain in the muscles and joints in his legs. Which of the following is most likely responsible for these symptoms?

(check all that apply) A. Deep venous thrombosis B. Disseminated intravascular coagulation (DIC) C. Fat embolization D. Hemorrhage into muscles and joints E. Nitrogen gas embolism

Option E (Nitrogen gas embolism) is correct.. The patient has decompression sickness (gas embolism), which is a complication of scuba diving. As a diver descends, the atmospheric pressure increases by 1 atmosphere (760 mm Hg) for every 33 feet of water. Under increased pressure, nitrogen gas moves from the alveoli through the blood and dissolves in tissue and blood. Rapid ascent forces nitrogen to move out of tissue and blood as bubbles. It forms gas emboli in the blood that obstruct blood vessels, causing ischemic damage to bone (e.g., aseptic necrosis of the femoral head), spinal cord (hemiparesis), and other tissues. Gas bubbles within skeletal muscle and supporting tissues around joints cause a painful condition called the bends. Treatment involves recompression in a compression chamber to force the gas bubbles back into solution followed by slow decompression to prevent them from re-forming. Option A (Deep venous thrombosis) is incorrect. Increased atmospheric pressure under water may cause stasis and thrombus formation in the deep veins of the leg, potentially causing pulmonary thromboembolism. Leg pain in deep venous thrombosis develops while under water. Option B (Disseminated intravascular coagulation (DIC)) is incorrect. DIC is caused by in vivo activation of the coagulation system, resulting in the formation of fibrin clots throughout the microvasculature. DIC is not a complication of scuba diving. Option C (Fat embolization) is incorrect. Fat embolization is most often caused by traumatic fractures of the long bones (e.g., femur) and pelvis. Microglobules of fat from the bone marrow and adipose lodge in the microvasculature throughout the body, producing ischemic damage to tissue. Fat embolization is not a complication of scuba diving. Option D (Hemorrhage into muscles and joints) is incorrect. Hemorrhage into muscles and joints is a complication of a severe coagulation factor deficiency (e.g., hemophilia A). Such hemorrhage is not associated with nitrogen gas embolism.

The photograph shows the hands of an 82-year-old woman. The lesions are nonpalpable and nonpruritic and are found only in this location. Lymphadenopathy and hepatosplenomegaly are not present on physical examination. The patient is not taking any prescription drugs or over-the-counter medications. The complete blood cell count is normal. Which of the following is the most likely cause of the patients disorder?

A. Antibodies directed against a platelet receptor B. Deficiency of von Willebrand factorcleaving protease

C. Immunocomplex vasculitis involving small vessels D. Increased capillary fragility in the elderly E. Infiltrative bone marrow disease with destruction of megakaryocytes

Option D (Increased capillary fragility in the elderly) is correct. The patient has senile purpura, which is characterized by large, irregular areas of hemorrhage on the dorsa of both hands. Senile purpura is a benign condition in the elderly. Increased cross-linking of collagen, a normal finding in the elderly, decreases vessel distensibility leading to increased fragility of vessels primarily in areas that are frequently traumatized, such as the dorsa of the hands. Option A (Antibodies directed against a platelet receptor) is incorrect. Antibodies directed against a platelet receptor are implicated in idiopathic thrombocytopenic purpura in children and chronic autoimmune thrombocytopenia in adults, where an IgG antibody is directed against the GpIIb/IIIa fibrinogen receptor. Thrombocytopenia is not present in this patient. Option B (Deficiency of von Willebrand factorcleaving protease) is incorrect. A deficiency of von Willebrand factor cleaving protease is found in thrombotic thrombocytopenic purpura, which produces a classic pentad of fever, thrombocytopenia, microangiopathic hemolytic anemia with schistocytes, central nervous system dysfunction, and renal failure. Option C (Immunocomplex vasculitis involving small vessels) is incorrect. Immunocomplex vasculitis involving small vessels produces palpable purpura referring to areas of subcutaneous hemorrhage with painful swelling of the overlying tissue, a sign of acute inflammation. Option E (Infiltrative bone marrow disease with destruction of megakaryocytes) is incorrect. Infiltrative bone marrow disease with destruction of megakaryocytes commonly occurs in leukemia, myelofibrosis, and metastatic disease to the bone marrow. Destruction of megakaryocytes produces thrombocytopenia.

A postmenopausal 55-year-old woman has vaginal bleeding. Speculum examination of the vagina and cervix is normal. Endometrial biopsy shows sheets of hyperchromatic cells, many of which have atypical mitotic spindles. There are focal areas of necrosis. Which of the following best characterizes this tumor?

A. Increased intercellular adhesion B. Polyclonal cells in culture C. Secretion of growth factors that inhibit angiogenesis

D. Upregulation of telomerase activity


Option D (Upregulation of telomerase activity) is correct. Upregulation of telomerase activity, which preserves the telomere length and ensures that there is no loss of genetic material after each cell division, is an indication of malignancy. The presence of hyperchromatic cells with atypical mitotic spindles in an endometrial biopsy specimen is also characteristic of malignancy. Option A (Increased intercellular adhesion) is incorrect. The cells in malignant tumors lose intercellular adhesion by decreasing the production of E-cadherin, an intercellular adhesion agent. Decreased intercellular adhesion must occur before these cells are able to invade through the basement membrane into the extracellular matrix. Option B (Polyclonal cells in culture) is incorrect. The cells in benign tumors are polyclonal in culture, whereas the cells in malignant cells are monoclonal. Option C (Secretion of growth factors that inhibit angiogenesis) is incorrect. Malignant cells secrete vascular endothelial growth factor and basic fibroblast growth factor, which stimulate angiogenesis. Angiogenesis is necessary to provide oxygen and nutrients to the malignant cells.

A 1-year-old child has mental retardation, blindness, and a cherry red macula. Which of the following types of mutations is most likely responsible for these findings?

A. Frameshift B. Missense C. Nonsense D. Silent E. Translocation

Option A (Frameshift) is correct. A frameshift mutation is due to insertion or deletion of one or more nucleotides that causes a shift in the reading frame of the DNA strand. The patient has Tay-Sachs disease (mental retardation, cherry red macula) due to a deficiency of the lysosomal enzyme hexosaminidase A, which normally degrades GM2 gangliosides in the lysosome. Deficiency of hexosaminidase A causes the gangliosides to accumulate. The insertion of four nucleotides into the reading frame of DNA produces a stop codon leading to decreased synthesis of hexosaminidase. Option B (Missense) is incorrect. In a missense mutation, a point mutation produces an altered codon that specifies a different amino acid, causing variable phenotypic effects, such as UUG (lysine) UCG (serine). Option C (Nonsense) is incorrect. In a nonsense mutation, a point mutation produces a stop codon (e.g., UAG) that causes premature termination of protein synthesis.

Option D (Silent) is incorrect. In a silent mutation, a point mutation produces an altered codon that specifies the same amino acid without altering the phenotypic effect (no disease), such as UUG (lysine) CUG (lysine). Option E (Translocation) is incorrect. In a translocation, one part of a chromosome is transferred to another chromosome. The resulting structural abnormality causes disease. For example, translocation of the ABL protooncogene from chromosome 9 to chromosome 22 causes chronic myelogenous leukemia. Which of the following preventive measures would most likely reduce the incidence of hepatocellular carcinoma in Southeast Asia?

A. Cessation of drinking alcohol B. Control of sexually transmitted disease C. Increase intake of dietary fiber D. Mass immunization against a virus E. Smoking cessation
Option D (Mass immunization against a virus) is correct. Hepatocellular carcinoma commonly develops from postnecrotic cirrhosis caused by hepatitis B, which is very common in Southeast Asia. Mass immunization with the hepatitis B vaccine would reduce the development of hepatitis B and its related complications. Option A (Cessation of drinking alcohol) is incorrect. Alcohol abuse may produce alcoholic cirrhosis, which increases the risk for developing hepatocellular carcinoma. However, alcoholic cirrhosis is not as common a cause of hepatocellular carcinoma as postnecrotic cirrhosis due to hepatitis B. Option B (Control of sexually transmitted disease) is incorrect. The only sexually transmitted disease that is also a carcinogenic virus is the human papillomavirus (HPV). It is a risk factor for squamous cell carcinoma of the vulva, vagina, and cervix. HPV is not a risk factor for hepatocellular carcinoma. The HPV vaccine will decrease the risk for all of these cancers. Option C (Increase intake of dietary fiber) is incorrect. Increasing dietary fiber reduces the risk of colorectal cancer. It does not reduce the risk of hepatocellular carcinoma. Option E (Smoking cessation) is incorrect. Smoking cessation reduces the risk of cancer in the lung, oropharynx, esophagus, pancreas, kidney, and urinary bladder. It does not reduce the risk for hepatocellular carcinoma.

A 45-year-old man developed fever, headache, and pain when moving his neck. The photograph shows the surface of the cerebral hemisphere removed at autopsy. Gram stain of the cerebrospinal fluid most likely shows which of the following types of bacteria?

A. Gram-negative diplococci B. Gram-negative rods C. Gram-positive cocci D. Gram-positive diplococci E. Gram-positive rods
Option D (Gram-positive diplococci) is correct. The patient has bacterial meningitis caused by Streptococcus pneumoniae, a Gram-positive diplococcus. This pathogen is the most common cause of bacterial meningitis in individuals over 18 years of age. The photograph shows engorged blood vessels and a purulent exudate covering the lateral surface of the cerebral hemispheres. Option A (Gram-negative diplococci) is incorrect. Bacterial meningitis caused by Neisseria meningitidis (Gram-negative diplococcus) most often occurs in individuals between 1 month and 18 years of age. Option B (Gram-negative rods) is incorrect. Meningitis caused by Escherichia coli usually occurs in newborns and in individuals over 50 years of age. Option C (Gram-positive cocci) is incorrect. Meningitis caused by Staphylococcus aureus usually occurs in postsurgical patients, patients with penetrating trauma to the brain, and intravenous drug abusers. Meningitis caused by Streptococcus agalactiae occurs in newborns. Option E (Gram-positive rods) is incorrect. Bacterial meningitis caused by Gram-positive rods is most often due to Listeria monocytogenes. It causes bacterial meningitis in newborns and in immunocompromised patients, particularly those with AIDS.

A 50-year-old man with polycythemia vera presents with a severe right upper quadrant pain. Physical examination reveals an enlarged, painful liver; ascites; and splenomegaly. Serum transaminases are markedly increased. What is the most likely diagnosis?

A. Centrilobular hemorrhagic necrosis B. Hepatic artery thrombosis C. Hepatic vein thrombosis D. Peliosis hepatis E. Portal vein thrombosis
Option C (Hepatic vein thrombosis) is correct. Hepatic vein thrombosis is a potential complication of any thrombosis state (e.g., polycythemia vera, oral contraceptive pills, antiphospholipid syndrome, protein C/S deficiency). In this case, polycythemia vera is the cause. It is an example of a posthepatic obstruction of blood flow. Blood builds up behind the obstructed hepatic vein and produces congestion of the central veins in the liver (painful hepatomegaly) with concomitant necrosis of the zone III hepatocytes (markedly increased serum transaminases). The increased pressure extends into the portal vein (portal vein hypertension) leading to ascites and splenomegaly. Ultrasound with pulsed Doppler is the first-line test. Magnetic resonance imaging is also useful in securing the diagnosis. Treatment modalities include anticoagulation, in-situ thrombolysis, stenting, and various shunts. There is a high mortality with approximately 75% mortality in the first year. Option A (Centrilobular hemorrhagic necrosis) is incorrect. Centrilobular hemorrhagic necrosis is an example of an intrahepatic obstruction of blood flow. It is most often due to a combination of left-sided heart failure (LHF) and rightsided heart failure (RHF). LHF decreases cardiac output causing hypoperfusion of the liver (produces liver cell necrosis) and RHF causes a back-up of systemic venous blood into the hepatic veins and then to the central veins and sinusoids producing an enlarged, painful liver with a mottled red appearance; and into the portal veins producing ascites and splenomegaly. It differs from hepatic vein thrombosis in that it is associated with biventricular failure and RHF signs of jugular neck vein distention and dependent pitting edema, findings that are not present in hepatic vein thrombosis. Option B (Hepatic artery thrombosis) is incorrect. Hepatic artery thrombosis is an example of prehepatic obstruction of blood flow. Liver infarction is uncommon because of a dual blood supply (hepatic artery and portal vein). Causes include liver transplant rejection and vasculitis due to polyarteritis nodosa. Portal hypertension, ascites, and splenomegaly are not present. Option D (Peliosis hepatis) is incorrect. Peliosis hepatis is an example of an intrahepatic obstruction of blood flow. It is characterized by the presence of sinusoidal dilation due to blood. Causes include anabolic steroids and Bartonella henselae causing bacillary angiomatosis in patients with AIDS. There is a potential for intraperitoneal hemorrhage. Ascites and splenomegaly are not present. Option E (Portal vein thrombosis) is incorrect. Portal vein thrombosis is an example of a prehepatic obstruction of

blood flow. It is most often caused by pylephlebitis (inflammation of the portal vein) secondary to acute appendicitis. Radiographs show air in the biliary tree from bacterial gas. Other causes include polycythemia vera and hepatocellular carcinoma (invasion of the portal vein). There is no hepatomegaly, portal hypertension, ascites, or splenomegaly.

A febrile 30-year-old woman sees her physician because of a rash on the thigh (see photograph) that developed at the site where she was bitten by an insect when on a camping trip about 4 weeks ago. The woman lives in the northeastern United States. Which of the following is the most likely causal pathogen?

A. Babesia microti B. Borrelia burgdorferi C. Borrelia recurrentis D. Ehrlichia chaffeensis E. Rickettsia rickettsii
Option B (Borrelia burgdorferi) is correct. The photograph shows an erythematous expanding rash with concentric circles separated by clear spaces. This is erythema chronicum migrans, which is the pathognomonic skin lesion of early Lyme disease. The disease is transmitted to humans by the bite of an Ixodes tick, which carries the Gram-negative spirochete B. burgdorferi. Option A (Babesia microti) is incorrect. B. microti is a protozoan carried by the Ixodes tick. It parasitizes RBCs and

causes a mild hemolytic anemia that may occur concurrently with Lyme disease. Option C (Borrelia recurrentis) is incorrect. B. recurrentis is a spirochete that is transmitted to humans by the bite of a tick. This tick also carries the rickettsial organisms Ehrlichia chaffeensis and Rickettsia rickettsii. B. recurrentis is the causative agent of relapsing fever, which is associated with a high fever and a petechial rash (not a rash with concentric circles) that covers the trunk and extremities. Option D (Ehrlichia chaffeensis) is incorrect. E. chaffeensis is a tick-transmitted rickettsial pathogen that is the causal agent of ehrlichiosis. The organism parasitizes monocytes and causes fever and multisystem disease. The rash in ehrlichiosis is an erythematous maculopapular rash unlike the rash in Lyme disease. Option E (Rickettsia rickettsii) is incorrect. R. rickettsii is a tick-transmitted pathogen that is the causative agent of Rocky Mountain spotted fever, which is characterized by fever, multisystem disease, and a petechial rash (not a rash with concentric circles) that begins on the palms and spreads to the trunk. The rash is due to a vasculitis.

Which of the following clinical disorders is most compatible with the distribution of affected patients shown in this pedigree?

A. Glucose-6-phosphate dehydrogenase (G6PD) deficiency B. Hereditary spherocytosis C. Hemochromatosis D. Neurofibromatosis E. Type 2 diabetes mellitus
Option C (Hemochromatosis) is correct. Hemochromatosis, a disorder involving excessive reabsorption of iron from the gastrointestinal tract, has an autosomal recessive inheritance pattern, as shown in the pedigree. In autosomal recessive

disorders, disease is present only in patients who are homozygous for the abnormal allele (e.g., aa), and both parents must have the abnormal allele to transmit the disease to their children. In most cases, the parents are asymptomatic heterozygous carriers (e.g., Aa). They have a 25% chance of having a normal child (AA), a 50% chance of having children who are asymptomatic carriers (Aa), and a 25% chance of having a child with the disease (aa), which is evident in the pedigree. Option A (G6PD deficiency) is incorrect. Deficiency of G6PD is an X-linked recessive disorder associated with hemolytic anemia induced by infection or oxidant drugs (e.g., primaquine). X-linked recessive disorders are expressed in males, whereas females with the abnormal allele are usually asymptomatic carriers. Option B (Hereditary spherocytosis) is incorrect. Hereditary spherocytosis is an autosomal dominant disorder associated with splenomegaly and hemolytic anemia. Autosomal dominant disorders are characterized by a dominant allele that expresses itself in either the homozygous or the heterozygous state. Only one parent must have the abnormal allele for the disease to be transmitted to the children. A heterozygous parent with disease will transmit it to 50% of the children. Option D (Neurofibromatosis) is incorrect. Neurofibromatosis is an autosomal dominant disorder characterized by caf-au-lait spots and pedunculated neurofibromas on the skin. Option E (Type 2 diabetes mellitus) is incorrect. Type 2 diabetes is an example of multifactorial (polygenic) inheritance. Multifactorial inheritance involves the additive effect of two or more gene mutations of small effect conditioned by environmental and other nongenetic factors.

A 6-year-old child has a primary malignant tumor arising in the para-aortic lymph nodes (see photograph). Chromosome studies show a t(8;14) translocation. Which of the following microbial pathogens has a causal relationship with this tumor?

A. Cytomegalovirus (CMV) B. Epstein-Barr virus (EBV) C. Helicobacter pylori D. Human herpesvirus-8 (HHV-8)

E. Human papillomavirus (HPV)


Option B (Epstein-Barr virus (EBV)) is correct. EBV is an oncogenic virus that infects B cells, causing them to undergo mitoses that predispose to a t(8;14) translocation. Translocation of the MYC proto-oncogene (nuclear transcriber) from chromosome 8 to chromosome 14 causes increased activity of the gene, eventually causing Burkitt's lymphoma. The photograph shows complete effacement of the lymph node by malignant lymphocytes with clear spaces containing reactive histiocytes containing phagocytic debris. At low power this has a starry sky appearance, with the lymphocytes representing the dark of night and the histiocytes the stars. Option A (Cytomegalovirus (CMV)) is incorrect. CMV is not an oncogenic virus. Option C (Helicobacter pylori) is incorrect. H. pylori increases the risk for developing adenocarcinoma of the stomach and low-grade malignant lymphoma of the stomach. Option D (Human herpesvirus-8 (HHV-8)) is incorrect. HHV-8 is an oncogenic virus that causes Kaposi sarcoma in AIDS. It is not a risk factor for malignant lymphoma. Option E (Human papillomavirus (HPV)) is incorrect. HPV is an oncogenic virus that produces protein products that inactivate the RB suppressor gene and the TP53 suppressor gene. HPV is associated with squamous cell carcinoma of the cervix, vagina, and vulva. HPV is not a risk factor for malignant lymphoma.

A 35-year-old man has cancer involving the lungs, brain, liver, spleen, and kidneys. He has a 10-pack-year history of smoking cigarettes. The photograph shows a lesion that was discovered on physical examination. Which of the following is the most likely cause of his metastatic disease?

A. Glioblastoma multiforme B. Hepatocellular carcinoma C. Malignant melanoma D. Primary lung cancer E. Renal cell carcinoma

Option C (Malignant melanoma) is correct. The pigmented lesion under the nail is an acral lentiginous malignant melanoma. This variant of melanoma is extremely aggressive and commonly causes widespread metastatic disease. Smoking is not a risk factor for this type of malignant melanoma. Option A (Glioblastoma multiforme) is incorrect. A glioblastoma multiforme does not metastasize outside the central nervous system. It can seed the spinal fluid and implant on other parts of the brain and spinal cord. Option B (Hepatocellular carcinoma) is incorrect. Hepatocellular carcinomas invade hepatic veins and portal veins and metastasize to the lungs and other sites. However, the nail bed is not a common location for metastasis. Option D (Primary lung cancer) is incorrect. Cigarette smoking is the most common cause of lung cancer, which metastasizes throughout the body. However, the nail bed is not a common location for metastasis. Option E (Renal cell carcinoma) is incorrect. Cigarette smoking is the most common cause of a renal cell carcinoma. These cancers invade the renal vein and metastasize to the lungs, skin, and many other sites. However, the nail bed is not a common location for metastasis.

A 6-year-old boy is admitted to the hospital for evaluation of a suspected immunodeficiency. Since infancy, he has had eczema involving his face and flexor creases in his arms and legs. He has had recurrent pneumonias, sinus infections, otitis media, and skin infections. Physical examination reveals chronic eczema, predominantly in flexor skin surfaces and face, and otitis media in the left ear. A chest x-ray is normal. A CBC shows thrombocytopenia and borderline low lymphocyte count. Serum IgG is normal; IgA and IgE are increased; and IgM is decreased. The Candida skin test is nonreactive. What is the most likely diagnosis?

A. Ataxia telangiectasia B. Brutons agammaglobulinemia C. Common variable immunodeficiency (CVID) D. DiGeorge syndrome E. Selective IgA immunodeficiency F. Severe combined immunodeficiency (SCID) G. Wiskott-Aldrich syndrome
Option G (Wiskott-Aldrich syndrome) is correct. Wiskott-Aldrich syndrome is an X-linked recessive disorder with progressive depletion of B and T cells. The classic triad is eczema, thrombocytopenia, and recurrent sinopulmonary infections. There is an increased risk for malignant lymphoma and leukemia. Immunoglobulins show a decreased IgM, normal IgG, and increased IgA and IgE. There is defective cell mediated immunity (anergic to Candida skin test). There is

an increased risk for developing malignant lymphoma. The treatment is bone marrow transplantation. Option A (Ataxia telangiectasia) is incorrect. Ataxia telangiectasia is an autosomal recessive disorder with a mutation in DNA repair enzymes leading to increased susceptibility for chromosomal mutations. Clinical findings include cerebellar ataxia and telangiectasias of the eyes and skin. Laboratory studies reveal an increase in serum -fetoprotein; decreased in IgA, IgE, low molecular weight IgM, and IgG2 or total immunoglobulins. There is defective cell mediated immunity and an increased risk for developing malignant lymphoma, leukemia, and adenocarcinoma. Treatment is intravenous immunoglobulins and deferoxamine. Option B (Brutons agammaglobulinemia) is incorrect. Brutons agammaglobulinemia is an X-linked recessive disorder where a mutation in tyrosine kinase causes failure of pre-B cells to become mature B cells. Patients have recurrent sinopulmonary infections and an increased incidence of Streptococcus pneumoniae infections. All immunoglobulins (Igs) are decreased and there is a flat -globulin peak in a serum protein electrophoresis. Treatment is intravenous immunoglobulins. Option C (Common variable immunodeficiency (CVID)) is incorrect. CVID has no inheritance pattern. It is the most common immunodeficiency in adults. It is characterized by recurrent sinopulmonary infections, malabsorption (celiac sprue), and an increased risk for autoimmune disease, non-Hodgkins lymphoma, and giardiasis. All immunoglobulins are decreased. Treatment is intravenous immunoglobulins. Option D (DiGeorge syndrome) is incorrect. In DiGeorge syndrome, there is failure of development of the 3rd and 4th pharyngeal pouches. This leads to a pure T cell immunodeficiency and hypoparathyroidism, the latter producing hypocalcemia and tetany. Cyanotic congenital heart disease due to a truncus arteriosus (common aorta and pulmonary artery) occurs in some cases. Treatment is calcium salts to correct hypocalcemia and bone marrow transplantation. Thymus transplantation has had mixed results. Option E (selective IgA immunodeficiency) is incorrect. Selective IgA deficiency is the most common hereditary immunodeficiency. It is characterized by failure of IgA B cells to mature into plasma cells leading to decreased serum IgA and secretory IgA. Sinopulmonary infections and giardiasis are common findings. They may develop anaphylaxis when exposed to blood products containing IgA. There is no treatment for the underlying disorder. Option F (severe combined immunodeficiency) is incorrect. . SCID is an autosomal recessive disease. In some cases, there is a deficiency of adenine deaminase leading to an accumulation of adenine in B and T cells resulting in death of the cells. All immunoglobulins are decreased and there is defective cell-mediated immunity (CMI). Treatment is gene therapy and bone marrow transplantation.

A 22-year-old asymptomatic black woman has had a mild microcytic anemia since early childhood. Her menstrual history is normal. Physical examination is normal. Laboratory studies show a slightly decreased hemoglobin concentration and an increased RBC count. Hemoglobin electrophoresis is normal. Serum ferritin is normal. Which of the following is the most likely diagnosis?

A. Iron deficiency

B. Sickle cell trait C. Sideroblastic anemia D. -Thalassemia E. -Thalassemia


Option D (-Thalassemia) is correct. -Thalassemia is an autosomal recessive disorder that involves decreased production of -globin chains, causing a microcytic anemia with decreased synthesis of hemoglobin A (22), hemoglobin A2 (22), and hemoglobin F (22). The RBC count is usually increased in the thalassemias, and it is decreased in all the other microcytic anemias (e.g., iron deficiency anemia). Hemoglobin electrophoresis is normal, because the proportion of each of the decreased hemoglobins remains the same. Serum ferritin is normal in thalassemia. Option A (Iron deficiency) is incorrect. Iron deficiency is the most common cause of a microcytic anemia. The RBC count is decreased and serum ferritin is decreased. Option B (Sickle cell trait) is incorrect. Sickle cell trait does not cause anemia. Option C (Sideroblastic anemia) is incorrect. Sideroblastic anemias are a group of microcytic anemias that are caused by a defect in the synthesis of heme (iron plus protoporphyrin) in the mitochondria of developing RBC normoblasts. Iron accumulates in the mitochondria of nucleated RBCs and produces ringed sideroblasts, which are easily identified in a bone marrow aspirate. RBC count is decreased and serum ferritin is increased in these anemias. Option E (-Thalassemia) is incorrect. In -thalassemia, hemoglobin A (22) is decreased, because -globin chain synthesis is decreased. There is a corresponding increase in hemoglobin A2 (22) and hemoglobin F (22), because -, -, and -chain synthesis is normal. The hemoglobin electrophoresis is normal in this patient, that is, excluding thalassemia.

A 49-year-old woman complains of muscle weakness. Physical examination shows diastolic hypertension. When the blood pressure is taken, the patients thumb adducts into the palm. A random urine potassium is increased. An ECG shows prominent U waves. Which of the following laboratory findings is most likely to be reported? A. Decreased serum HCO3 B. Decreased serum K+ C. Decreased serum Na+ D. Decreased serum PO42

E. Increased serum Ca2+


Option B (Decreased serum K +) is correct. The hypertension is caused by an adrenal adenoma that secretes aldosterone (primary aldosteronism). Hyperaldosteronism leads to an increased exchange of Na+ for K+ in the late distal and collecting tubules, causing an increase in serum Na+ and a decrease in serum K+ (hypokalemia). Hypokalemia produces prominent U waves (positive wave after the T wave) on an ECG and causes muscle weakness because of lack of muscle repolarization. Once K+ is unavailable, Na+ exchanges for H+, leading to regeneration of HCO3 and an increase in serum HCO3 (metabolic alkalosis). Metabolic alkalosis increases the number of negative charges on albumin, the binding protein for calcium. The extra calcium that binds to the negative charges comes from the ionized calcium fraction. This causes tetany (adduction of the thumb into the palm), due to a decrease in ionized calcium without altering the total serum calcium, which is the sum of calcium bound to albumin and ionized calcium. Option A (Decreased serum HCO3) is incorrect. Bicarbonate is increased (metabolic alkalosis) in primary aldosteronism, not decreased (metabolic acidosis). Option C (Decreased serum Na+) is incorrect. Serum sodium is increased (not decreased) in primary aldosteronism, due to increased renal reabsorption of sodium. Option D (Decreased serum PO42) is incorrect. The serum phosphorus level is normal in primary aldosteronism, because aldosterone is not involved in phosphorus reabsorption in the kidneys. Option E (Increased serum Ca2+) is incorrect. The serum calcium is normal in primary aldosteronism, because the increase in negative charges on albumin causes a shift of calcium from the ionized fraction to the bound fraction without altering the total serum calcium.

After taking penicillin, a 32-year-old woman developed generalized palpable purpura and sudden onset of oliguria associated with hematuria, RBC casts, and proteinuria. She progressed into renal failure and died. The photograph shows one of many glomeruli in a kidney biopsy. Which of the following is most likely involved in the pathogenesis of the glomerular disease?

A. Antineutrophil cytoplasmic antibodies (p-ANCA) B. Immunocomplexes

C. Loss of negative charge in the basement membrane D. Nonenzymatic glycosylation E. Osmotic damage
Option A (Antineutrophil cytoplasmic antibodies (p-ANCA)) is correct. The patient has microscopic polyangiitis. It is a small vessel type of vasculitis (palpable purpura) that can by precipitated by drugs (penicillin in this case), infections, and immune disorders. Whatever the triggering event is in this disease, it results in increased production of p-ANCA antibodies in more than 80% of cases. These antibodies are directed against myeloperoxidase in neutrophils, which cause degranulation and thus the release of toxins causing endothelial injury to small vessels throughout the body as well as to those in the glomeruli. Renal failure is most often due to rapidly progressive crescentic glomerulonephritis, which is associated with a nephritic presentation (hematuria, RBC casts, mild proteinuria), as seen in this patient. The photograph shows proliferation of parietal epithelial cells in Bowmans capsule forming a crescent around the glomerulus, the base of which is located at the 3 oclock position. As the name implies, it is a rapidly progressive glomerulonephritis that produces renal failure in a few weeks and is associated with a very poor prognosis. Option B (Immunocomplexes) is incorrect. The damage to vessels and glomeruli in microscopic polyangiitis is due to the p-ANCA antibodies, not immunocomplexes. Option C (Loss of negative charge in the basement membrane) is incorrect. Loss of the negative charge in the basement membrane is the mechanism for minimal change disease (lipoid nephrosis), which is the most common cause of the nephrotic syndrome in children. Option D (Nonenzymatic glycosylation) is incorrect. Nonenzymatic glycosylation is the binding of glucose to amino acids in the basement membrane of arterioles and the glomerular basement membrane. Glycosylation products render the membrane permeable to proteins. It produces glomerular damage in diabetic nephropathy leading, in some cases, to the nephrotic syndrome. Option E (Osmotic damage) is incorrect. Osmotic damage is the conversion of glucose to sorbitol by aldose reductase. Sorbitol is osmotically active and draws water into cells. Osmotic damage is responsible for tissue damage in diabetes mellitus (e.g., retinopathy, cataracts, peripheral neuropathy).

A 52-year-old woman had a modified radical mastectomy 12 years ago. She now has pain in the pelvic girdle and point tenderness over the lower lumbar vertebra. A complete blood cell count shows a mild normocytic anemia, a slightly increased total WBC count, and a normal platelet count. There is no hepatosplenomegaly. The photograph shows a representative section of the peripheral blood smear. Which of the following is the most likely diagnosis?

A. Acute myeloblastic leukemia B. Leukemoid reaction C. Malignant plasma cell disorder D. Metastatic disease to the bone marrow E. Myelofibrosis and myeloid metaplasia (MMM)
Option D (Metastatic disease to the bone marrow) is correct. The patient has breast cancer metastatic to the bone marrow, with subsequent reactive fibrosis (myelofibrosis) causing displacement of normal bone marrow cells into the peripheral blood. The peripheral blood in this patient shows a nucleated normoblast (small nucleated cell) and immature myeloid cells (large nucleated cell), which are normal in the bone marrow but abnormal in the peripheral blood. The tear-drop shape of many of the RBCs in the peripheral blood smear is caused by membrane damage that occurs when they exit the fibrotic bone marrow. Breast cancer commonly metastasizes to bone, often to the vertebral column, as in this patient. Option A (Acute myeloblastic leukemia) is incorrect. Although acute myeloblastic leukemia occurs in patients the same age as this patient, it usually causes thrombocytopenia, generalized lymphadenopathy, and hepatosplenomegaly and is characterized by the presence of myeloblasts with Auer rods (red, splinter to rod-shaped inclusions) in a peripheral blood smear. Option B (Leukemoid reaction) is incorrect. A leukemoid reaction is a benign increase in mature leukocytes (e.g., neutrophils, lymphocytes) often >50,000/mm3. It usually occurs in response to infection (e.g., tuberculosis) and does not cause anemia or produce tear-drop RBCs. Option C (Malignant plasma cell disorder) is incorrect. The most common malignant plasma cell disorder is multiple myeloma. Evidence of multiple myeloma in the peripheral blood is the presence of rouleaux (RBCs stacked like coins). In addition, lytic lesions in bone are commonly present, which are not mentioned in this case. Option E (Myelofibrosis and myeloid metaplasia (MMM)) is incorrect. MMM is a clonal myeloproliferative disease due to a mutation of the JAK2 gene on the short arm of chromosome 9. It produces ineffective erythropoiesis, immature granulocytes in the peripheral blood, and reactive myelofibrosis. Extramedullary hematopoiesis occurs spleen, liver, and

other sites. Peripheral blood findings are similar to metastasis to the bone marrow that is present in this patient; however, in this patient, there is a history of breast cancer and there is point tenderness limited to the vertebral column. In addition, there is no hepatosplenomegaly, which would definitely be present in MMM.

The photograph shows a 28-year-old man, who is infertile. He does not recall ever having had erections during the night. Findings on physical examination include slightly tender subareolar masses and diminished volume of both testicles. Which of the following laboratory findings is most likely to be reported?

A. Decreased concentration of follicle-stimulating hormone (FSH) B. Decreased concentration of luteinizing hormone (LH) C. Increased concentration of serum estradiol D. Normal concentration of serum testosterone E. Normal sperm count
Option C (Increased concentration of serum estradiol) is correct. The patient has Klinefelters syndrome. Characteristic findings in Klinefelters syndrome include an XXY phenotype with 47 chromosomes and signs of hyperestrinism (gynecomastia, female distribution of pubic hair) as noted in the photograph. Diminished testicular volume is due to testicular atrophy. The testicles show fibrosis of the seminiferous tubules and hyperplasia of the Leydig cells. Loss of the seminiferous tubules leads to absence of spermatogenesis (azoospermia) and absence of Sertoli cells, which normally

contain the hormone inhibin. Inhibin has a negative feedback relationship with FSH; therefore, a decrease in inhibin causes an increase in FSH. FSH, in turn, increases the synthesis of aromatase in the hyperplastic Leydig cells leading to conversion of all the testosterone to estradiol, which produces the signs of hyperestrinism that are present in the patient. Serum testosterone levels are decreased, which leads to a decrease in libido (sexual desire) and impotence (failure to sustain an erection). Option A (Decreased concentration of follicle-stimulating hormone (FSH)) is incorrect. In Klinefelters syndrome, destruction of Sertoli cells in the seminiferous tubules leads to a decrease in inhibin and a corresponding increase in FSH. Option B (Decreased concentration of luteinizing hormone (LH)) is incorrect. In Klinefelters syndrome, decreased testosterone causes a corresponding increase in LH. Option D (Normal concentration of serum testosterone) is incorrect. In Klinefelters syndrome, serum testosterone is decreased, because increased aromatase activity in the Leydig cells converts all the androgens to estrogens. Option E (Normal sperm count) is incorrect. In Klinefelters syndrome, destruction of the seminiferous tubules prevents spermatogenesis, causing a total absence of sperm (azoospermia).

A 22-year-old man has fever, fatigue, and a sore throat. Findings on physical examination include exudative pharyngitis, hepatosplenomegaly, and painful generalized lymphadenopathy. Laboratory findings show an increased WBC count, normal hemoglobin concentration, and normal platelet count. The photograph shows one of many similar WBCs in the peripheral blood of the patient. Which of the following laboratory studies would be most useful in confirming the diagnosis in this patient?

A. Bone marrow aspiration B. Heterophil antibody test

C. Lymph node biopsy D. Serum antibody screening E. Serum transaminase tests


Option B (Heterophil antibody test) is correct. This patient has infectious mononucleosis, which is caused by the Epstein-Barr virus (EBV). Infectious mononucleosis is usually transmitted by saliva (kissing disease). The photograph shows atypical lymphocytes with dark chromatin and abundant blue-gray cytoplasm. In mononucleosis, these are antigenically stimulated T cells that are responding to B cells infected by the virus. Recall that EBV has CD21 receptors on B cells and causes them to produce plasma cells that generate IgM heterophile antibodies that are used to screen for the infection (87% sensitivity). The heterophil antibodies that are specific for mononucleosis are IgM antibodies against horse, sheep, or bovine RBCs. Antibodies that are also useful diagnosing mono, particularly if the heterophile antibody test is negative, are anti-viral capsid antigen antibodies and antibodies against early antigen, both of which have >90% sensitivity and specificity. Hepatosplenomegaly and painful generalized lymphadenopathy are invariably present in mononucleosis. Option A (Bone marrow aspiration) is incorrect. Bone marrow aspiration biopsies are most often performed to rule out leukemias or primary disorders of the bone marrow (e.g., aplastic anemia, myelofibrosis). Option C (Lymph node biopsy) is incorrect. A lymph node biopsy is usually performed when a neoplastic process (e.g., malignant lymphoma, metastatic disease) is suspected. A neoplastic process produces painless enlargement of lymph nodes, which is not present in this case. This patient's painful lymphadenopathy indicates a benign, reactive process. Option D (Serum antibody screening) is incorrect. Standard serum antibody screening (indirect Coombs test) detects antibodies directed against Rh and other antigens that occur normally on the surface of human RBCs. It does not detect the antibodies that are present in infectious mononucleosis. Option E (Serum transaminase tests) is incorrect. Tests for serum transaminases (e.g., alanine aminotransferase) are useful in the diagnosis of hepatitis, which is invariably present in patients with infectious mononucleosis. Thus, markedly increased levels of serum transaminases would be expected in this patient, but they would not help diagnose the cause of the hepatitis. The hepatitis in mononucleosis is self-limited and does not progress into chronic hepatitis.

A 62-year-old man is taking NSAIDs for treatment of chronic low back pain due to osteoarthritis. Which of the following sets of laboratory test results is most likely to be reported? Platelet countBleeding timePT aPTT A.Decreased Prolonged Normal Normal B. Normal Normal Normal Prolonged C. Normal Normal ProlongedProlonged D.Normal Prolonged Normal Normal E. Normal Prolonged Normal Prolonged PT, prothrombin time; aPTT, activated partial thromboplastin time.

A. platelet count decreased, bleeding time prolonged, PT normal, aPTT normal B. platelet count normal, bleeding time normal, PT normal, aPTT prolonged C. platelet count normal, bleeding time normal, PT prolonged, aPTT prolonged D. platelet count normal, bleeding time prolonged, PT normal, aPTT normal E. platelet count normal, bleeding time prolonged, PT normal, aPTT prolonged

Option D (platelet count normal, bleeding time prolonged, PT normal, aPTT normal) is correct. A prolonged bleeding time associated with a normal platelet count, normal PT, and normal aPTT indicates a defect in platelet function (adhesion and/or aggregation). The most common cause of a prolonged bleeding time is aspirin or other NSAIDs. Option A (platelet count decreased, bleeding time prolonged, PT normal, aPTT normal) is incorrect. A decreased platelet count (thrombocytopenia) prolongs the bleeding time, because the end of the bleeding time is marked by the formation of a temporary platelet plug. Signs of thrombocytopenia include petechiae and ecchymoses. Both PT and aPTT are normal in thrombocytopenia, because coagulation factors are normal. Option B (platelet count normal, bleeding time normal, PT normal, aPTT prolonged) is incorrect. A normal platelet count, normal bleeding time, normal PT, and a prolonged aPTT indicate a factor deficiency in the intrinsic coagulation system (e.g., factor VIII deficiency). A normal PT indicates that there are no coagulation factor deficiencies in the final common pathway. Aspirin and other NSAIDs do not produce coagulation factor deficiencies; therefore, the PT and aPTT are normal. Option C (platelet count normal, bleeding time normal, PT prolonged, aPTT prolonged) is incorrect. Prolongation of both the PT and PTT with a normal platelet count and bleeding time is most often due to warfarin and/or heparin therapy, which produce multiple coagulation factor deficiencies. Warfarin inhibits epoxide reductase, which normally converts inactive vitamin K to active vitamin K in the liver. Lack of active vitamin K renders vitamin Kdependent coagulation factors, such as prothrombin, factor VII, factor IX, and factor X, nonfunctional. Heparin enhances antithrombin III, which neutralizes factors XII, XI, IX, X, and II. The PT evaluates the activity of coagulation factors in the extrinsic system (factor VII) to the formation of a fibrin clot in the final common pathway (factor X, factor V, prothrombin, fibrinogen). The aPTT evaluates the activity of the coagulation factors in the intrinsic system (factor XII, factor XI, factor IX, factor VIII) to the formation of a fibrin clot. The PT and aPTT are prolonged in both warfarin and heparin therapy, because both produce deficiencies of coagulation factors in the final common pathway. Option E (platelet count normal, bleeding time prolonged, PT normal, aPTT prolonged) is incorrect. The most common cause of a prolonged bleeding time and prolonged aPTT is von Willebrands disease, which is an autosomal dominant disease with absence of the von Willebrand factor (loss of platelet adhesion) and decrease in factor VIII:coagulant (prolonged aPTT).

A 45-year-old woman with systemic lupus erythematosus (SLE) complains of fatigue and yellow discoloration of her skin. Her hemoglobin concentration is 5 g/dL, the mean corpuscular volume is normal, and a direct Coombs test is positive. The photograph shows a peripheral blood smear stained with new methylene blue (supravital stain). The arrow points to one of many RBCs with cytoplasmic inclusions. Which of the following best characterizes the clinical significance of these cells?

A. Effective erythropoiesis B. Extramedullary hematopoiesis (EMH) C. Hypocellular bone marrow D. Intravascular hemolysis E. Splenic dysfunction

Option A (Effective erythropoiesis) is correct. The patient's clinical and laboratory findings (fatigue, jaundice, low hemoglobin concentration, positive Coombs test) are compatible with a diagnosis of an immune hemolytic anemia secondary to SLE. In this type of immune hemolytic anemia, the patients RBCs are coated by IgG with or without C3b and are being phagocytosed by splenic macrophages that have receptors for IgG and C3b (extravascular hemolysis). In the macrophages, the end-product of degradation of hemoglobin is unconjugated bilirubin, which enters the blood producing jaundice. A direct Coombs test detects the presence of IgG and/or C3b on the surface of the RBCs. Most immune hemolytic anemias are caused by SLE. The arrow points to one of many reticulocytes, which are young RBCs that retain threadlike RNA filaments to synthesize hemoglobin. In hemolytic anemias, erythropoietin stimulates RBC hyperplasia in the bone marrow to replace those RBCs that are being hemolyzed. This effective erythropoiesis is indicated by the presence of increased numbers of reticulocytes in the peripheral blood and is a good sign that the bone marrow is responding appropriately to an anemia. A lack of an appropriate reticulocyte response to an anemia is a sign of ineffective erythropoiesis.

Option B (Extramedullary hematopoiesis (EMH)) is incorrect. EMH is the synthesis of blood cells outside the bone marrow, usually in the spleen and liver. Evidence of EMH is the presence of mature and immature WBCs and nucleated RBCs, which are not present in the peripheral blood of this patient. EMH usually occurs in patients who have fibrosis of the bone marrow (myelofibrosis) or have such a severe anemia that EMH is required (e.g., sickle cell anemia; thalassemia major). Option C (Hypocellular bone marrow) is incorrect. An increased reticulocyte count usually indicates increased RBC synthesis (RBC hyperplasia) due to the release of erythropoietin in response to blood loss or a hemolytic anemia, or as a response of the bone marrow to treatment of iron deficiency with iron. The bone marrow in these conditions is hypercellular rather than hypocellular. A hypocellular marrow has a decrease in the reticulocyte count, indicating ineffective erythropoiesis. Option D (Intravascular hemolysis) is incorrect. Hemolytic anemias are usually either extravascular (macrophage removal of RBCs) or intravascular (destruction of RBCs within the vascular compartment). In either type of hemolytic anemia, there is a reticulocyte response to the anemia; therefore, the presence of reticulocytes does not distinguish the type of hemolytic anemia that is present. Option E (Splenic dysfunction) is incorrect. Splenic dysfunction (e.g., sickle cell disease) is frequently associated with reticulocytosis, because reticulocytes are normally converted to mature RBCs by splenic macrophages. If splenic macrophages cannot perform this function, then reticulocytes may increase in the peripheral blood. However, splenic dysfunction is marked by other peripheral blood findings, the most important of which is the presence of Howell Jolly bodies (RBCs with nuclear remnants). Howell Jolly bodies are not present in the peripheral blood of this patient; therefore, it can be assumed that the reticulocytosis in this patient is a sign of effective erythropoiesis and not splenic dysfunction.

A 65-year-old man with a history of coronary artery disease complains of severe substernal chest pain for the past 24 hours. He states that the pain also radiates down the left arm. A blood sample is drawn for electrolytes and serum creatine kinase (CK) isoenzyme MB. Owing to technical difficulties in collecting the blood, the sample is visibly hemolyzed. The serum potassium and CK isoenzyme MB are both increased. An ECG shows a Q wave in the anterior leads; however, there are no T-wave abnormalities present. Which of the following correctly describes the test results? Serum potassiumSerum CK-MB A.False negative True positive B. False positive False positive C. False positive True positive D.True positive False positive E. True positive True positive

A. false negative, true positive B. false positive, false positive

C. false positive, true positive D. true positive, false positive E. true positive, true positive

Option C (false positive, true positive) is correct. A true positive (TP) test result is a positive test result in a person with disease, while a false positive (FP) test result is a positive test result in a person without disease. The patient has chest pain for 12 hours with radiation of the pain down the arm and into the jaw and a Q wave in an ECG. These changes are consistent with an acute myocardial infarction (AMI). A hemolyzed blood sample falsely increases the serum potassium (pseudohyperkalemia), because potassium is the major intracellular cation. Further confirmation that it is a FP test result is that the ECG does not show peaked T waves, which is a sign of pathologic hyperkalemia. The CK-MB is a true positive test result, because it begins increasing approximately 6 to 8 hours after an AMI and peaks in 24 hours. It is not falsely increased by hemolysis. Option A (false negative, true positive) is incorrect. A false negative (FN) test result is a negative test result in a person with disease. The serum potassium is a FP test result (not a FN) and the serum CK-MB is correctly classified as a TP test result. Option B (false positive, false positive) is incorrect. The serum potassium is correctly classified as a FP test result; however, the serum CK-MB is a TP test result (not a FP). Option D (true positive, false positive) is incorrect. The serum potassium is a FP test result (not a TP), and the serum CK-MB is a TP test result (not a FP). Option E (true positive, true positive) is incorrect. The serum potassium is a FP test result (not a TP), and the serum CK-MB is correctly classified as a TP test result.

A sexually active 19-year-old woman complains of increased frequency and a burning sensation on urination. Pelvic examination shows inflammation of the exocervix and an exudate in the cervical os. Urinalysis shows numerous neutrophils but no bacteria. A cervical Pap smear shows numerous lymphocytes and metaplastic squamous cells with vacuoles in the cytoplasm containing an inclusion. Which of the following pathogens is the causal agent?

A. Candida albicans B. Chlamydia trachomatis C. Human papillomavirus (HPV) D. Neisseria gonorrhoeae

E. Trichomonas vaginalis

Option B (Chlamydia trachomatis) is correct. The patient has follicular cervicitis due to C. trachomatis. Two distinct forms of the organism that develop in vacuoles within metaplastic squamous cells are the elementary body (metabolically inert but infective) and the reticulate body (metabolically active but not infective). Binary fission of the reticulate bodies in the vacuoles of the infected cell results in the production of numerous elementary bodies. Urinalysis findings suggest acute urethral syndrome, which is also due to C. trachomatis. The treatment is a 1-gm oral dose of azithromycin. Option A (Candida albicans) is incorrect. Candida causes vaginal mucosal inflammation with a white discharge containing yeasts and pseudohyphae. Irritation of the urethra can cause dysuria. Option C (Human papillomavirus (HPV)) is incorrect. HPV produces koilocytotic atypia in squamous cells, which is characterized by a halo surrounding a pyknotic (dense) nucleus. These types of cells are not described in the cervical Pap smear. Option D (Neisseria gonorrhoeae) is incorrect. The gram-negative diplococci are not visible with Pap stains. Furthermore, the pathogen does not produce vacuoles in metaplastic squamous cells. Option E (Trichomonas vaginalis) is incorrect. Trichomonas causes cervicitis and urethritis. The pear-shaped organisms have flagella.

A 36-year-old man has a history of chronic liver disease and a movement disorder. The photograph shows an abnormality in the cornea of the eye. Which of the following laboratory findings would most likely be reported?

A. Decreased serum ceruloplasmin B. Increased percent iron saturation

C. Increased serum iron D. Increased total serum copper E. Normal serum prothrombin time

Option A (Decreased serum ceruloplasmin) is correct. The patient has Wilsons disease. A Kayser-Fleischer ring (rusty-colored pigment around the perimeter of the cornea), chronic liver disease, and a movement disorder are features of Wilsons disease, an autosomal recessive disease. Ceruloplasmin is an enzyme that is synthesized in the liver. It contains 6 atoms of copper in its structure. Ceruloplasmin is secreted into the plasma where it represents 90% to 95% of the total serum copper concentration. The remaining 5% to 10% of copper is free copper that is loosely bound to albumin. Ceruloplasmin is eventually taken up and degraded by the liver. The copper that was bound to ceruloplasmin is excreted into the bile. The gene defect in Wilsons disease affects a copper transport system that produces a dual defectdecreased incorporation of copper into ceruloplasmin in the liver and decreased excretion of copper into bile. Accumulation of copper in the liver increases the formation of free radicals causing damage to hepatocytes. Liver disease progresses from acute hepatitis to cirrhosis. In a few years, unbound copper is released from the liver into the circulation (increased in blood and urine) where it damages the brain, kidneys, cornea, and other tissues. Excess copper deposits in Descemets membrane of the eye and in the basal ganglia, particularly the putamen, result in parkinsonism, or choreiform movements, in some cases. Option B (Increased percent iron saturation) is incorrect. Movement disorders and deposition of iron in the cornea are not associated with disorders of iron metabolism (e.g., hemochromatosis). Hemochromatosis (not Wilsons disease) is associated with excess iron reabsorption from the gastrointestinal tract causing an increase in serum iron, decrease in total iron-binding capacity, and increase in percent iron saturation of transferrin, the binding protein for iron. Option C (Increased serum iron) is incorrect. Hemochromatosis (not Wilsons disease) is associated with an increase in serum iron. Option D (Increased total serum copper) is incorrect. In Wilsons disease, the total serum copper level is decreased, because ceruloplasmin is decreased. There is an increase in free copper in the serum and urine. Option E (Normal serum prothrombin time) is incorrect. The patient has chronic liver disease (chronic hepatitis or cirrhosis). Therefore, the prothrombin time is most likely increased due to decreased synthesis of coagulation factors in the liver.

A 48-year-old man with postnecrotic cirrhosis due to hepatitis B virus develops fever and diffuse abdominal pain. Abdominal examination reveals rebound tenderness and absent bowel sounds. A peritoneal tap shows markedly increased neutrophils and an increase in total protein. Many of the neutrophils contain phagocytosed bacteria. A Gram stain would most likely show which of the following types of bacteria?

A. Gram-negative diplococci B. Gram-negative rods C. Gram-positive cocci D. Gram-positive diplococci E. Gram-positive rods
Option B (Gram-negative rods) is correct. The patient has developed spontaneous bacterial peritonitis (SBP; diffuse abdominal pain, rebound tenderness, absent bowel sounds), a complication of ascites that is associated with nephrotic syndrome or cirrhosis (this case) in 10% to 30% of cases. The most common pathogen in SBP in cirrhosis is Escherichia coli, a Gram-negative rod, while in nephrotic syndrome, SBP is caused by Streptococcus pneumoniae. Treatment is cefotaxime. Peritoneal fluid protein concentration in SBP > 2.5 g/dL and a WBC count > 300 cells/mm3 with neutrophils > 25% of the total count. Option A (Gram-negative diplococci) is incorrect. Neisseria gonorrhoeae and meningitidis are gram-negative diplococci. They do not cause SBP. Option C (Gram-positive cocci) is incorrect. Gram-positive cocci include pathogens like Staphylococcus aureus and Streptococcus pyogenes. Neither one of these pathogens is a cause of SBP. Option D (Gram-positive diplococci) is incorrect. S. pneumoniae is the most common cause of SBP in nephrotic syndrome, not cirrhosis. Option E (Gram-positive rods) is incorrect. Grampositive rods include Bacillus species and Listeria monocytogenes, neither one of which have been implicated in SBP.

A 35-year-old man, who is positive for HIV, develops nonpruritic lesions (see photograph) that spread when he scratches them. Which of the following is the most likely diagnosis?

A. Basal cell carcinoma

B. Herpes simplex virus type 1 C. Keratoacanthoma D. Molluscum contagiosum E. Varicella


Option D (Molluscum contagiosum) is correct. The patient has molluscum contagiosum, which is characterized by multiple, small, domed, shiny lesions with central areas of umbilication. A granular material is present in the area of umbilication and contains the cells infected by the causal poxvirus (molluscum bodies). The virus is often spread by selfinoculation. The lesions may occur in the early symptomatic phase of HIV infections when the CD4 helper T-cell count is 200 to 500 cells/mm3. Option A (Basal cell carcinoma) is incorrect. Basal cell carcinomas are malignant tumors that develop on areas of the body that are often exposed to the sun. The lesions are dome-shaped and have telangiectasias on their walls. Central ulceration is a characteristic finding. Basal cell carcinomas are not commonly associated with HIV and do not spread by self-inoculation. Option B (Herpes simplex virus type 1) is incorrect. Herpes simplex virus type 1 causes groupings of vesicles and pustules (not crateriform lesions with umbilication). Option C (Keratoacanthoma) is incorrect. A keratoacanthoma is a rapidly growing crateriform benign tumor with a central keratin plug. It develops in areas of increased sun exposure. Option E (Varicella) is incorrect. The varicella-zoster virus is the causal agent of chickenpox, which appears as macules, vesicles, and pustules on the skin (not crateriform lesions with umbilication).

A 38-year-old Native American woman develops explosive, watery diarrhea and abdominal distention after eating dairy products. A fecal smear for leukocytes is negative. Which of the following is the most likely cause of the diarrhea?

A. Autoimmune destruction of intestinal villi B. Enterocolitis due to microbial pathogens C. Intraluminal osmotically active solutes D. Toxin activation of adenylate cyclase E. Toxin activation of guanylate cyclase

Option C (Intraluminal osmotically active solutes) is correct. The patient has a deficiency of lactase, which is a disaccharidase that hydrolyzes lactose in dairy products into glucose and galactose. Anaerobes in the colon degrade lactose into lactic acid and hydrogen gas. The gas causes distention of the bowel and explosive diarrhea. Undigested lactose is osmotically active and causes the movement of a hypotonic salt solution from the bowel mucosa into the bowel lumen. The diarrhea is noninflammatory; therefore, the fecal smear for leukocytes is negative. Lactase deficiency is a common genetic defect in Native Americans, Asians, and blacks. Option A (Autoimmune destruction of intestinal villi) is incorrect. In celiac disease there is autoimmune destruction of intestinal villi by antigliadin, endomysial, and tissue transglutaminase antibodies due to reaction against the gliadin fraction in gluten in wheat products. Loss of the villous surface causes malabsorption of fat, proteins, and carbohydrates. The stool is greasy, not watery. Option B (Enterocolitis due to microbial pathogens) is incorrect. Invasive diarrhea (enterocolitis) caused by microbial pathogens (e.g., Campylobacter jejuni) produces a low-volume diarrhea with blood and neutrophils in the stool. The fecal smear for leukocytes is negative in this patient. Option D (Toxin activation of adenylate cyclase) is incorrect. Toxins that stimulate adenylate cyclase (produced by Vibrio cholerae and some strains of enterotoxigenic Escherichia coli) cause a secretory diarrhea (loss of isotonic fluid) by stimulating ionic pumps in the small intestine. The diarrhea is noninflammatory, and the fecal smear for leukocytes is negative. Toxin induced secretory diarrhea is excluded, because the diarrhea only follows ingestion of dairy products. Option E (Toxin activation of guanylate cyclase) is incorrect. Toxins that stimulate guanylate cyclase (toxin produced by some strains of enterotoxigenic E. coli) cause a secretory diarrhea (loss of isotonic fluid) by stimulating ionic pumps in the small intestine. The diarrhea is noninflammatory, and the fecal smear for leukocytes is negative. Toxin induced secretory diarrhea is excluded, because the diarrhea only follows ingestion of dairy products.

The photograph shows the face of a 40-year-old woman who complains of muscle pain, particularly in the shoulders. Purple papules are also noted over her knuckles and proximal interphalangeal joints. What is the most likely diagnosis?

A. CREST syndrome B. Dermatomyositis C. Mixed connective tissue disease (MCTD) D. Systemic lupus erythematosus (SLE) E. Systemic sclerosis
Option B (Dermatomyositis) is correct. The patient has dermatomyositis, which occurs predominantly in women 40 to 60 years of age. Muscle pain and atrophy commonly occur, with the shoulders being particularly involved. Heliotrope eyelids, or raccoon eyes, is very characteristic in this disease. Note the swelling and red-mauve discoloration below both eyes in this patient. The patient also has Gottrons patches, which are purple-colored papules over the knuckles and proximal interphalangeal joints (see Fig. 3-2E in Rapid Review Pathology, 3rd edition). The serum ANA is positive in <30% of cases. There is an increase in serum creatine kinase, due to antibody destruction of muscle. Muscle biopsy shows muscle damage with a lymphocytic infiltrate. Option A (CREST syndrome) is incorrect. CREST syndrome is a limited variant of systemic sclerosis. Clinical findings are correlated with the name of the syndrome: Ccalcification, centromere antibody; RRaynauds phenomenon; E esophageal dysmotility; Ssclerodactyly (i.e., tapered, clawlike fingers); and Ttelangiectasis (i.e., multiple punctate blood vessel dilations). Note that none of these findings is in the patient. Option C (Mixed connective tissue disease (MCTD)) is incorrect. MCTD has signs and symptoms similar to systemic lupus erythematosus, systemic sclerosis, and polymyositis (muscle but no skin involvement). Antiribonucleoprotein antibodies are positive in almost 100% of cases. Heliotrope eyes and Gottrons patches are not present in MCTD. Option D (Systemic lupus erythematosus (SLE)) is incorrect. SLE is a connective tissue disease that mainly affects the blood, joints, skin, and kidneys. Heliotrope eyes and Gottrons patches are not features of the disease. Option E (Systemic sclerosis) is incorrect. Systemic sclerosis is characterized by excessive production of collagen that primarily targets the skin (scleroderma), gastrointestinal tract, lungs, and kidneys. Heliotrope eyes and Gottrons patches are not characteristics of the disease.

A 5-year-old child has an enlarged liver and kidneys as well as fasting hypoglycemia. When glucagon is administered, the serum glucose does not increase. The patient most likely has a deficiency of which of the following enzymes?

A. Aldolase B B. Fructokinase

C. Galactokinase D. Galactose 1-phosphate uridyltransferase (GALT) E. Glucose-6-phosphatase


Option E (Glucose-6-phosphatase) is correct. The patient has von Gierkes disease, which is an autosomal recessive glycogen storage disease characterized by a deficiency of glucose-6-phosphatase, a gluconeogenic enzyme that converts glucose 6-phosphate to glucose in the fasting state. Deficiency of glucose-6-phosphatase causes fasting hypoglycemia that does not respond to stimulators of gluconeogenesis (e.g., glucagon), because it is the last step in gluconeogenesis. There is an accumulation of glucose 6-phosphate proximal to the enzyme block. The excess glucose 6-phosphate is used to synthesize normal glycogen in sites of gluconeogenesis, including the liver and kidneys causing hepatorenomegaly. Option A (Aldolase B) is incorrect. Aldolase B catalyzes a reaction that converts fructose 1-phosphate to glyceraldehyde 3-phosphate and dihydroxyacetone phosphate, which are substrates for gluconeogenesis. Deficiency of aldolase B is associated with hereditary fructose intolerance and causes liver disease and fasting hypoglycemia. Glucagon stimulation increases serum glucose, because there is no enzyme block in gluconeogenesis. Option B (Fructokinase) is incorrect. Fructokinase converts fructose to fructose 1-phosphate. Deficiency of fructokinase is characterized by an accumulation of fructose in the blood and urine. There is no liver or kidney enlargement and no fasting hypoglycemia. Option C (Galactokinase) is incorrect. Deficiency of galactokinase is an autosomal recessive disorder causing an accumulation of galactose in the blood and urine and increased conversion of galactose to galactitol (sugar alcohol) in the lens, causing cataracts. There is no liver or kidney enlargement and no fasting hypoglycemia. Option D (Galactose 1-phosphate uridyltransferase (GALT)) is incorrect. GALT converts galactose 1-phosphate to glucose 1-phosphate, which is used as a substrate for gluconeogenesis (deficiency causes fasting hypoglycemia). Deficiency of GALT is present in galactosemia, an autosomal recessive disease that causes an accumulation of galactose 1-phosphate, which is toxic to the brain (mental retardation) and liver (cirrhosis). Glucagon stimulation increases serum glucose, because there is no enzyme block in gluconeogenesis.

A 28-year-old woman with Crohns disease has a macrocytic anemia associated with pancytopenia. A peripheral blood smear shows hypersegmented neutrophils. Which of the following is a likely complication?

A. Achlorhydria B. Chronic gastritis C. Hypergastrinemia D. Neurologic dysfunction

E. Stomach cancer
Option D (Neurologic dysfunction) is correct. Crohns disease is an inflammatory condition that involves the terminal ileum in approximately 80% of cases; therefore, vitamin B12 deficiency may occur, because the terminal ileum is the primary site for reabsorption of the intrinsic factorvitamin B12 complex. Vitamin B12 deficiency causes a macrocytic anemia (delayed DNA maturation), pancytopenia (anemia, neutropenia, thrombocytopenia), and the formation of hypersegmented neutrophils in the peripheral blood. Vitamin B12 is involved in propionate metabolism (odd-chain fatty acids). Propionyl CoA is converted to methylmalonyl CoA, which is then converted to succinyl CoA, a reaction that uses vitamin B12 as a cofactor. Therefore, vitamin B12 deficiency causes an increase in both methylmalonyl CoA and propionyl CoA. Propionyl CoA replaces acetyl CoA in neuronal membranes resulting in demyelination of the posterior columns (loss of vibratory sensation and proprioception) and lateral corticospinal tract (upper motor neuron disease). Option A (Achlorhydria) is incorrect. Pernicious anemia (not Crohns disease) is associated with autoimmune destruction of the parietal cells in the body and fundus of the stomach (chronic gastritis), causing deficiency of intrinsic factor (vitamin B12 deficiency) and acid (achlorhydria). Chronic gastritis may progress to stomach cancer. Option B (Chronic gastritis) is incorrect. Pernicious anemia (not Crohns disease) is associated with autoimmune destruction of the parietal cells in the body and fundus of the stomach (chronic gastritis), causing deficiency of intrinsic factor (vitamin B12 deficiency) and chronic atrophic gastritis. Option C (Hypergastrinemia) is incorrect. Pernicious anemia (not Crohns disease) is associated with autoimmune destruction of the parietal cells in the body and fundus of the stomach (chronic gastritis), causing deficiency of intrinsic factor (vitamin B12 deficiency) and acid (achlorhydria). Achlorhydria causes an increase in gastrin production by G cells in the pylorus and antrum. Option E (Stomach cancer) is incorrect. Pernicious anemia (not Crohns disease) is associated with autoimmune destruction of the parietal cells in the body and fundus of the stomach causing chronic atrophic gastritis, which may progress to stomach cancer.

Which of the following clinical disorders is most compatible with the distribution of affected patients shown in this pedigree?

A. 1-antitrypsin deficiency B. Hemophilia A C. Sickle cell trait D. Type 2 diabetes mellitus E. Vitamin Dresistant rickets
Option E (Vitamin Dresistant rickets) is correct. Vitamin Dresistant rickets is an X-linked dominant disorder that involves a defect in the reabsorption of phosphate from the gastrointestinal tract and kidneys. Decreased phosphate leads to decreased mineralization of bone and rickets (osteomalacia). X-linked dominant disorders are characterized by a dominant allele that causes both male and female carriers of the abnormal allele to express the disease. As with Xlinked recessive disorders, affected males transmit the abnormal allele to all their daughters, as shown in the pedigree. Furthermore, symptomatic carrier females transmit the abnormal allele to 50% of their sons and 50% of their daughters, as is also shown. Option A (1-antitrypsin deficiency) is incorrect. 1-antitrypsin deficiency is an autosomal dominant disorder that involves a defect in either the synthesis of 1-antitrypsin in the liver or its secretion from the liver. Autosomal dominant disorders are characterized by a dominant allele that expresses itself in either the homozygous or the heterozygous state. Only one parent must have the abnormal allele for the disease to be transmitted to the children. A heterozygous parent with the disease transmits the disease to 50% of the children. Option B (Hemophilia A) is incorrect. Hemophilia A is an X-linked recessive disorder characterized by a deficiency of factor VIII. X-linked recessive disorders are expressed in males, whereas females with the abnormal allele are usually asymptomatic carriers. Since females do express disease in the pedigree, hemophilia A is not compatible with this inheritance pattern. Option C (Sickle cell trait) is incorrect. Sickle cell trait is an autosomal recessive disorder in which the patients are asymptomatic heterozygote carriers of the sickle cell allele. There is no anemia and sickle cells are not present in the peripheral blood. Option D (Type 2 diabetes mellitus) is incorrect. Type 2 diabetes mellitus is a disorder with multifactorial (polygenic) inheritance. Multifactorial inheritance involves the additive effect of two or more gene mutations of small effect conditioned by environmental and other nongenetic factors.

A 25-year-old man sees his physician because of recurrent swelling of the face and larynx that results in inspiratory stridor. He states that his father and maternal aunt have a similar problem. Initial laboratory studies show normal concentrations of complement C3 and factor B and a decreased concentration of complement C4. Which of the following studies is most indicated?

A. C1 esterase inhibitor assay B. Chromosome study C. Cutaneous skin testing to rule out anergy D. Quantitative immunoglobulin assay for IgG, IgA, and IgM E. Serum antinuclear antibody test
Option A (C1 esterase inhibitor assay) is correct. The patient has hereditary angioedema, an autosomal dominant disease characterized by a deficiency of C1 esterase inhibitor. The resultant continued activation of C1 decreases the concentration of C2 and C4 in the classic pathway and increases the concentration of their cleavage products (C2a and C4a), both of which have anaphylatoxic activity. Anaphylatoxins directly stimulate the release of histamine from mast cells, resulting in increased vascular permeability and swelling of the face and larynx. Inspiratory stridor is a symptom of upper airway obstruction. Factor B is present in the alternative complement pathway, which explains why it is normal. There is no activation of C3 in hereditary angioedema. Option B (Chromosome study) is incorrect. Chromosome studies are usually ordered when a genetic disorder involving chromosome number (e.g., Down syndrome) or structure (e.g., translocation) is suspected. It would be normal in hereditary angioedema. Option C (Cutaneous skin testing to rule out anergy) is incorrect. Cutaneous skin testing to rule out anergy is ordered when a defect in cellular immunity is suspected (e.g., T-cell deficiency in DiGeorge syndrome). Injection of Candida into the skin does not produce an inflammatory reaction when T cells are deficient. It would be normal in hereditary angioedema. Option D (Quantitative immunoglobulin assay for IgG, IgA, and IgM) is incorrect. Total immunoglobulins are ordered when B-cell immunodeficiency is suspected (e.g., Brutons agammaglobulinemia). It would be normal in hereditary angioedema. Option E (Serum antinuclear antibody test) is incorrect. A serum antinuclear antibody test is ordered when autoimmune disease is suspected (e.g., systemic lupus erythematosus). It would be normal in hereditary angioedema.

An afebrile 48-year-old woman with a 20-year history of chronic headaches suddenly develops right ipsilateral colicky pain with radiation into the right groin. Urinalysis shows RBCs, WBCs, and mild proteinuria. There are no bacteria, casts, or crystals. Urine-specific gravity is now 1.01, and it is unchanged the following morning. An intravenous pyelogram shows a ring deformity in the right kidney. What is the most likely diagnosis?

A. Acute drug-induced tubulointerstitial nephritis B. Chronic glomerulonephritis

C. Chronic pyelonephritis D. Renal papillary necrosis E. Ureteral stone


Option D (Renal papillary necrosis) is correct. The patient has renal papillary necrosis, which is a chronic drug-induced tubulointerstitial nephritis (analgesic nephropathy) caused by the cumulative effect of ingestion of aspirin and acetaminophen over many years. Aspirin inhibits renal production of prostaglandin E2, which is a vasodilator of the afferent arteriole, leaving angiotensin II, a vasoconstrictor of the efferent arteriole, in control of the blood flow into the peritubular capillaries. Free radicals from acetaminophen injure renal tubular cells in the renal papillae. The combination of ischemia and damage from the free radicals causes renal papillary necrosis. The resultant sloughing of the renal papillae into the urine cause the patients right-sided colicky pain (see Fig. 19-11 in Rapid Review Pathology, 3rd edition). The ring deformity in the right kidney shown on the intravenous pyelogram indicates absence of the renal papillae. Due to the loss of collecting tubules in the renal papillae, the ability to concentrate and dilute urine is lost and the specific gravity remains fixed at 1.01. Option A (Acute drug-induced tubulointerstitial nephritis) is incorrect. Acute drug-induced tubulointerstitial disease is characterized by a sudden onset of oliguria (renal failure), fever, and a rash shortly after taking a drug (e.g., synthetic penicillin). Other findings include eosinophilia, eosinophiluria (eosinophils in the urine), hematuria, pyuria (WBCs in the urine), and WBC casts. Withdrawal of the drug reverses these findings. Renal papillary necrosis is not a complication of the disease. Option B (Chronic glomerulonephritis) is incorrect. Chronic glomerulonephritis usually arises from preexisting acute glomerulonephritis. Glomerular sclerosis, tubular atrophy, and renal failure eventually occur. Renal papillary necrosis is not a complication of chronic glomerulonephritis. Option C (Chronic pyelonephritis) is incorrect. Chronic pyelonephritis is a complication of vesicoureteral reflux (reflux of urine from the bladder into the ureter) or postrenal obstruction caused by prostate hyperplasia or a ureteral stone. An intravenous pyelogram shows blunting of the renal calyces underlying cortical scars. Renal papillary necrosis is not a complication of chronic pyelonephritis. Option E (Ureteral stone) is incorrect. A ureteral stone presents with sudden onset colicky pain with radiation to the groin; however, the intravenous pyelogram in this patient showed a ring deformity due to loss of the renal papillae, not a ureteral stone.

A 55-year-old woman complains of headaches. She is a non-smoker. Which of the following is the most likely primary site of the cancer?

A. Brain B. Breast C. Colon D. Kidney E. Lung

Option B (Breast) is correct. The MRI shows multiple nodular enhancing masses of varying sizes. Since the patient is a non-smoker, breast cancer would be the most likely primary site. Option A (Brain) is incorrect. Glioblastoma multiforme is the most common primary cancer of the brain. It usually develops in the frontal lobes and commonly extends across the corpus callosum to the opposite lobe. Option C (Colon) is incorrect. Colorectal cancer can metastasize to the brain; however, mortality due to breast cancer is much greater than mortality due to colorectal cancer. Option D (Kidney) is incorrect. Renal cell carcinoma commonly metastasizes to the brain. However, it is most commonly associated with smoking and the patient is a non-smoker. Option E (Lung) is incorrect. The patient is a non-smoker; therefore, lung cancer is an unlikely cause of metastasis to the brain. If the patient were a smoker, then lung cancer would have been the correct answer.

The diet of this 48-year-old woman consists primarily of cola drinks and potato chips. Her tongue is sore and her gums bleed after brushing her teeth. The bones in her legs hurt when walking. Physical findings include pinpoint hemorrhages around the hair follicles, corkscrew-appearing hairs, and scattered ecchymoses. Which of the following metabolic processes is defective?

A. Carboxylation of coagulation factors B. Collagen synthesis C. Elastic tissue synthesis D. Platelet aggregation E. Platelet production
Option B (Collagen synthesis) is correct. Defective collagen synthesis is associated with scurvy, which is caused by a dietary deficiency of vitamin C (ascorbic acid). The photograph shows inflammation and bleeding of the gingival papillae. Other characteristic findings include painful glossitis (sore tongue) and the presence of corkscrew hairs and perifollicular hemorrhage. Vitamin C is important in the hydroxylation of proline and lysine. Normally, hydroxylated lysyl residues become the binding site for the cross-links that bind -chains together to increase the tensile strength of collagen. In scurvy, deficiency of vitamin C leads to diminished tensile strength of collagen, causing venules and capillaries to rupture (causing ecchymoses and pinpoint hemorrhages) and bleed into joints and subperiosteal tissue (causing pain with movement). Option A (Carboxylation of coagulation factors) is incorrect. A defect in the carboxylation of coagulation factors is associated with vitamin K deficiency, which may produce bleeding diatheses (ecchymoses, gingival bleeding) but not glossitis and gum inflammation. The vitamin Kdependent coagulation factors are prothrombin, factor VII, factor IX, factor X, and protein C and S. Most vitamin K is synthesized by colonic bacteria; therefore, a poor diet is not likely to produce a deficiency. Option C (Elastic tissue synthesis) is incorrect. Marfan syndrome is characterized by defects in elastic tissue synthesis and in the synthesis of fibrillin, a major component of elastic fibers. Marfan syndrome is not associated with bleeding diatheses (ecchymoses, pinpoint hemorrhages, gingival bleeding). Elastic tissue defects include aortic dissection, dislocated lenses, and hypermobile joints. Option D (Platelet aggregation) is incorrect. A defect in platelet aggregation (e.g., taking aspirin) may produce petechiae (pinpoint hemorrhages) and ecchymoses but not glossitis and gum inflammation. Option E (Platelet production) is incorrect. A defect in platelet production causes thrombocytopenia and may produce petechiae (pinpoint hemorrhages) and ecchymoses, but not glossitis and gum inflammation.

A 75-year-old man was admitted to the hospital with severe substernal chest pain that radiated into the left arm and jaw. On day 5 of hospitalization, he developed acute mitral valve regurgitation and died. The photograph shows a transverse section of the heart at autopsy with the anterior portion of the heart at the top. Which of the following coronary arteries was most likely responsible for the gross changes in the heart?

A. Left anterior descending coronary artery (LAD) B. Left circumflex coronary artery C. Left main stem coronary artery D. Right coronary artery
Option D (Right coronary artery) is correct. The patient had an acute myocardial infarction secondary to a coronary artery thrombosis. An infarct is the gross manifestation of coagulation necrosis that is present in underlying tissue. The gross specimen shows an extensive pale yellow infarct in the left ventricle, which involves the posterior wall and posteromedial papillary muscle (the round structure projecting into the ventricular lumen) of the mitral valve. The posteromedial papillary muscle most likely ruptured producing the acute mitral valve regurgitation and death. This area of the left ventricle is in the distribution of the right coronary artery. The infarct is pale because of the increased density of myocardial tissue, which prevents the infiltration of blood into surrounding dead tissue from necrotic blood vessels. Option A (Left anterior descending coronary artery (LAD)) is incorrect. The LAD supplies the anterior portion of the left ventricle and anterior two-thirds of the interventricular septum. Option B (Left circumflex coronary artery) is incorrect. In general, the left coronary artery supplies the anterior portion of the heart, not the posterior portion of the heart. Option C (Left main stem coronary artery) is incorrect. The left circumflex coronary artery supplies the lateral portion of the left ventricle.

A 50-year-old man with ischemic heart disease has signs of both left- and right-sided heart failure. Which of the following is characteristic of both types of heart failure?

A. Bibasilar inspiratory crackles B. Decreased cardiac output C. Dependent pitting edema D. Jugular neck vein distention E. Paroxysmal nocturnal dyspnea
Option B (Decreased cardiac output) is correct. The heart fails when it cannot pump blood delivered to it by the venous system. Therefore, cardiac output is decreased whether the heart failure is left-sided or right-sided. Option A (Bibasilar inspiratory crackles) is incorrect. Bibasilar inspiratory crackles are a sign of left-sided heart failure, a forward heart failure causing a decreased cardiac output and backup of blood in the left ventricle, left atrium, and pulmonary capillaries. Increased pulmonary capillary hydrostatic pressure causes fluid (transudate) to enter the interstitium of the lung and eventually the alveoli (pulmonary edema). Air entering alveoli containing fluid produces inspiratory crackles that are best heard at the base of both lungs. Option C (Dependent pitting edema) is incorrect. Dependent pitting edema is a sign of right-sided heart failure, a backward heart failure causing systemic venous congestion. The increase in hydrostatic pressure in the venous system causes fluid (transudate) to leak into the interstitial space through the venules, leading to dependent pitting edema. Option D (Jugular neck vein distention) is incorrect. Jugular neck vein distention is a sign of right-sided heart failure. In right-sided heart failure, blood builds up in the venous system causing distention of the jugular veins (see Fig. 10-3 in Rapid Review Pathology, 3rd edition). Option E (Paroxysmal nocturnal dyspnea) is incorrect. Paroxysmal nocturnal dyspnea is a sign of left-sided heart failure, which occurs primarily at night when the patient is supine in bed. At this time, gravity does not impede blood flow to the right side of the heart, and fluid from the interstitial space enters the venous system. Excess blood enters the failed left ventricle and backs up into the lungs, causing pulmonary edema and dyspnea, which awakens the patient. Symptoms resolve when the patient stands up and gravity decreases venous return to the right side of the heart.

A 3-year-old girl with an upper respiratory infection lapses into a coma after receiving aspirin for fever. Physical examination shows scleral icterus, papilledema, and hepatomegaly. Serum aminotransferases are increased; alanine aminotransferase (ALT) is higher than aspartate aminotransferase (AST). Total bilirubin is increased. Which of the following histologic findings is most likely present in the liver?

A. Apoptotic bodies B. Lymphocytic infiltrate in the portal triads

C. Mallory bodies D. Microvesicular fatty change E. Periodic acidSchiff-positive granules in hepatocytes


Option D (Microvesicular fatty change) is correct. The patient has Reyes syndrome, which usually develops in children < 4 years of age who receive aspirin (acetylsalicylic acid) to reduce fever associated with a viral respiratory infection or chickenpox. Salicylates damage the mitochondria in the liver, which disrupts the urea cycle and -oxidation of fatty acids. Disruption of the urea cycle causes an increase in serum ammonia, which produces an encephalopathy characterized by cerebral edema, papilledema, and coma. Disruption of -oxidation of fatty acids in the liver causes microvesicular fatty change, in which small globules of fat in the cytoplasm surround a centrally located nucleus. Aminotransferases and total bilirubin are increased (ALT > AST). Option A (Apoptotic bodies) is incorrect. Apoptotic bodies (deeply eosinophilic bodies with or without a pyknotic nucleus) are present in viral hepatitis. The prodrome associated with viral hepatitis does not include signs of an upper respiratory infection. Furthermore, fatty change is not a finding in viral hepatitis. Option B (Lymphocytic infiltrate in the portal triads) is incorrect. A lymphocyte-predominant infiltrate in the portal triads is present in viral hepatitis. Option C (Mallory bodies) is incorrect. Mallory bodies are eosinophilic inclusions in the cytosol of hepatocytes. They are commonly present in patients with alcoholic liver disease. Option E (Periodic acidSchiff-positive granules in hepatocytes) is incorrect. Periodic acidSchiff-positive granules in hepatocytes are present in 1-antitrypsin (AAT) deficiency, an autosomal dominant disorder. The type of AAT deficiency associated with liver disease in children involves defective secretion of AAT by the hepatocyte. This disorder usually presents at birth with jaundice caused by cholestatic hepatitis, which progresses to chronic hepatitis, cirrhosis, and hepatocellular carcinoma.

A 70-year-old man complains of generalized bone pain and fatigue. The CBC exhibits a normocytic anemia with prominent rouleau. The serum blood urea nitrogen (BUN) is 60 mg/dL (718) and serum creatinine is 6 mg/dL (0.61.2). A urinalysis reveals a +1 reagent strip reaction for protein and a +4 sulfosalicylic acid reaction. The results of a bone marrow aspirate are pending. Which type of proteinuria is most likely present?

A. Functional B. Glomerular C. Overflow D. Tubular

Option C (Overflow) is correct. In overflow proteinuria, the protein loss is variable (0.2 to >10g/24 hours). It is a low molecular weight proteinuria in which the amount filtered exceeds the tubular capacity to reabsorb it. The patient most likely has multiple myeloma, a malignant plasma cell disorder associated with Bence-Jones proteinuria (light chains in the urine). He has anemia and renal failure (BUN:creatinine ratio is 10). There is a disparity in the protein readings for the reagent strip and sulfosalicylic acid (SSA). This occurs in multiple myeloma with Bence-Jones proteinuria, because the reagent strip for protein detects albumin and not globulins (e.g., light chains), while the SSA reaction detects albumin and globulins. A bone marrow aspirate confirms the presence of malignant plasma cells, while a serum and urine immunoelectrophoresis determine the immunoglobulin and light chain that is involved. Option A (Functional) is incorrect. A functional proteinuria is a proteinuria that is not associated with an underlying renal disease. Causes include fever, stress, sepsis, and orthostatic proteinuria. The patient has renal failure and proteinuria due to the presence of Bence-Jones protein, which damages the tubular epithelium and blocks tubular lumens with proteinaceous material. Option B (Glomerular) is incorrect. Glomerular proteinuria is associated with a loss of protein ranging from 150 mg/24 hours to >3 g/24 hours. It is subdivided into selective and nonselective proteinuria. Selective proteinuria refers to a loss of albumin and not globulins. It is due to loss of the negative charge in the glomerular basement membrane (e.g., nephrotic syndrome caused by minimal change disease). Nonselective proteinuria refers to the loss of plasma proteins (e.g., albumin and globulins) in urine. It is due to damage of the glomerular basement membrane (e.g., poststreptococcal glomerulonephritis). Glomerular proteinuria is unlikely in this patient, because no abnormal casts are present in the urine (e.g., RBC or fatty casts). Option C (Tubular) is incorrect. Tubular proteinuria is associated with a protein loss < 2 g/24 hours. It is due to a defect in proximal tubule reabsorption of low molecular weight proteins (e.g., 2-microglobulin, amino acids) at normal filtered loads (e.g., heavy metal poisoning). Tubular proteinuria is unlikely in this patient, because Bence-Jones protein damages the collecting tubules rather than the proximal renal tubules.

A test for coronary artery disease is positive in 180 of 200 people with known coronary artery disease and negative in 140 of 200 people who do not have disease. If the test result returns positive, what is the likelihood that coronary artery disease is present?

A. 50% B. 70% C. 75% D. 87.5% E. 90%

Option C (75%) is correct. In this question, a test is performed on people with known coronary artery disease (disease group) and on people who do not have coronary artery disease (control group). People with disease either have a true positive (TP) or a false negative (FN) test result. A TP test result is a positive test in a person with disease. A FN test result is a negative test result in a person with disease. People in a control group, who do not have disease, either have a true negative (TN) or a false positive (FP) test result. A TN test result is a negative test in a person without disease, while a FP test result is a positive test result in a person without disease. The question is asking what is the predictive value (or likelihood) that a positive test result (PV+) is a TP and not a FP. The formula for PV+ is TP/TP + FP. In this question, the PV+ is 180/180 + 60 = 75%. In other words, there is a 75% chance that the person has a TP test result and a 25% chance that it is a FP test result.

Coronary artery disease Control group Total Positive test True positive 180 False positive 60 240 Negative testFalse negative 20 True negative 140160 Total 200 200 400

Option A (50%) is incorrect. The prevalence of coronary artery disease is 50%. Prevalence refers to the number of people with a disease in the population under study, which includes all the people with coronary artery disease and all the people in the control group. The formula for prevalence is TP + FN/TP + FN + TN + FP. The prevalence of coronary artery disease is 180 + 20/180 + 20 + 140 + 60 = 50%. Option B (70%) is incorrect. The specificity of the test is 70%. The specificity of a test refers to the likelihood of having negative test results in people without disease. Since people without disease either have TN or FP test results, the formula for calculating the specificity of a test is TN/TN + FP. The specificity of the test in this question is 140/140 + 60 = 70% Option D (87.5%) is incorrect. The predictive value (or likelihood) that a negative test result (PV-) is a TN and not a FN is equal to 87.5%. The formula for PV- is TN/TN + FN; therefore, 140/140 + 20 = 87.5%. Option E (90%) is incorrect. The sensitivity of the test is 90%. The sensitivity of a test refers to the likelihood of having positive test results in people with disease. Since people with disease either have TP or FN test results, the formula for calculating the sensitivity of a test is TP/TP + FN. The sensitivity of the test is 180/180 + 20 = 90%.

A 32-year-old woman with a 15-year history of cigarette smoking has an abnormal cervical Pap smear. The photograph shows a representative histologic section of a cervical biopsy. Which of the following is the most likely pathogenesis of this lesion?

A. Cigarette smoking B. Herpes simplex virus type 2 (HSV-2) infection C. Human papillomavirus (HPV) infection D. Malignant transformation of a stem cell E. Unopposed estrogen exposure

Option C (Human papillomavirus (HPV) infection) is correct. The biopsy shows squamous cells with pyknotic (condensed chromatin) nuclei surrounded by a clear halo, representing koilocytic atypia due to HPV infection. Both lowrisk types of HPV (e.g., types 6 and 11) and high-risk types of HPV (e.g., types 16 and 18) can initiate these changes in the squamous epithelium, with the latter having the greatest risk for evolution into squamous cancer. Option A (Cigarette smoking) is incorrect. Carcinogens in cigarette smoke can produce cervical cancer. However, the biopsy shows koilocytotic atypia (not cancer) due to HPV. Option B (Herpes simplex virus type 2 (HSV-2) infection) is incorrect. HSV-2 causes eosinophilic intranuclear inclusions in multinucleated squamous cells. Option D (Malignant transformation of a stem cell) is incorrect. There is no evidence of dysplasia or cancer in the biopsy. Option E (Unopposed estrogen exposure) is incorrect. Unopposed estrogen increases the number of normal-appearing superficial squamous cells in a cervical Pap smear. It does not produce koilocytic atypia.

A 63-year-old man with a 10-year history of osteoarthritis of the hips and knees develops urinary retention due to benign prostatic hyperplasia. He takes a nonsteroidal anti-inflammatory drug to relieve pain in his hips and knees. A transurethral resection of the prostate is performed to relieve the obstruction. Shortly after the procedure, there is profuse bleeding from the penis. Coagulation studies show a partial thromboplastin time (activated, aPTT) of 38 seconds, a prothrombin time (PT) of 14 seconds, a bleeding time > 15 minutes, and a platelet count of 350,000/mm3. The d-dimer assay is negative. Which of the following is most likely responsible for the bleeding disorder?

A. Antiphospholipid antibodies B. Circulating anticoagulant C. Coagulation factor deficiency D. Platelet function disorder E. Primary fibrinolytic disorder
Option D (Platelet function disorder) is correct. A qualitative platelet disorder is present due to the patients use of NSAIDs for his chronic osteoarthritis. NSAIDs inhibit platelet cyclooxygenase, which prevents the synthesis of thromboxane A2 (TXA2), a platelet aggregator and vasoconstrictor of small vessels. TXA 2 is mainly responsible for producing a temporary hemostatic plug composed of platelets that stop bleeding in injured small vessels. Hence, blocking the synthesis of TXA2 causes severe bleeding from small vessels that lack temporary platelet plugs. The aPTT evaluates the intrinsic coagulation system, and the PT evaluates the extrinsic coagulation system. They are normal because they do not detect platelet abnormalities but only coagulation factor deficiencies. The bleeding time, which evaluates platelet function up to the formation of the temporary platelet plug, is prolonged because the temporary platelet plug is not formed. The platelet count is a quantitative measurement of platelets, not a means for evaluating platelet function. It is normal in patients taking NSAIDs. Option A (Antiphospholipid antibodies) is incorrect. Antiphospholipid antibodies, such as anticardiolipin antibodies and lupus anticoagulant, are directed against phospholipids bound to plasma proteins. These antibodies are more often associated with thrombosis syndromes (e.g., strokes) than with bleeding disorders. They do not produce qualitative platelet defects. Option B (Circulating anticoagulant) is incorrect. Circulating anticoagulants are antibodies that inhibit specific coagulation factors (e.g., antibodies against factor VIII), thus simulating a coagulation factor deficiency. A normal aPTT and PT exclude a circulating anticoagulant from the differential diagnosis. Option C (Coagulation factor deficiency) is incorrect. In a coagulation factor deficiency, the aPTT and/or the PT should be prolonged. These tests are normal in this patient. Option E (Primary fibrinolytic disorder) is incorrect. Primary fibrinolysis is uncommon; however, it may complicate a radical prostatectomy or transurethral resection because of the increased release of urokinase, which can cleave plasminogen and release plasmin. Plasmin can act upon fibrinogen and produce fibrinogen degradation products (FDPs) that can interfere with platelet aggregation causing a prolonged bleeding time. FDPs can also be measured and would

be present in primary fibrinolysis. Plasmin also degrades coagulation factors causing multiple-factor deficiencies leading to a prolonged PT and aPTT, which is not present in this patient. When plasmin breaks down fibrin clots (e.g., coronary thrombosis), D-dimers are released (cross-linked fibrin monomers). In primary fibrinolysis there are no fibrin clots; therefore, the test is negative. The platelet count is normal in primary fibrinolysis. Hence, the only finding that eliminates primary fibrinolysis as a potential cause of this patients problem is that the PT and aPTT are normal in this patient.

The photograph shows the cut section of the liver removed at autopsy of a 54-year-old man with chronic hepatitis B caused by intravenous drug abuse. A few months before his death, the man developed weight loss, abdominal pain, bloody ascitic fluid, and a rapidly enlarging liver. Which of the following tests would have been most useful in defining the cause of the rapid decline in this patient prior to his death?

A. Antimitochondrial antibody B. -fetoprotein C. Hepatitis B surface antigen D. Prothrombin time E. Serum transaminases

Option B (-fetoprotein) is correct. The patient developed a hepatocellular carcinoma as a complication of postnecrotic cirrhosis secondary to hepatitis B virus. Classic findings include weight loss, abdominal pain, bloody ascitic fluid, and a rapidly enlarging liver. The photograph shows multiple, large, hemorrhagic, and necrotic tumor masses in the liver. Microscopic sections from the liver showed invasion of portal vein and hepatic vein tributaries. -fetoprotein is a tumor marker that is increased in 70% of cases of hepatocellular carcinoma. Option A (Antimitochondrial antibody) is incorrect. Antimitochondrial antibodies are present in primary biliary cirrhosis, an autoimmune disease characterized by granulomatous destruction of the bile ducts in the portal triads.

Option C (Hepatitis B surface antigen) is incorrect. Hepatitis B surface antigen is present in chronic hepatitis B. However, it provides no additional information about the possibility of an underlying hepatocellular carcinoma. Option D (Prothrombin time) is incorrect. The prothrombin time is invariably prolonged in chronic liver disease. However, it does not provide any information concerning the presence of a hepatocellular carcinoma. Option E (Serum transaminases) is incorrect. Serum aspartate aminotransferase is often increased in hepatocellular carcinomas. However, it does not provide any information concerning the presence of a hepatocellular carcinoma.

The photograph shows a skin lesion on a 28-year-old-man from Africa. The lesion is insensitive to pinprick. Which of the following best characterizes this patients disease?

A. Absence of granulomas B. Acid-fast organisms in macrophages C. Highly contagious D. Leonine facies E. Positive lepromin skin test

Option E (Positive lepromin skin test) is correct. The patient has the tuberculoid type of leprosy. It initially presents with anesthetic macules with hypopigmentation (photograph) and nerve involvement (often can be palpated) and eventually progresses to autoamputation of the digits. Cellular immunity is intact in this type of leprosy, hence the positive lepromin skin test. Granulomas are present in sites of involvement with very few acid-fast bacteria in the granulomas. Treatment is dapsone plus rifampin. Option A (Absence of granulomas) is incorrect. Absence of granulomas characterizes the lepromatous type of leprosy. They lack cellular immunity (negative lepromin skin test); hence, the lack of granuloma formation. Option B (Acid-fast organisms in macrophages) is incorrect. Lepromatous leprosy has numerous acid-fast organisms in foamy macrophages, usually located below a clear zone in the skin (called a Grenz zone).

Option C (Highly contagious) is incorrect. Neither type of leprosy is highly contagious. Option D (Leonine facies) is incorrect. Nodular lesions produce the classic leonine facies in the lepromatous type of leprosy.

An afebrile 28-year-old medical student, who spent spring break in Tijuana, Mexico, develops travelers diarrhea. Physical examination demonstrates dry mucous membranes, poor skin turgor, and hypotension. Which of the following serum Na+, total body sodium (TBNa+) is most likely present? Serum Na+TBNa+ A.Decreased Decreased B. Decreased Increased C. Decreased Normal D.Increased Decreased E. Normal Decreased

A. serum Na+ decreased, TBNa+ decreased B. serum Na+ decreased, TBNa+ increased C. serum Na+ decreased, TBNa+ normal D. serum Na+ increased, TBNa+ decreased E. serum Na+ normal, TBNa+ decreased

Option E (serum Na + normal, TBNa+ decreased) is correct. Travelers diarrhea is most often due to enterotoxigenic Escherichia coli, which produces a toxin that activates adenylate or guanylate cyclase in enterocytes in the small intestine causing an isotonic loss of diarrheal fluid (secretory type of diarrhea). Loss of isotonic fluid does not alter the serum Na+ concentration (serum Na+ = TBNa+/TBW); however, the decrease in TBNa+ causes signs of volume depletion (dry mucous membranes, poor skin turgor, hypotension). Regarding the fluid compartments, the extracellular fluid (ECF) compartment is contracted (loss of fluid) and the intracellular (ICF) compartment is unchanged (no osmotic effect). Option A (serum Na+ decreased, TBNa+ decreased) is incorrect. This occurs when there is a hypertonic loss of fluid (serum Na+ = TBNa+/TBW), which often occurs with a loop diuretic (inhibition of the Na+, K+, 2Cl cotransporter), Addisons disease (loss of mineralocorticoids), or 21-hydroxylase deficiency (loss of mineralocorticoids). Regarding the fluid compartments, the ECF compartment is contracted (loss of fluid) and the ICF compartment is expanded (osmosis effect of hyponatremia). Option B (serum Na+ decreased, TBNa+ increased) is incorrect. This is an example of a hypotonic gain of sodium-

containing fluid. Examples include the edema states (e.g., cirrhosis, right-sided heart failure). In these conditions there are alterations in Starling pressuresdecreased plasma oncotic pressure and/or increase in plasma hydrostatic pressurewhich force fluid out of the vascular compartment into the interstitial space and body cavities. The cardiac output is decreased in these conditions, because fluid is trapped in the interstitial space and body cavities by the Starling pressure abnormalities. Decreased renal blood flow causes the kidney to reabsorb a slightly hypotonic saltcontaining fluid, which produces hyponatremia (serum Na+ = TBNa+/TBW). The fluid is redirected into the interstitial space (decreased plasma oncotic pressure) and body cavities (decreased plasma oncotic pressure and increased plasma hydrostatic pressure). Regarding the fluid compartments, the ECF compartment is expanded (addition of fluid) and the ICF compartment is expanded (osmosis effect of hyponatremia). Option C (serum Na+ decreased, TBNa+ normal) is incorrect. This occurs when there is a hypotonic gain of pure water, which is present in the syndrome of inappropriate antidiuretic hormone (ADH) due to excessive stimulation of ADH release (e.g., ectopic secretion of ADH by a small-cell carcinoma of lung). ADH reabsorbs electrolyte-free water from the collecting tubules causing a dilutional hyponatremia (serum Na+ = TBNa+/TBW). Since Na+ is not reabsorbed by ADH, the TBNa+ is normal; therefore, there are no signs of pitting edema. Regarding the fluid compartments, the ECF compartment is expanded (addition of fluid) and the ICF compartment is expanded (osmosis effect of hyponatremia). Option D (serum Na+ increased, TBNa+ decreased) is incorrect. This occurs when there is a hypotonic loss of sodiumcontaining fluid (serum Na+ = TBNa+/TBW)). Examples include excessive sweating and osmotic diuresis (e.g., glucosuria). The patient has signs of volume depletion (TBNa+) from the loss of sodium, mainly dry mucous membranes and a positive tilt test (blood pressure dropped and pulse rate increased when sitting up). Regarding the fluid compartments, the ECF compartment is contracted (loss of fluid) and the ICF compartment is contracted (osmosis effect of hypernatremia).

A 5-year-old boy with a family history of seasonal allergies develops erythema, itching, and swelling of the skin after a subcutaneous injection of ragweed pollen. Which of the following chemical mediators is most responsible for this skin reaction?

A. Bradykinin B. Complement C. Histamine D. Nitric oxide (NO) E. Prostaglandins

Option C (Histamine) is correct. Histamine is the primary chemical mediator of type I hypersensitivity reactions. It is stored and released by mast cells, the primary effector cell of type I hypersensitivity reactions. Histamine contracts the venular endothelial cells and exposes the basement membrane, causing increased vessel

permeability and swelling of the tissue. Histamine causes arteriolar vasodilation, which produces redness (rubor) of the skin and increased heat (calor). Histamine also causes pruritus. Option A (Bradykinin) is incorrect. Activated coagulation factor XII converts high-molecular-weight kininogen to bradykinin. Bradykinin causes vasodilation of arterioles, increased vessel permeability, and pain in acute inflammation. Bradykinin is not released from mast cells and is not operative in type I hypersensitivity reactions. Option B (Complement) is incorrect. Complement is synthesized by the liver and has many functions in acute inflammatory reactions. Anaphylatoxins C3a and C5a, for example, directly stimulate mast cell release of histamine; however, this is not the mechanism for histamine release from mast cells in type I hypersensitivity reactions. Option D (Nitric oxide (NO)) is incorrect. NO, a free radical gas produced mainly by macrophages and endothelial cells, is released during conversion of arginine to citrulline by NO synthase. NO causes arteriolar vasodilation and has microbicidal activity; however, it is not a key chemical mediator in type I hypersensitivity reactions. Option E (Prostaglandins) is incorrect. Prostaglandins are produced in leukocytes, endothelial cells, and platelets. Arachidonic acid is converted to PGG2 by cyclooxygenase, and PGG2 is then converted to PGH2, the major precursor of prostaglandins that produce vasodilation (PGE 2, PGI2), pain (PGE2), and fever (PGE2) in acute inflammation. In type I hypersensitivity reactions, prostaglandins further enhance the inflammatory reaction after they are synthesized and released by mast cells.

A 14-year-old boy develops fever, nuchal rigidity, and petechial lesions. Examination of the cerebrospinal fluid (CSF) shows increased protein, decreased glucose, increased neutrophils, and neutrophils with phagocytosed bacteria (see photograph). Which of the following pathogens is most likely responsible for his clinical findings?

A. Escherichia coli B. Listeria monocytogenes C. Neisseria meningitidis D. Streptococcus agalactiae E. Streptococcus pneumoniae

Option C (Neisseria meningitidis) is correct. The patient has N. meningitidis (photograph shows a gramnegative diplococcus), which is the most common cause of meningitis from 1 month to 18 years of age. The CSF findings (i.e., decreased glucose and increased protein and neutrophils) are characteristic of bacterial meningitis. Sepsis caused by N. meningitidis is commonly associated with petechial lesions (small-vessel vasculitis), which often coalesce into large ecchymotic blotches on the skin. There is increased risk that the patient could develop disseminated intravascular coagulation leading to adrenal hypofunction from hemorrhagic infarction of both adrenal glands (Waterhouse-Friderichsen syndrome). Option A (Escherichia coli) is incorrect. Meningitis caused by E. coli usually occurs in newborns and in individuals > 50 years of age. Option B (Listeria monocytogenes) is incorrect. Bacterial meningitis caused by gram-positive rods is most often due to L. monocytogenes. It causes bacterial meningitis in newborns and in immunocompromised patients, particularly those with AIDS. Option D (Streptococcus agalactiae) is incorrect. Meningitis caused by S. agalactiae, a gram-positive coccus, occurs in newborns. It is the most common cause of meningitis in this age group. Option E (Streptococcus pneumoniae) is incorrect. S. pneumoniae, a gram-positive diplococcus, is the most common cause of meningitis in patients > 18 years old.

A 28-year-old man with AIDS develops focal epileptic seizures. The photograph shows a CT of the brain. The CD4 helper T-cell count is 50 cells/mm3. Treatment with an antibiotic causes some resolution of the lesions. Which of the following is the most likely pathogen?

A. Cryptococcus neoformans B. Cytomegalovirus (CMV)

C. Epstein-Barr virus (EBV) D. Herpes simplex virus type 1 (HSV-1) E. Toxoplasma gondii
Option E (Toxoplasma gondii) is incorrect.T. gondii is the most common cause of space-occupying lesions in the brain in AIDS patients, especially when the CD4 helper T-cell count is about 50 cells/mm3. The CT shows multiple enhancing lesions. (Pseudocysts or individual tachyzoites of T. gondii are located at the periphery of the lesions in the brain at autopsy.) Option A (Cryptococcus neoformans) is incorrect. C. neoformans is the most common systemic fungus causing chronic meningitis in patients with AIDS. It does not cause space-occupying lesions in the brain. Option B (Cytomegalovirus (CMV)) is incorrect. CMV causes encephalitis in immunocompromised patients. It does not cause space-occupying lesions in the brain. Option C (Epstein-Barr virus (EBV)) is correct. EBV is responsible for the genesis of most AIDS-related primary B-cell lymphomas of the brain. MRI shows multifocal, ring-enhancing lesions located in the same areas as T. gondii encephalitis. However, the lesions associated with T. gondii respond to antibiotic therapy (pyrimethamine), while cancer does not. Option D (Herpes simplex virus type 1 (HSV-1)) is incorrect. HSV-1 produces an encephalitis characterized by hemorrhagic necrosis of the temporal lobes and orbital frontal areas. This pattern of inflammation is not present in this patient.

A 32-year-old woman, who is 5 days postpartum, develops fever, low back pain, and a yellow discoloration in her eyes. Immediately after her delivery she had massive hemorrhage due to a retained placenta for which she received 3 units of packed RBCs. Her pretransfusion antibody screen was negative and all units of blood were compatible. A similar obstetrical mishap occurred during her first pregnancy 14 years ago, which required 3 units of packed RBCs to normalize her blood loss. Her current CBC demonstrates a 2 g/dL drop in Hb concentration when compared to the posttransfusion level prior to discharge from the hospital. Both direct and indirect Coombs tests are positive. A pink discoloration is noted in the serum. The total bilirubin is 3.5 mg/dL with predominantly unconjugated bilirubin present on fractionation. The urine has a positive dipstick for blood, but no RBCs are noted. The serum alanine aminotransferase (ALT) concentration is 20 U/L. Which of the following best describes the mechanism for jaundice?

A. Donor IgG antibodies are destroying patient RBCs B. Patient anti-human leukocyte antibodies (HLA) antibodies are destroying patient leukocytes

C. Patient IgG antibodies are destroying transfused donor RBCs D. Posttransfusion fever due to cytomegalovirus E. Posttransfusion hepatitis due to hepatitis C
Option C (Patient IgG antibodies are destroying transfused donor RBCs) is correct. The patient has a delayed hemolytic transfusion reaction. She has positive direct and indirect Coombs tests, free Hb in her serum (pink staining), and a significant drop in her posttransfusion Hb concentration after leaving the hospital. It is likely that she was sensitized during her first pregnancy 14 years ago, but the antibody titers were too low to detect in the antibody screen she had prior to transfusion during her recent hospitalization. One or more of the units she was transfused with during her most recent hospitalization must have had the antigen to which she had memory B cells. Once these memory B cells were antigenically stimulated, development of plasma cells producing the antibody can be extremely quick or, as in this case, extremely slow before signs of a hemolytic anemia become evident. Option A (Donor IgG antibodies are destroying patient RBCs) is incorrect. Although donor packed RBCs do contain small amounts of plasma containing natural antibodies, they are not responsible for producing hemolytic transfusion reactions. Option B (Patient anti-human leukocyte antibodies (HLA) antibodies are destroying patient leukocytes) is incorrect. This describes the pathogenesis of a febrile type of hemolytic transfusion reaction. In these reactions, the indirect and direct Coombs tests are negative and there is no hemolysis of RBCs. Option D (Posttransfusion fever due to cytomegalovirus) is incorrect. Cytomegalovirus is the most common pathogen transmitted by blood transfusion; however, this patients fever is related to a hemolytic transfusion reaction. Option E (Posttransfusion hepatitis due to hepatitis C) is incorrect. The time interval from her transfusion is too short to develop signs of hepatitis C. In addition, her serum ALT was normal and would have been markedly elevated if she had a hepatitis and the bilirubin fractions elevated would be a mixture of conjugated and unconjugated bilirubin (not predominantly unconjugated).

The photograph shows the arm of a 35-year-old man who works on an estate as a gardener. He says he was initially pricked by sphagnum moss on his hand. Which of the following is the causal agent?

A. Fonsecaea pedrosoi B. Nocardia asteroides C. Pseudallescheria boydii D. Sporothrix schenckii E. Trichophyton rubrum
Option D (Sporothrix schenckii) is correct. The patient has lymphocutaneous sporotrichosis (linear subcutaneous nodules). S. schenckii is a saprophyte that is commonly found on shrubs and thorny plants (e.g., roses). It is introduced into the subcutaneous tissue by trauma (e.g., thorn prick; sphagnum moss) and then spreads via the lymphatics (nodular, suppurative lesions). It is an occupational hazard of gardeners and florists. It is treated with oral itraconazole or a saturated solution of potassium iodide. Option A (Fonsecaea pedrosoi) is incorrect. F. pedrosoi is one of a group of dematiaceous (pigmented) fungi that enters the subcutaneous tissue by traumatic implantation (e.g., splinter of wood), where it produces a warty skin lesion (chromomycosis). Microscopic sections show a granulomatous reaction with pigmented fungi located within multinucleated giant cells. Option B (Nocardia asteroides) is incorrect. N. asteroides and Pseudallescheria boydii are organisms associated with the formation of a mycetoma (Madura foot). A mycetoma is a chronic infection caused by subcutaneous fungi (e.g., P. boydii) or actinomycetes (e.g., filamentous bacteria, such as N. asteroides). The organisms are present in soil and are introduced into the subcutaneous tissue (usually the foot) by traumatic implantation. This results in progressive subcutaneous swelling of the tissue and formation of numerous abscesses and draining sinus tracts containing granules that harbor the organisms. Infection usually extends to the subjacent bones. Amputation frequently is required for cases that are resistant to antibiotic therapy. Option C (Pseudallescheria boydii) is incorrect. See the description of a mycetoma. P. boydii usually extends to the subjacent bones.

Option E (Trichophyton rubrum) is incorrect. T. rubrum is a superficial dermatophyte that is associated with infections limited to the stratum corneum (e.g., tinea corporis).

The photograph shows the foot of a 21-year-old man who has a family history of premature death due to stroke and myocardial infarction. Which of the following best explains the pathogenesis of the finding on the foot?

A. Decreased activation of capillary lipoprotein lipase B. Deficiency of apolipoprotein C-II C. Deficiency of apolipoprotein E D. Deficiency of low-density lipoprotein (LDL) receptors E. Increased synthesis of very low density (VLDL) lipoprotein

Option D (Deficiency of low-density lipoprotein (LDL) receptors) is correct. The patient has familial hypercholesterolemia, an autosomal dominant disorder associated with a deficiency of LDL receptors. A decrease in LDL receptors causes increased levels of serum cholesterol. Excess cholesterol deposits in tendons (Achilles tendon in this case) and in other sites (e.g., eyelids; see Fig. 9-2B in Rapid Review Pathology, 3rd edition). It also causes premature atherosclerosis, resulting in strokes and myocardial infarctions between 30 and 40 years of age. Patients are placed on HMG-CoA reductase inhibitors (statins) at an early age. Option A (Decreased activation of capillary lipoprotein lipase) is incorrect. Deficiency of capillary lipoprotein lipase, an enzyme that normally hydrolyses chylomicrons, causes an increase in chylomicrons in children (type I hyperlipoproteinemia). Hyperchylomicronemia leads to hypertriglyceridemia, causing pancreatitis, hepatosplenomegaly, and papular skin lesions (eruptive xanthomas, not tendon xanthomas). Option B (Deficiency of apolipoprotein C-II) is incorrect. Deficiency of apolipoprotein C-II, the activator of capillary lipoprotein lipase, causes an increase in chylomicrons in children (type I hyperlipoproteinemia). Hyperchylomicronemia leads to hypertriglyceridemia, causing pancreatitis, hepatosplenomegaly, and papular skin lesions (eruptive xanthomas, not tendon xanthomas). Option C (Deficiency of apolipoprotein E) is incorrect. Chylomicron remnants and intermediate-density lipoproteins (IDL) normally have apolipoprotein E on their surface. Receptors for apolipoprotein E in

hepatocytes remove these remnants from the blood. Deficiency of apolipoprotein E leads to an increase in chylomicron remnants and IDL in the blood, which is designated type III hyperproteinemia (remnant disease; dysbetalipoproteinemia). Both cholesterol and triglyceride are increased in the blood causing deposition of these lipids into the creases of the palms (palmar xanthomas; see Fig. 9-2C in Rapid Review Pathology, 3rd edition). Option E (Increased synthesis of very low density (VLDL) lipoprotein) is incorrect. Increased synthesis of endogenous triglyceride in the liver increases the VLDL fraction in blood (type IV hyperlipoproteinemia). There may be deposits of VLDL fraction and triglycerides in yellow papular skin lesions (eruptive xanthomas, not tendon xanthomas; see Fig. 9-2D in Rapid Review Pathology, 3rd edition).

A 1-month-old has a diagnosis of neonatal hepatitis. Liver functions studies show that the conjugated bilirubin fraction > 50% of the total bilirubin. Serum alkaline phosphatase adjusted for age is markedly increased. A liver biopsy is performed and the tissue is stained with periodic acid-Schiff (PAS; see photograph). What is the most likely diagnosis?

A. Alpha1-antitrypsin (AAT) deficiency B. Congenital cytomegalovirus (CMV) C. Extrahepatic biliary atresia D. Viral hepatitis
Option A (Alpha1-antitrypsin (AAT) deficiency) is correct. AAT deficiency is an autosomal dominant disease, whose alleles are inherited codominantly (each allele expresses itself). The normal allele is M (95% frequency in United States), and MM is the normal genotype with AAT in the normal range. Deficient variants (decreased AAT levels) involve the Z allele and the S allele. Severe deficiency most commonly occurs in the homozygous ZZ variant. It is associated with panacinar emphysema and cirrhosis of the liver. In children, who are homozygous for ZZ, in 50% of cases, AAT is not secreted properly from hepatocytes. Pathologic accumulation of AAT in hepatocytes causes liver damage. Liver biopsies with PAS stains show red cytoplasmic granules (see photograph). It presents as neonatal hepatitis with intrahepatic cholestasis, with the hepatitis progressing into cirrhosis. In other cases, where there is no problem with secreting AAT, but levels of AAT are decreased, panacinar emphysema may develop later in life. Option B (Congenital cytomegalovirus (CMV)) is incorrect. Congenital CMV is one of many causes of neonatal hepatitis; however, the classic basophilic intranuclear inclusions are not present in the biopsy.

Option C (Extrahepatic biliary atresia) is incorrect. Extrahepatic biliary atresia is a cause of neonatal jaundice. Liver biopsies show inflammatory destruction of all or part of the extrahepatic bile ducts and bile duct proliferation in the portal triads. These features are not present in the patients liver biopsy. Option D (Viral hepatitis) is incorrect. Viral hepatitis in neonates is usually associated with congenital infections (e.g., CMV). The findings of PAS positive granules in the biopsy exclude viral hepatitis as a possibility.

A 45-year-old man, who is under treatment for acute sinusitis, develops a sudden onset of fever, a diffuse erythematous rash, and oliguria. The serum blood urea nitrogen (BUN) is 50 mg/dL and creatinine is 5 mg/dL. A CBC reveals a mild normocytic anemia and eosinophilia. A urinalysis shows mild proteinuria, hematuria, and presence of WBCs and WBC casts. Some of the WBCs in the urine have refractile granules consistent with eosinophils. What is the most likely diagnosis?

A. Acute drug-induced tubulointerstitial nephritis B. Acute glomerulonephritis C. Acute pyelonephritis D. Prerenal azotemia E. Renal papillary necrosis
Option A (Acute drug-induced tubulointerstitial nephritis) is correct. The patient has acute sinusitis and is likely taking an antibiotic (e.g., amoxicillin) that has produced acute drug-induced tubulointerstitial disease. Key clinical features of the disease are an abrupt onset of fever, oliguria, and a rash. Laboratory findings include a serum BUN:creatinine ratio < 15 (renal azotemia), WBCs and WBC casts, proteinuria, eosinophiluria, and the presence of eosinophilia in the peripheral blood. Cessation of the drug causes reversal of the renal failure. Option B (Acute glomerulonephritis) is incorrect. Acute glomerulonephritis does not present with an abrupt onset of acute renal failure, fever, and rash, as in this patient. Furthermore, the type of urine cast is different, mainly RBC casts in the nephritic syndrome or fatty casts in the nephrotic syndrome. Option C (Acute pyelonephritis) is incorrect. Acute pyelonephritis is an example of an acute tubulointerstitial nephritis and is associated with WBC casts; however, it does not present with acute renal failure and is not associated with rash, eosinophilia, and eosinophiluria. Option D (Prerenal azotemia) is incorrect. Prerenal azotemia refers to oliguria that is due to hypoperfusion of the kidneys from a decrease in cardiac output. The serum BUN:creatinine ratio > 15 and the urine sediment is normal. The patient has renal azotemia with a serum BUN:creatinine ratio of 10. Option E (Renal papillary necrosis) is incorrect. Renal papillary necrosis refers to sloughing off of the renal papillae. It

produces gross hematuria, proteinuria, and colicky flank pain. An intravenous pyelogram shows a ring defect where one or more papillae used to reside. Other causes of renal papillary necrosis include diabetes mellitus, sickle cell trait/disease, and acute pyelonephritis.

A 28-year-old woman has a history of colicky right lower quadrant pain and diarrhea. The photograph shows a resection of a portion of the terminal ileum with attached cecum and appendix. Which of the following best characterizes this disease?

A. Begins in the rectum B. Limited to the colon C. Inflammation without skip areas D. Noncaseating granulomas E. Primarily mucosal and submucosal ulceration
Option D (Noncaseating granulomas) is correct. The photograph shows a resection of terminal ileum with attached cecum and appendix. The appendix is located on the left side of the photograph. Thickening of the terminal ileal wall causes an area of narrowing at the junction of the ileum with the cecum. The ileal mucosa has a cobblestone appearance due to linear ulcerations (aphthous ulcers) that cut into the underlying submucosa. The gross findings, plus the history of recurrent right lower quadrant colicky pain and diarrhea, are characteristic of Crohns disease. Crohns disease is a chronic granulomatous ulceroconstrictive disease that peaks between the second and third decades of life. Transmural inflammation involving the terminal ileum causes narrowing of the lumen and signs of obstruction (colicky pain). The terminal ileum is involved either alone or in association with colonic involvement in 80% of cases. Option A (Begins in the rectum) is incorrect. Ulcerative colitis is a chronic ulceroinflammatory disease that begins in the rectum and may spread in continuity to the ileocecal valve. It is characterized by a recurrent history of crampy (not colicky) left lower quadrant pain associated with bloody diarrhea, which is due to ulceration involving the mucosa and submucosa. Crohns disease produces anal fistulas and abscesses. Option B (Limited to the colon) is incorrect. Ulcerative colitis is an inflammatory bowel disease that is limited to the

colon and rectum. Crohns disease commonly involves the terminal ileum as well as the colon. Option C (Inflammation without skip areas) is incorrect. Ulcerative colitis is continuous throughout the bowel, whereas Crohns disease is discontinuous throughout the gastrointestinal tract. Option E (Primarily mucosal and submucosal ulceration) is incorrect. Ulcerative colitis produces mucosal and submucosal ulceration, whereas Crohns disease has transmural inflammation of all layers of the intestinal wall.

The photograph shows a facial lesion in a febrile 42-year-old man. The lesion is raised and warm to the touch. Physical examination shows enlarged and tender cervical lymph nodes. What is the Gram stain morphology of the offending pathogen?

A. Gram-negative diplococcus B. Gram-positive cocci in chains C. Gram-positive cocci in clusters D. Gram-positive diplococcus E. Gram-positive rod
Option B (Gram-positive cocci in chains) is correct. The patient has erysipelas (raised erythematous rash) complicated by orbital cellulitis (eyelid swollen shut). Erysipelas is a cellulitis caused by Streptococcus pyogenes (group A streptococcus), which is a Gram-positive coccus that forms chains. Hyaluronidase produced by the pathogen causes the infection to spread through the subcutaneous tissue. Enlarged, tender cervical lymph nodes indicate an inflammatory reaction secondary to the infection. Treatment is intravenous vancomycin if located on the face or intravenous penicillin

G if located on the extremities. Option A (Gram-negative diplococcus) is incorrect. Gram-negative diplococci include Neisseria gonorrhoeae and meningitidis. N. gonorrhoeae is a sexually transmitted disease. It can produce pustules in disseminated gonococcemia, but does not produce a cellulitis. N. meningitidis produces meningitis in children and young adults. It produces petechial lesions on the skin when it disseminates, but is not associated with a cellulitis. Option C (Gram-positive cocci in clusters) is incorrect. Staphylococcus aureus is a Gram-positive coccus that forms clusters due to the production of coagulase, which converts fibrinogen to fibrin. This localizes the infection causing abscesses, not a cellulitis. Option D (Gram-positive diplococcus) is incorrect. Streptococcus pneumoniae is a Gram-positive diplococcus that produces community-acquired typical pneumonias, otitis media, and sinusitis. It does not produce skin infections. Option E (Gram-positive rod) is incorrect. Bacillus anthrax is a Gram-positive rod that can produce cutaneous anthrax through direct contact with infected or contaminated animal products. The skin lesion resembles an insect bite that eventually swells to form a black scab (due to edema factor), or eschar, with a central area of necrosis. This is called a malignant pustule. Other Gram-positive rods include Bacillus cereus, Clostridia species, and Listeria monocytogenes, none of which produce a cellulitis.

A 23-year-old man is moving logs on his woodpile when he is bitten by a spider (see photograph). He feels a sharp pain in his right index finger soon after being bitten. Which of the following is the clinical outcome of this bite?

A. Ascending paralysis and hypertension B. Disseminated intravascular coagulation (DIC) C. Muscle cramping and severe abdominal pain D. Mydriasis and muscle paralysis E. Ulceration with extensive necrosis

Option C (Muscle cramping and severe abdominal pain) is correct. Muscle cramping and abdominal pain are due to a neurotoxin produced by the black widow spider (Latrodectus mactans). The photograph shows a glossy black spider with a red hourglass on the ventral aspect of the abdomen. Paralysis and death may occur in very young and in very old individuals. Option A (Ascending paralysis and hypertension) is incorrect. Ascending paralysis and hypertension are due to a neurotoxin produced by the scorpion (Centruroides species). Option B (Disseminated intravascular coagulation (DIC)) is incorrect. DIC is associated with activation of the intrinsic and extrinsic coagulation system. It is a common complication of crotalid bites (e.g., rattlesnake). The venom has cytotoxic, hemotoxic, and neurotoxic components. DIC is very uncommon in a black widow bite. Option D (Mydriasis and muscle paralysis) is incorrect. Mydriasis and muscle paralysis are due to a neurotoxin produced by the coral snake. The neurotoxin binds to presynaptic nerve terminals and acetylcholine receptors. These findings do not occur with black widow bites. Option E (Ulceration with extensive necrosis) is incorrect. Ulceration with necrosis is characteristic of the necrolytic toxin produced by the brown recluse spider (Loxosceles reclusa). Initial symptoms from the bite are usually minimal and are followed within a few hours by severe pain and swelling and erythema in the area of the bite. A central area of vesiculation usually is followed by extensive ulceration and necrosis of the skin and subcutaneous tissue. Skin ulceration does not occur with black widow bites.

A 2-year-old child suddenly develops midepigastric colicky abdominal pain and bloody stools. Physical examination shows a palpable sausage-shaped mass in the midepigastrium. Which of the following is the most likely diagnosis?

A. Acute appendicitis B. Congenital pyloric stenosis C. Duodenal atresia D. Intussusception E. Meckels diverticulitis
Option D (Intussusception) is correct. Intussusception has a peak incidence between ages 1 and 5. In children, the terminal ileum invaginates into the cecum. Mounds of hyperplastic lymphoid tissue in Peyers patches serve as the nidus for the intussusception. The entrapped bowels show signs of obstruction and ischemia (colicky pain with bloody diarrhea, respectively). An oblong mass is frequently palpated in the midepigastrium (Dances sign). It usually selfreduces, but may require air reduction under fluoroscopy or ultrasound. In adults, a polyp or cancer is the nidus for intussusception. Option A (Acute appendicitis) is incorrect. Acute appendicitis presents with right lower quadrant pain and rebound

tenderness. Option B (Congenital pyloric stenosis) is incorrect. Congenital pyloric stenosis presents in the second or third week of life with persistent vomiting of non-bile-stained fluid and a walnut-shaped mass in the left upper quadrant. Bloody stools are not present. Option C (Duodenal atresia) is incorrect. Duodenal atresia presents at birth with projectile vomiting of bile-stained fluid. The area of atresia begins distal to the entrance of the common bile duct into the duodenum. Air is present in the stomach and the proximal duodenum (double-bubble sign). Bloody stools are not present. There is an increased association with Down syndrome. Option E (Meckels diverticulitis) is incorrect. Meckels diverticulitis presents with right lower quadrant pain and rebound tenderness (cannot be distinguished from acute appendicitis). A palpable mass and bloody stools are not present. A radionuclide scan to identify parietal cells in the diverticulum is the screening test of choice.

A 3-year-old black child with sickle cell anemia has a high fever. Physical examination shows nuchal rigidity, scleral icterus, and hepatosplenomegaly. Complete blood cell count shows a normocytic anemia and a WBC count of 41,000 cells/mm3. The corrected reticulocyte count is 8%. Lumbar puncture shows increased protein, decreased glucose, and numerous neutrophils with phagocytosed bacteria. Which of the following bacteria are most likely to be found on Gram stain of the cerebrospinal fluid (CSF)?

A. Gram-negative diplococci B. Gram-negative rods C. Gram-positive cocci D. Gram-positive diplococci E. Gram-positive rods
Option D (Gram-positive diplococci) is correct. The child has sickle cell anemia complicated by bacterial meningitis (nuchal rigidity, neutrophils in CSF with phagocytosed bacteria). In sickle cell anemia, the spleen is typically dysfunctional, which causes the patient to be susceptible to sepsis and meningitis resulting from Streptococcus pneumoniae, a Gram-positive diplococcus. Sepsis due to S. pneumoniae is the most common cause of death in children with sickle cell disease. This emphasizes the importance of Pneumovax in preventing this complication. Option A (Gram-negative diplococci) is incorrect. Neisseria meningitidis, a Gram-negative diplococcus, is the most common cause of meningitis in patients 1 month to 18 years of age, whether the spleen is present or absent; however, it is not a common cause of sepsis in sickle cell disease. Option B (Gram-negative rods) is incorrect. Meningitis caused by Gram-negative rods (e.g., Escherichia coli) most commonly occurs in newborns.

Option C (Gram-positive cocci) is incorrect. Meningitis caused by Gram-positive cocci (e.g., Streptococcus agalactiae) most commonly occurs in newborns. Option E (Gram-positive rods) is incorrect. Meningitis caused by Gram-positive rods (e.g., Listeria monocytogenes) most commonly occurs in newborns and immunocompromised hosts.

A 39-year-old man who has recently returned from a business trip to Mexico City sees his physician because of fatigue and yellow discoloration of the eyes. The photograph shows a histologic section of the liver obtained by needle biopsy. Which of the following is the most likely diagnosis?

A. Alcoholic hepatitis B. Metastatic liver disease C. Obstructive liver disease D. Primary liver cancer E. Viral hepatitis
Option E (Viral hepatitis) is correct. Clinical manifestations of acute viral hepatitis include fatigue and jaundice, which is indicated by the yellow discoloration of the patients eyes. Hepatitis A is commonly associated with travel outside the United States. Characteristic histologic changes include apoptosis of hepatocytes, swelling of hepatocytes, and the presence of lymphocytic infiltrate, which are shown in the photograph. An apoptotic (Councilmans) body is apparent along the lower border of the biopsy. It has a dense black nuclear remnant surrounded by dense-staining eosinophilic cytoplasm and has moved away from nearby cells. Swollen cells and a prominent lymphocytic infiltrate, also indicative of hepatitis, are apparent in the subjacent hepatic parenchyma. Option A (Alcoholic hepatitis) is incorrect. Histologic features of alcoholic hepatitis include a neutrophilic infiltrate, Mallorys bodies (pink-staining keratin intermediate filaments in hepatocytes), and fatty change in hepatocytes. Option B (Metastatic liver disease) is incorrect. Metastatic disease does not commonly produce jaundice. Furthermore, there are malignant cells present in the tissue. Option C (Obstructive liver disease) is incorrect. Obstructive liver disease is characterized by obstruction of intrahepatic

or extrahepatic bile ducts. If the patient had obstructive liver disease as opposed to acute viral hepatitis, the biopsy would show dilated ducts filled with bile. Option D (Primary liver cancer) is incorrect. Hepatocellular carcinoma (primary liver cancer) usually occurs in a setting of underlying cirrhosis. There is no evidence of fibrosis to indicate cirrhosis or malignant cells to indicate hepatocellular carcinoma.

A 45-year-old man, who is a pharmacist, has a 20-year history of alcohol abuse. He complains of recurrent episodes of dizziness, confusion, and inability to concentrate. Laboratory studies when he is symptomatic show a serum glucose level of 20 mg/dL and an increase in serum insulin and serum C-peptide. What is the most likely diagnosis?

A. Alcohol-induced hypoglycemia B. Benign tumor of -islet cells C. Ectopic secretion of an insulin-like factor D. Malignant tumor of -islet cells E. Patient injection of human insulin
Option B (Benign tumor of -islet cells) is correct. Benign tumors of the -islet cells, or insulinomas, synthesize excess insulin, resulting in a fasting hypoglycemia. Since the brain requires glucose for fuel during the fasting state, the signs of fasting hypoglycemia include dizziness, confusion, headache, inability to concentrate, seizures, and the potential for coma. When preproinsulin in the -islet cells is delivered to the Golgi apparatus, proteolytic reactions generate insulin and a cleavage peptide called C-peptide. Hence, C-peptide is a marker for endogenous synthesis of insulin. Both serum insulin and serum C-peptide levels are increased in this patient; hence, confirming the presence of an insulinoma. Option A (Alcohol-induced hypoglycemia) is incorrect. Alcohol is a common cause of hypoglycemia in the fasting state. The increase in nicotinamide adenine dinucleotide (NADH) in alcohol metabolism causes pyruvate to be converted to lactic acid. This reduces the amount of pyruvate to use as a substrate for gluconeogenesis, which is the primary source of glucose in the fasting state. Alcohol has no direct effect on insulin or C-peptide levels; however, hypoglycemia would decrease both serum insulin and C-peptide. Option C (Ectopic secretion of an insulin-like factor) is incorrect. An insulin-like factor that causes hypoglycemia is most often produced by a hepatocellular carcinoma. Hypoglycemia suppresses -islet cells, resulting in a decrease in serum insulin and C-peptide. Option D (Malignant tumor of -islet cells) is incorrect. Tumors of -islet cells secrete glucagon, which produces hyperglycemia (not hypoglycemia) by stimulating gluconeogenesis. Option E (Patient injection of human insulin) is incorrect. Injection of human insulin increases serum insulin and

produces hypoglycemia. Hypoglycemia suppresses -islet cells, causing a decrease in endogenous synthesis of insulin and a corresponding decrease in serum C-peptide.

A 49-year-old man has fatigue and postural hypotension. Physical examination reveals dry mucous membranes, an increase in heart rate and drop in blood pressure when moved from the supine to sitting position, and increased pigmentation of the buccal mucosa and skin. Laboratory studies show a decrease in serum cortisol and an increase in serum adrenocorticotropic hormone (ACTH). A peaked T wave is present on an ECG. Which one of the following electrolyte profiles is most likely to be present in this man? Serum Na+ Serum K+ Serum Cl Serum HCO3 (135147 mEq/L) (3.55.0 mEq/L) (95105 mEq/L) (2228 mEq/L) A.128 5.9 96 20 B. 146 5.5 104 18 C. 138 2.2 114 14 D.130 2.9 80 36 E. 152 2.8 110 33

A. serum Na+ 128, serum K+ 5.9, serum Cl 96, serum HCO3 20 B. serum Na+ 146, serum K+ 5.5, serum Cl 104, serum HCO3 18 C. serum Na+ 138, serum K+ 2.2, serum Cl 114, serum HCO3 14 D. serum Na+ 130, serum K+ 2.9, serum Cl 80, serum HCO3 36 E. serum Na+ 152, serum K+ 2.8, serum Cl 110, serum HCO3 33
Option A (serum Na+ 128, serum K+ 5.9, serum Cl 96, serum HCO3 20) is correct. The patient has Addisons disease, which is most often due to autoimmune destruction of the adrenal cortex. Destruction of the zona glomerulosa produces a deficiency of aldosterone, which is responsible for the electrolyte abnormalities. There is inhibition of Na+ reabsorption and K+ secretion by the aldosterone-dependent Na+ and K+ channels located in the distal tubule and collecting ducts causing a hypertonic loss of Na+ in the urine (hyponatremia) and retention of K+ (hyperkalemia with peaked T waves). Due to dysfunction of the aldosterone-dependent H+/K+ ATPase pump in the collecting tubules, there is retention of H+ ions and a subsequent lack of synthesis of HCO3, which produces metabolic acidosis. The excess H+ ions combine with Cl anions producing a normal anion gap type of metabolic acidosis and the calculation is as follows: anion gap = serum Na+ 128(serum Cl 96 + serum HCO320) = 12 mEq/L. The decrease in serum cortisol causes an increase in serum ACTH due to the loss of a negative feedback on ACTH by cortisol. Since ACTH has melanocytestimulating properties, there is diffuse pigmentation of the buccal mucosa and skin. Option B (serum Na+ 146, serum K+ 5.5, serum Cl 104, serum HCO3 18) is incorrect. These findings are most compatible with chronic renal failure. In chronic renal failure there is tubular cell dysfunction resulting in retention of K + (hyperkalemia with peaked T waves on an ECG) and an increased anion gap type of metabolic acidosis due to retention of organic acids like sulfuric and phosphaturic acid. The anion gap in this case is calculated as follows: anion gap = serum Na+ 146(serum Cl 104 + serum HCO318) = 24 mEq/L (12 mEq/L 2).

Option C (serum Na+ 138, serum K+ 2.2, serum Cl 114, serum HCO3 14) is incorrect. These findings are most compatible with a secretory type of diarrhea (e.g., travelers diarrhea due to enterogenic Escherichia coli, cholera). In these conditions a toxin activates adenylate cyclase in enterocytes causing the ion pumps in the small intestine to secrete isotonic fluid. Loss of isotonic fluid does not alter the serum Na+ concentration. Diarrheal fluid is rich in K+ and HCO3, the former resulting in hypokalemia and the latter a normal anion gap type of metabolic acidosis. The anion gap calculation is as follows: anion gap = serum Na+ 138(serum Cl 114 + serum HCO314) = 10 mEq/L (12 mEq/L 2). The serum Cl is increased, because it replaces the HCO3 anions that are lost in the diarrheal fluid to maintain electroneutrality. Option D (serum Na+ 130, serum K+ 2.9, serum Cl 80, serum HCO3 36) is incorrect. This profile is compatible with metabolic alkalosis due to loop diuretics, thiazide diuretics, or vomiting. Thiazide and loop diuretics inhibit channels in the nephron causing decreased reabsorption of Na+. This causes a hypertonic loss of Na+ (hyponatremia) and Cl (hypochloremia). Increased delivery of Na+ to the late distal and collecting tubules results in augmented exchange of Na+ for K+ in the aldosterone-dependent Na+ channels resulting in increased urinary loss of K+ (hypokalemia with U waves on an ECG). When K+ is depleted, Na+ exchanges with H+ ions causing increased synthesis and reabsorption of HCO3 causing metabolic alkalosis. In vomiting, hydrochloric acid is lost. For every H + ion lost in the vomitus, there is a corresponding HCO3left unneutralized in the blood causing metabolic alkalosis (HCO3> 28 mEq/L). Option E (serum Na + 152, serum K+ 2.8, serum Cl 110, serum HCO3 33) is incorrect. This profile is compatible with mineralocorticoid excess (e.g., primary aldosteronism; 11-hydroxylase deficiency). Enhanced function of aldosteronemediated Na+-H+ channels in the late distal and collecting ducts increases the synthesis of HCO3 leading hypernatremia, hypokalemia (initially exchanged with Na+ until depleted), and metabolic alkalosis (Na+ exchanges with H+; for every H+ lost in the urine there is a corresponding HCO3 entering the blood).

A 24-year-old woman has hereditary spherocytosis and jaundice. Which of the following serum and urine laboratory test abnormalities would you expect? % CB Urine BilirubinUrine UBGAST ALT ALP GGT Normal Negative NormalNormalNormalNormal A <20 Negative NormalNormalNormal B >50 Negative C 2050 D 2050 E <20 Negative Normal NormalNormalNormalNormal

Number of arrows relates to the degree of elevation. ALP, alkaline phosphatase; ALT, alanine aminotransferase; AST, aspartate aminotransferase; CB, conjugated bilirubin; GGT, -glutamyltransferase; UBG, urobilinogen.

A. A B. B C. C D. D E. E
Option A is correct. In hereditary spherocytosis, an autosomal dominant disease with a membrane defect due to a lack of ankyrin, spherocytes are removed extravascularly by macrophages leading to an increase in unconjugated bilirubin (UCB) and a CB < 20%. AST is present in RBCs; therefore, there is a slight increase in serum AST. Urine UBG is increased due to increased production of CB by the liver and increased conversion of CB to UBG in the colon. Fecal UBG gives the color to stool. Normally, a small portion of UBG in the colon is reabsorbed back into the blood (enterohepatic circulation) where most is taken up by the liver and the remainder is filtered into the urine to give the color to urine. Therefore, if there is more UBG in the stool (e.g., extravascular hemolytic anemia), then proportionately more UBG ends up in the urine (darker yellow than normal). Option B is incorrect. The laboratory findings indicate an obstructive type of jaundice, either due to intrahepatic cholestasis (e.g., drug-induced oral contraceptives) or extrahepatic cholestasis. Since bile is predominantly CB, in obstructive jaundice this is the predominant fraction that enters the bloodstream (CB > 50%) through ruptured intrahepatic bile ductules. CB is water soluble; therefore, it enters the urine to produce a positive dipstick test for bilirubin. Since UBG normally present in the urine derives from enterohepatic circulation of UBG initially produced in the liver, absence of UBG in the stool in obstructive liver disease (light-colored stools) leads to an absence of UBG in the urine. Enzyme markers for obstructive liver disease are serum ALP and serum GGT. Serum AST and ALT are also increased but to a lesser degree. Option C is incorrect. The laboratory findings are compatible with alcoholic hepatitis, which produces a mixed jaundice (CB 2050%) due to decreased uptake and conjugation of UCB and destruction of bile ductules among liver cells leading to an increase in CB in the blood. Urine UBG is increased, because UBG that is normally recycled back to the liver is redirected into the urine. Serum transaminases are increased with serum AST (mitochondrial enzyme) greater than ALT (cytosol enzyme). Since AST is normally located in the mitochondria of hepatocytes, alcohol, a mitochondrial toxin, causes preferential release of AST into the blood and is higher that ALT. Serum ALP is slightly increased; however, serum GGT is markedly increased, because alcohol induces enzyme synthesis in the hepatocyte cytochrome P450 system in the smooth endoplasmic reticulum (SER). GGT is also located in the SER and is increased as well. Therefore, key markers for alcoholic hepatitis are AST > ALT and marked increase in GGT. Option D is incorrect. The laboratory findings are most consistent with hepatitis in which there is diffuse necrosis of

hepatocytes (e.g., hepatitis A). Hepatitis produces a mixed jaundice (CB 2050%), due to decreased uptake and conjugation of UCB and destruction of bile ductules among liver cells leading to an increase in CB in the blood. Urine UBG is increased, because UBG that is normally recycled back to the liver is redirected into the urine. Urine bilirubin is increased, because CB is water soluble. Serum transaminases are markedly increased because of massive liver cell necrosis; serum ALT is greater than AST. Serum ALP and GGT are only slightly increased. Option E is incorrect. The laboratory findings indicate an increase in UCB. This could be due to hemolytic anemias; however, destruction of RBCs also increases AST and urine UBG is increased. Hence, the most common cause of this profile is Gilberts disease, which is an autosomal dominant disease characterized by decreased uptake and conjugation of bilirubin particularly exacerbated by the fasting state. Baseline UCB levels are increased to over twice normal in the fasting state leading to visible evidence of jaundice (CB < 20%). Next to viral hepatitis, it is the second overall most common cause of jaundice in the United States. All enzyme studies and urine studies are normal. It has no clinical significance.

A 70-year-old man requires a radical prostatectomy for prostate cancer. Prior to surgery, his platelet count, bleeding time, prothrombin time (PT), and partial thromboplastin time (PTT) time are all normal. Following surgery he develops severe bleeding from the penis that requires blood transfusions. Which of the following sets of hemostasis studies is most likely to be present in this patient? Platelet CountBleeding TimePT PTT A.Normal Normal B. Normal Normal Normal C. D.Normal E. Normal Normal PT, prothrombin time; PTT, partial thromboplastin time.

A. platelet count normal, bleeding time increased, PT normal, PTT increased B. platelet count normal, bleeding time normal, PT normal, PTT increased C. platelet count decreased, bleeding time increased, PT increased, PTT increased D. platelet count normal, bleeding time increased, PT increased, PTT increased E. platelet count normal, bleeding time normal, PT increased, PTT increased
Option D (platelet count normal, bleeding time increased, PT increased, PTT increased) is correct. The patient has primary fibrinolysis, which may occur in radical prostate surgery. In this setting, urokinase is released from the tissue causing activation of plasminogen and the release of plasmin. Plasmin degrades multiple coagulation factors (e.g., V, VIII, fibrinogen). This increases the PT and PTT, because factors V and VIII are in the final common pathway. The degradation products of fibrinogen interfere with platelet aggregation causing an increase in the bleeding time. The platelet count is not affected. The test for fibrinogen degradation products is positive; however, d-dimers are negative because fibrin clots are not present in primary fibrinolysis.

Option A (platelet count normal, bleeding time increased, PT normal, PTT increased) is incorrect. This set of studies is most often associated with classic von Willebrand disease. In this autosomal dominant disorder, there is a deficiency of von Willebrand factor (vWF), which is necessary for platelet adhesion to areas of endothelial injury. This prolongs the bleeding time without affecting the platelet count. In the circulation, vWF also complexes with factor VIII coagulant (VIII:C), which prevents degradation of VIII:C. Therefore, deficiency of vWF automatically leads to decreased factor VIII:C activity and prolongation of the PTT. The PT is normal because it does not evaluate VIII:C activity. Option B (platelet count normal, bleeding time normal, PT normal, PTT increased) is incorrect. This set of studies is consistent with a coagulation factor deficiency in the intrinsic coagulation pathway: XII, XI, IX, or VIII. Of the four choices, factor VIII would be the most common deficiency in a patient with hemophilia A. Option C (platelet count decreased, bleeding time increased, PT increased, PTT increased) is incorrect. This set of data in which all the tests are abnormal is most often seen in disseminated intravascular coagulation (DIC). Tissue thromboplastin activates the extrinsic coagulation system causing the formation of fibrin thrombi within the microcirculation. Factors I, II, V, and VIII are consumed in the fibrin clots, which results in anticoagulation (PT and PTT are both increased). Fibrin thrombi also trap platelets causing thrombocytopenia, which, in turn, is responsible for producing a prolonged bleeding time. Option E (platelet count normal, bleeding time normal, PT increased, PTT increased) is incorrect. This set of data is most consistent with a patient that is taking warfarin and/or heparin. Rat poison contains warfarin, which blocks epoxide reductase, rendering vitamin K inactive. This prevents further -carboxylation of the vitamin Kdependent coagulation factors: factors II (prothrombin), VII, IX, and X. Since factors X and II are in the final common pathway, both the PT and PTT are prolonged. The PT evaluates factors VII, X, V, II, and I (fibrinogen), while the PTT evaluates factors XII, XI, IX, VIII, X, V, II, and I. The platelet count and bleeding time are not affected by warfarin. Heparin enhances antithrombin III activity, which neutralizes activated serine proteases (XII, XII, IX, X, and II). Because factor X and II are in the final common pathway, both the PT and PTT are prolonged in a patient on heparin.

An 8-year-old boy with minimal change disease in the kidneys has generalized pitting edema and effusions. There is no evidence of tetany. Which of the following total serum calcium and serum-ionized calcium levels is most likely present in this patient? The square represents the normal reference interval for total serum calcium and ionized serum calcium.

A. A B. B C. C D. D
Option D (decreased total serum calcium, normal ionized serum calcium) is correct. The patient has nephrotic syndrome with a loss of albumin. Components of the total serum calcium include the bound and free fraction. Calcium is bound to albumin (40%) and phosphorus and citrate (13%). Albumin is the most important binding protein, because it has the most acidic amino acids that have COOH and COO- groups on them depending on the pH. At a normal pH of 7.40, approximately 40% of the acidic groups are COO- and can bind to positively charged calcium. The free component (not bound to anything) in the total serum calcium is ionized calcium (47%). It is the metabolically active fraction and has a negative feedback with parathyroid hormone (PTH). In this patient, the patient has nephrotic syndrome due to minimal change disease in which there is loss in the negative charge in the glomerular basement membrane with a selective loss of albumin in the urine. Albumin is also important in maintaining the plasma oncotic pressure; therefore, hypoalbuminemia results in a loss of a protein-poor, cell-poor fluid into the interstitial space causing dependent pitting edema and body effusions. The decrease in serum albumin automatically lowers the total serum calcium, but it has no effect on the ionized calcium level. Hence, the patient does not have tetany (muscle contractions). Option A (decreased total serum calcium, decreased ionized serum calcium) is incorrect. These findings are present in primary hypoparathyroidism (e.g., autoimmune destruction; DiGeorge syndrome). Patients will have tetany (e.g., thumb adducts into the palm; twitching of facial muscles with tapping of facial nerve).

Option B (normal total serum calcium, decreased ionized serum calcium) is incorrect. These findings occur in an alkalotic state (e.g., respiratory alkalosis, hysterical person hyperventilating). At an alkaline pH, there are more COOgroups on the acidic amino acids in albumin due to less H+ ions. Therefore, some of the ionized calcium is bound to these groups, which lowers the ionized calcium level (produces tetany) without altering the total serum calcium level. Option C is incorrect.

A 20-year-old man with chronic cholecystitis has a family history of anemia. Physical examination reveals splenomegaly. A CBC shows a normocytic anemia with an increased corrected reticulocyte count. The urinalysis is normal. The direct Coombs test is negative. The photograph shows the peripheral blood smear. What is the most likely diagnosis?

A. Glucose 6-phosphate dehydrogenase (G6PD) deficiency B. Hereditary spherocytosis C. Paroxysmal nocturnal hemoglobinuria (PNH) D. Pyruvate kinase (PK) deficiency E. Warm-autoimmune hemolytic anemia
Option D (Pyruvate kinase (PK) deficiency) is correct. PK deficiency is an autosomal recessive hemolytic disease with extravascular hemolysis. PK normally converts phosphoenolpyruvate to pyruvate leading to a net gain of 2 adenosine triphosphate (ATP). In PK deficiency, lack of ATP damages the membrane causing a loss of K + and dehydration of the RBC (echinocytes with thorny projections, arrow in the photograph). In homozygous variants, hemolytic anemia with jaundice begins at birth. There is an increase in 2,3 bisphosphoglycerate (BPG) proximal to the enzyme block, which right-shifts the O2-binding curve causing increased release of O2 to tissue. This somewhat offsets the deleterious effects of the anemia. Chronic extravascular hemolysis increases the risk for developing calcium bilirubinate stones leading to cholecystitis. An RBC enzyme assay is the confirmatory test for the anemia. Option A (Glucose 6-phosphate dehydrogenase (G6PD) deficiency) is incorrect. G6PD deficiency is an X-linked recessive disease. Due to the lack of G6PD, there is a corresponding deficiency of glutathione (GSH), which is necessary

to neutralize oxidants like hydrogen peroxide. The peripheral blood shows bite cells (RBCs with membrane missing) and occasional spherocytes, unlike the RBCs present in this patient. Furthermore, splenomegaly and chronic cholecystitis are not features of the disease. Option B (Hereditary spherocytosis) is incorrect. The patient has diastolic hypertension that is complicated by a druginduced hemolytic anemia (reticulocyte index > 3%). Methyldopa is the most likely drug he is taking, because it is commonly associated with a positive direct Coombs test and with a hemolytic anemia in a small percentage of cases. The drug alters Rh antigens on the surface of RBCs. IgG autoantibodies develop against the altered Rh antigens, attach to RBCs, and are phagocytosed by splenic macrophages (extravascular hemolysis). Patients develop fever, an unconjugated hyperbilirubinemia, spherocytes in the peripheral blood (membrane removed by macrophages), and polychromasia (blue-tinted young reticulocytes). Option C (Paroxysmal nocturnal hemoglobinuria (PNH)) is incorrect. PNH is an acquired membrane defect involving common myeloid stem cells. A gene mutation causes loss of the anchor for decay accelerating factor (DAF), which normally neutralizes complement attached to RBCs, neutrophils, and platelets at night. Loss of DAF causes intravascular complement destruction of RBCs, neutrophils, and platelets leading to pancytopenia and hemoglobinuria. There is no abnormal cell morphology in PNH and no splenomegaly. Option E (Warm-autoimmune hemolytic anemia) is incorrect. In warm-type (IgG-mediated) autoimmune hemolytic anemia, the RBCs are coated by IgG and C3b causing them to be phagocytosed by splenic and liver macrophages. They are removed extravascularly. The direct Coombs test is positive. This patient has a negative direct Coombs test.

A 25-year-old woman at 12 weeks gestation develops proteinuria, hypertension, and pitting edema of the legs. The photograph shows tissue passed through the vagina. Which of the following best describes this tissue?

A. Benign neoplasm of the chorionic villus with a 46,XX karyotype B. Benign neoplasm of the chorionic villus with a triploid karyotype C. Malignancy associated with diethylstilbestrol (DES) exposure D. Malignancy derived from trophoblastic tissue

E. Malignancy originating from striated muscle


Option A (Benign neoplasm of the chorionic villus with a 46,XX karyotype) is correct. The photograph shows enlarged, edematous chorionic villi interconnected with cordlike structures. No embryo is present. The findings indicate a complete hydatidiform mole, which is a benign neoplasm of the chorionic villus. Hydatidiform moles do not contain embryos and have a 46,XX karyotype, which is produced when an egg with no chromosomes is fertilized by two spermatozoa with a 23,X karyotype. Histologically, the villi are devoid of blood vessels and are surfaced by trophoblastic tissue (syncytiotrophoblast and cytotrophoblast). Hydatidiform moles have the capacity to transform into a choriocarcinoma, a malignancy of trophoblastic tissue. Hydatidiform moles develop in the first trimester. In approximately 2% of cases, they can present with signs of preeclampsia (proteinuria, hypertension, pitting edema), as in this patient. In all cases of complete moles, the uterus is too large for gestational age and human chorionic gonadotropin levels are higher than normal for gestational age. An ultrasound shows a snowstorm appearance (see Fig. 21-32 from Rapid Review Pathology, 3rd edition). The treatment is to remove all of the tissue from the uterus. Option B (Benign neoplasm of the chorionic villus with a triploid karyotype) is incorrect. A benign neoplasm with a triploid karyotype describes a partial hydatidiform mole, in which only some of the villi are neoplastic and an embryo usually is present. The neoplastic chorionic villi contain 69 chromosomes and have an XXY karyotype, which is produced when a haploid egg with a 23,X karyotype is fertilized by two spermatozoa, one having a 23,X and the other a 23,Y karyotype. Partial hydatidiform moles do not progress into a choriocarcinoma. Option C (Malignancy associated with diethylstilbestrol (DES) exposure) is incorrect. A daughter of a woman who received DES while pregnant is predisposed to clear cell adenocarcinoma involving the upper third of the vagina and cervix. Option D (Malignancy derived from trophoblastic tissue) is incorrect. A choriocarcinoma is a malignancy derived from syncytiotrophoblastic and cytotrophoblastic tissue. Unlike hydatidiform moles, choriocarcinomas do not contain chorionic villi. This woman would be at risk for developing a choriocarcinoma if all of the molar material is not removed from the uterus. Option E (Malignancy originating from striated muscle) is incorrect. A malignancy originating from striated muscle describes an embryonal rhabdomyosarcoma. It presents as a grapelike, necrotic mass protruding from the vagina of a girl younger than 2 years of age.

A 52-year-old man complains of weight loss, a dragging sensation in the right upper quadrant, and constipation and diarrhea. The photograph shows a partial resection of the sigmoid colon. Which of the following is the greatest risk factor for developing the colon lesion?

A. Dysplastic tubular adenoma B. Hamartomatous colonic polyp C. Human papillomavirus D. Inflammatory bowel disease E. Sigmoid diverticulosis
Option A (Dysplastic tubular adenoma) is correct. The photograph shows an annular and ulcerating growth that is causing a constriction in the sigmoid colon. The patients history of weight loss and change in stool habits are compatible with a colorectal carcinoma. The dragging sensation in the right upper quadrant suggests liver metastasis. Tubular adenomas (adenomatous polyps; see Figs. 8-1A and 17-37 in Rapid Review Pathology 3rd edition) are the most common premalignant polyps in the colon. Risks for malignancy include size > 2 cm and the presence of multiple polyps. Option B (Hamartomatous colonic polyp) is incorrect. Hamartomatous polyps (e.g., hyperplastic polyp, juvenile polyp) are non-neoplastic; therefore, their risk of becoming malignant is extremely low. Option C (Human papillomavirus) is incorrect. Anal squamous cell carcinomas are associated with the human papillomavirus types 16 and 18, especially in men who engage in unprotected anal intercourse. Option D (Inflammatory bowel disease) is incorrect. Ulcerative colitis and Crohns disease are both risk factors for colorectal cancer; however, the incidence of cancer is much less than for tubular adenomas. Option E (Sigmoid diverticulosis) is incorrect. Sigmoid diverticulosis is not a risk factor for colorectal cancer.

A 42-year-old woman has a painless mass on the right side of the lower neck. She has a 10-year history of smoking cigarettes. The photograph shows a histologic section of the lymph node, which was completely replaced by the tumor. Which of the following is the most likely primary site for this metastatic tumor?

A. Larynx B. Lung C. Nasopharynx D. Oropharynx E. Thyroid


Option E (Thyroid) is correct. The photograph shows a papillary lesion with numerous blue-stained psammoma bodies (calcified bodies) in the connective tissue stroma. The patient has metastatic papillary adenocarcinoma of the thyroid. Papillary cancers are the most common primary cancer of the thyroid and usually metastasize to subjacent cervical lymph nodes. There is no increased risk of this type of cancer in individuals who smoke cigarettes. However, there is an increased risk for papillary cancer if there is exposure to ionized radiation. Option A (Larynx) is incorrect. Cancers in the larynx are usually squamous cell carcinomas. Smoking is the most common cause of squamous cell carcinoma in this site. Option B (Lung) is incorrect. Primary adenocarcinomas of the lung do not have psammoma bodies and are more often mucin-producing, rather than papillary tumors. Option C (Nasopharynx) is incorrect. Nasopharyngeal tumors are either squamous cell or undifferentiated carcinomas (not papillary cancers with psammoma bodies). The Epstein-Barr virus has been implicated in their carcinogenesis. Option D (Oropharynx) is incorrect. Cancers in the oropharynx are usually squamous cell carcinomas. Smoking is the most common cause of squamous cell carcinoma in this site.

A 25-year-old woman is a type 1 diabetic in ketoacidosis. She has yellow papular lesions on her buttocks and trunk. After refrigeration of a turbid blood sample, there is a turbid supranate and infranate. Which of the following best explains the mechanism for her hyperlipoproteinemia?

A. Absent low density lipoprotein (LDL) receptors B. Decreased activation and release of capillary lipoprotein lipase (CPL) C. Deficiency of apolipoprotein (Apo) E D. Increased synthesis of very low density lipoprotein (VLDL)
Option B (Decreased activation and release of capillary lipoprotein lipase (CPL)) is correct. The lack of insulin in diabetic ketoacidosis decreases activation and release of CPL, which is required to hydrolyze chylomicrons from the diet into a chylomicron remnant and VLDL into intermediate density lipoprotein (IDL) and LDL. Both chylomicrons and VLDL markedly increase in the blood producing turbidity. After refrigeration, chylomicrons floats the surface of plasma forming a turbid supranate (lowest density of all lipoproteins) and VLDL produces a turbid infranate (more density than chylomicrons). This is called a type V hyperlipoproteinemia. With such high triglyceride levels, eruptive xanthomas develop all over the body (yellow papular lesions; see Fig. 9-2D in Rapid Review Pathology, 3rd edition). Other complications include lipemia retinalis (turbid retinal vessels) and acute pancreatitis. Option A (Absent low density lipoprotein (LDL) receptors) is incorrect. Absence of LDL receptors produces an increase in serum LDL (>190 mg/dL) and serum cholesterol (>260 mg/dL). This is called a type II hyperlipoproteinemia. Hypercholesterolemia does not produce plasma turbidity. Patients with familial hypercholesterolemia, an autosomal dominant disorder cause of type II hyperlipoproteinemia, commonly have Achilles tendon xanthomas, which are pathognomonic for the disease. Option C (Deficiency of apolipoprotein (Apo) E) is incorrect. Deficiency of Apo E is associated with familial dysbetalipoproteinemia, or remnant disease, which has an autosomal recessive inheritance pattern. It produces a type III hyperlipoproteinemia. In normal conditions, Apo E receptors in the liver remove some of the IDL remnants and chylomicrons from the blood. However, if there is a deficiency of Apo E leads, the IDL and chylomicron remnants markedly increase in the blood to produce palmar xanthomas located in the flexor creases of the hands (see Fig. 9-2C in Rapid Review Pathology, 3rd edition). Both serum cholesterol (CH) and thyroglobulin (TG) > 300 mg/dL. Option D (Increased synthesis of very low density lipoprotein (VLDL)) is incorrect. Increased synthesis of VLDL by the liver produces hypertriglyceridemia (>300 mg/dL), which is called a type IV hyperlipoproteinemia. It produces a turbid infranate but no supranate. Markedly increases levels of triglyceride can produce eruptive xanthomas similar to those discussed in type V hyperlipoproteinemia.

A 35-year-old woman complains of colicky abdominal pain and vomiting. She has had previous surgery for endometriosis involving the small bowel. Physical exam reveals abdominal distention, tympany to percussion, absence of rebound tenderness, and high-pitched tinkling sounds on auscultation. The photograph shows a plain abdominal radiograph that is taken with the patient in an erect position. Which of the following is the most likely cause of the abdominal pain?

A. Direct inguinal hernia B. Intussusception C. Large bowel infarction D. Small bowel adhesions E. Volvulus
Option D (Small bowel adhesions) is correct. The patients history of colicky pain (pain followed by a pain-free interval) is characteristic of a small bowel obstruction. The history of previous abdominal surgery for endometriosis involving small bowel most likely produced adhesions causing small bowel obstruction. Adhesions from previous surgery are the most common cause of bowel obstruction. The physical findings of abdominal distention, tympany to percussion, absences of rebound tenderness, and high-pitched tinkling sounds on auscultation are classic findings of bowel obstruction. The photograph shows the classic radiograph of small bowel obstruction, mainly multiple air-fluid levels with a stepladder configuration, and absence of air distal to the obstruction. Option A (Direct inguinal hernia) is incorrect. Direct inguinal hernias produce a bulge in the middle of the triangle of Hesselbach, which is located above the inguinal ligament. The bulge appears when the patient is standing and disappears when the patient is lying down. This type of hernia is not associated with entrapment of bowel leading to small bowel obstruction. Option B (Intussusception) is incorrect. An intussusception, or telescoping of a portion of bowel into another portion of bowel, is uncommon in adults. The nidus for intussusception in adults usually is a polyp or cancer. Obstruction and ischemic damage with bloody diarrhea usually are present. Option C (Large bowel infarction) is incorrect. Large bowel infarctions cause bloody diarrhea and localized, noncolicky abdominal pain. Atherosclerosis or embolism to the superior mesenteric artery is the most common causes of a large bowel infarction.

Option E (Volvulus) is incorrect. Volvulus occurs when bowel (sigmoid colon or cecum) twists around the mesenteric root, resulting in obstruction and strangulation. The affected bowel is distended and visible on a plain abdominal radiograph.

The photograph shows characteristic leukocyte alterations in peripheral blood obtained from a 23-year-old man with fever. Which of the following conditions is the most compatible with these changes?

A. Acute myeloblastic leukemia B. Infectious mononucleosis C. Invasive helminth infection D. Streptococcal tonsillitis E. Viral gastroenteritis
Option D (Streptococcal tonsillitis) is correct. The peripheral blood smear shows leukocyte alterations that characterize acute bacterial infections, such as streptococcal tonsillitis. These changes include the presence of band neutrophils (without segmentation) and of multilobed neutrophils that show toxic granulation (prominent azurophilic granules) and cytoplasmic vacuolization (phagolysosomes). When band neutrophils exceed 10% of the total leukocytes, the term left shift is used, indicating that more immature cells in the neutrophil series are being released from the bone marrow. These cells are released from the post-mitotic neutrophil pool in the bone marrow due to the presence of interleukin-1 and tumor necrosis factor-. Option A (Acute myeloblastic leukemia) is incorrect. A peripheral blood smear from a patient with acute myeloblastic leukemia would show myeloblasts containing rodlike structures (Auer rods) in the cytoplasm and other immature myeloid elements. Option B (Infectious mononucleosis) is incorrect. A peripheral blood smear from a patient with infectious mononucleosis would show atypical lymphocytes, which have abundant cytoplasm and enlarged nuclei with prominent

nucleoli. Option C (Invasive helminth infection) is incorrect. A peripheral blood smear from a patient with invasive helminths (e.g., Strongyloides stercoralis) would show eosinophils, which contain bright red granules in the cytoplasm that are larger than the azurophilic granules seen in neutrophils. Option E (Viral gastroenteritis) is incorrect. A peripheral blood smear from a patient with viral gastroenteritis would show absolute lymphocytosis (increased total lymphocyte count), the leukocyte alteration that characterizes viral infections. Lymphocytes have round nuclei and are usually surrounded by a thin rim of cytoplasm.

A 48-year-old man is admitted to the hospital for an acute myocardial infarction (MI). Four days later, he suddenly becomes short of breath. Physical examination shows a pansystolic murmur at the apex that does not increase in intensity with deep, held inspiration. Bibasilar inspiratory crackles are heard in both lung bases. There is no neck vein distention. Laboratory studies show absence of serum creatine kinase MB (CK-MB) and increased serum troponin-I (cTnI) and troponin-T (cTnT). Which of the following complications has most likely occurred in this patient?

A. Anterior wall rupture B. Interventricular septum rupture C. Posterior wall rupture D. Posteromedial papillary muscle rupture E. Rupture of a ventricular aneurysm
Option D (Posteromedial papillary muscle rupture) is correct. The posteromedial papillary muscle in the left ventricle is supplied by the right coronary artery (RCA) (see Fig. 10-5, Rapid Review Pathology, 3rd edition). Rupture of this muscle causes acute mitral regurgitation (pansystolic murmur at the apex that does not increase in intensity with deep, held inspiration), which leads to left-sided heart failure (pulmonary edema with bibasilar inspiratory crackles). Serum CK-MB peaks in 24 hours and returns to normal within 72 hours; therefore, it is not expected to be increased on day 4. Both cTnI and cTnT peak in 24 hours and return to normal within 7 to 10 days, which is an expected finding. Option A (Anterior wall rupture) is incorrect. The entire anterior portion of the heart and the anterior two-thirds of the interventricular septum is supplied by the left anterior descending (LAD) coronary artery. Rupture of the anterior wall causes cardiac tamponade and rapid death. There are no murmurs with this type of rupture. Option B (Interventricular septum rupture) is incorrect. The entire anterior portion of the heart and the anterior twothirds of the interventricular septum are supplied by the LAD coronary artery. Rupture of the interventricular septum causes a systolic murmur with a left-to-right shunt, resulting in right-sided heart failure. Signs of right-sided heart failure are not present in this patient (e.g., neck vein distention, dependent pitting edema). Option C (Posterior wall rupture) is incorrect. The entire posterior portion of the left ventricle, the posterior one-third

of the interventricular septum, and the right ventricle are supplied by the RCA. Rupture of the posterior wall of the heart is uncommon and causes cardiac tamponade and rapid death. It is not associated with heart murmurs. Option E (Rupture of a ventricular aneurysm) is incorrect. Ventricular aneurysms occur 4 to 8 weeks after a myocardial infarction. They produce a precordial bulge that correlates with systole (not present in this patient). The aneurysms rarely rupture because their walls are composed of scar tissue. Congestive heart failure is the most common complication.

A 22-year-old medical student only develops jaundice whenever he stays up all night studying for an exam. His physical examination is normal. All hepatitis serology tests are negative. Which of the following serum and urine laboratory test abnormalities would you expect? % CB Urine BilirubinUrine UBGAST ALT ALP GGT Normal Negative NormalNormalNormalNormal A. <20 Negative NormalNormalNormal B. >50 Negative C. 2050 D. 2050 E. <20 Negative Normal NormalNormalNormalNormal Number of arrows relates to the degree of elevation. ALP, alkaline phosphatase; ALT, alanine aminotransferase; AST, aspartate aminotransferase; CB, conjugated bilirubin; GGT, -glutamyltransferase; UBG, urobilinogen.

A. A B. B C. C D. D E. E
Option E is correct. The patient has Gilberts disease, which is an autosomal dominant disease characterized by decreased uptake and conjugation of bilirubin particularly exacerbated by the fasting state. Baseline unconjugated bilirubin (UCB) levels are increased to over twice normal in the fasting state leading to visible evidence of jaundice (CB < 20%). Next to viral hepatitis, it is the second overall most common cause of jaundice in the United States. All serum enzyme and urine findings are normal. It has no clinical significance. Option A is incorrect. The laboratory studies are most consistent with extravascular hemolytic anemias (e.g., hereditary spherocytosis, antibodies blood group hemolytic disease of the newborn), where RBCs are removed extravascularly by macrophages leading to an increase in UCB and a CB < 20%. AST is present in RBCs; therefore, there is a slight increase

in serum AST. Urine UBG is increased due to increased production of CB by the liver and increased conversion of CB to UBG in the colon. Fecal UBG gives the color to stool. Normally, a small portion of UBG in the colon is reabsorbed back into the blood (enterohepatic circulation) where most is taken up by the liver and the remainder is filtered into the urine to give the color to urine. Therefore, if there is more UBG in the stool (e.g., extravascular hemolytic anemia), then proportionately more UBG ends up in the urine (darker yellow than normal). Option B is incorrect. The laboratory findings indicate an obstructive type of jaundice, either due to intrahepatic cholestasis (e.g., drug-induced oral contraceptives) or extrahepatic cholestasis. Since bile is predominantly CB, in obstructive jaundice this is the predominant fraction that enters the bloodstream (CB > 50%) through ruptured intrahepatic bile ductules. CB is water soluble; therefore, it enters the urine to produce a positive dipstick test for bilirubin. Since UBG normally present in the urine derives from enterohepatic circulation of UBG initially produced in the liver, absence of UBG in the stool in obstructive liver disease (light-colored stools) leads to an absence of UBG in the urine. Enzyme markers for obstructive liver disease are serum ALP and serum GGT. Serum AST and ALT are also increased but to a lesser degree. Option C is incorrect. The laboratory findings are compatible with alcoholic hepatitis, which produces a mixed jaundice (CB 2050%), due to decreased uptake and conjugation of UCB and destruction of bile ductules among liver cells leading to an increase in CB in the blood. Urine UBG is increased, because UBG that is normally recycled back to the liver is redirected into the urine. Serum transaminases are increased with serum AST (mitochondrial enzyme) > ALT (cytosol enzyme). Since AST is normally located in the mitochondria of hepatocytes, alcohol, a mitochondrial toxin, causes preferential release of AST into the blood and is higher that ALT. Serum ALP is slightly increased; however, serum GGT is markedly increased, because alcohol induces enzyme synthesis in the hepatocyte cytochrome P450 system in the smooth endoplasmic reticulum (SER). GGT is also located in the SER and is increased as well. Therefore, key markers for alcoholic hepatitis are AST higher than ALT and marked increase in GGT. Option D is incorrect. The laboratory findings are most consistent with hepatitis in which there is diffuse necrosis of hepatocytes (e.g., hepatitis A). Hepatitis produces a mixed jaundice (CB 2050%), due to decreased uptake and conjugation of UCB and destruction of bile ductules among liver cells leading to an increase in CB in the blood. Urine UBG is increased, because UBG that is normally recycled back to the liver is redirected into the urine. Urine bilirubin is increased, because CB is water soluble. Serum transaminases are markedly increased because of massive liver cell necrosis; serum ALT is greater than AST. Serum ALP and GGT are only slightly increased.

A 48-year-old man with a 20-year history of alcohol abuse complains of tiredness and inability to remember time and dates. Physical examination reveals a nodular liver, splenomegaly, spider angiomas on the face and chest, and bilateral gynecomastia. The photograph is a liver biopsy with a special stain. Which of the following laboratory tests results would likely be present in this patient?

A. Increased serum ammonia B. Increased serum bicarbonate C. Increased serum blood urea nitrogen (BUN) D. Increased serum calcium E. Increased serum glucose F. Increased serum potassium G. Increased serum sodium
Option A (Increased serum ammonia) is correct. The patient has alcoholic cirrhosis. The microscopic section shows regenerative nodules (red) surrounded by dense bands of fibrous tissue (blue). Ammonia derived from normal amino acid metabolism or from conversion of amine groups in amino acids to ammonia by urease producing colonic bacteria is normally metabolized by the urea cycle in the liver. In cirrhosis, the urea cycle is dysfunctional; hence, ammonia is not converted into urea and increases in the blood. It is most likely responsible for the tiredness and mental status abnormalities in the patient, who is in an early stage of hepatic encephalopathy. Option B (Increased serum bicarbonate) is incorrect. In cirrhosis, there is an accumulation of lactic acid, because the hepatocytes can no longer convert lactate to pyruvate. Lactic acidosis, an example of increased anion gap metabolic acidosis, is associated with a decrease in serum bicarbonate, not an increased bicarbonate, which is the definition of metabolic alkalosis. Option C (Increased serum blood urea nitrogen (BUN)) is incorrect. In cirrhosis, the urea cycle is dysfunctional; hence, urea synthesis is decreased. Option D (Increased serum calcium) is incorrect. Decreased liver synthesis of albumin automatically results in a decrease in the bound fraction of calcium and a decreased total serum calcium. Furthermore, vitamin D deficiency occurs because of decreased liver 25-hydroxylation of vitamin D. Hypovitaminosis D leads to decreased gastrointestinal and renal reabsorption of calcium and hypocalcemia, not hypercalcemia. Option E (Increased serum glucose) is incorrect. Hypoglycemia is more likely to occur in cirrhosis than hyperglycemia.

Due to liver dysfunction in cirrhosis, glycogen stores are depleted and there is defective gluconeogenesis, both of which lead to hypoglycemia in the fasting state. Option F (Increased serum potassium) is incorrect. In cirrhosis, aldosterone is increased for two reasons: decreased metabolism of aldosterone by the liver and decreased cardiac output activating the renin-angiotensin-aldosterone system. Secondary aldosteronism increases renal exchange of Na+ for K+ leading to hypokalemia (not hyperkalemia). Option G (Increased serum sodium) is incorrect. In cirrhosis, there is a decreased cardiac output, causing the kidneys to reabsorb a slightly hypotonic fluid: serum Na+ = total body Na+/ total body water.

A newborn child coughs during and after breast feeding. The stomach is distended and tympanitic. The mother had polyhydramnios during pregnancy. Which of the following is the most likely diagnosis?

A. Choanal atresia B. Congenital pyloric stenosis C. Duodenal atresia D. Esophageal web E. Tracheoesophageal fistula

Option E (Tracheoesophageal fistula) is correct. In a tracheoesophageal fistula, the proximal esophagus ends blindly. The distal esophagus arises from the trachea, causing the stomach to distend with air. When the infant breast-feeds, milk refluxes into the trachea, causing coughing due to aspiration of milk into the lungs. Polyhydramnios occurs during pregnancy, because the fetus cannot swallow the amniotic fluid and reabsorb it in the duodenum. Option A (Choanal atresia) is incorrect. Choanal atresia is caused by a bony septum between the nose and the pharynx, which forces the infant to breathe only through the mouth. The cyanosis that develops when the infant is breastfeeding ceases when the infant breaks from the breast and begins crying. Gastric distention or polyhydramnios is not associated with choanal atresia. Option B (Congenital pyloric stenosis) is incorrect. Hypertrophy of the pylorus does not present at birth but at approximately 2 to 4 weeks of life. Projectile vomiting of nonbile-stained fluid occurs. No polyhydramnios or cough during or after breast feeding is associated with congenital pyloric stenosis. Option C (Duodenal atresia) is incorrect. Duodenal atresia (lack of a lumen) occurs just distal to the entry of the common bile duct into the duodenum. Projectile vomiting of bile-stained fluid occurs at birth. Air is present in the stomach and in the duodenum proximal to the opening of the common bile duct. Polyhydramnios occurs during pregnancy, because there is not enough duodenum to reabsorb the amniotic fluid. There is an increased association

with Down syndrome. Option D (Esophageal web) is incorrect. Esophageal webs are uncommon in newborns. They produce dysphagia for solids but not liquids. No polyhydramnios or cough during or after breast feeding is associated with esophageal webs.

Which of the following laboratory test results is expected in a 35-year-old man with AIDS? A. Abnormal phytohemagglutinin assay B. Absolute lymphocytosis C. Hypergammaglobulinemia D. Increase in CD4 helper T-cell/CD8 suppressor T-cell ratio E. Normal skin reaction to intradermal injection of Candida
Option C (Hypergammaglobulinemia) is correct. AIDS is the most common acquired immunodeficiency in the United States. It is caused by human immunodeficiency virus-1 (HIV-1), which is an RNA retrovirus containing reverse transcriptase. The virus infects CD4 T cells and destroys the cells. Epstein-Barr virus and cytomegalovirus infections are common in AIDS. They are potent polyclonal stimulators of B cells, which produce a polyclonal gammopathy (benign increase in -globulins). However, because patients are unable to mount an antibody response to a new antigen, they are still susceptible to bacterial infections in spite of the increase in -globulins. Option A (Abnormal phytohemagglutinin assay) is incorrect. The virus attacks and destroys CD4 helper T cells, hence impairing cell-mediated immunity (type IV hypersensitivity). Therefore, in vitro stimulation of T-cell response to phytohemagglutinin, a potent T-cell mitogen, is impaired (not normal) in HIV. Option B (Absolute lymphocytosis) is incorrect. Since T cells account for 60% to 70% of the total lymphocyte count, destruction of CD4 helper T cells results in a decreased lymphocyte count (not increased). Option D (Increase in CD4 helper T-cell/CD8 suppressor T-cell ratio) is incorrect. The virus attacks and destroys CD4 helper T cells; therefore, decreasing the CD4 helper T-cell count (<200 cells/mm3) and reversing the CD4 helper Tcell/CD8 suppressor T-cell ratio from a normal of 2:1 to less than 1:2. Option E (Normal skin reaction to intradermal injection of Candida) is incorrect. Cellular immunity is impaired in AIDS. Therefore, intradermal injections of Candida does not elicit the expected T-cell immune response. This lack of immune response is called anergy

The arrow in the photograph shows one of many arterioles in the kidneys of a 55-year-old man with proteinuria > 3.5 g/24 hours, and fatty casts in the urine. Which of the following renal disorders is the most likely cause of this vascular abnormality?

A. Acute glomerulonephritis B. Acute pyelonephritis C. Analgesic nephropathy D. Diabetic glomerulopathy E. Renovascular hypertension
Option D (Diabetic glomerulopathy) is correct. The arrow points to homogeneous, pink-staining proteinaceous material in the wall of an arteriole (hyaline arteriolosclerosis) causing narrowing of the vessel lumen. Diabetic glomerulopathy, which is often associated with proteinuria in the nephrotic range (>3.5 g/24 hrs), is the most likely cause of this finding. Arteriolar and glomerular disease in poorly controlled diabetes is due to nonenzymatic glycosylation (NEG). NEG occurs when glucose attaches to amino acids in the basement membrane of arterioles/capillaries leading to the production of advanced glycosylation products that render the membrane permeable to plasma proteins. The proteins diffuse into the wall of the vessel, narrowing the lumen causing ischemic damage to the tissue. Both the afferent and efferent arterioles are hyalinized in diabetic glomerulopathy. Hypertension can also produce hyaline arteriolosclerosis by forcing plasma proteins through the basement membrane via the increased intraluminal pressure. Option A (Acute glomerulonephritis) is incorrect. Acute glomerulonephritis is associated with inflammation involving glomerular structures (i.e., capillaries, epithelial cells, mesangium). It is associated with hematuria, RBC casts, and mild to moderate proteinuria that is not in the nephrotic range. Option B (Acute pyelonephritis) is incorrect. Acute pyelonephritis is associated with microabscesses in the renal tubules and interstitium. Neutrophils and WBC casts are present in the urine. Vascular abnormalities and proteinuria in the nephrotic range are not features of acute pyelonephritis. Option C (Analgesic nephropathy) is incorrect. Analgesic nephropathy is associated with chronic ingestion of aspirin and acetaminophen. Tubular and interstitial damage results from a combination of ischemia and free radical injury of the tubule cells. Ischemia is due to aspirin inhibition of renal prostaglandin synthesis, which normally produces vasodilation of the afferent arterioles. Free radical injury is due to liver conversion of acetaminophen into a drug free radical. Analgesic nephropathy may cause renal papillary necrosis, but does not produce hyaline arteriolosclerosis or the nephrotic syndrome. Option E (Renovascular hypertension) is incorrect. Renovascular hypertension in older men is most often due to

atherosclerosis of the renal arteries causing decreased renal blood flow and activation of the renin-angiotensinaldosterone system. It is not associated with hyaline arteriolosclerosis or the nephrotic syndrome.

A 1-year-old child has mental retardation, chronic liver disease, and fasting hypoglycemia caused by a deficiency of galactose 1-phosphate uridyltransferase (GALT). Which of the following should be eliminated from the diet?

A. Dairy products B. Gluten C. Phenylalanine D. Table sugar E. Tyrosine


Option A (Dairy products) is correct. The patient has galactosemia, an autosomal recessive disease characterized by a deficiency of GALT. Galactose derives from the metabolism of lactose. Lactase converts lactose to glucose and galactose. Galactose is converted by galactase to galactose 1-phosphate, which is converted to glucose 1-phosphate by GALT. Glucose 1-phosphate is converted to glucose 6-phosphate, which is used as a substrate for gluconeogenesis (deficiency causes fasting hypoglycemia). Deficiency of GALT causes accumulation of galactose 1-phosphate, which is toxic to the brain (mental retardation) and to the liver (cirrhosis). Dairy products must be eliminated from the diet. Option B (Gluten) is incorrect. Gluten is present in wheat products. Patients who have antibodies against gliadin (alcohol extract of gluten) develop celiac disease, which is characterized by immunologic destruction of the villi in the small intestine, causing malabsorption of fat, carbohydrates, and proteins. Option C (Phenylalanine) is incorrect. Phenylalanine is increased in phenylketonuria, an autosomal recessive disease characterized by a deficiency of phenylalanine hydroxylase, which converts phenylalanine to tyrosine. Phenylalaninecontaining foods should be eliminated (e.g., certain sweeteners) from the diet to prevent mental retardation. Option D (Table sugar) is incorrect. Table sugar, which contains sucrose, is converted to glucose and fructose by sucrase. It should be avoided in hereditary fructose intolerance, an autosomal recessive disorder, that is characterized by a deficiency of aldolase B. Clinical findings are similar to those of galactosemia. Option E (Tyrosine) is incorrect. Tyrosine-containing foods should be eliminated from the diet in patients with tyrosinosis, an autosomal recessive disease caused by a deficiency of fumarylacetoacetate hydrolase. Excess tyrosine causes chronic liver disease, which may lead to hepatocellular carcinoma. Tyrosine should be added to the diet in phenylketonuria.

A 15-year-old boy, who recently recovered from scarlet fever, develops hypertension, oliguria, and periorbital edema. His urine is smoke colored. The photograph shows an electron micrograph of a representative glomerulus from a kidney biopsy. Which of the following casts is most likely present in the urine?

A. Fatty casts B. Hyaline casts C. RBC casts D. Renal tubular cell casts E. Waxy casts
Option C (RBC casts) is correct. The patient has poststreptococcal glomerulonephritis, secondary to scarlet fever, which is caused by an erythrogenic strain of group A streptococcus. The electron micrograph shows a glomerulus with two RBCs in the capillary lumen. Multiple electron-dense deposits (immunocomplexes; type III hypersensitivity) are present directly beneath the visceral epithelial cells (subepithelial location). Poststreptococcal glomerulonephritis presents as a nephritic syndrome with mild to moderate proteinuria, hematuria, and RBC casts (see Fig. 19-2E in Rapid Review Pathology, 3rd edition). Option A (Fatty casts) is incorrect. Fatty casts are the most common casts in the nephrotic syndrome (heavy proteinuria with pitting edema). They are not present in the nephritic syndrome. Option B (Hyaline casts) is incorrect. Hyaline casts are acellular casts that occur in any proteinuric state. In the absence of proteinuria, they have no clinical significance. They do not define any particular type of glomerular disease. Option D (Renal tubular cell casts) is incorrect. Renal tubular casts are present in acute tubular necrosis, the most common cause of acute renal failure. Renal failure rarely occurs in poststreptococcal glomerulonephritis. Option E (Waxy casts) is incorrect. Waxy casts are present in chronic renal failure, which rarely occurs in poststreptococcal glomerulonephritis.

An afebrile 22-year-old woman, a college student, develops intermittent complaints of crampy left and right lower quadrant abdominal pain and distention associated with alternating periods of diarrhea and constipation. She states that having a bowel movement relieves the pain. The stool guaiac is negative; small bowel endoscopy and colonoscopy are normal. What is the most likely diagnosis?

A. Celiac disease B. Inflammatory bowel disease C. Irritable bowel syndrome (IBS) D. Laxative abuse E. Volvulus
Option C (Irritable bowel syndrome (IBS)) is correct. IBS is an intrinsic colonic motility disorder that may be due to possible loss of tolerance to gastrointestinal flora, possible genetic factors, or various environmental triggers (e.g., stress, certain foods). It is the most common functional bowel disorder and occurs more often in women than in men. Risk factors include history of childhood sexual abuse, domestic abuse in women, increased stress, depression, and personality disorders. It is characterized by alternating bouts of diarrhea and constipation. Abdominal pain and bloating are relieved by defecation. Stools are often coated by mucus. There is abnormal defecation (e.g., straining, sense of incomplete evacuation). Flexible sigmoidoscopy and colonoscopy are normal. Nonpharmacologic treatment is adequate fiber intake and elimination of foods that may aggravate the condition (e.g., coffee, fatty foods, dairy products). Pharmacologic treatment involves the use of antispasmodics-anticholinergics (e.g., dicyclomine); loperamide (serotonin type 3 receptor antagonist) if diarrhea is present. Lubiprostone (chloride channel activator) is effective if constipation is the primary symptom. Option A (Celiac disease) is incorrect. Celiac disease is an inappropriate immune response to gluten in wheat products as well as related proteins in rye and barley. Antibody destruction of villi in the small intestine produces malabsorption with increased fat in the stool (not present in this patient). Endoscopy of the small bowel shows flattening of the villi (endoscopy was normal in this patient). Option B (Inflammatory bowel disease) is incorrect. Ulcerative colitis and Crohns disease, the two most common inflammatory bowel diseases, are excluded by normal endoscopy studies of small and large bowel. Option D (Laxative abuse) is incorrect. Laxative abuse is a common problem. They produce a secretory type of diarrhea, unlike this patient who has alternating bouts of constipation and diarrhea. Phenanthracene types of laxatives often produce melanosis coli (bowel with decreased motility and black mucosa from phenanthracene pigment). Option E (Volvulus) is incorrect. A volvulus is where the bowel twists around mesenteric root producing obstruction and strangulation. The sigmoid colon is the most common site in elderly patients and the cecum in young patients. Risk factors include chronic constipation (most common), pregnancy, and laxative abuse.

A 69-year-old man has a dragging sensation in the right upper quadrant and crampy abdominal pain in the left lower quadrant. Over the past 6 months, he has experienced weight loss and alternating bouts of constipation and diarrhea. He has noticed blood coating and mixed in with stools when they are more solid. Physical examination shows an enlarged, nodular liver and external hemorrhoids. Rectal examination reveals blood mixed with stool. Laboratory studies show hemoglobin 9 g/dL, WBC count 6500/mm3, platelet count 500,000/mm3, mean corpuscular volume 75 (m3, and serum ferritin 3 ng/mL. Which of the following disorders best explains the presence of blood in the stool?

A. Angiodysplasia B. External hemorrhoids C. Hepatocellular carcinoma with metastasis to the colon D. Peptic ulcer disease E. Primary colorectal cancer
Option E (Primary colorectal cancer) is correct. The patient has primary colorectal cancer complicated by iron deficiency anemia (microcytic anemia and decreased serum ferritin) and metastasis to the liver (enlarged, nodular liver). Clinical findings highly predictive of colorectal cancer in this patient include weight loss, change in consistency of stools, and blood mixed in with stools. Option A (Angiodysplasia) is incorrect. Angiodysplasia commonly causes passage of large volumes of blood per rectum (hematochezia) that are capable of producing dizziness due to hypotension or hypovolemic shock. Angiodysplasia refers to the presence of dilated vascular channels in the submucosa of the cecum, which often rupture and produce massive lower gastrointestinal bleeding. The patient has blood mixed in with stool and signs of metastatic disease to the liver, which excludes angiodysplasia. Option B (External hemorrhoids) is incorrect. External hemorrhoids usually thrombose and do not commonly bleed. In addition, they would not explain the weight loss, liver findings, and change in bowel habits in the patient. Option C (Hepatocellular carcinoma with metastasis to the colon) is incorrect. Metastasis to the colon from any site is extremely rare and usually manifests as signs of obstruction rather than bleeding. Option D (Peptic ulcer disease) is incorrect. Peptic ulcer disease produces epigastric distress with no changes in bowel habits. Black tarry stools (melena) are more likely to occur than blood coating and mixing with stools.

A 45-year-old pig farmer complained of sudden jerking movements in his extremities and a constant headache. He has a 40-pack-year history of smoking cigarettes. Physical examination showed no focal neurologic deficits. Calcified nodular masses within the muscles of his extremities and chest were palpated. A complete blood cell count showed a marked peripheral blood eosinophilia. The photograph shows a section of brain removed at autopsy. Which of the following is the most likely diagnosis?

A. Cerebral infarctions B. Cysticercosis C. Echinococcosis D. Metastatic lung carcinoma E. Trichinosis


Option B (Cysticercosis) is correct. Cysticercosis is caused by the larval form of the pork tapeworm Taenia solium. The patient most likely contracted the infection from a family member who ate improperly cooked pork containing the larvae, which developed into adults that laid eggs (family member is the definitive host). The patient must have ingested food contaminated with eggs of the adult T. solium. The eggs develop into larvae (cysticerci) in the intestinal tract (patient is an intermediate host), and the larvae gain access to the systemic circulation, resulting in lesions in the muscle and brain and eosinophilia. The photograph shows multiple cysts located at the junction of the gray and white matter. The cysts contain the scolex of the immature worm. Cysts frequently undergo calcification and also cause focal or generalized seizures. Option A (Cerebral infarctions) is incorrect. A cerebral infarction is most often due to atherosclerosis involving the middle cerebral artery or internal carotid artery. The infarction usually is pale and occurs at the periphery of the brain. The brain undergoes liquefactive necrosis resulting in the formation of a cystic space. Option C (Echinococcosis) is incorrect. Echinococcosis is caused by the tapeworm Echinococcus granulosus or E. multilocularis, which infects sheep. It is transmitted to humans via contact with a dog that harbors the adult worms and eggs of the Echinococcus. The eggs are ingested by humans and develop into larvae that enter the liver, where they form cysts. Option D (Metastatic lung carcinoma) is incorrect. Metastatic foci usually are located at the junction of gray and white

matter and are more likely to show hemorrhage and necrosis. Option E (Trichinosis) is incorrect. Trichinosis is caused by the nematode Trichinella spiralis, which is present in its larval form in pigs (pigs are the intermediate host). Ingestion of improperly cooked pork results in the formation of adult worms in the human host (the human is the definitive host). The adult worms produce larvae that gain access to the systemic circulation where they encyst in skeletal muscle. The cysts frequently calcify and produce muscle pain. Although larvae can enter the central nervous system, this is extremely uncommon.

For the past 4 months, a 59-year-old man with a 30-year history of smoking cigarettes complained of persistent headaches. For several days, he had projectile vomiting and bilateral blurry vision. Physical examination showed bilateral papilledema. An MRI revealed a mass lesion involving the corpus callosum with spread to both cerebral hemispheres. The patient died a few days later of a transtentorial herniation. The photograph shows the gross appearance of the brain at autopsy. Which of the following underlying conditions is the most likely cause of the lesion?

A. Cerebral infarction B. Glioblastoma multiforme (GBM) C. Intracerebral hemorrhage D. Medulloblastoma E. Metastatic lung carcinoma
Option B (Glioblastoma multiforme (GBM)) is correct. The photograph shows a hemorrhagic and necrotic mass extending across the splenium of the corpus callosum and into the adjacent cerebral hemisphere bilaterally. The distribution of this lesion and the history of headache, projectile vomiting, and bilateral papilledema are signs and symptoms of a glioblastoma multiforme (GBM). This is the most common primary cancer of the brain in adults and arises de novo or from a preexisting low-grade astrocytoma. The tumors are located predominantly in the cerebral hemispheres and are classified as grade IV astrocytomas. There is an increased risk for developing a GBM in smokers.

Option A (Cerebral infarction) is incorrect. Cerebral infarctions are most commonly caused by a thrombus overlying an atheromatous plaque, which causes a pale infarction that extends to the periphery of the cerebral cortex. Option C (Intracerebral hemorrhage) is incorrect. Intracerebral hemorrhages usually are due to hypertension, which produces small-vessel aneurysms that result in rupture of the vessel and hemorrhage into the brain. The basal ganglia area is the most common site for the hemorrhage. Option D (Medulloblastoma) is incorrect. Medulloblastomas usually are located in the midline of the cerebellum in children. Option E (Metastatic lung carcinoma) is incorrect. Metastasis to the brain usually is multifocal and primarily occurs at the junction of the gray and white matter near the periphery of the brain.

The photograph shows a lung removed at autopsy from a 72-year-old woman, who died suddenly 5 days after a total hip replacement. A few hours before she died, she told her daughter that she felt like she was going to die. Which of the following is the most likely cause of death?

A. Acute pulmonary infarction B. Acute right-sided ventricular strain C. Aspiration of gastric contents D. Disseminated metastasis

E. Nosocomial pneumonia
Option B (Acute right-sided ventricular strain) is correct. The photograph shows a pulmonary artery thromboembolism (saddle embolus) with extension into all the main pulmonary artery trunks. The site of origin for an embolus of this size is usually one of the femoral veins in the lower extremity. A venous thrombus first developed in the deep veins of the leg and then continued to propagate and eventually extend into the femoral vein, where it dislodged and embolized to the lungs. Thromboembolism is common in the postoperative setting, particularly after a total hip replacement. Sudden occlusion of the main stem pulmonary arteries produces an acute increase in pulmonary artery pressure and acute right-sided ventricular strain, leading to sudden death (acute cor pulmonale). For unexplained reasons, there is frequently a premonition of impending death. Option A (Acute pulmonary infarction) is incorrect. The photograph does not show acute changes in the lung parenchyma, such as a hemorrhagic pulmonary infarction. A massive saddle embolus causes death so quickly, that there is not enough time for an infarction to occur. Option C (Aspiration of gastric contents) is incorrect. Aspiration of gastric contents is common postoperatively. It often precipitates acute respiratory distress syndrome, but does not cause sudden death. Option D (Disseminated metastasis) is incorrect. There are no gray-white lesions in the lung parenchyma to suggest either a primary lung cancer or metastatic lung disease as the cause of death. Option E (Nosocomial pneumonia) is incorrect. There are no patchy yellow-gray areas of alveolar consolidation in the lung parenchyma to suggest a hospital-acquired pneumonia as the cause of death.

A 75-year-old woman with a history of chronic ischemic heart disease and severe osteoarthritis had a total hip replacement 5 days ago. During the surgery, she received three units of packed RBCs. She currently complains of pain in her left calf. Physical exam reveals swelling of the left calf and pain on gentle dorsiflexion of the foot and compression of the calf. Which of the following is the greatest risk factor for her current problem?

A. Decreased cardiac output B. Decreased hemoglobin concentration C. Her advanced age D. Hypercoagulable state E. Immobilization

Option E (Immobilization) is correct. The patient has a deep vein thrombosis (DVT; swelling, pain with dorsiflexion of the foot, compression of the calf). The three main causes of intravascular thrombus formation are endothelial cell injury (e.g., cigarette smoking), stasis of blood flow (e.g., postsurgery), and

hypercoagulability (e.g., use of oral contraceptives, hereditary factor deficiencies). Stasis of blood flow is the most common cause of venous thrombus formation, which most often occurs in the deep veins below the knee. Stasis of blood causes endothelial cell injury and local activation of the coagulation system, resulting in the formation of an adherent, occlusive, firm, dark red fibrin clot that entraps RBCs, WBCs, and platelets. The clot propagates toward the heart and may break off once it reaches the femoral vein, which is the most common site of pulmonary thromboembolic disease. The treatment of DVT is low molecular weight heparin, compression stockings, and long-term treatment (3 to 6 months) to prevent recurrent DVTs with warfarin. Option A (Decreased cardiac output) is incorrect. Decreased cardiac output may occur in ischemic heart disease complicated by congestive heart failure, which causes stasis of venous blood, predisposing the patient to deep venous thrombosis. There is no indication that the patient is in heart failure at the present time. Option B (Decreased hemoglobin concentration) is incorrect. Decreased hemoglobin concentration decreases the viscosity of blood and reduces (not increases) the risk of thrombosis. Furthermore, the patient was transfused during surgery. Option C (Her advanced age) is incorrect. The incidence of venous clots does not increase with age. Option D (Hypercoagulable state) is incorrect. A hypercoagulable state, due to acquired causes (e.g., estrogen, antiphospholipid syndrome) or genetic causes (e.g., antithrombin III deficiency, protein C or S deficiency), do cause deep vein thrombosis; however, these are far less common causes of a DVT than immobilization.

An 18-year-old black woman with a history of dysfunctional uterine bleeding complains of fatigue when exercising. Laboratory studies show a mild microcytic anemia and increased RBC distribution width (RDW). The photograph shows a representative section of the peripheral blood smear. Which of the following investigative studies would be most useful in defining the type of anemia that is present?

A. Bone marrow aspiration biopsy B. Hemoglobin electrophoresis C. Osmotic fragility test

D. Serum ferritin test E. Sickle cell screening


Option D (Serum ferritin test) is correct. The patients clinical and laboratory findings are consistent with the diagnosis of an iron-deficiency anemia, which can be confirmed by the serum ferritin test. The peripheral blood smear shows RBCs with a notable increase in the central areas of pallor, which indicate decreased hemoglobin concentration. Strong evidence of iron deficiency is given by the increased RBC distribution width (RDW, increased size variation in RBCs) and the patients history of dysfunctional uterine bleeding (menorrhagia). The RDW is increased in iron deficiency, because there is a mixed population of normocytic and microcytic RBCs. The other causes of microcytic anemia (e.g., anemia of chronic disease, thalassemia) are not as likely to have this size variation in RBCs and have a normal RDW. The serum ferritin test is the best screening test for iron-related disorders, because serum ferritin levels correlate with the amount of iron stored in the bone marrow macrophages. Option A (Bone marrow aspiration biopsy) is incorrect. The patient has a microcytic anemia. A bone marrow aspiration biopsy is rarely indicated in workups for microcytic anemias, because serum tests (e.g., ferritin, hemoglobin electrophoresis) are available to identify the various causes of microcytic anemia. Option B (Hemoglobin electrophoresis) is incorrect. Hemoglobin electrophoresis identifies changes in the concentration of normal and abnormal forms of hemoglobin. Of the microcytic anemias, the only one that is likely to show an abnormal hemoglobin electrophoresis is -thalassemia where there is a decrease in hemoglobin A (2, 2) and a corresponding increase in hemoglobin A2 (2, 2) and hemoglobin F (2, 2). The patient does not have thalassemia, because the clinical history is more compatible with iron deficiency from menorrhagia and the RDW is increased (normal RDW in thalassemia). Option C (Osmotic fragility test) is incorrect. The osmotic fragility test is used to confirm a diagnosis of hereditary spherocytosis, in which the osmotic fragility of RBCs is increased. Hereditary spherocytosis is a normocytic (not microcytic) anemia. Spherocytes are not present in the peripheral blood in this patient. Option E (Sickle cell screening) is incorrect. Although the patient is black, sickle cell anemia is unlikely, because it is usually normocytic (not microcytic) and because sickle cells are not present in the peripheral blood.

A 29-year-old man is found unconscious under a bridge and is brought to the emergency department. No history is available on the patient, although it is suspected that he is a homeless individual. Initial laboratory studies show: Serum Bicarbonate 14 mEq/L Blood urea nitrogen (BUN) 40 mg/dL Chloride 90 mEq/L Creatinine 4 mg/dL Potassium 5.0 mEq/L Sodium 136 mEq/L Urine dipstick test: negative for ketone bodies and glucos Sediment: numerous calcium oxalate crystals

Which of the following is the most likely diagnosis? A. Diabetic ketoacidosis B. Ethanol intoxication C. Ethylene glycol poisoning D. Isopropyl alcohol poisoning E. Methyl alcohol poisoning

Option C (Ethylene glycol poisoning) is correct. Laboratory findings show an increased serum BUN and creatinine with a BUN:creatinine ratio of 10 indicating renal azotemia (uremia). Electrolytes show an increased anion gap metabolic acidosis. Anion gap = serum sodium (serum chloride + serum bicarbonate); 136 (90 + 14) = 32 mEq/L (normal, 12 2). The patient most likely drank antifreeze (ethylene glycol), which is converted to oxalic acid by alcohol dehydrogenase. Oxalic acid combines with calcium to form calcium oxalate crystals, which obstruct the kidney tubules and cause acute renal failure. In acute renal failure, the decreased glomerular filtration rate initially causes a proportionate increase in both serum BUN and serum creatinine. Normally, the ratio of serum BUN:creatinine is 15. However, in renal failure, creatinine is also lost in extrarenal sites (e.g., skin, gastrointestinal tract), causing the ratio to be <15, as in this case (e.g., serum BUN 40 mg/dL and serum creatinine 4 mg/dL; ratio 10). Option A (Diabetic ketoacidosis) is incorrect. The urine dipstick test is negative for ketone bodies and glucose; therefore, diabetic ketoacidosis is excluded. Option B (Ethanol intoxication) is incorrect. Ethanol does produce an increased anion gap metabolic acidosis due to an increase in lactic acid and -hydroxybutyric acid. The former is due to an increase in NADH in alcohol metabolism, which causes pyruvate to convert to lactic acid. The latter is due to liver conversion of acetyl CoA, the end-product of alcohol metabolism, to ketoacids in the liver. The specific ketoacid is hydroxybutyric acid, because NADH causes acetoacetate to convert to -hydroxybutyrate. Calcium oxalate crystals in the urine are not present in ethanol intoxication. Option D (Isopropyl alcohol poisoning) is incorrect. Isopropyl alcohol is rubbing alcohol. When ingested, it is converted to acetone. No acids are produced; therefore, metabolic acidosis does not occur. In addition, calcium oxalate crystals are not present in the urine. Option E (Methyl alcohol poisoning) is incorrect. Methyl alcohol is present in solvents such as window washing fluid. When ingested, it is converted to formic acid by alcohol dehydrogenase producing an increased anion gap metabolic acidosis. Formic acid damages the optic nerve, resulting in optic neuritis and possibly blindness. It does not produce acute renal failure or calcium oxalate crystals in the urine.

The photograph shows a lung removed at autopsy from a 70-year-old man, who was a roofer for 50 years. Which of the following is the most likely cause of the cancer involving the lung?

A. Chronic exposure to asbestos B. Chronic exposure to carbon monoxide C. Chronic exposure to coal dust D. Chronic exposure to polycyclic hydrocarbons E. Chronic exposure to silica dust
Option A (Chronic exposure to asbestos) is correct. This patient has a malignant mesothelioma, which is most frequently due to chronic exposure to asbestos. Roofing material during the life span of the patient used to have asbestos in it for insulation. Mesotheliomas have no etiologic relationship with smoking. They arise from the serosal cells lining the pleura and encase and locally invade the subpleural lung tissue, which is evident in the autopsy lung specimen. There is a long latent period of 25 to 45 years for the development of these tumors. Primary bronchogenic carcinomas of the lung are the most common asbestos-related tumors; however, they do not encase the lung like a mesothelioma. Option B (Chronic exposure to carbon monoxide) is incorrect. Carbon monoxide is the product of incomplete combustion of carbon containing compounds. Carbon monoxide is not a risk factor for any cancer. Option C (Chronic exposure to coal dust) is incorrect. Coal workers pneumoconiosis (inhalation of coal dust) is not a risk factor for malignant mesothelioma or any other type of cancer. Option D (Chronic exposure to polycyclic hydrocarbons) is incorrect. Polycyclic hydrocarbons are the primary carcinogens in cigarette smoke. They are not a risk factor for malignant mesothelioma.

Option E (Chronic exposure to silica dust) is incorrect. Silicosis is a pneumoconiosis characterized by inhalation of crystalline silica (quartz) dust. It is not a risk factor for malignant mesothelioma; however, it does increase the risk for developing a primary lung cancer.

A 55-year-old alcoholic, who also has chronic bronchitis secondary to cigarette smoking, is taking theophylline to improve breathing. His wife states that he is taking the medicine as prescribed and has not missed taking a single dose. A therapeutic drug level of serum theophylline is reported to be in a suboptimal range. A repeat test on a different sample shows the same result. Which of the following best explains this test result?

A. Decreased gastrointestinal reabsorption of the drug B. Decreased renal excretion of the drug C. Increased liver metabolism of the drug D. Instrument error E. Patient noncompliance
Option C (Increased liver metabolism of the drug) is correct. Alcohol induces the liver microsomal mixed-function oxidase system, which is located in the smooth endoplasmic reticulum (SER). This enzyme system is responsible for drug and chemical metabolism. Drugs like alcohol and chemicals like polycyclic aromatic hydrocarbons in cigarette smoke induce this system by enhancing the rate of synthesis of NADPH-cytochrome P450 reductase and cytochrome P450 oxidase, which are the two key enzymes in the system. The increase in enzyme activity increases the metabolism of drugs, in this case, theophylline, leading to a suboptimal concentration of the drug. Option A (Decreased gastrointestinal reabsorption of the drug) is incorrect. Although decreased drug reabsorption in the gastrointestinal tract is a potential cause of a suboptimal theophylline level, the most likely cause in this patient with a drinking and smoking history is increased metabolism of the drug in the liver. Option B (Decreased renal excretion of the drug) is incorrect. Decreased excretion in the kidneys would more likely produce drug toxicity rather than suboptimal levels of the drug. Option D (Instrument error) is incorrect. The test was repeated with a different patient sample, which eliminates instrument error as a possible cause. Option E (Patient noncompliance) is incorrect. Although patient noncompliance is the most common cause of a lack of an appropriate therapeutic response to a drug, the statement by the wife that the patient is taking the medicine as prescribed is sufficient to rule out noncompliance as a possibility.

A 23-year-old man is scuba diving in 60 feet of water and suddenly develops stabbing chest pain in the left side with dyspnea while coming slowly to the surface. Physical examination of the left lung shows hyper-resonance to percussion, elevation of the left diaphragm, absent tactile fremitus, and absent breath sounds. Which of the following is the most likely diagnosis?

A. Decompression sickness B. Pleural effusion C. Pulmonary infarction D. Spontaneous pneumothorax E. Tension pneumothorax

Option D (Spontaneous pneumothorax) is correct. A pulmonary complication of scuba diving is rupture of a preexisting intrapleural bleb or a subpleural bleb, causing a hole in the pleura and a spontaneous pneumothorax. A hole in the pleura causes a loss of negative pressure in the pleural cavity, which collapses all or part of the lung. Physical findings include hyper-resonance to percussion, tracheal deviation to the side of the collapse (if the whole lung is collapsed), elevation of the diaphragm, absent breath sounds, and absent vocal tactile fremitus. Option A (Decompression sickness) is incorrect. Decompression sickness (gas embolism) is a complication of scuba diving. As a diver descends, nitrogen gas under increased pressure moves from the alveoli and dissolves in tissue and blood. Rapid ascent forces nitrogen to come out of the tissue and blood in the form of bubbles, causing ischemic damage. A spontaneous pneumothorax is not a complication of gas embolism. Option B (Pleural effusion) is incorrect. Although a pulmonary thromboembolism leading to a pulmonary infarction and pleural effusion is a complication of scuba diving, it usually occurs when the diver is stationary and in deep water. Physical findings of a pleural effusion include dullness to percussion, deviation of the trachea to the contralateral side, and absent breath sounds. Option C (Pulmonary infarction) is incorrect. Although a pulmonary thromboembolism leading to a pulmonary infarction and pleural effusion is a complication of scuba diving, it usually occurs when the diver is stationary and in deep water. Clinical findings in a pulmonary infarction are pleuritic chest pain and dyspnea. Physical examination would not show hyper-resonance to percussion and elevation of the diaphragm. Option E (Tension pneumothorax) is incorrect. A tension pneumothorax is not a common cause of dyspnea in scuba diving. Unlike a spontaneous pneumothorax, a tension pneumothorax is associated with a flaplike pleural tear. Inspiration causes the flap to open and allow air to enter the pleural cavity. However, the flap closes on expiration and prevents the air from leaving the cavity. Increased intrapleural pressure (greater than the atmosphere) causes compression of the lung (atelectasis) and deviation of the trachea to the contralateral side. The diaphragm is depressed (not elevated).

The photograph shows a biopsy of a nonpruritic lesion located on the vulva of a sexually active 28-year-old woman. Which of the following best describes this lesion?

A. Association with vulvar lymphedema B. Koilocytic atypia C. Latent virus with recurrence D. Precursor of squamous cell cancer E. Progression to secondary syphilis
Option C (Latent virus with recurrence) is correct. The patient has herpes simplex virus type 2 (HSV-2), a sexually transmitted disease. The figure shows a multinucleated squamous cell with ground glass nuclei, which will eventually develop into more discrete eosinophilic inclusions. The virus remains latent in sacral sensory ganglia and recurs in the same location. Acyclovir decreases the number of recurrences but does not cure the infection. Option A (Association with vulvar lymphedema) is incorrect. A subtype of Chlamydia trachomatis causes lymphogranuloma venereum. This sexually transmitted disease is associated with papules, granulomatous microabscesses in inguinal nodes, and scarring of lymphatics, resulting in vulvar lymphedema. No specific nuclear changes are present in lymphogranuloma venereum. Option B (Koilocytic atypia) is incorrect. Human papillomavirus causes koilocytotic atypia in squamous cells. Infected cells have pyknotic (condensed chromatin) nuclei surrounded by a clear halo. Option D (Precursor of squamous cell cancer) is incorrect. HSV-2 is not an oncogenic virus Option E (Progression to secondary syphilis) is incorrect. Syphilis, due to Treponema pallidum, does not produce vesicular lesions. The chancre of primary syphilis is raised, ulcerated, and painless. If untreated, it may progress to secondary syphilis.

The photograph shows the pupillary light reflex in the right eye of a 1-year-old boy. Which of the following cancers may develop in this child in the second to third decades of life?

A. Embryonal rhabdomyosarcoma B. Neuroblastoma C. Osteogenic sarcoma D. Retinoblastoma E. Wilms' tumor


Option C (Osteogenic sarcoma) is correct. The child has a white pupillary reflex (leukokoria) due to a retinoblastoma, which is caused by inactivation of the RB (retinoblastoma) suppressor gene located on chromosome 13. In the sporadic type of cancer, both normal alleles must be inactivated by a point mutation after birth to produce a retinoblastoma (two-hit theory). In the autosomal dominant type of cancer, one allele already is inactivated in the germ cells and the other allele is normal. After birth, only a single mutation on the remaining allele is necessary to produce a retinoblastoma. Children with the autosomal dominant type of retinoblastoma have an increased incidence of osteogenic sarcoma associated with inactivation of the RB suppression gene. Option A (Embryonal rhabdomyosarcoma) is incorrect. Embryonal rhabdomyosarcoma (sarcoma botryoides) is the most common sarcoma in children and develops in the vagina of girls and the urogenital tract in boys. It is not associated with inactivation of the RB suppressor gene or a retinoblastoma. Option B (Neuroblastoma) is incorrect. Neuroblastomas are a common cancer found in children. They are due to amplification of the N-MYC proto-oncogene located on chromosome 1. They can metastasize to the orbit. Option D (Retinoblastoma) is incorrect. Retinoblastomas usually occur before the age of 4 years and would not be expected to occur in the second and third decades of life. Option E (Wilms' tumor) is incorrect. Wilms tumors are due to inactivation of the WT1 suppressor gene located on chromosome 11. It is not associated with inactivation of the RB suppressor gene or a retinoblastoma.

Physical examination of a 72-year-old man shows severe hypertension, an epigastric bruit, and diminished amplitude of the pedal pulses. An angiogram of the renal arteries is ordered (see photograph; the arrow points to the right renal artery). Which of the following best characterizes the findings in this patient?

A. Activation of the renin-angiotensin-aldosterone (RAA) system in the left kidney B. Aldosterone levels are decreased in the blood C. Plasma renin activity (PRA) is increased in the left renal vein D. Plasma renin activity (PRA) is increased in the right renal vein E. Renal artery stenosis is caused by fibromuscular hyperplasia
Option D (Plasma renin activity (PRA) is increased in the right renal vein) is correct. The patient has renovascular hypertension. In men older than 50 years of age, it is most often caused by an atherosclerotic plaque narrowing the orifice of the renal artery. The photograph shows an arrow pointing to proximal stenosis of the right renal artery with poststenotic dilation. A decrease in renal blood flow activates the RAA system, resulting in hypertension caused by renal retention of sodium by aldosterone and vasoconstriction of the peripheral resistance arterioles by angiotensin II. PRA is increased in the right renal vein and should be decreased in the left renal vein. This is due to an increase in plasma volume related to increased retention of sodium by aldosterone. An increase in plasma volume increases renal blood flow to the uninvolved kidney leading to suppression of the RAA system. Narrowing of the orifice of the renal artery produces an epigastric bruit, causing atrophy of the affected kidney. Option A (Activation of the renin-angiotensin-aldosterone (RAA) system in the left kidney) is incorrect. The RAA system is suppressed in the left kidney because there is an increase in plasma volume related to increased retention of sodium by aldosterone. An increase in plasma volume increases renal blood flow to the uninvolved kidney leading to suppression of the RAA system. Option B (Aldosterone levels are decreased in the blood) is incorrect. Aldosterone levels are increased due to activation of the RAA system in the right kidney. Option C (Plasma renin activity (PRA) is increased in the left renal vein) is incorrect. PRA is suppressed in the left kidney due to the increase in renal blood flow to the left renal artery. Option E (Renal artery stenosis is caused by fibromuscular hyperplasia) is incorrect. Fibromuscular hyperplasia of the

renal arteries is the primary cause of renovascular hypertension in women between the ages of 30 and 50 years. Atherosclerosis is the most common cause in men.

Study the following data on a test performed on 100 patients, who are known to have disease Y, and a control group containing 100 people who do not have disease Y. Which of the following conclusions can be drawn from this study? Disease YControl Group Positive test 70 0 Negative test30 100

A. A negative test result excludes disease Y. B. A positive test result confirms disease Y. C. It is a poor test for confirming disease Y. D. The predictive value of a negative test result is 100%. E. The prevalence of disease is 60%.

Option B (A positive test result confirms disease Y.) is correct. People with disease either have a true positive (TP) or a false negative (FN) test result. A TP test result is a positive test result in a person with disease, while a FN test result is a negative test result in a person with disease. People in a control group, who do not have disease, either have a true negative (TN) or a false positive (FP) test result. A TN test result is a negative test in a person without disease, while a FP test result is a positive test in a person without disease. Specificity of a test refers to the likelihood of having negative test results in people without disease. Since people without disease have test results that are either TN or FP, the formula for calculating the specificity of a test is TN/TN+FP. A test with no FPs has 100% specificity. The predictive value of a positive test result (PV+) is the likelihood that a positive test result is a TP rather than a FP; therefore, the formula for calculating the PV+ is TP/TP+FP. Tests with 100% specificity always have a PV+ of 100%; therefore, they are most useful for confirming disease. The specificity of the test for disease Y is 100/100+0 = 100%, and the PV+ is 70/70+0 = 100%. Disease YControl Group Positive test 70 TP 0 FP Negative test30 FN 100 TN

Option A (A negative test result excludes disease Y.) is incorrect. The sensitivity of a test is the likelihood of having positive test results in patients who have a selected disease. Since people with disease either have TP or FN test results, the formula for calculating the sensitivity of a test is TP/ TP+FN. The less the FN rate, the greater the sensitivity of the test. Tests with 100% sensitivity are most often used as screening tests for disease.

If the test result is negative, the likelihood (predictive value of a negative test result, PV) is a TN rather than a FN is 100%, because a test with 100% sensitivity has no FNs. Since tests with 100% sensitivity always have a PV (TN/TN+FN) of 100%, a negative test result excludes disease. The sensitivity of the test for disease Y is 70/70+30 = 70%, and the PV is 100/100+30 = 77%; therefore, a negative test result does not exclude disease Y (likelihood of a FN is 23%). Option C (It is a poor test for confirming disease Y.) is incorrect. The test is an excellent test to confirm disease, because the specificity is 100% and the PV+ is 100%. Option D (The predictive value of a negative test result is 100%.) is incorrect. The PV is 100/100+30 = 77% (not 100%). Option E (The prevalence of disease is 60%.) is incorrect. Prevalence is the total number of people with disease in the population under study. The formula is TP+FN/TP+FN+TN+FP. The prevalence of disease in this study is 100+0/100+0+70+30 = 50% (not 60%).

A mother notes in her 1-month-old baby girl clear fluid in the umbilicus. She brings this to the attention of her pediatrician, who not only verifies the presence of the fluid but also palpates a midline mass. At surgery a cystic mass is noted that connects to the urinary bladder. What is the most likely diagnosis?

A. Exstrophy of the bladder B. Omphalocele C. Persistence of vitelline duct D. Urachal sinus and cyst

Option D (Urachal sinus and cyst) is correct. The urachus is an embryologic canal that connects the urinary bladder of the fetus with the umbilical cord. Usually the lumen of the urachus becomes obliterated during embryonic development and becomes a solid cord. However, if the lumen remains patent, then urine (clear fluid in this patient) will leak from the umbilicus. This patient also developed a urachal cyst. If remnants of the urachus remain behind in the bladder, there is a danger for developing an adenocarcinoma; hence, the importance for the surgeon to remove the remnants. Option A (Exstrophy of the bladder) is incorrect. Exstrophy of the bladder is a developmental failure of the anterior abdominal wall and bladder where bladder mucosa is exposed to the body surface. Complications include inflammation predisposing to glandular metaplasia with a predisposition for developing adenocarcinoma of the bladder. Option B (Omphalocele) is incorrect. An omphalocele is a protrusion at birth of part of the intestine through a large defect in the abdominal wall at the umbilicus. It is usually covered by a transparent membrane composed of amnion and peritoneum.

Option C (Persistence of vitelline duct) is incorrect. The vitelline (omphalomesenteric) duct is an opening to the digestive tract that arises from the yolk sac. Usually the duct obliterates in the embryonic period; however, if its proximal part persists, it does so as a diverticulum from the small intestine called a Meckel diverticulum. Persistence of the duct can also appear as a draining fistula (fecal material) at the umbilicus.

A 20-year-old man complains of muscle fatigue when exercising. Occasionally, after an especially strenuous workout, his urine is red. The serum glucose and serum lactate are normal. After the patient exercises on a treadmill, the serum lactate value is unchanged from the pre-exercise value. The patient most likely has a deficiency of which of the following enzymes?

A. -1,4-Glucosidase B. Glucose-6-phosphatase C. Glucosyl-4,6 transferase D. Liver phosphorylase E. Muscle phosphorylase

Option E (Muscle phosphorylase) is correct. The patient has McArdles disease, which is an autosomal recessive glycogen storage disease characterized by a deficiency of muscle phosphorylase. This enzyme releases glucose from glycogen stored in muscle. Deficiency of muscle phosphorylase deprives the muscle of glucose for energy, and after exercise, lactic acid is not produced as an end-product of anaerobic glycolysis. Strenuous exercise may damage muscle, causing myoglobinuria (red-colored urine). There is no hypoglycemia in these patients, because glycogenolysis in the liver is still intact. Glucose obtained by glycogenolysis in muscle is used only by the muscle and does not enter the bloodstream. Option A (-1,4-Glucosidase) is incorrect. -1,4-Glucosidase, a lysosomal enzyme that degrades glycogen, is deficient in Pompes disease. Glycogen accumulates in cardiac muscle, resulting in death at an early age. Option B (Glucose-6-phosphatase) is incorrect. Glucose-6-phosphatase, a gluconeogenic enzyme that normally converts glucose 6-phosphate to glucose, is deficient in von Gierkes disease. Fasting hypoglycemia occurs, and normal glycogen (with branches) accumulates in the liver and kidneys, not in muscle. Option C (Glucosyl-4,6 transferase) is incorrect. Glucosyl-4,6 transferase, a branching enzyme, is used in the synthesis of glycogen. Abnormal glycogen (no branches) is produced. Cirrhosis and liver failure develop, and death occurs by 2 years of age. Option D (Liver phosphorylase) is incorrect. Absence of liver phosphorylase, which degrades glycogen in the liver, leads to fasting hypoglycemia. This is not present in this patient, because the liver (not muscle) normally helps maintain blood glucose during the fasting state.

A professional bodybuilder has a serum creatinine of 6 mg/dL (0.61.2 mg/dL) and a normal serum blood urea nitrogen (BUN). His urine volume in 24 hours is normal. A complete urinalysis is reported as normal. Which of the following best explains the increased serum creatinine?

A. Acute renal failure (ARF) B. Creatine supplementation C. Excess protein intake D. Excess vitamin intake E. Prerenal azotemia
Option B (Creatine supplementation) is correct. Creatinine is the metabolic end-product of creatine in muscle. Creatine phosphate is the main storage depot for energy in muscle in that it is a ready source of phosphate for adenosine triphosphate (ATP) synthesis. Creatine supplementation provides increased energy for bodybuilders and causes an increase in the serum creatinine. Option A (Acute renal failure (ARF)) is incorrect. ARF is associated with an increase in serum creatinine and serum BUN and the urine sediment shows renal tubular cell casts. The patient has a normal serum BUN, the urinalysis is normal, and there is no oliguria, hence excluding ARF as a diagnosis. Option C (Excess protein intake) is incorrect. Bodybuilders commonly increase protein intake to supply amino acids for muscle growth. However, the end-product of amino acid metabolism is urea not creatinine. Option D (Excess vitamin intake) is incorrect. Bodybuilders commonly take numerous vitamins to increase energy; however, vitamins have no effect on increasing serum creatinine. Option E (Prerenal azotemia) is incorrect. In prerenal azotemia there is oliguria and an increased serum BUN and creatine with a ratio > 15.

A 23-year-old man is scuba diving in 60 feet of water and suddenly develops stabbing chest pain in the left side with dyspnea while coming slowly to the surface. Physical examination of the left lung shows hyper-resonance to percussion, elevation of the left diaphragm, absent tactile fremitus, and absent breath sounds. Which of the following is the most likely diagnosis?

A. B. C.

Decompression sickness Pleural effusion Pulmonary infarction

D. E.

Spontaneous pneumothorax Tension pneumothorax

Option D (Spontaneous pneumothorax) is correct. A pulmonary complication of scuba diving is rupture of a preexisting intrapleural bleb or a subpleural bleb, causing a hole in the pleura and a spontaneous pneumothorax. A hole in the pleura causes a loss of negative pressure in the pleural cavity, which collapses all or part of the lung. Physical findings include hyper-resonance to percussion, tracheal deviation to the side of the collapse (if the whole lung is collapsed), elevation of the diaphragm, absent breath sounds, and absent vocal tactile fremitus. Option A (Decompression sickness) is incorrect. Decompression sickness (gas embolism) is a complication of scuba diving. As a diver descends, nitrogen gas under increased pressure moves from the alveoli and dissolves in tissue and blood. Rapid ascent forces nitrogen to come out of the tissue and blood in the form of bubbles, causing ischemic damage. A spontaneous pneumothorax is not a complication of gas embolism. Option B (Pleural effusion) is incorrect. Although a pulmonary thromboembolism leading to a pulmonary infarction and pleural effusion is a complication of scuba diving, it usually occurs when the diver is stationary and in deep water. Physical findings of a pleural effusion include dullness to percussion, deviation of the trachea to the contralateral side, and absent breath sounds. Option C (Pulmonary infarction) is incorrect. Although a pulmonary thromboembolism leading to a pulmonary infarction and pleural effusion is a complication of scuba diving, it usually occurs when the diver is stationary and in deep water. Clinical findings in a pulmonary infarction are pleuritic chest pain and dyspnea. Physical examination would not show hyper-resonance to percussion and elevation of the diaphragm. Option E (Tension pneumothorax) is incorrect. A tension pneumothorax is not a common cause of dyspnea in scuba diving. Unlike a spontaneous pneumothorax, a tension pneumothorax is associated with a flaplike pleural tear. Inspiration causes the flap to open and allow air to enter the pleural cavity. However, the flap closes on expiration and prevents the air from leaving the cavity. Increased intrapleural pressure (greater than the atmosphere) causes compression of the lung (atelectasis) and deviation of the trachea to the contralateral side. The diaphragm is depressed (not elevated).

S-ar putea să vă placă și